You are on page 1of 197

AMERICAN PSYCHIATRIC ASSOCIATION

FOCUS Psychiatry Review:


400 Self-Assessment Questions

A workbook with questions covering the


ABPN* outline of topics for recertification:

Anxiety Disorders
Bipolar Disorder
Child and Adolescent Psychiatry
Clinical Neuroscience and Genetics
Forensic and Ethical Issues in Psychiatry
Gender, Race, and Culture
Geriatric Psychiatry
Major Depressive Disorders
Personality Disorders
Posttraumatic Stress Disorder
Psychopharmacology
Psychosomatic Medicine
Psycho therapy
Schizophrenia
Sleep, Sex, and Eating Disotders
Substance-Related Disorders

Editors

Deborah']. Hales, M.D.


Mark Hyman Rapaport, M.D.
AMERICAN PSYCHIATRIC ASSOCIATION A V . J > *

FOCUS Psychiatry Review: . ^JJJ


400 Self-Assessment Questions

A workbook with questions covering the


ABPN outline of topics for recertification:

Anxiety Disorders
Bipolar Disorder
Child and Adolescent Psychiatry
Clinical Neuroscience and Genetics
Forensic and Ethical Issues in Psychiatry
Gender, Race, and Culture
Geriatric Psychiatry
Major Depressive Disorders
Personality Disorders
Posttraumatic Stress Disorder
Psychopharmacology
Psychosomatic Medicine
Psychotherapy
Schizophrenia
Sleep, Sex, and Eating Disorders
Substance-Related Disorders

'-Editors

Deborah" J. Hales, M.D.


Mark Hyman Rapaport, M.D.

1844
Note: The authors have worked to ensure that all information in this';book concerning drug dosages, schedules, and routes of
administration is accurate as of the time of publication and consistent with standards set by the U.S. Food and Drug
Administration and the general medical community. As medical research and practice advance, however, therapeutic standards
may change. For this reason and because human and mechanical errors sometimes occur, we recommend that readers follow the
advice of a physician who is directly involved in their care or the care of a member of their family.

Books published by American Psychiatric Association represent the views and opinions of the individual authors and do not
necessarily represent the policies and opinions of the American Psychiatric Association.

Copyright © 2007 American Psychiatric Association

ALL RIGHTS RESERVED


• • i t

Manufactured in the United States of America on acid-free paper

American Psychiatric Association .


1000 Wilson Boulevard
Arlington, VA 22209-3901
www.psych.org

ISBN-13: 978-0-89042-297-7
Contents
>

Introduction vii

C M E Form Ix

Section 1: Self-Assessrherit Questions 1

Section 2: Answers and Explanations ( 59

Answer Sheet 185

Blank Answer Sheet 187

Editors and Editorial Board Affiliations and Disclosures 189

Index : 191
Introduction
In todays world it is hard to keep up with the explosive growth in knowledge. Psychiatric practice is rap-
idly improving thanks to developments in evidence-based practice and advances in neuroscience
research. The editors of FOCUS developed the FOCUS Psychiatry Review as an aid for psychiatrists in
lifelong learning in the .field. This workbook contains 400 board-type multiple-choice questions from
FOCUS'S annual Self-Assessment Examinations that can help psychiatrists prepare for examinations and
identify areas for further study. The questions, developed by the FOCUS self-assessment board, are con-
sistent in form and process with the questions used by high-stakes examinations. They cover important
clinical areas of psychiatric practice and closely follow the American Board of Psychiatry and Neurology
(ABPN) outline of topics for the recertification examination in psychiatry.

The FOCUS Psychiatry Review is designed to test current knowledge and its clinical application. The
workbook is flexible in format, allowing readers to use the educational approach that works best for
them. Readers can review resource materials prior to answering questions, or they can use the workbook
to review the references listed in the critiques after scoring test sections.

The workbook will be useful for anyone committed to lifelong learning in the field—psychiatric resi-
dents, practicing psychiatrists, and psychiatrists preparing for examinations.

• The FOCUS Psychiatry Review contains 400 clinical questions that can be used to identify areas
of strength and weakness.
• It provides up-to-date critiques and current references to facilitate further study.
• It is a complementary component to a larger overall program of lifelong learning for the psychia-
trist who wants to keep current in the field.

This edition covers the following topics:


Anxiety Disorders Personality Disorders
Bipolar Disorder Posttraumatic Stress Disorder
Child and Adolescent Psychiatry Psychopharmacology
Clinical Neuroscience and Genetics Psychosomatic Medicine
Forensic and Ethical Issues in Psychiatry Psychotherapy ^
Gender, Race, and Culture Schizophrenia
Geriatric Psychiatry Sleep, Sex, and Eating Disorders
Major Depressive Disorders Substance-Related Disorders

The FOCUS Psychiatry Review provides up to 50 hours of Continuing Medical Education Credit.

APA is accredited by ACCME to provide continuing medical education for physicians.


APA designates this educational activity for a maximum of 50 MIA PRA Category 1 credits. Physicians should
claim credit commensurate with the extent of their participation in the activity.

Educational Objectives

At the completion of the activity, participants will


1) have an increased understanding of new developments in psychiatric diagnosis and treatment;
2) be aware of resources available for learning more about these developments; and
3) recognize areas of strength and areas where more study is needed.

vu
To obtain Continuing Medical Education Credit, complete ana sena UKS yayc ^ .

American Psychiatric Association


Department of C M E >
1000 Wilson Blvd., Suite 1825
Arlington, VA 22209
Fax:703-907-7849
plee@psych.org

Begin date: J u l y 2006 , •


E n d date: J u l y 2009

AMERICAN PSYCHIATRIC ASSOCIATION

FOCUS Psychiatry Review:


400 Self-Assessment Questions'
I have participated for hours (up to 50) in completion of this CME activity.

Name (please print):

Address:

Address:

City/State/Zip: ;

E-mail Fax

Please send my certificate by. Mail Fax E-mail

1. The quality of the FOCUS Psychiatry Review workbook was excellent.


Strongly agree Agree Disagree Strongly disagree

2. The FOCUS Psychiatry Review workbook was useful to me in preparing for examinations.
Strongly agree Agree Disagree Strongly disagree

3. The workbook was useful in helping me understand my areas of strength and weakness.
Strongly agree Agree Disagree Strongly disagree

4. The workbook will be helpful to me in my clinical practice.


Strongly agree Agree Disagree- Strongly disagree

5. The material in the FOCUS Psychiatry Review was presented without bias.

Strongly agree Agree Disagree Strongly disagree

6. The questions were: too hard , just right , too easy .

Comments:
Section 1: Self-Assessment Questions
(£) EEFL

A forensic psychiatric evaluation differs from a genera! W h i c h of the following antidepressants w o u l d be the
psychiatric evaluation in that a forensic evaluation: best choice for a patient concerned about erectile
dysfunction?
(A) typically includes a mental status examination.
(B) does not hove a doctor-patient relationship. (A) Bupropion
(C) requires a completed written report. (B) Fluoxetine
( 0 ) requires the presence of a lawyer during the evaluation. (C) Nortriptyline
(D) Imipramine
(E) Venlafaxine

A 30-year-old man reports that he is U J I A ^ l e j o j e e p


a n d h e j j ^ n p j s e s ^ n d j / o j c ^ at Jiight even though he
rm
lives alone. T h e symptoms started abruptly on the d a y The N a t i o n a l Comorbidity Survey identified a number
preceding the visit. During the interview, he repeat- of g e n d e r differences in exposure a n d in the develop-
edly brushes,off hjs _arms, m u j t e r j n g _ 5 b p j j t ^ The ment of posttraumatic stress disorder (PTSD). C o m -
information that would be most helpful in determining p a r e d with females, males h a v e :
initial interventions would be the history of:
(A) higher trauma exposure, and higher prevalence of PTSD.
(A) family disorders. (B) lower trauma exposure, and lower prevalence of PTSD.
(B) medical problems. 1 (C) higher trauma exposure, and lower prevalence of PTSD.
(C) psychiatric hospitalization. (D) lower trauma exposure, and higher prevalence of PTSD.
(D) recent stresses. (E) the some trauma exposure, and the some prevalence of PTSD.

@ 0 3 0 ,.

T h e practice of obtaining informed consent from an A cancer patient with significant nausea requires an
individual prior to initiating a n y treatment fulfills antidepressant. W h i c h of the following medications
w h i c h of the following ethical principles? w o u l d be the best choice?

(A) Nonmaleficence (A) Bupropion


(B) Autonomy (B) Duloxetii.?
(C) Justice (C) Mirtazapine
(D) Competence (D) Paroxetine
(E) Venlafaxine

W h i c h of the following psychotherapies has the best m


documented effectiveness in the treatment of major W h e n non-substance abusing men a n d w o m e n drink
depressive disorder? the same amount of alcohol, the w o m e n are likely to
have higher alcohol blood levels than the men. The best
(A) Supportive
explanation for this is that compared with men, w o m e n :
(B) Psychodynamic
(C) Interpersonal (A) have a larger volume of distribution.
(D) Psychoeducational (B) have lower excretion rates.
(E) Family (C) only metabolize by first-order kinetics.
(D) metabolize less alcohol nj the gut.
(E) are deficient in acetaldehyde dehydrogenase.

4 FOCUS Psychiatry Review 400 Self-Assessment Quest?, is


BjB
W h i c h of the following situations best describes w h e n W h i c h of the following disorders has the highest rela-
weight considerations should determine hospitalization tive ris,k for first-degree relatives?
for anorexia nervosa in children and young adolescents?
(A) Alcoholism
(A) Weight is less than 20% of recommended healthy body (B) Anorexia
weight. (C) Bipolar disorder
(B) Weight is less than 25% of ideal body weight. (D) Panic disorder
(C) Weight is being rapidly lost and outpatient efforts are inef- (E) Somatization disorder
fective, regardless of actual weight.
(D) The family asks for hospitalization.
(E) Weight is fluctuating unpredictably over 2-3 months.

A 68-year-old man has a g r a n d mal seizure that is

m attributed to an abrupt hyponatremia, with a serum


sodium concentration of 110 m m o l / L W h i c h of the
W h i c h of the following antipsychotic drugs is most following medications is the most likely cause?
likely to be associated with hyperprolactinemia?
(A) Gabapentin
(A) Aripiprazole (B) lithium
(B) Clozapine (C) Oxcarbazepine
(C) Olanzapine (D) Topiramate
(D) Quetiapine (E) Valproate
(E) Risperidone

m Social rhythm therapy, w h i c h is designed specifically


W h i c h of the following atypical antipsychotic drugs is for bipolar disorder, is based on which of the follow-
a D receptor partial agonist?
2 ing models?

(A) Aripiprazole (A) Psychoeducation


(B) Olanzapine (B) Object relations and self psychology theory
(C) Quetiapine (C) Orcadian regulation and interpersonal psychotherapy
(D) Risperidone (D) Cognitive therapy techniques to cddress social dysfunction
(E) Ziprasidone (E) Supportive psychotherapy

m
A 33-year-old man started twice-weekly psychody- Rapid cycling in bipolar I or II disorder is associated
namic psychotherapy 6 months a g o with the goal of with:
exploring issues stemming from his distant relationship
(A) menopause.
with his father and his inability to form adequate men-
(B) antidepressant use.
toring relationships in "his w o r k as a research chemist.
(C) cocaine abuse.
He reports an increasing preoccupation with his ther-
(D) early onset.
apist's unwillingness to see him more frequently. The
(E) alcohol abuse.
patient has been speaking in therapy of his wish that
the therapist see him on Sunday. He believes that the
therapist refuses to have extra sessions because he
prefers other patients. W h i c h of the following best
explains the patient's behavior?

(A) Transference neurosis


(B) Delusional system
(C) Obsessional diathesis
(0) Erotomania
(E) Psychotic distortion

Secrior. I: Self-Assessrr.sn: Questions 5


® 33
A psychiatrist attends a dinner lecture sponsored by a W h i c h of the following diseases associated with
major pharmaceutical c o m p a n y the maker of a n e w l y dementia characteristically has early changes in per-
a p p r o v e d drug for major depression. T h e c o m p a n y ' s sonality a n d a late decline in memory?
representative approaches the psychiatrist after the
(A) HIV infection
lecture a n d says "I hope we can count on y o u to pre-
(

scribe our medication. This is a great medication!" (B) Creutzfeldt-Jakob disease


The psychiatrist does not know w h a t to say a n d later (C) Parkinson's disease
1
feels troubled by this encounter. W h i c h of the follow- (D) Lewy body dementia
ing statements reflects the psychiatrist's ethical obli- (E), Pick's disease
gation in this situation?

(A) The psychiatrist can accept dinners and "repay" the com-
pany with favorable prescribing practices if the psychiatrist 34
chooses to do so.
' W h i c h of the following features best distinguishes
(B) The psychiatrist should report the pharmaceutical represen-
anorexia nervosa from bulimia nervosa?
tative's behavior to the local APA branch's ethics committee.
(C) The psychiatrist should be aware that "strings attached" (A) Amenorrhea
industry-sponsored activities are unethical. (B) Decreased body weight
(D) The psychiatrist must repay the representative for the cost (C) Calluses on the dorsum of the hand
of the dinner, since there are apparent, though unstated, (D) Dental enamel erosion
ethical conflicts. (E) Enlarged parotid glands

31 35
More severe and prolonged forms of conduct disorder W h i c h of the following aspects of cognitive performance
are most often associated with w h i c h of the following is most likely to decline in the course of normal aging?
comorbid disorders?
(A) Short-term memory
(A) Anxiety disorders (B) Speed of performance
(B) Attention deficit hyperactivity disorder (C) Store of knowledge
((} Depression (D) Syntax
(D) Eating disorder (E) Vocabulary
(E) Tic disorder

36
32
Disorders with significant psychiatric symptoms that
A 62-year-old man is taking desipramine for depres- can be linked to a single gene include:
sion. He presents with marked sedation, tachycardia,
(A) attention deficit hyperactivity disorder.
and postural hypotension about 10 days after the
(B) bipolar disorder.
addition of a second antidepressant. W h i c h of the fol-
(C) fragile X syndrome.
lowing medications is most likely responsible?
(D) major depression.
(A) Venlafaxine (E) schizophrenia.
(B) Mirtazapine
(C) Citalopram.
(D) Sertraline
(E) Fluoxetine 37
The rule of confidentiality is w a i v e d in a psychiatrist-
pctient interaction w h e n the treatment or evaluation
includes:

(A) a minor.
(B) a forensic consultation.
(C) an impaired physician.
(D) a patient who reveals a past felony.

Section 1: Sdf-Assissmcnc Questions 7


38 B23 .
T h e highest percentage of persons with mental retar- W h i c h of the following medications is considered first-
dation h a v e an intelligence quotient of: line monotherapy for posttraumatic stress disorder?

(A) <20. (A) Clonazepam


IB] 20 to 35. (B) Sertraline
(C) 35 to 50. (C) Olanzapine
(D) 50 to 70. (D) Valproate
(E) 70 to 90. (E) Propranolol

39 m
A 4-year-old girl w h o has been c a r e d for in seven dif- A 50-year-old w o m a n has a long history of difficulty
ferent foster homes since the a g e of 6 months, n o w with driving because she worries that she might hit a
exhibits excessive familiarity with strangers. Her cur- car or a person accidentally. She also worries exces-
rent foster parents, with w h o m she has lived for the sively about her son getting hurt or attacked w h e n he
past 5 months, state that she does not seem to be par- qoes out. Her husband can often reassure her. W h i c h
ticularly close to them. T h e girl's biological mother is of the following diagnoses is most appropriate?
reported to have used alcohol in a binge pattern dur-
(A) Agoraphobia •
ing her pregnancy. W h i c h of the following is the most
(B) Delusional disorder
likely diagnosis?
(C) Generalized anxiety disorder
(A) Attention deficit hyperactivity disorder (D) Obsessive-compulsive disorder
(B) Fetal alcohol syndrome (E) Panic disorder
(C) Oppositional defiant disorder
(D) Pervasive developmental disorder
(E) Reactive attachment disorder

03?
A 40-year-old w o m a n with chronic headaches has
undergone trials with several narcotic a n d nonnar-
40
cotic agents with variable success. H e r physician
An 8-year-old girl teiists on keeping a rigid routine elects to try her on a newer antidepressant medica-
w h e n dressing, will w e u ?nly certain clothes, insists tion. W h i c h of the following medications is most likely
on recopying her h o m e w o r k if there are a n y mistakes, to be effective?
a n d has temper tantrums w h e n the items on her desk
a r e m o v e d . During a discussion of the diagnosis and (A) Bupropion ^
treatment options, her parents express reluctance to (B) Mirtazapine
use medication a n d w a n t to e x p l o r e other options. (C) Nefazodone
T h e first recommendation w o u l d be: (D) Sertraline
(E) Veniofoxine
(A) cognitive behavior therapy.
(B) family therapy.
(C) interpersonal psychotherapy.
(D) parent training.
(E) supportive psychotherapy.
m
Echolalia a n d e c h o p r a x i a are most likely manifesta-
tions of w h i c h of the following disorders?

(A) Hypochondriasis
10 _ (B) Bipolar disorder, mixed episode
(C) Depression with catatonic features
W h i c h of the following are c o m m o n hyperarousa
(D) Lewy body dementia •
symptoms in posttraumatic stress disorder (£TSD|?
(E) Frontolemporal dementia
(A) Intense psychological distress at exposure to external cues
resembling the trauma
(B) Difficulty falling or staying asleep
(C) Intrusive images of the event *
(D) Feelings of estrangement from others

8 FOCUS Psychiatry Review. 400 Self-Assessment Questions


'<S2 A 75-year-old retired physicist w h o is suffering from
A 65-year-old w o m a n has a history of a left frontal
lobe stroke. W h i c h of the following psychiatric symp- metastatic cancer is referred to a psychiatrist by the pri-
toms is most commonly associated with a stroke in this mary care physician because the patient wants to die
area of the brain? and has requested assistance in suicide. On evaluation,
the psychiatrist finds that the patient's cognition is intact.
(A) Panic The most appropriate next step for the psychiatrist is to:
(B) Mania
((] Depression (A) be as persuasive as possible so that the patient accepts the
(D) Obsessions cancer treatment.
(E) Anxiety (B) find out whether there are areas of suffering that can be
addressed by available palliative care measures.
(C) tell the referring physician that the patient can be given assis-
tance in suicide because the patient is a competent adult.
(D) tell the referring physician that even though the patient's
cognition appears intact, the patient is probably incapacitated
Genetic studies of obsessive-compulsive disorder have
by virtue of the unreasonable choice that is being mode.
revealed linkages to w h i c h of the following disorders?

(A) Alcohol dependence


(B) Schizophrenia
(C) Shared psychotic disorder
(§2
Of the following ethnic groups, which is at lowest risk
(D) Somotoform disorder
of completed suicide?
(E) Tourette s syndrome
(A) African Americans
(B) Asian Americons
•(C) Caucasian Americans
(D) Hispanic Americans
(E) Native Americans
A psychiatrist proposes to use an F D A - a p p r o v e d drug
r o t previously used for the treatment of mania
because it has biochemical properties similar to
known antimanic agents. T h e psychiatrist has also
read several articles describing open-label studies
A 15-year-old boy is referred for psychiatric evaluation
suggesting efficacy of the drug. T h e patient in ques-
after taking an overdose of an over-the-counter cold
tion has not responded to a n y agent thus far. T h e psy-
medication. The patient denies that this w a s a suicide
chiatrist must do which of the following?
attempt. The patient acknowledges that he hcs been
(A) Get an institutional review board approval, since what is having difficulties for about a year, since the separation
proposed is clinical research. of his parents. He often feels a n g r y and irritable, has
(B) Notify the FDA, since the drug is being used for a non-FDA- difficulty sleeping, has little appetite, has lost weight,
approved purpose. has little interest in his usual activities, and often wishes
(C) Obtain informed consent from the patient or from an he w a s dead. His grades have dropped to the point
appropriate proxy agent. that he is failing his courses. O v e r the past year, he has
(D) Wait until there is higher-quality data to support this use of been smoking 1-2 packs of cigarettes a day, drinking
the drug. to the point of intoxication on the weekends, and taking
over-the-counter cold medication to enhance the effects
of the alcohol. His past psychiatric history is significant
for attention deficit hyperactivity disorder ( A D H D ) , for
which he has a prescription for a stimulant medication.
He has not taken his medication as prescribed. Instead,
he.hoards the medication ,and then takes large quanti-
ties to experience a euphoric effect. W h i c h of the fol-
lowing medications would be the most efficacious in
addressing this patient's symptom constellation?

(A} Bupropion
(B) Citalopram
(C) Desipramine.
(D) Trazodone
(E) Venlafaxine

6 FOCUS Psychiatry Review: 400 Self-Assessment Questions


An adolescent female took an unknown drug at an all- W h i c h of the following psychotherapies has the great-
night dance party. She w a s brought to the e m e r g e n c y est b o d y ^ f evidence demonstrating efficacy for social
department for evaluation of altered mental status a n d phobia?
marked hyperthermia. W h i c h of the following w a s
(A) Insight-oriented psychotherapy
most likely the drug that w a s ingested?
(B) Interpersonal psychotherapy
(A) Ketcmine (C) Brief psychodynamic psychotherapy
(B) Methylenedioxymethamphetamire (MDMA) (D) Cognitive behavior psychotherapy
(C) Flunitrozepom (E). Supportive psychotherapy
(D) Gamma-hydroxybutyrate (GHB) X
(E) Phencyclidine(PCP)

@
m ' A patient w h o is completely deaf arrives with an
interpreter at the outpatient clinic for an evaluation of
A 23-year-old man w h o is hospitalized for psychosis depressed m o o d . You w i s h to k n o w about the
displays prominent, excessive, a n d purposeless motor patient's sleep quality. Of the following, which is the
activity together with peculiar voluntary movements. most appropriate w a y to w o r k with the interpreter a n d
On one occasion, he stands in the middle of the w a r d the patient?
immobile and mute. He demonstrates w a x y flexibility.
(A) Ask the interpreter, "How is she sleeping?"
T h e appropriate medical intervention is:
(B) Ask the interpreter, "Please ask her how she is sleeping."
(A) benztropine. (C) Look at the patient and ask, "How are you sleeping?"
(B) clonidine. (D) Loudly enunciate "How are you sleeping?" to the patient.
(C) lorazepam. (E) Write out "How are you sleeping?" and give it to the patient.
(D) propranolol.
(E) ziprasidone.

m An internist consults a psychiatrist because of his frus-


tration with an elderly patient w h o has a diagnosis of
A 49-year-old w o m a n is r e f e r e d for treatment of hypochondriasis. Medical tests are negative, but the
chronic, severe major depression. W h i c h of the follow- patient is unable to accept that he is not ill. T h e psy-
ing treatment approaches is most likely to be associ- chiatrist confirms the diagnosis of hypochondriasis.
ated with sustained improvement in her symptoms? W h i c h of the following is the best management strat-
e g y for a patient with hypochondriasis?
(A) Antidepressant medication plus psychotherapy
(B) Psychotherapy alone (A) Refer the patient to a more psychologically minded
(C) Antidepressant medication alone internist colleague.
(D) EG alone (B) Have regularly scheduled appointments with limited reas-
(E) ECF plus psychotherapy surance.
(C) See the patient as needed, but for a limited time.
(D) Instruct the patient to call only for urgent matters.
(E) Refer the patient for psychotherapy.

w
In people with typical left-brain dominance, the ability
to interpret the emotional tone of speech is a function
of the:

(A) left premotor cortex (Broca's area).


(B) right premotor cortex.
(C) left parietotemporal cortex (Wernicke's area).
(D) right parietotemporal cortex.
(E) anterior cingulate gyrus.

Section 1: Self-Asiessrr.e.ic Questions 9


@1
A 29-year-old woman presents for an initial evaluation. The Child Behavior Checklist is a commonly used
She describes periods of mood lability and unstable instrument completed by parents about their children's
interpersonal relationships, particularly with men. behaviors. In a study comparing the results from sub-
During periods of stress, she reports feeling a n g r y and ject g r o u p s obtained from multiple cultures, girls
" e m p t y " a n d sometimes scratches herself with sharp scored higher than boys across all cultures on w h i c h
items. Sleep is often a problem, and alprazolam has behavior scale?
b e e n helpful. In developing a treatment plan, which of
(A) Aggression
the following principles would be most appropriate?
(B) Anxious/depressed
(A) Restrict pharmacotherapy to antidepressants and mood (C) Attention problems
stabilizers (D) Delinquency
(B) Treat with multiple classes of medications for potential (E) Thought problems
future symptoms
(C) Target specific symptoms that are currently causing disruption
(D) Refuse to prescribe a benzodiazepine
(E) Withhold medications if the patient engages in acting out
behavior
Characteristic cognitive processes in persons with
obsessive-compulsive disorder include:

(A) above average spatial recognition.


(B) better memory for pleasant events.
(C) decreased capacity for selective attention.
A 45-year-old w o m a n with bipolar disorder w h o has
(D) impaired reality testing.
b e e n successfully maintained on lithium presents at
(E) normal confidence in one's own memory.
the clinic with the complaint of swelling in her ankles.
Examination reveals 2+ pitting e d e m a . Her serum
lithium level is 0.8 m E q / L The physician prescribes a
thiazide diuretic. Four days later the patient presents at
the emergency department with confusion, a coarse
tremor in her extremities, and ataxia. Her serum lithium In family studies of patients with schizophrenia, the
level is n o w 2.6 mEq/L. Urinalysis reveals a slightly ele- personality disorder that has been found to occur most
vated specific gravity and an absence of blood, frequently in first-degree relatives is:
ketones, a n d protein. W h i c h of the following best ex-
plains the patient's lithium toxicity? (A) borderline.
(B) histrionic.
(A) Acute nephrogenic diabetes insipidus (C) paranoid.
(B) Increased reabsorption in the proximal tubules (D) schizoid.
(C) Decreased glomerular filtration rate (E) schizotypal.
(D) Glomerulonephritis
(E) Tubulointerstitial nephropathy

The following vignette applies to questions 59 and 60.

A 25-year-old w o m a n presents to the e m e r g e n c y


department with the chief complaint, "I think I'm hav-
A r a n d o m community sample contains 100 individu- ing a heart attack." She reports that while g r o c e r y
als w h o meet diagnostic criteria for borderline per- shopping she suddenly felt "scared to d e a t h . " H e r
sonality disorder. W h i c h of the following is the. best heart w a s racing, she felt short of breath a n d d i z z y ,
estimate of the gender ratio of the sample? , a n d she w a s nauseated a n d broke out in a sweat. H e r
fingers a n d hands a n d the area around her mouth felt
(A) 50% men and 50% women numb. T h e episode lasted about 10 minutes a n d dis-
(B) 40% men and 60% women sipated on its o w n . She m a n a g e d to drive herself to
(C) 25% men and 75% women the e m e r g e n c y department. Physical examination a n d
(D) 10% men and 90% women ' laboratory studies, including a chest X-ray, blood
chemistries, cardiac e n z y m e s , a n d electrocardio-
g r a m , a r e normal.

10 FOCUS Psvchiatrv Review: 400 Self-Awssmenr Oupwnn-t


E33
In the lab, w h i c h of the following substances would be Expert consensus suggests that the length of time for a
most likely to induce an episode with these symptoms? pharmacological trial in obsessive-compulsive disor-
der should be at leash
(A) Carbon monoxide
(B) Sodium lactate (A) 3 weeks.
(C) Physostigmine (B) 6 weeks.
(D) Propranolol (C)9 week's.
(E) Sodium pyruvate (D) 12 weeks.

® EH
T h e medication that is most likely to be effective in the A 35-year-old man presents with a 4-week history of
long-term treatment of her condition with the best tol- low mood, crying spells, poor sleep with early morn-
erance of side effects is: ing a w a k e n i n g , poor appetite with a 12-pound
weight loss, and difficulty in concentrating at w o r k . At
(A) alprazolam.
a g e 27 he had been hospitalized with an episode of
(B) buspirone.
mania, but shortly thereafter he decided not to con-
(C) paroxetine.
tinue in outpatient follow-up treatment. He has no
(D) propranolol. medical problems a n d takes no medications. As initial
(E) imipramine. pharmacotherapeutic treatment, which of the follow-
ing is most appropriate?

(A) Lamofrigine
m (B) Nortriptyline
(C) Sertraline
A 38-year-old man with migraine headaches had suc-
(D) Valproate
cessfully obtained relief by taking codeine. Recently'
(E) Venlafaxine
his physician started him on a trial of paroxetine for
suspected depression. T h e patient notes improvement
in his symptoms of depression a n d n o w has head-
aches less frequently, but w h e n he does have one, he
$ 5 1
must take twice the amount of codeine for pain relief.
W h i c h of the following best describes this drug inter- A patient with borderline personality disorder is in
action? dialectical behavior therapy. She has left messages on
the therapist's voice-maii while he is on vacation
(A) Cytochrome P450 enzymes: inhibition despite an agreement that she would not call him at
(B) Cytochrome P450 enzymes: induction all during hij vacation and would go to the emer-
(C) Increased protein binding g e n c y department if she became suicidal. T h e best
(D) Decreased absorption approach in dialectical behavior therapy is for the
(E) Increased excretion therapist to:

(A) explain that a treatment boundary has been violated and


therapy will have to end.
555T (B) wait for the patient to bring up the issue before discussing
the implications for therapy.
W h i c h of the following antidepressants is most likely
(C) explain to the patient that the treatment plan will have to
to be associated with substantial weight gain?
change if she cannot keep the agreement.
(A) Bupropion (D) make an exception since there is a history of serious
(B) Fluoxetine attempts and safety is an issue.
(C) Sertraline
(D) Venlafoxine
(E) Mirtazapine

Section 1: Self-Assessment Questions II


resa
A patient being treated with interferon for hepatitis C A 27-year-old w o m a n has had five hospitalizations
complains of depression, anxiety, a n d irritability. over the 3-year period since she w a s initially diag-
W h i c h of the following pharmacological agents has nosed with schizophrenia. On each occasion, recur-
the most evidence for efficacy in treating those symp- rent psychotic symptoms have been associated with
toms? treatment nonadherence. W h i c h of the following
strategies is supported by the greatest body of
(A) Trazodone research evidence as the most likely to improve med-
(B) Haloperidol ication adherence for this patient?
(C) Risperidone
(D) Nefazodone (A) Cognitive-motivational interventions
(E) Sertraline (B) Insight-oriented psychotherapy
(C) Psychoeducational interventions
(D) Family therapy
(E) Supportive group psychotherapy
m
An 11-year-old girl is referred for an evaluation of
school problems. Her teachers a n d parents describe
her as argumentative, hostile, disrespectful, a n d diffi- w
cult. T h e girl often refuses to listen, will not obey W h i c h of the following is the most likely symptom in
instructions, does not do her w o r k , Has temper cocaine intoxication?
tantrums, and insists on having her o w n w a y . She has
been this w a y since preschool. T h e most likely diag- (A) Paranoid delusions
nosis is: (B) Hypotension
(C) Bradycardia
(A) antisocial personality disorder. • (D) Depersonalization
(B) attention deficit hyperactivity disorder.
(C) conduct disorder.
(D) intermittent explosive disorder.
(E) oppositional defiant disorder.

A consultation-liaison psychiatrist, on arriving on the


internal medicine hospital unit, learns that the patient's
nurse requested the consultation a n d that the attending
internist does not w a n t the consultation. Of the fol-
Patients with end-stage renal disease w h o are on lowing, the best action for the psychiatrist would be to:
hemodialysis are most likely to present with which of
the following psychiatric symptoms? (A) talk briefly with the nurse about why he or she considered
the consultation important.
(A) Major depression (B) apologize to the attending internist and leave the unit.
(B) Delirium (C) talk with the nurse's supervisor about the correct way to
(C) Psychosis request a consultation.
(D) Panic attacks (D) proceed with the consultation and make treatment recom-
(E) Generalized anxiety mendations.
(E) ask to have a case conference about the patient with the
physician and nursing staff.

12 FOCUS Psychiatry Review: 4 0 0 Self-Assessment OM«IVWK


W h i c h of the following accurately describes the major W h i c h of the following differentiates Lewy body
quality that fundamentally distinguishes brief dynamic dementia from dementia of the A l z h e i m e r ' s type?
psychotherapy from long-term d y n a m i c psychother-
(A) Apraxia
a p y ? Brief therapy has:
(B) Choreiform movements
(A) no more than five sessions. (C) Executive dysfunction
(B) limited focus and goals. . (0) Gradual progression of deficits
(C) less demonstrated efficacy. , (E) Recurrent visual hallucinations
(D) no transference or countertransference phenomena. .
(E) fewer demands on the therapist.

The symptom of "flashbacks" is a manifestation of


which of the following psychological states?
cm
A 68-year-old man with bipolar I disorder has been (A) Psychosis
adequately maintained on lithium. His most recent (B) Fugue
serum lithium level w a s 0.8 m E q / L He has a variety (C) Hyperarousal
of medical problems fonwfilcfThe takes several med- (D) Dissociation
ications. He n o w presents with pressured speech, rac-
ing thoughts, increased energy, a n d little sleep. His
serum lithium level is 0.3 m E q / L . His wife reports that
the patient has been adherent to his medication regi-
men, but she began to notice a c h a n g e 2 weeks after Response prevention is a useful psychotherapeutic
his primary care physician started him on a n e w med- technique for w h i c h of the following disorders?
ication. W h a t w a s the most likely class of medication
(A) Generalized anxiety disorder
a d d e d to his regimen?
(B) Intermittent explosive disorder
(A) Angiotensin-converting enzyme inhibitors (C) Obsessive-compulsive disorder
(B) Beta-blockers (D) Pedophilia
(C) Nonsteroidal anti-inflammatory drugs (E) Schizophrenia
(D) Thiazide diuretic
(E) Xanthine bronchodilators

(§5
A patient with alcoholism wants a psychiatrist to bill
the patient's insurance c o m p a n y under another diag-
A patient with an alcohol problem is ambivalent nosis because the patient is afraid of the stigma
about starting acamprosate. T h e psychiatrist explores attached to the diagnosis. T h e psychiatrist should:
the patient's thoughts about the advantages and dis-
(A) tell the patient that this would be lying and refuse to comply.
advantages of taking and not taking the medication,
(B) comply with the request because stigmas are inherently
attempting to tip the patient's decisional balance in
unfair to patients.
favor of taking the medication. W h i c h of the following
(C) comply with the request provided the patient's fears are
techniques is the physician using?
adequately addressed.
(A) Cognitive reframing (D) explore the reasons behind the request and explain why
(B) Contingency management this is something the psychiatrist is reluctant to do.
(C) Motivational enhancement
(D) Pessimistic anticipation
(E) Rational emotion 79
In a patient experiencing bereavement, w h i c h of the
following suggests the diagnosis of major depression?

(A) A poor appetite


(B) Initial insomnia
(C) A feeling of worthlessness
(D) Hallucinations of the deceased
(E) Sadness

Section 1: Self-Assessment Questions 13


W h i c h C N S structure is most responsible for arousal A psychiatrist decides that a patient with alcohol
a n d sleep-wake cycles? dependence w o u l d benefit from regular laboratory
monitoring. W h i c h of the following single tests w o u l d
(A) Amygdala best provide information about h e a v y alcohol use
(B) Hippocampus over the preceding 7 to 10 days?
(C) Hypothalamus
(0) Reticular activating system (A) Aspartate aminotransferase (AST)
(E) Ventral striatum (B) Carhohydrate-defident transferrin (CDT)
(C) Exhaled ethanol concentration [e.g., Breathalyzer)
(D) Mean corpuscular volume (MCV)

A 38-year-old patient provides a 12-year history of


obsessive concerns about dirt, germs, a n d contamina-
tion and spends more than 3 hours a d a y with w a s h -
A patient with major depression shows no improve-
ing and cleaning rituals. W h i c h of the following would
ment after an adequate trial (in dose and duration) of
be preferred as an initial medication treatment?
an antidepressant. T h e best next step is to:
(A) Desipramine
(A) augment the antidepressant with thyroid hormone.
(B) Duloxetine
(B) augment with lithium.
(C) Paroxetine
(C) augment with both thyroid hormone and lithium.
(D) Phenelzine
(D) switch to a different class of antidepressant.
(E) Venlafaxine
(E) conduct a "washout" by stopping all medication for
4 weeks, and then reassess.

A 59-year-old w o m a n is seen for an initial outpatient psy-


chiatric assessment. Her husband says that increasingly
over the past 2 years she has seemed less like her usual Of the following, w h i c h is the most common reason
outgoing self. She has been increasingly apathetic and psychiatrists a r e sued for malpractice?
uninterested in her usual activities, and more recently she
has behaved inappropriately in social interactions, mak- (A) Sexual improprieties with patients
ing unusual comments and returning home with items that (B) Suicide
do not belong to her. Recently, her husband has had to (C) Failure to obtain informed consent
begin helping her dress in the morning, and he notes that (D) Tardive dyskinesia
she is occasionally incontinent of urine. On mental status (E) Unnecessary commitment
examination, her affect is blunted and her speech is
sparse, although she does not report specific psychotic
symptoms or changes in mood. She knows the y e a r and
the season but not the month or date, and she has par-
ticular difficulty in naming objects. MRI shows prominent
A n o r e x i a nervosa is most commonly comorbid with
frontal and some temporal atrophy with relative sparing
which of the following personality disorders?
of other cortical regions. W h i c h of the following diag-
noses is most likely in this patient? (A) Dependent
(B) Paranoid
(A) Dementia of the Alzheimer's type
(C) Schizotypal
(B) Creutzfeldt-Jakob disease
(D) Obsessive-compulsive
(C) Dementia associated with Huntington's disease ,
(E) Histrionic
(D) Dementia associated with Parkinson's disease
(E) Pick's disease

14 FOCUS Psychiatry Review. 400 Self-Assessment Questions


W h i c h of the following accurately describes the major W h i c h of the following differentiates Lewy body
quality that fundamentally distinguishes brief dynamic dementia from dementia of the A l z h e i m e r ' s type?
psychotherapy from long-term dynamic psychother-
(A) Apraxia
apy? Brief therapy has:
(B) Choreiform movements
(A) no more than five sessions. (0 Executive dysfunction
(B) limited focus and goals. , (D) Gradual progression of deficits
(C) less demonstrated efficacy. , (E) Recurrent visual hallucinations
(D) no transference or countertransference phenomena. .
(E) fewer demands on the therapist.

C76). '
The symptom of "flashbacks" is a manifestation of
which of the following psychological states?

A 68-year-old man with bipolar I disorder has been (A) Psychosis


adequately maintained on lithium. His most recent (B) Fugue
serum lithium level w a s 0.8 m E q / L He has a variety [Q Hyperarousal
of medical problems for"whlclThe takes several med- (D) Dissociation
ications. He now presents with pressured speech, rac-
ing thoughts, increased energy, a n d little sleep. His
serum lithium level is 0.3 m E q / L . His wife reports that
the patient has been adherent to his medication regi-
77)
men, but she began to notice a c h a n g e 2 weeks after Response prevention is a useful psychotherapeutic
his primary care physician started him on a n e w med- technique for w h i c h of the following disorders?
ication. W h a t w a s the most likely class of medication
(A) Generalized anxiety disorder
a d d e d to his regimen?
(B) Intermittent explosive disorder
(A) Angiotensin-converting enzyme inhibitors (C) Obsessive-compulsive disorder
(B) Beta-blockers (D) Pedophilia
(C) Nonsteroidal anti-inflammatory drugs (E) Schizophrenia
(D) Thiazide diuretics
(E) Xanthine bronchodilators

cm
A patient with alcoholism wants a psychiatrist to bill
the patient's insurance c o m p a n y under another diag-
A patient with an alcohol problem is ambivalent nosis because the patient is afraid of the stigma
about starting acamprosate. T h e psychiatrist explores attached to the diagnosis. The psychiatrist should:
the patient's thoughts about the advantages and dis-
(A) tell the patient that this would be lying and refuse to comply.
advantages of taking and not taking the medication,
(B) comply with the request because stigmas are inherently
attempting to tip the patient's decisional balance in
unfair to patients.
favor of taking the medication. W h i c h of the following
(C) comply with the request provided the patient's fears are
techniques is the physician using?
adequately addressed.
(A) Cognitive reframing (D) explore the reasons behind the request and explain why
(B) Contingency management this is something the psychiatrist is reluctant to do.
(C) Motivational enhancement
(D) Pessimistic anticipation
(E) Rational emotion 79
In a patient experiencing bereavement, w h i c h of the
following suggests the diagnosis of major depression?

(A) A poor appetite


(B) Initial insomnia
(C) A feeling of worthlessness
(0) Hallucinations of the deceased
(E) Sadness

Seccion 1: Self-Assessment Questions 13


W h i c h C N S structure is most responsible for arousal A psychiatrist decides that a patient with alcohol
a n d sleep-wake cycles? dependence w o u l d benefit from regular laboratory
monitoring. W h i c h of the following single tests w o u l d
(A) Amygdala best provide information about heavy alcohol use
(!) Hippocampus over the preceding 7 to 10 days?
(C) Hypothalamus
(D) Reticular activating system (A) Aspartate aminotransferase (AST)
(E) Ventral striatum (B) Carbohydrate-deficient transferrin (CDT)
(C) Exhaled ethanol concentration [e.g., Breathalyzer)
(D) Mean corpuscular volume (MCV)

A 38-year-old patient provides a 12-year history of


obsessive concerns about dirt, germs, and contamina-
tion and spends more than 3 hours a d a y with wash-
2
A patient with major depression shows no improve-
ing and cleaning rituals. W h i c h of the following would
ment after an adequate trial (in dose and duration) of
be preferred as an initial medication treatment?
an antidepressant. T h e best next step is to:
(A) Desipramine
(A) augment the antidepressant with thyroid hormone.
(B) Ouloxetine
(B) augment with lithium.
(C) Paroxetine
(C) augment with both thyroid hormone and lithium.
(D) Phenelzine
(D) switch to a different class of antidepressant.
(E) Venlafaxine
(E) conduct a "washout" by stopping a I medication for
4 weeks, and then reassess.

A 59-year-old w o m a n is seen for an initial outpatient psy-


chiatric assessment. Her husband says that increasingly
over the past 2 years she has seemed less like her usual Of the following, w h i c h is the most common reason
outgoing self. She has been increasingly apathetic and psychiatrists are sued for malpractice?
uninterested in her usual activities, and more recently she
has behaved inappropriately in social interactions, mak- (A) Sexual improprieties WITH patients
ing unusual comments and returning home with items that (B) Suicide
do not belong to her. Recently, her husband has had to (C) Failure to obtain informed consent
begin helping her dress in the morning, and he notes that (D) Tardive dyskinesia
she is occasionally incontinent of urine. On mental status (E) Unnecessary commitment
examination, her affect is blunted and her speech is
sparse, although she does not report specific psychotic
symptoms or changes in mood. She knows the y e a r and
the season but not the month or date, and she has par-
ticular difficulty in naming objects. MRI shows prominent
A n o r e x i a nervosa is most commonly comorbid with
frontal and some temporal atrophy with relative sparing
which of the following personality disorders?
of other cortical regions. W h i c h of the following diag-
noses is most likely in this patient? (A) Dependent
(B) Paranoid
(A) Dementia of the Alzheimer's type
(C) Schizotypal
(B) (reutzfeldt-Jakob disease
(D) Obsessive-compulsive
(C) Dementia associated with Huntington's disease ,
(E) Histrionic
(D) Dementia associated with Parkinson's disease
(E) Pick's disease

14 FOCUS Psychiatry Review: 400 Self-Assessment Qu


91
T h e use of which of the following has been associated A 48-year-old m a n with a medical history of gastro-
with hyperparathyroidism? esophageal reflux disease (GERD] is referred for a
psychiatric evaluation of his anxiety. For the past
(A) Lamotrigine
month, since the patient's initial evaluation and treat-
(B) Divalproex
ment for G E R D , he complains'of an increasing sense
(0 lithium of unease, nervousness, restlessness, a n d inability to
(D) Topiramate sit and read the paper. His medications include
2 0 m g / d a y of e s o m e p r a z o l e , 10 mg of metoclo-
pramide q.i.d., a n d 0.5 mg of l o r a z e p a m t.i.d. orally
or as needed. He is v e r y concerned about his condi-
tion because a sibling w h o had a similar problem
N a r c o l e p s y is characterized by w h i c h of the following died from e s o p h a g e a l carcinoma. Other than being
signs and symptoms? noticeably fidgety, his mental status e x a m is unre-
markable. W h a t is the most likely explanation?
(A) Daytime nonrefreshing sleep episodes
(B) Bouts of urinary incontinence (A) Development of generalized anxiety disorder
(C) Early morning awakening (B) Adjustment disorder with anxious features
(D) Sleepwalking (C) Somatoform disorder not otherwise specified [i.e., "sympa-
(E) Sudden episodes of muscle tone loss thy symptoms" with deceased sibling)
(D) Akathisia from metoclopramide
(E) Benzodiazepine withdrawal

T h e four major components of a psychodynamic v i e w


92
of personality disorders are a biologically based tem-
peramenr, a set of internalized object relations, an A 30-year-old patient with no prior history of mental
enduring sense of self, a n d : health treatment presents with a major depressive
episode. W h i c h of the following elements would be the
(A) an assessment of realrty testing. most important in choosing a medication for treatment?
(B) a punitive superego.
(C) an intact ego ideal. (A) Co-occurring diagnosis of alcohol dependence in full sus-
(D) a specific constellation of defense mechanisms. tained remission
(B) Good antidepressant response to fluoxetine in a first-
degree relative
(C) History of a hypomanic episode
(D) Inactive hepatitis C infection
(E) Suicide attempt by aspirin overdose at age 16
T h e antidepressant duloxetine m a y simultaneously
improve mood a n d :

(A) panic attacks.


(B) chronic pain. 93
(C) flashbacks. A 32-year-old w o m a n with bipolar I disorder has
(D) psychotic symptoms. been adequately maintained on lamotrigine. Recently
(E) night terrors. she has e x p e r i e n c e d an exacerbation of her manic
symptoms, a n d her physician elects to add a second
mood stabilizer. Instead of improving, the patient's
symptoms w o r s e n . H e r serum lamotrigine levels are
nearly undetectable. W h a t w a s the most likely mood
stabilizer that w a s added?

(A) Olanzapine
(B) Carbamazepine
(C) Valproate
(D) Topiramate
(E) Lithium

Section 1: Self-Assessment Questions 15


94 98
Obsessive-compulsive disorder is hypothesized to in- A patient is being treated for a cat phobia. T h e ther-
volve a neural circuit connecting the cortex and striatum apist encourages the patient to pass by a pet store
that has cats in the w i n d o w . From w h i c h of the fol-
with the:
lowing psychotherapy approaches does this strategy
(A) amygdala. derive?
(B) hippocampus.
(() hypothalamus. (A) Cognitive behavior
(D) mammillary body. (B) Insight oriented
(E) thalamus. (C) Interpersonal
(D) Short-term anxiety-regulating
(E) Supportive

95
W h i c h of the following psychiatric disorders occurs
most commonly as a comorbid disorder with a n o r e x i a 99
nervosa?
W h i c h of the following cognitive functions is most
(A) Somatization disorder likely to remain stable with normal aging?
(B) Generalized anxiety disorder
(A) Language syntax
(C) Major depressive disorder
(B) Recent memory
(D) Obsessive-compulsive disorder
(C) Speed of information processing
(E) Social phobia
(D) Topographic orientation
(E) Working memory

96
W h i c h of the following is the LEAST problematic for 100
the psychiatrist according to ethical principles?
A consultation is requested for a 22-year-old m a n
(A) A psychiatrist in a metropolitan area agrees to treat her because of a gradual onset of b e h a v i o r a l symptoms
financial adviser's child. that include irritability, a g g r e s s i o n , a n d personality
(B) A psychiatrist in a remote orea with no other psychiatrists is c h a n g e . Associated findings include mild jaundicv..
involved in a romantic relationship with a patient's adult dysarthria, a n d choreiform movements. T h e consul-
grandchild. tation-liaison psychiatrist also notices a golden-
(C) A psychiatrist hires a current patient to perform clerical b r o w n discoloration of the c o r n e a . T h e most likely
work in the psychiatrist's office. diagnosis is:
(D) A psychiatrist convinces a patient who was sexually abused
(A) Huntington's disease.
by a former clinician to file a suit against that former clini-
(B) Wilson's disease.
cian and serves as the forensic expert for the patient.
(C) Parkinson's disease.
(D) progressive supranuclear palsy.
(E) adrenoleukodystrophy.

97
In the initial assessment, a psychiatrist is consulted by
a lesbian couple seeking help for some problems in
their long-standing committed relationship. W h i c h of
According to DSM-IV-TR, a patient with recurrent hypo-
the following is the best a p p r o a c h for the psychiatrist
manic episodes without intercurrent depressive features
to take in assessing the possibility of domestic vio-
would receive which of the following diagnoses?
lence within the couple?
1
(A) Bipolar I disorder
(A) Ask about it only when material is presented that suggests
(B) Bipolar II disorder
the problem. *
(C) Cyclothymic disorder
(B) Ask routine questions about battering while taking the his-
(D) Bipolar disorder, not'Otherwise specified
tory.
(C) Obtain information from collateral sources.
(D) The topic need not be raised because domestic viojence is
low in lesbian couples.
(E) Wait until the therapy is well established before asking about it.

16 FOCUS Psychiatry Peview 400 Self-Assessment Ouwrion*


@
T h e ventral tegmentum, the nucleus accumbens, a n d Biological relatives of individuals with antisocial per-
the prefrontal cortex a r e brain structures or regions sonality disorder have an increased risk of having anti-
most involved in the neurobiology of: social personality disorder a n d substance-related
disorders. These relatives, especially if they are female,
(A) alcohol dependence.
are also at greater risk of:
( 8 ) anorexia nervosa.
(C) bipolar disorder. (A) autism.'
(D) panic disorder. , (B) narcissistic personality disorder.
(E) schizophrenia. 1
' (C) .bipolar disorder.
(D) schizophrenia.
(E) somatization disorder.

©
A 32-year-old man with panic disorder treated with
l o r a z e p a m for several y e a r s begins combination ther- (!§)
a p y (which includes ritonavir) for H I V infection. T w o C o m p a r e d with younger adults, the elderly require
weeks later, his panic attacks increase in frequency. lower doses of lithium to achieve a given serum lithium
W h a t is the most likely explanation? concentration because of:

(A) An HIV-related brainstem lesion (A) impaired hepatic metabolism.


1
(B) An HIV-related lung infection (B) more complete absorption.
(() A direct side effect of one of his HIV medications (C) reduced fat storage.
(D) Ritonavir is decreasing blood lorazepam levels (D) reduced renal excretion.
(E) Failure to take lorazepam as directed (E) reduced serum protein binding.

A 24-year-old man w h o lives with his parents is being W h i c h of the following is the best description of the
treated for schizophrenia in a continuing d a y treat- therapist's empathy?
ment p r o g r a m . Since the onset of his illness at a g e
(A) Envisioning what it would be like for the therapist to be in
20, he hcis had three hospitalizations for recurrent
the patient's situation
psychosis. He is currently on quetiapine 300 mg
(B) Mirroring the patient's presentations of a vulnerable self
b.i.d., a n d his auditory hallucinations h a v e resolved,
(() Understanding the patient's inner experience from the
but he still has some concerns that a government con-
patient's perspective
spiracy m a y be operating a n d spying on him. Apart
( 0 ) Maintaining an attitude of compassion and sympathy
from his family a n d the d a y treatment p r o g r a m , he
(E) Avoiding making the patient anxious or uncomfortable
has f e w interactions with others and no outside inter-
ests. If family therapy w e r e instituted with this patient's
parents, w h i c h of the following outcomes w o u l d be
most likely to be observed?

(A) Improved employability


(B) Improved social functioning
(C) Reduced likelihood of psychotic relapse and rehospitalization
(D) Reduced number and severity of negative symptoms
(E) Reduced number and severity of positive symptoms

Section 1: Self-Assessment Questions 17


( T I T )

A 39-year-old secretary must do everything meticu- A 76-year-old w o m a n presents with weakness, fatigue,
lously. H e r w o r k area is extremely neat and organ- somnolence, and depression. Her husband has also
i z e d . H o w e v e r , she is not v e r y productive, because noticed that there has been some cognitive slowing
she will restart any project if she makes an error. She and her voice is hoarse. W h i c h of the following
typically works through lunch and rarely socializes endocrine disorders is the most likely diagnosis?
with her coworkers. At home, she is in constant con-
(A) Cushing's disease
flict with her children about the tidiness of their rooms,
(B) Hyperparathyroidism
the neatness of their schoolwork, a n d the need to be
(C) Hypoparathyroidism
frugal. H e r children a n d coworkers tell her that her
(D) Hypothyroidism
behaviors "drive them nuts." She does not believe she
(E) Pheochromocytoma
has a problem a n d in fact thinks her habits represent
"strong moral values." W h i c h term best describes the
w o m a n ' s lack of distress about her problems?

(A) Ambivalence
( 8 ) Denial
Early-onset A l z h e i m e r ' s dementia due to mutations in
(C) Ego-syntonic
the a m y l o i d precursor protein genes, presenilin-1
(D) La belle indifference
and presenilin-2, a r e transmitted by w h a t mode of
(E) Projection
inheritance?

(A) Autosomal dominant


(B) Autosomal recessive
(C) X-linked
(D) Trinucleotide repeat
An 18-year-old female patient w h o is being evaluated
(E) Polygenic
for depression reveals that she worries excessively
about her Weight. She states that she is unable to diet
a n d consumes large quantities of food about once a
month. She appears to h a v e normal weight for her D H
height. W h a t is the most likely diagnosis?
A 27-year-old man has a long-standing history of
(A) Anorexia nervosa marked discomfort in social situations a n d avoids
(B) Body dysmorphic disorder group discussions, parties, dating, a n d speaking at
(C) Bulimia nervosa meetings. He clso has a history of binge alcohol use,
(D) Eating disorder not otherwise specified particularly w h e n he has to e n g a g e in social activi-
(E) Factitious disorder ties. T h e class of medication preferred for treatment of
this patient w o u l d be:

(A) benzodiazepines.
(B) beta-blockers.
0 3 2
(C) tricyclics.
Patients with bulimia nervosa who e n g a g e in (D) second-generation antipsychotics.
b i n g e / p u r g e behaviors are at risk for w h i c h of the fol- (E) selective serotonin reuptake inhibitors.
lowing medical disorders?

(A) Hyperkalemia
(B) Decreased serum amylase
(C) Cardiomyopathy
(D) Hypothyroidism The consultation-liaison psychiatrist is called to the emer-
(E) Osteopenia gency department to evaluate a 17-year-old patient
w h o is highly agitated and floridly psychotic with find-
i

ings of ataxia, nystagmus,' dysarthria, miosis, and ele-


vated blood pressure. Intoxication with w h i c h of the
following substances best explains this presentation?

(A) Heroin
(B) Psilocybin
(C) Cannabis .
(D) LSD
(E) Phencyclidine

FOCUS Psychiatry Review: 400 Self-Assessment Questions


A 35-year-old nurse is admitted to the medical service W h i c h of the following is an e x a m p l e of an instru-
with numerous ecchymoses on her body a n d a com- mental activity of daily living that becomes impaired
plaint of tarry stools. H e r profhrombin time w a s 4 INR in the mild to moderate stages of dementia?
(international normalized prothrombin ratio) units
(A) Ambulating
(normal, 0 . 7 8 - 1 . 2 2 ) . Several d a y s after admission
her prothrombin time w a s normal. A medical w o r k u p (B) Dressing
failed to identify the cause of her abnormal clotting (C) Feeding oneself
time. Her stool w a s w e a k l y positive for blood. Four (D) Remembering appointments
days after admission, more ecchymoses appeared (E) Toileting
a n d her prothrombin time w a s a g a i n elevated. The
patient expressed c o n c e r n that she might have
leukemia and inquired if she w o u l d need a bone mar-
r o w biopsy. On the fifth d a y of admission, a warfarin a
pill w a s found beneath her b e d . The patient signed ' A 66-year-old patient w h o is being treated for bipolar
out of the hospital that evening. W h i c h of the follow- disorder presents comatose with a serum sodium con-
ing is the most likely diagnosis? centration of 112 m m o l / L . W h i c h of the following is
most likely to be the cause of the sodium imbalance?
(A) Somatization disorder
(B) Malingering (A) Divalproex
(C) Hypochondriasis (B) Carbamozepine
(D) Factitious disorder (C) Lithium
(E) Body dysmorphic disorder (D) Olanzapine

m
Patients w h o suffer from depression after a myocar- • W h i c h of the following features differentiates delirium
dial infarction should be treated with which of the fol- from dementia of the A l z h e i m e r ' s type?
lowing antidepressants?
(A) Acuity of onset and level of consciousness
(A) A monoamine oxidase inhibitor (B) Level of consciousness and orientation
(B) Bupropion (C) Acuity of onset and orientation
(C) Trazodone (D) Visual hallucinations and memory
(D) A tricyclic antidepressant (E) Memory and level of consciousness
(E) AnSSRI

m *
<m W h i c h of the following sleep disorders is more com-
Cocaine-induced euphoria is most highly associated mon in males than females during childhood?
with w h i c h of the following neurotransmitters?
(A) Breathing-related sleep disorder
(A) Serotonin (B) Nightmare disorder
(B) Dopamine (C) Primary insomnia
(C) Norepinephrine (D) Sleep terror disorder
(D) Gamma-aminobutyric acid (E) Sleepwalking disorder
(E) Acetylcholine

Section 1: Self-Assessment Questions


126
@)
A physician elects to treat a depressed patient with A 70-year-old w o m a n presents with a depression that
imipramine. Four days after the start of treatment, the has not responded to treatment with sertraline, parox-
physician receives a call from the e m e r g e n c y depart- etine, or escitalopram. She has said that she w o u l d
ment reporting that the patient has fallen. T h e staff like to die, a n d she has a history of an overdose in the
report that the patient stood up quickly after being in past 3 months. Although a b d o m i n a l computerized
bed overnight, felt d i z z y , and then lost consciousness, t o m o g r a p h y shows no abnormalities, she is convinced
falling to the floor. Examination reveals a pulse of that a hole in her liver is causing her to lose weight.
76 bpm; blood pressure is 1 3 6 / 8 2 mm Hg lying and Mental status examination is also significant for
8 4 / 4 6 mm Hg standing. An electrocardiogram is severe psychomotor retardation, a n d physical exami-
unremarkable. W h i c h of the following best explains nation shows evidence of d e h y d r a t i o n . She is cur-
the patient's symptoms? rently being treated with 150 m g / d a y of venlafaxine.
W h i c h of the following recommendations is most
(A) A-Adrenergic receptor blockade appropriate at the present time?
(B) Cholinergic receptor blockade
(C) Histamine receptor blockade (A) Increase the dose of venlafaxine
(D) First-degree atrioventricular block (B) Recommend ECT
(E) Prolongation of the QIc interval (C) Change to mirtazopine
(D) Add lamotrigine
(E) Obtain a liver scan to assess for evidence of carcinoma

Soon after ECT, a patient is most likely to have prob-


]27
lems with which of the following items on the Mini-
Mental State Examination? A 29-year-old unmarried w o m a n is admitted to an
acute inpatient unit after police spotted her w a n d e r i n g
(A) Reporting the dote
along a busy h i g h w a y gesturing a n d muttering to her-
(B) Spelling "WORLD" backwards
self. On admission, she w a s disheveled and bizarrely
(C) Repeating "no ifs onds or buts"
clothed. H e r speech w a s tangential, and she reported
(D) Following a three-step command
auditory hallucinations commenting on her behavior
(E) Writing a sentence
and telling her that "criminal elements" w e r e watch-
ing her. She had recently been residing with her par-
ents a n d g a v e permission for staff to contact them.
Her parents report that her first hospitalization w a s at
(t24) a g e 25, just after she b e g a n w o r k i n g on her thesis for
Imaging genetics is a form of a P h . D . in mathematics. She responded rapidly to
treatment with risperidone 3 mg daily, a n d several
(A) association study. months later, with the support of her adviser, she w c s
(B) double-blind study. able to resume w o r k on her thesis. O v e r the past
(C) linkage study. 6 months, after she decided to stop her medication,
(D) randomized study. her symptoms have returned. In responding to the par-
ents' questions about her prognosis, w h i c h of the fol-
lowing factors w o u l d be the best predictor of a g o o d
prognosis for this patient?
0 2 5 2

(A) Age at onset of illness


W h i c h of the following is the most appropriate indi-
(B) Initial response to medication
cation for E C T in a patient with borderline perspnality
(C) Marital status
disorder?
(D) Number and duration of remissions between psychotic
(A) Comorbid major depression es
(B) Severe mood instability (E) Premorbid cognitive? functioning
(C) Poor response to valproate
(D) Noncompliance with medications
(E) Recurrent transient psychotic episodes

20 FOCUS Psychiatry Review: 400 Self-Assessn-ru Questions


128 132
A patient with a history of "manic and major depres- A 10-year-old b o y has a well-documented episode of
sive episodes" w h o has persistent delusions or halluci- moderately severe non-bipolar major depression.
nations even w h e n prominent mood symptoms are W h i c h of the following medications is F D A - a p p r o v e d
absent, would have which of the following diagnoses? for use in this patient?

(A) Bipolar I disorder (A) Bupropion


(B) Delusional disorder, grandiose type iB) Duldxetine
((} Schizoaffective disorder (C) Fluoxetine
(D) Schizophrenia, disorganized type (D) Sertraline
(E) Venlafaxine

129
133
A patient with borderline personality disorder reports
prominent lability, sensitivity to rejection, anger, out- A 35-year-old man has a 10-year history of schizo-
bursts, a n d " m o o d crashes." As an initial approach to phrenia a n d poor adherence with outpatient treat-
pharmacotherapy, which of the following would be ment. He has been stabilized on 20 mg of o l a n z a p i n e
most appropriate? in the hospital, a n d he has previously done well on
10 mg of oral haloperidol. He has a g r e e d to switch
(A) Gabapentin
to haloperidol decanoate injections once a month. He
(B) Sertraline
is given an initial injection of 50 mg. W h i c h of the fol-
(C) Quetiapine lowing is the most likely amount of time he will need
(D) Phenelzine to continue taking the oral olanzapine?
(E) Valproic acid
(A) Two days
(B) Two weeks
(C) One month
130 (D) Three months
(E) One year
W h i c h of the following schools of therapy has its base
in the idea that family problems are due to structural
imbalances in family relationships and symptoms are
communications? ;»
134
(A) Cognitive behavior A 51-year-old w o m a n presents to her physician with
(B) Insight oriented the chief complaint of feeling depressed over the past
(C) Psychoeducational month. She has no energy, is disinterested in her chil-
(D) Solution focused dren, and Ijas lost 25 pounds. She is unable to fall
(E) Strategic asleep until the early morning hours. She has b e g u n
to feel that she is u n w o r t h y of her family. W i t h the
onset of these symptoms, she is quite certain that she
has d e v e l o p e d a degenerative nerve condition,
13] although all investigations have been negative. T h e
C o m p a r e d with depressed elderly individuals w h o most appropriate first step in treating this patient is to
had a first episode of depression in y o u n g adulthood, start her o n :
individuals with a first episode of depression in late
(A) a serotonin reuptake inhibitor alone.
life are more likely to have:
(B) a serotonin-norepinephrine reuptake inhibitor alone.
(A) brain imaging findings suggesting dementia. (C) a serotonin reuptake inhibitor and an antipsychotic.
(B) comorbid personality disorder. (D) a serotonin-norepinephrine reuptake inhibitor and a benzo-
{() first-degree relatives with depression. diazepine.
(D) good response to treatment.
(E) suicidal ideation.

Section 1: Self-Assessment Questions 21


135 138
W h i c h of the following is N O T a predisposing risk fac- A man reports that he avoids public urinals even
tor for the development of posttraumatic stress disorder w h e n he has great u r g e n c y to urinate. This type of
(PTSD) after a traumatic event? chief complaint is most consistent with a diagnosis of:

(A) Recent life stressors (A) body dysmorphic disorder.


(B) Female gender (B) obsessive-compulsive disorder.
(C) Internal locus of control (C) panic disorder with agoraphobia.
(D) Past history of depression (D) social phobia.
(E) posttraumatic stress disorder.

136
139
A patient with schizophrenia, paranoid type, and
methamphetamine dependence receives mental A patient with mild dementia of the Alzheimer's type
health care through a community mental health clinic is brought in by his wife, w h o is also his primary care-
( C M H C ) . T h e patient has a p p e a r e d to clinically dete- giver, for follow-up evaluation. She brings along a list
riorate over a period of 6 weeks a n d is hospitalized of his medications. T h e patient is taking donepezil,
with a psychotic decompensation. A drug screen on hydrochlorothiazide, a n d warfarin. The use of which
admission shows methamphetamine a n d ampheta- of the following herbal or over-the-counter products by
mine in the patient's urine. After a 3-da^ hospital stay, this patient w o u l d be of the most concern?
the patient is r e a d y for discharge. T h e outpatient psy-
(A) Ginkgo biloba
chiatrist should do w h i c h of the following?
(B) Ginseng
(A) Resume psychiatric care through the CMHC, deferring sub- (C) Hawthorn
stance dependence treatment unless the patient resumes (D) Vitamin C
methamphetamine use. (E) Yitamin E
(B) Resume psychiatric care at the CMHC, with increased
emphasis on the provision of substance dependence treat-
ment by the mental health team.
(C) Enroll the patient in a separate program specifically for 140
substance dependence and continue to provide psychiatric
W h a t diagnostic specifier w o u l d be most appropriate
care through the CMHC.
lor a depressed patient w h o complains of a sense of
(D) Enroll the patient in a separate program specifically for
leaden paralysis a n d difficulty being around other
substance dependence and resume psychiatric care at the
people but is able to enjoy himself w h e n g o o d things
CMHC once a period of sobriety is achieved.
happen?

(A) With atypical features


(B) With catatonic features
137 (C) With melancholic features
(D) With psychotic features
W h i c h of the following is the most accurate w a y to
diagnose early-stage dementia of the Alzheimer's type?

(A) Apolipoprotein E genotyping


(B) Brain MRI
(C) History of stepwise memory decline
(D) Neuropsychological testing
(E) Patient self-report of memory difficulties

22 FOCUS Psycl (try Review: 400 Self-Assessment Questions


U5
A 34-year-old w o m a n with two previous episodes of Of the following, w h i c h is the most important factor in
major depressive disorder is treated with 100 m g / d a y determining whether a patient is suited to brief psy-
of sertraline. Her most recent episode occurred 9 chodynamic psychotherapy?
months a g o . At that time sertraline w a s initiated, with
(A) Ability to recognize and discuss feelings
the dose titrated up to 150 m g / d a y o v e r a 2-week
(B) Existence of multiple conflicts'
period. Her symptoms remitted after 6 w e e k s , but she
(C) Compliance with medication
experienced significant sexual side effects that resolved
with a decrease in the sertraline, dose to 100 m g / d a y . (D) Lack of meaningful relationships
O v e r the past 7 months she has remained free of (E) Failure of long-term psychodynamic psychotherapy
depressive symptoms and now inquires about decreas-
ing her dose of medication. W h i c h of the following rec-
ommendations is most appropriate?
H6
(A) Continue sertraline at 100 mg/day.
C o m p a r e d with men, w o m e n with schizophrenia tak-
(B) Decrease sertraline to 50 mg/day and continue at that dose.
ing equivalent doses per weight of antipsychotics are
(C) Discontinue sertraline after tapering the dose.
less likely to h a v e which of the following?
(D) Initiate psychotherapy and then decrease sertraline to
50 mg/day. (A) Acute dystonia
(E) Initiate psychotherapy and then gradually discontinue sertra- (B) Drug-drug interactions
line. (C) Lower serum drug levels
(D) Sedation

142
W h i c h of the following antipsychotic drugs has the 147
greatest effect on prolonging the QT interval on the
electrocardiogram? A 22-year-old man presents at the e m e r g e n c y depart-
ment with agitated, g u a r d e d behavior, paranoid delu-
(A) Aripiprazole sional thoughts, and a 7-month history consistent with
(B) Ha operidol a diagnosis of schizophrenia, paranoid type. Under-
(C) Olanzapine standing the man's cultural background w o u l d be
(D) Thioridazine most helpful for:
(E) Ziprasidone
(A) choosing his acute and maintenance medications.
(B) determining the cause of his disorder.
(C) determining safety issues and the need for hospitalization.
(D) understanding the content of the delusions and hallucinations.
143
The therapeutic benefit of acamprosate is best estab-
lished for which of the following conditions?
148
(A) Alcohol dependence
(B) Barbiturate dependence Hypertension is most associated with which of the fol-
(C) Cocaine withdrawal lowing medications?
(D) Heroin addiction
(A) Bupropion
(E) Methamphetamine abuse
(B) Fluvoxamine
(C) Mirtazapine
(D) Paroxetine
(E) Venlafaxine
144
A 25-year-old w o m a n with bipolar disorder is about
to be started on lamotrigine for maintenance therapy.
She should receive one-half of the usual starting dose
if she is taking w h i c h of the following medications?

(A) Carbamazepine
(B) Lithium
(C) Oral contraceptive
(0) Phenytoin (E) Valproate

Section 1: Self-Assessment Questions 23


149 153
A patient reports regularly taking a drug bought on In the initial assessment of a depressed patient, w h a t
the street. Its effect is pleasurable, but it sometimes is the most critical decision that the psychiatrist must
causes n a u s e a , restlessness, and teeth grinding. The make?
drug is most likely:
(A) Type of psychotherapy
(A) methamphetamine. (B) Choice of medication
(B) flunitrazepam (Rohypnol). (C) Level of care
(C) methylenedioxymethomphetomine (MDMA). (D) Medical workup
(D) cocaine. (E) Involvement of family

]50 1 5 4

Indications of caffeine withdrawal: A 27-year-old patient announces that she is pregnant


despite having taken an oral contraceptive for 4
(A) are evident following an average intake of 50 mg/day. years. W h i c h of the following medications might
(B) include symptoms that typically last 3 weeks. account for the failure of her oral contraceptive?
(C) include a flushed face and diuresis.
(D) include headache. (A) lithium
(B) Divalproex
(C) Corbamazepine
(D) Lamotrigine
151 (E) Gabopentin

A 23-year-old college student from Ethiopia is admit-


ted to the hospital for a psychotic disorder. His symp-
toms include paranoid ideation, hallucinations, and
1 5 5
d i s o r g a n i z e d thinking. He is initially started on
0.5 mg of risperidone twice daily, w h i c h is rapidly Screening for hepatitis C ( H C V ) infection is LEAST
titrated to 4 m g / d a y . Despite this treatment, there is important in patients with:
no improvement in his symptoms. No detectable
(A) methamphetamine dependence.
plasma levels of the parent compound are noted.
(B) marijuana dependence.
W h a t is the most likely explanation for these results?
(C) heroin dependence.
(A) Binding of the drug to fatty tissue (D) history of blood transfusion (before 1992).
(B) First-pass effect (E) hemodialysis.
(C) Impaired absorption of the drug
(D) Increased excretion ,
(E) Ultrarapid metabolism of the drug
1 5 6

W h i c h of the following is best characterized as a


degenerative dementia?
152
(A) Systemic lupus erythematosus
T h e preferred initial pharmacological treatment for a
(B) Korsakoff's syndrome
26-year-old w o m a n w h o w a s violently assaulted 3
(C) Parkinson's disease
years a g o a n d suffers from recurrent nightmares
about the event, hypervigilance, difficulty concentrat- (D) HIV disease
ing, a n d constricted range of affect w o u l d . b e : (E) Cerebrovascular accident

(A) alprazolam.
I
(B) clomipramine.
(C) clonidine.
(D) valproate.
(E) sertraline.

74 FOCT'K V .-h'wv RRVIRW: 4 0 0 SELF-ASSESSMENT QUESTIONS


157 161
In a urine test for phencyclidine, a false positive test W h i c h of the following medicction classes is the pre-
result can occur if a person has ingested: ferredjtreatment for obsessive-compulsive disorder?

(A) dextromethorphan. (A) Atypical antipsychotics


(B) ibuprofen. (B) Anticonvulsants
(C) tonic water. (C) Mood stabilizers
(D) phenylephrine. (0) SS&is
(E) diphenhydramine. (E) Benzodiazepines

158 162
A 48-year-old man is admitted to the hospital with W h i c h of the following interventions is the best first step
cholecystitis, a n d after diagnosis he consents to and in the management of agitation in the elderly patient?
undergoes a cholecystectomy. On the third hospital
(A) Haloperidol, 5 mg twice a day, whatever the cause of the
d a y he becomes a n g r y at the nursing staff and wishes
agitation
to leave the hospital against medical advice. In asses-
(B) Physical restraints
sing this patient's capacity to refuse further medical
(C) Evaluation of the patient's surroundings and daily schedule
care, w h i c h of the following questions would be most
(D) Diazepam, 5 mg every 6 hours, or until the patient is asleep
useful for the psychiatrist to ask?
(E) Seclusion until the behavior ceases
(A) Have you discussed with your family your decision to leave?
(B) What is the danger of your going home at this time?
(C) Have you been troubled by depression?
(D) Are you able to name all of your medications? 1 6 3

(E) When did you first become ill, and do you remember your
W h i c h of the following will double the blood level of
symptoms?
lamotrigine?

(A) Carbamazepine
(B) Divalproex

]59 (C) Phenytoin


(D) Phenobarbital
W h i c h of the following disorders has been shown to
h a v e genetic or familial links?
(A) Autistic disorder 164
(B) Dissociative amnesia
Buspirone has been found to be most consistently
(C) Factitious disorder
effective iri the treatment of which of the following
(D) Hypoactive sexual desire disorder
(E) Pyromania anxiety disorders?

(A) Generalized anxiety disorder


(B) Obsessive-compulsive disorder
(C) Panic disorder with agoraphobia
160 (D) Panic disorder without agoraphobia
A 45-year-old man and his monozygotic twin have (E) Social phobia
been diagnosed as having the same personality disor-
der. W h i c h of the following diagnoses is most likely?

(A) Histrionic personality disorder


1 6 5
(B) Obsessive-compulsive personality disorder
(C) Narcissistic personality disorder W h i c h of the following pcrental transmissions of the
(D) Antisocial personality disorder fragile X trinucleotide repeat is most likely to result in
(E) Avoidant personality disorder an affected child?

(A) Mother-to-daughter
(B) Mother-to-son
(C) Father-to-daughter
(D) Father-to-son
(E) No parental gender difference

Section I: Self-Assessment Questions 25


266 ; 170
W h i c h of the following is N O T a relative contraindi- W h i c h of the following treatments is most effective for
cation for the use of disulfiram as an adjunct in the patients with bulimia nervosa and major depressive
treatment of alcohol dependence? disorder?

(A) Impulsive behavior (A) Cognitive behavior therapy


(B) Psychotic symptoms (B) Fluoxetine
(() Suicidal thoughts {() Imipramine
(D) A family history of alcoholism (D) Bupropion
(E) Impaired judgment (E) Combined cognitive behavior therapy and fluoxetine

167 171
W h i c h of the following signs and symptoms is more W h i c h of the following describes the A m e r i c a n
likely to occur in females than in males at first presen- A c a d e m y of Pediatrics statement regarding maternal
tation of psychosis? lithium use during breast-feeding?

(A) Amotivation (A) Associated with significant side effects in some nursing
(B) Cognitive impairment infants; use with caution
(C) Dysphoric mood stote (B) Unknown effects on nursing infants; may be of concern
(D) Paranoid ideation (C) Absolutely contraindicated
(E) Social isolotion (D) Usually compatible

168 172
A 45-year-old w o m a n complains of blurred vision, Treatment with which of the following cytokines has
ocular p o i n , and headache a n d w a s noted to have been linked to suicidal behavior?
increased intraocular pressure. W h i c h of the recently
(A) Erythropoietin
started medications is the most likely cause?
(B) Granulocyte colony-stimulating factor
(A) Lamotrigine (C) Interferon-a
(B) Oxcorbazepine (D) lnterleukin-1 receptor agonist
' (C) Tiagabine (E) Anti-tumor-necrosis-factor antibodies
(D) Topiramate
(E) Valproate

173
The a m y g d a l a is most specifically involved in w h i c h of
16?
the following brain functions?
A married 50-year-old w o m a n is admitted to the hos-
(A) Determining social behavior
pital with an acute myocardial infarction. It is recom-
(B) Emotional coding of sensory cues
mended that she have a cardiac catheterization with a ,
(C) Generating normal sleep patterns
possible procedure based on the findings. T h e patient
refuses. W h i c h of the following w o u l d be a reason- (D) Recalling previously learned material
able a p p r o a c h to the patient at this time? • (E) Signaling reward by exogenous substances

(A) Discharge the patient to home so that she can make up her
mind.
(B) Call for a family meeting with her husband and adult chil-
dren to discuss the options.
(C) Tell her that there is no guarantee that she wouldn't die if
she leaves the hospital without treatment.
(D) Consider treatment for depression since she does not seem
to want to live.
(E) Recommend treatment against her will because of the seri-
ousness of the condition.
174 178
A 22-year-old w o m a n wants to take an antidepressant Despite intensive psychosocial treatment for alcohol
for treatment of her major depression but is concerned dependence, a patient continues to drink alcohol. T h e
about possible sexual side 'effects. W h i c h of the fol- psychiatrist decides to recommend adjunctive med-
lowing medications is the best choice for her? ication. W h i c h of the following medications w o u l d
N O T b e a n acceptable treatment?
(A) Bupropion
(B) Clomipramine (A) DisuHiram
(C) Escitaopram (B) Bromocriptine
(D) Sertraline \ ' (C) Naltrexone
(E) Venlafaxine (D) Ondansetron
(E) Accfmprosate

175
179
W h i c h of the following factors is U N R E L A T E D to a
positive treatment outcome for cocaine dependence? The anxiety disorder that includes a dissociation-like
phenomenon in its criteria is:
(A) Counseling rapport
(B) Treatment retention (A) generalized anxiety disorder.
(C) Patient choice of program type (B) obsessive-compulsive disorder.
(D) Comorbid depressive symptoms (C) panic disorder.
(D) posttraumatic stress disorder.
(E) social phobia.

176
W e i g h t gain is most likely to occur with which of the
180
following antipsychotic drugs?
Patients with w h i c h of the following personality disor-
(A) Aripiprazole
ders w o u l d be expected to benefit most from adjunc-
(B) Clozapine tive pharmacotherapy?
(C) Haloperidol
(D) Ziprasidone (A) Borderline
(E) Risperidone (B) Schizoid
(C) Antisocial
(D) Obsessive-compulsive
177 (E) Dependent

A 16-year-old female patient is admitted to the hospital


for gastric rupture. On interview, she reports that in the
past 2 months, whenever she has felt anxious, angry, or
sad, she has eaten unusually large quantities of cookies,
181
candy, crackers, "or whatever I can get my hands on W h e n assessing a patient's suitability for short-term
like I'm a maniac, totally out of control." She reports that psychodynamic psychotherapy, of the following fac-
after gorging, she feels better emotionally but is afraid tors, which is the most important?
that she will "get fat." She gags herself to induce vom-
iting, and recently she has begun drinking ipecac as an (A) The DSM-IV-TR diagnosis
emetic after such episodes. This morning, after an (B) Family psychiatric history
episode of self-induced vomiting, she developed severe (C) Level of education
abdominal pain and was brought to the emergency (D) An identifiable focus
department by her parents. On physical examination, (E) Need for psychoactive medication
her height is 5'6" and her weight is 120 pounds. The
psychotherapy that has been found to be most effective
in treating her eating disorder is:

(A) cognitive behavior therapy.


(B) family therapy.
(C) group therapy.
(D) psychodynamically oriented individual therapy.

Section 1: Sct-Ai-essaent Questions 27


182 186
A psychiatrist is treating an 8-year-old child of a Treatments s h o w n to be effective for smoking cessa-
divorced single parent w h o is the child's custodial par- tion include all of the following E X C E P T :
ent. The noncustodial parent wishes to be informed of
(A) bupropion.
the child's source of problems and progress of treat-
(B) brief advice intervention.
ment. The psychiatrist should share clinical informa-
(C) 12-step programs.
tion with the noncustodial parent:
(D) nicotine replacement therapy.
(A) without consent of the custodial parent or the child.
(B) only with the informed consent of the custodial parent and
the child.
(C) only with informed consent of the custodial parent. 187
(D) only with the informed consent of the child.
Based on the mental status examination, the psychia-
trist believes that a patient is delirious. The examination
reveals disorientation, changing levels of con-
sciousness, and visual illusions. W h i c h of the following
183 tests has the greatest evidence supporting its use in con-
Cerebral ventricular enlargement, one of the most firming a diagnosis of delirium?
consistent structural brain findings in patients with
(A) Positron emission tomography
schizophrenia, is most closely associated with:
(B) Magnetic resonance imaging
(A) prominent negative symptoms. (C) Computerized tomography
(B) rapid onset of the disorder. (D) EEG
(C) improved response rates to atypical antipsychotics.
(D) retained memory- and language-processing capabilities.
(E) increased risk of developing tardive dyskinesia.
]88
The diagnosis of shared psychotic disorder is most
commonly found in w h i c h of the following groups?
184
(A) Couple relationships
A patient with heroin dependence purchases a drug
(B) Groups larger than two people
on the street. T h e patient feels a mild opiate high but
(C) Groups of men, rather than women
than, despite continued injection of a sizable volume
(D) Family blood relations
of drug., feels opiate withdrawal coming o n . T h e drug
(E) Children and adolescents
injected is most likely:

(A) buprenorphine.
(B) heroin. (

(C) methadone. 189


(D) naloxone.
A colleague w h o is a cardiac surgeon asks for a psy-
chiatrist's help in raising funds for a n e w w i n g of the
local hospital. T h e cardiac surgeon asks the psychia-
trist to solicit patients for charitable contributions. T h e
185
psychiatrist's ethical response should be to a g r e e to
W h i c h of the following w o u l d be the most important solicit funds from:
consideration w h e n evaluating an individual for a
personality disorder? > (A) only former patients, because there is no longer a doctor-
patient relationship.
(A) Culture (B) only wealthy patients who have the means to contribute.
(B) Intelligence (C) no patients, because of th,e nature of the psychiatrist-
(C) Gender patient relationship. 1

(D) Socioeconomic status (D) both current and former'patients, since patients can make
(E) Education autonomous decisions.

28 POO''•>' P w h i a r T v REVIEW 4FLFL S R ! f - A s « K s m e n t OTI«RMN<


190 193
A m a n a g e d care o r g a n i z a t i o n ( M C O ) is refusing to A 29-year-old w o m a n presents to the emergency
p a y for additional treatment d a y s for a patient in an department complaining of migraine headache. A re-
inpatient psychiatric facility, t h e attending psychiatrist view of her medical file reveals one brief admission for
believes that the additional treatment days m a y be a transient psychotic episode and depression within the
needed to ensure the patient's safety. W h i c h of the fol- past 3 years. She is noted to be dressed in odd cloth-
lowing statements is correct regarding this situation? ing. She insists that she is clairvoyant and telepathic.
Her speech is noted to be metaphorical, overelaborate,
(A) THE PSYCHIATRIST IS LEGALLY RESPONSIBLE TO ABIDE BY THE MCO'S
and stereotyped. She says she has no close friends or
DECISION. ' '
confidants other than her mother and father, and that
( B ) THE PSYCHIATRIST IS RESPONSIBLE FOR MAKING PROVISIONS FOR
this has been the case since she was a teenager. She is
CONTINUITY OF NEEDED CARE EVEN IF ADDITIONAL DAYS ARE NOT
not particularly bothered about her lack of companion-
COVERED BY THE MCO.
ship because she has fears of being harmed in rela-
(C) AS LONG AS THE PSYCHIATRIST DOCUMENTS THAT THE MCO WILL NOT
tionships. Her presentation is most consistent with which
PAY, THE PSYCHIATRIST MAY DISCHARGE THE PATIENT.
of the following personality disorders?
( D ) THE PSYCHIATRIST MAY INFORM THE PATIENT OF HIS OR HER RIGHT

TO APPEAL THE MCO'S DECISION ONLY IF THERE ARE NO "GAG (A) AVOIDANT

CLAUSES" THAT LIMIT WHAT THE PSYCHIATRIST IS ALLOWED TO SAY. ( B ) HISTRIONIC

(C) PARANOID

( D ) SCHIZOID

(E) SCHIZOTYPAL

191
The parents of a 14-year-old b o y bring him to a clinic
because he has been refusing to go to school. In ele-
194
mentary and middle school, he w a s in a special edu-
cation class for mildly mentally retarded students. W h i c h of the following w o u l d be most appropriate as
W h e n he has been at school, he ruminates about initial pharmacotherapy for a patient with borderline
"something really b a d " happening to his mother or personality disorder w h o is exhibiting impulsivity and
father. Recently, he has been a w a k e n i n g with night- behavioral dyscontrol?
mares that his parents have been killed. His parents
(A) SERTRALINE
have had to stay with their son in order for him to get
(B) CLOZAPINE
back to sleep. His medical history is significant for stra-
(C) HALOPERIDOL
bismus and scoliosis. Physical examination reveals a
( D ) NALTREXONE
long face with promir-ent ears and jaw, a high arched
(E) ALPRAZOLAM
palate, hyperextensible finger joints, macroorchidism,
and flat feet. This boy's overall presentation is most
consistent with:
i
(A) ANGELMAN SYNDROME. 195
( B ) FRAGILE X SYNDROME.
A 30-year-old man with schizophrenia has made sev-
(C) PRADER-WILLI SYNDROME.
eral significant suicide attempts over the past 10
( D ) STURGE-WEBER SYNDROME.
years in response to auditory command hallucina-
(E) WILLIAMS SYNDROME.
tions. W h i c h of the following has been shown in stud-
ies to be most likely to reduce his risk for further
suicidal behaviors?

192 (A) ARIPIPRAZOLE

( B ) CLOZAPINE
W h i c h of the following therapies explicitly gives the
(C) LITHIUM
patient permission to be in the sick role?
( D ) OLANZAPINE

(A) BRIEF PSYCHOTHERAPY (E) RISPERIDONE

(B) COGNITIVE BEHAVIOR THERAPY

(C) INSIGHT-ORIENTED THERAPY

( D ) INTERPERSONAL PSYCHOTHERAPY

(E) RATIONAL-EMOTIONAL THERAPY

Section 1: Self-Assessment Questions


196 199
A 23-year-old patient with chronic schizophrenia com- A 32-year-old man is brought to the emergency depart-
plains of a milky discharge from her nipples. M e d - ment by his family, w h o notes that he has been spend-
ication-induced antagonism of w h i c h of the following ing a lot of time sitting motionless in his room and
receptors is responsible? appears to be losing weight. In the past, he had been
fearful that family members w e r e poisoning his food,
(A) Acetylcholine but his parents state that he has not expressed those
(B) Dopomine concerns recently. On examination, he is disheveled
(C) 6ABA and poorly g r o o m e d , and he sits quietly in his chair
(D) Norepinephrine except for intermittent grimacing. He has minimally
(E) Serotonin spontaneous speech but will occasionally repeat the
last few w o r d s of a question posed by the interviewer.
His affect is generally restricted in range, and he does
not answer questions about his mood, hallucinations,
1 9 7 delusions, and suicidal or homicidal ideation. W h i c h
of the following subtypes of schizophrenia would best
W h i c h of the following statements is most accurate
describe this patient's current presentation?
regarding the current status of g e n e therapy for the
clinical treatment of psychiatric disorders?
(A) Catatonic
(A) Gene therapy will be clinically applicable within the next (B) Disorganized
2 years. (C) Paranoid
(B) Rnding vectors to transfer genes into the nelvous system is (D) Residual
a challenge. (E) Undifferentiated
(C) Neurons are among the easiest cells into which to insert
new genes.
(D) Target genes for gene therapy have been clearly defined.
(E) Viral vectors quickly spread novel genes throughout the 200
nervous system.
W h i c h of the following is most predictive of a favor-
able response to lithium in bipolar disorder?

(A) Comorbid substance abuse


198 (B) Depression-mania-euthymia course
(C) Euphoric mania
W h i c h of the following agents w o u l d be most appro-
(D) Psychotic features
priate for a geriatric patient w h o has Parkinson's dis-
ease a n d agitation?

(A) Risperidone
(B) Diazepam (
201
(C) Quetiapine
A female patient reveals during a psychotherapy ses-
(D) Haloperidol
sion that she does not e n j o y sexual intercourse. She
(E) Lithium
states that she is aroused by her partner but has
sharp pains throughout intercourse. She cannot relax
and enjoy sex and has b e g u n to avoid sex because
of the anticipation of the pain. W h a t is the most likely
diagnosis?

(A) Dyspareunia
(B) Female orgasmic disorder
(C) Sexual mosochism
(D) Sexual sadism , ,
(E) Sexual aversion disorder

30 FOCUS TV.rhiatrv Review: £ W Scif-Assessm-nt 0'.IH;PT<


202 205
A 25-year-old w o m a n presents with severe anxiety A husband and wife present for treatment because the
after finding out that her biological mother w a s wife if concerned. Her husband recently told her that he
recently diagnosed with Huntington's disease. There believes he w a s born a w o m a n . He states that he has
is no family history of the disease on her father's side. always felt this w a y but can't fight it anymore. He has
She wishes to k n o w if she is affected. T h e probability started wearing dresses around the house after he
that she is affected is: arrives home from work at the end of the day. He says
that he loves his wife and kids but that he needs to be
(A) OX. ' , happy as well. W h a t is the most likely diagnosis?
(B) 25%. \ •
(C) 50%. (A) Exhibitionism
(D) 75%. (B) Gender identity disorder
(E) 100%. (C) Sexual arousal disorder
(D) Transvestic fetishism
(E) Voyeurism

203
A 9-year-old b o y is referred for evaluation because he 206
is having "temper tantrums" in school. He cannot sit
W h i c h of the following describes the pharmacokinet-
still, constantly disrupts the class, runs out in the hall
ics of children y o u n g e r than 12 years old?
without permission and refuses to o b e y directives from
the teacher. He frequently fights with his peers, and if (A) Children have a smaller volume of distribution than adults.
he does not get w h a t he wants, he yells, screams, (B) Children have more efficient renal function than adults.
throws objects, a n d flails about on the floor. Edu- (C) Children metabolize through hepatic pathways more slowly
cational testing reveals borderline intellectual function- than adults.
ing a n d significant delays in reading, writing, spelling, (D) Children absorb medications more slowly than adults.
end mathematics. On physical examination, the b o y is
noted to be in the fifth percentile for head circumfer-
ence. He has short palpebral fissures, a thin upper lip,
207
and a smooth philtrum. The b o y w a s most likely
exposed to which of the following drugs in utero? A 32-year-old w o m a n develops anorgasmia while tak-
ing paroxetine. Switching to which of the following
(A) Alcohol medications is most likely to resolve this problem?
(B) Cocaine
(C) Marijuana (A) Gtalopram
(D) Nicotine (B) Venlafaxine
(E) Opiates (C) Sertraline
(D) Bupropion
(E) Fluoxetine

204
W h i c h of the following is the most accurate statement 208
regarding psychotherapy for posttraumatic stress dis- A 4-year-old b o y is brought to the clinic by his par-
order (PTSD)? ents with the chief complaint that "he keeps having
nightmares." His parents report that for the past
(A) The therapist should be as nondirective as possible for the
month, during the first one-third of the night, the b o y
psychotherapy to be effective.
awakens from his sleep with a startled scream. W h e n
(B) Multiple modalities of psychotherapy have proven effective
they enter the room, they find that he has broken out
for PTSD.
in c sweat, is difficult to a w a k e n , and looks " s c a r e d
(C) Psychotherapy must be combined with pharmacotherapy to
to death." T h e next morning he has no recall of the
be effective.
event. These episodes a r e most likely occurring dur-
(D) Cognitive behavioral therapy (CBT) is of little value for
ing which stage of sleep?
patients with PTSD.
(A) REM
(B) Stage 0—non-REM
(C) Stage 1-non-REM
(0) Stage 2-non-REM
(E) Stage 3 or 4-non-REM

Section 1: Self-Assessment Questions 31


209 212
A 25-year-old male with a history of schizophrenia is According to the A m e r i c a n Psychiatric Association
hospitalized a n d treated with haloperidol a n d benz- guidelines, w h i c h of the following is true regarding a
tropine. T h e patient becomes distressed, has a tem- psychiatrist e n g a g i n g in a sexual relationship with a
C
perature of 103 F and has labile blood pressure. former patient?
Physical examination reveals hypertonicity, diaphore-
(A) Acceptable provided at least 2 years have passed since the
sis, a n d tachycardia. Laboratory studies reveal a cre-
termination of the doctor-patient relationship
atine kinase of 55,000 I U / L . W h a t is the most likely
(B) Acceptable provided at least 5 years have passed since the
diagnosis?
termination of the doctor-patient relationship
(A) Anticholinergic syndrome ((} Acceptable provided the former patient initiates the rela-
(B) CNS infection tionship and it is clear to both parties that no exploitation is
(C) Malignant hyperthermia taking place
(D) Neuroleptic malignant syndrome (D) Unethical no matter how long it has been since the termi-
(E) Serotonin syndrome nation of the doctor-patient relationship

210 213 .
A previousfy well 24-year-old w o m a n presented with a A patient with schizophrenia begins treatment with
4-week history of progressively worsening expansive clozapine. T h e baseline white blood cell count
3
irritable m o o d , pressured speech, racing thoughts, ( W B C ) is 8 1 0 0 ( n o r m a l = 4 5 0 0 - l l , 0 0 0 / m m ) . T h e
grandiosity, a n d distractibility. M o r e recently she heard absolute neutrophil count (ANC] is 6 2 0 0 (nor-
3
the voice of G o d proclaiming her to be a special mes- m a l 1 5 0 0 - 8 0 0 0 / m m ) . T h e tests remain normal in
senger. W h i c h of the following is the most likely diag- w e e k l y monitoring. After 3 months, the patient has
nosis? had significant clinical improvement, but the W B C
drops to 3 2 0 0 , the A N C drops to 2100, and imma-
(A) Brief psychotic disorder without marked stressor ture cell forms a r e present on peripheral blood smear.
(B) Bipolar-disorder with psychotic features Repeat tests show a W B C of 3100, an A N C of
(C) Schizoaffective disorder, bipolar type 1900, a n d no immature cell forms. T h e physical
(D) Schizophrenia, catatonic subtype examination is normal, with no fever, sore throat, or
(E) Schizophreniform disorder other sign of infection. W h a t w o u l d be the best next
step in the management of this patient?

(A) Continue current dosoge of clozapine and begin twice-


2n weekly monitoring of the WBC and differential.
(B) Immediately and permanently discontinue clozapine.
A c c o r d i n g to the principles of dialectical behavior
(C) Interrupt clozapine therapy until the WBC is normal, and
therapy, the core deficit in borderline personality dis-
then resume treatment.
order is in:
(D) Reduce the dose of clozapine and begin weekly monitoring
(A) regulation of affect. of the WBC and differential.
(B) capacity for attachment. (E) Routinely monitor the WBC and differential unless the
(C) object constancy^ patient develops signs and symptoms of infection.
(D) self-integration.
(E) impulsive aggression.

214
According to DSM-IV-TR, w h i c h personality disorder
cannot be d i a g n o s e d in children a n d adolescents?

(A) Paranoid
(B) Dependent
(C) Schizotypal
(D) Borderline
(E) Antisocial

32 FOCUS Psychiatry Review; 400 Self-Assessment Questions


215 219
T h e first-line treatment of choice (determined by expert A 55-year-old man presents with depressed m o o d ,
consensus) for acute posttraumatic stress disorder poor concentration, poor appetite, feelings of w o r t h -
(PTSD) milder severity is: lessness, a n d insomnia 4 w e e k s after alcohol cessa-
tion. There is no history of m a n i a . W h i c h of t h e '
(A) low-dose venlafaxine.
following is the best next step?
(B) psychotherapy.
(C) combination of a mood stabilizer and psychotherapy. (A) Becjin an antidepressant.
(D) any selective serotonin reuptake inhibitor (SSRl). (B) Begin a sleep aid.
,(C) Begin an anticonvulsant.
(D) Begin to phase-advance sleep onset.
(E) Wait 7-10 days, then reassess.
216
A 54-year-old w o m a n is hospitalized with hyperther-
mia, myoclonus, delirium, and autonomic instability.
220
W h i c h of the following medication combinations would
be most likely to cause this clinical presentation? A 16-year-old girl with depression has suicidal ide-
ation. W h i c h of the following characteristics is the most
(A) Bupropion and venlafaxine
strongly associated with a greater risk of completed sui-
(B) Desipramine and escitalopram
cide?
(C) Duloxetine and fluoxetine
(D) Paroxetine and phenelzine (A) Limited cognitive abilities
(E) Sertraline and buspirone (B) Perfectionist characteristics
(C) Previous suicide attempt
(D) Strong religious beliefs
(E) Superficial cutting of forearms
2U
In addition to lithium, which of the following is rec-
ommended as a first-line monotherapy for bipolar I
221
disorder, depressed m o o d , in the revised APA Practice
Guideline for the Treatment of Patients With Bipolar A psychiatrist routinely receives free golf outings,
Disorder (2002]? concert tickets, and dinners as gifts from a local phar-
maceutical representative. W h i c h of the following
(A) Lamotrigine
statements most adequately describes the ethics of
(B) Divalproex
this practice?
(C) Gabapentin
(D) Bupropion (A) It is ethical if no single gift is worth more than $250.
(B) Self-monitoring and self-regulation are the most effective
woysW minimizing harm from conflicts of interest.
(C) There is no evidence that pharmaceutical company market-
218 ing to physicians influences physicians' behavior.
(D) This is a conflict of interest for the psychiatrist.
A 15-year-old African American male high school fresh-
man is referred to a psychiatrist because of increasing
oppositional behavior at school. In middle school he
was an honor roll student, played soccer, and w a s on
222
student council, all of which he continued in his first
9 weeks of high school. On the weekends, he volunteers The process of gene m a p p i n g , performed to deter-
at a local Boys and Girls Club and plays the keyboard mine whether or not a particular allele occurs more
at his church. After a couple of sessions, he finally admits frequently than by c h a n c e in affected individuals, is
that he needed to "prove myself to my boys because known as w h i c h type of study?
they said I was 'acting white'." W h i c h of the following
(A) Twin
is the most likely reason for his peers' denigration?
(B) Linkage
(A) Being on student council (C) Association
(B) Doing volunteer work (D) Family
(C) Having honor roll grades (E) Segregation analysis
(D) Playing soccer
(E) Playing the keyboard

Section 1: Self-Assessment Questions


223 228
W h i c h of the following is the most common psychiatric There is accumulating evidence suggesting that all of
disturbance among adolescents w h o die by suicide? the following psychotherapies are beneficial in bipo-
lar I disorder EXCEPT:
(A) Schizophrenia
(B) Depressive disorders (A) interpersonal and social rhythm therapy.
(C) Antisocial behavior/conduct disorder (B) cognitive behavioral therapy.
(D) Anxiety disorders (C) family therapy.
(E) Alcohol dependence (D) psychoanalysis.

224 229
W h i c h of the following is the best medication treat- A 45-year-old patient with heroin dependence is
ment for premature ejaculation? admitted to the infectious disease service for intra-
venous antibiotic treatment of bacterial endocarditis.
(A) Bupropion
An H I V test is negative. There is no other past psychi-
(B) Lorazepam
atric history. O p i a t e w i t h d r a w a l is adequately con-
(C) Paroxetine
trolled with oral methadone. On hospital d a y 3, the
(D) Risperidone
patient becomes acutely anxious, has moderate tachy-
(E) Trazodone
cardia, a n d asks to be discharged from the hospital.
1 A low-grade fever develops, but blood cultures are
negative and a complete blood count shows no signif-
icant increase or shift in leukocytes. The most likely
225 explanation for the c h a n g e in the patient's condition is:
T h e highest rates of posttraumatic stress disorder
(A) an occult infection.
(PTSD) h a v e been reported to be induced by:
(B) alcohol or sedative-hypnotic withdrawal.
(A) combat. (C) an undiagnosed anxiety disorder.
(B) sexual assault. (D) a medication reaction, most likely to the antiUtic.
(C) natural disasters.
(D) motor vehicle accidents.

230
Long-term treatment with w h i c h of the following med-
226
ications has been demonstrated to reduce suicide risk
W h i c h of the following laboratory test results is ele- in bipolar disorder?
v a t e d in some patients with a n o r e x i a nervosa?
(A) Carbamazepine
(A) Amylase (B) Divalproex
(B) Magnesium (C) Lithium
(C) Phosphate (D) Olanzapine
(D) Potassium
(E) Zinc

227 •_
. W h i c h of the following is the most common side effect
of cholinesterase inhibitors?

(A) Anorexia
(B) Muscle cramps '
(C) Nausea
(D) Somnolence
(E) Syncope

FOCUS Psychiatry Review: 400 Self-Assessment Questions


231 234
A psychiatric referral is requested to evaluate a 25- A 42-year-old morbidly obese man is seen for chronic
year-old w o m a n w h o wishes to undergo a second fatigue. Findings on polysomnography indicate
rhinoplasty because, she states, "the first one left my obstructive sleep a p n e a . If the sleep a p n e a is left
nose too big." In tears, the patient states that her dis- untreated over a prolonged p e r i o d , w h i c h of the fol-
comfort about the appearance of her nose prevents lowing conditions is most likely to develop?
her from having an active social life. She pleads with
the psychiatrist to render an opinion that will permit (A) Cataplexy
the surgery. The patient doe:; not a p p e a r psychotic. (B] Catalepsy
She does not express a n y other obsessional thoughts. ,(C) Pulmonary hypertension
In the psychiatrist's opinion, the patient's nose is (D) Obstructive pulmonary disease
unremarkable. W h i c h of the following disorders is the (E) Sleep paralysis
most likely diagnosis for this patient?

(A) Delusional disorder, somatic type


(B) Obsessive-compulsive disorder 235
(C) Body dysmorphic disorder
A patient in psychotherapy believes that her therapist
(D) Hypochondriasis
wants to help her, she characteristically trusts him with
(E) Somatization disorder
v e r y private material, and she has at times expressed
her feeling that they have many things in common a n d
that in many w a y s she views him as a role model. This
patient's alliance is best characterized as:
232
(A) erotic.
A 50-year-old man is treated with several trials of sin-
(B) idealized.
gle antidepressants. His unipolar depression has been
(C) positive.
only partially responsive. W h i c h of the following
(D) primitive.
cgents has the best evidence from randomized con-
(E) mirroring.
trolled trials to support its use in augmenting his anti-
depressant?

(A) Bupropion
(B) Buspirone 236
(C) Lithium f
For which of he anxiety disorders does c l o n a z e p a m
(D) Methylphenidate
have an F D A indication?
(E) Triiodothyronine (T )
3

(A) Generalized anxiety disorder


(B) Obsessive-compulsive disorder
(C) Panic disorder
233 (D) Posttraumatic stress disorder
(E) Social phobia
A 65-year-old man seen in the emergency department
is agitated, tachycardic, hypertensive, a n d tremulous.
He sees fish swimming on the wall: "It's just like
watching television." T h e most likely diagnosis is:
237
(A) delirium.
A 22-year-old female presents with symptoms of
(B) delusional disorder.
depression. She is a l w a y s thinking that the worst will
(C) depression.
occur in her relationships and employment, and she
(D) obsessive-compulsive disorder. feels powerless to alter or control these events. She is
(E) schizophrenia. seeking treatment with a therapist w h o provides cog-
nitive behavior therapy. W h a t is the most likely focus
of the therapy for this patient?

(A) Anger turned inward


(B) Early deprivation
(C) Difficulties in relationships
(D) Self-image
(E) Maladaptive thought patterns

Section 1: Self-Assessment Quesr.orj


238 242
A 36-yecr-old female graduate student presents with A small-town newspaper's reporter calls a psychiatrist
atypical depression that has not responded to selec- to get "a professional's opinion" on the publicized
tive serotonin reuptake inhibitors (SSRls). W h i c h of the misbehavior of the district's elected representative,
following medications has the most evidence for effi- w h o m a y or may not have bipolar disorder. T h e
c a c y in this situation? reporter asks the psychiatrist, w h o has never met the
representative, " W h y do y o u think the representative
(A) Bupropion misbehaved, doctor?" The psychiatrist's ethical obliga-
(B) Phenelzine tions w o u l d lead to which of the following responses?
(C) Valproic odd
(D) Trazodone (A) Inform the public about the representative's bipolar disor-
(E) Imipromine der, since this person is a public figure.
(B) Comment on the representative's condition only if the rep-
resentative has not been a patient.
(C) Comment on either childhood dynamic origins or brain
239 abnormality as the possible cause of the representative's
problems, if public records contain information that is con-
"Guevodoces," which translates as "penis at 12," re-
sistent with such possibilities.
fers to Dominican children with a female a p p e a r a n c e
(D) Comment on the general nature of psychiatric illnesses.
at birth, reared as girls, w h o at puberty develop male
secondary sexual characteristics and male-typical sex-
ual urges and behaviors. T h e genetic condition they
h a v e is called:
243
(A) androgen insensitivity syndrome.
In psychodynamic psychotherapy, a boundary cross-
(B) cloocal extrophy.
ing—unlike a b o u n d a r y violation—is: t .
(C) congenital adrenal hyperplasia.
(D) Klinefelter syndrome. (A) discussed with the patient.
(E) 5-a reductase deficiency. (B) an exploitative break in the therapeutic frame.
(C) generally not examined in the therapy.
(D) harmful to the therapy.
(E) a repeated occurrence.
240
Rapid cycling is LEAST likely to respond to:

(A) divalproex. 244


(B) corbomozepine.
A male psychiatrist has been conducting weekly psy-
(C) haloperidol.
chotherapy for the last 4 months with a female patient.
(D) lithium.
T h e patient has serious financial problems due to over-
spending. O n e day, the patient brings in a gift-wrapped
box to the session a n d , handing the box to the psychi-
atrist, blurts out, "It's a $100 tie ... I couldn't help
241 myself, it just looked like something y o u ' d w e a r a n d
During an office follow-up visit, a 19-year-old w o m a n - I'm so grateful for all of y o u r help. Please accept it!"
with schizophrenia reports no improvement in her W h i c h of the following is an appropriate response for
symptoms despite being on an appropriate dose of an ' the psychiatrist to give to this patient?
antipsychotic medication for 3 w e e k s . T h e most rea-
(A) Accept the gift but donate it to charity without telling the
sonable initial approach w o u l d be to: ,
patient.
(A) add on anticonvulsant medicotion. (B) Accept the gift but make it clear that the psychiatrist is uncom-
(B) add another antipsychotic medication. fortable doing so, given jhe patient's financial difficulties.
(C) change to another antipsychotic medication. (C) Acknowledge the patient's gratitude, discuss the implica-
(D) explore potential nonadherence. tions, but state that as a general policy the psychiatrist does
(E) increase the dose of the patient's current medication. not accept gifts from patients.
(D) Decline the gift without further explanation.

36 FOCUS Psychiatry Review; 400 Self-Assessment Questions


245 248
A consultation is requested for a 16-year-old male Methylphenidate has its greatest action on which of
w h o has been in detention for the past 2 months on the fojlowing neurotransmitter systems?
charges of possession of cocaine. T h e detention cen-
(A) Acetylcholine
ter staff describe the youth as hyperactive, inattentive,
(B) Dopamine
impulsive, and easily distracted. A r e v i e w of his edu-
(C) Gamma-aminobutyric acid
cational history indicates that the youth has been in
(D) Glutomate
special education classes since the first g r a d e
because of attention deficit hyperactivity disorder and (E) Serotonin
a mixed expressive-receptive' language disorder. On
examination, there is no evidence of a m o o d or anxi-
ety disorder or current substance abuse. W h i c h of the
following medications w o u l d be most appropriate for 249
this patient? W h i c h mental disorder is the most frequent cause of
first-onset psychosis after a g e 60?
(A) Atomoxetine
(B) (lonidirie (A) Dementia of the Alzheimer's type
(C) Desipramine (B) Bipolar disorder
(0) Mixed salts of amphetamines (C) Delusional disorder
(E) Pemoline (D) Major depression
(E) Very late onset schizophrenia

246
A 78-year-old patient with major depressive disorder is
250
being treated with atorvastatin and metoprolol for car- Psychotic features do N O T occur during which of the
diovascular disease. W h i c h of the following antide- following?
pressants is best used with these two other medications?
(A) Manic episode
(A) Bupropion (B) Mixed episode
(B) Escitolopram (C) Hypomanic episode
(C) Fluoxetine (D) Major depressive episode
(D) Nefazodone
(E) Paroxetine

251
The parents of a 7-year-old b o y express concern
247
about his bed-wetting. T h e b o y seems well adjusted,
A 23-year-old w o m a n presents to the clinic with a and the family has developed a nonstigmatizing sys-
chief complaint of having sexual problems. She tem to care for his bed and personal hygiene. He has
reports that she gets aroused and enjoys intercourse no medical problems. After explaining the natural his-
but is unable to have an o r g a s m . She believes the tory of enuresis, the most reasonable initial approach
problem started about a month a g o , w h e n her physi- would be to:
cian prescribed a medication for her "anxiety
(A) start desmopressin.
attacks." The medication w a s most likely:
(B) start imipramine.
(A) bupropion. (C) provide observation and follow-up.
(B) buspirone. (D) order a bell and pad.
(C) citalopram. (E) start psychotherapy.
(D) mirtazapine.
(E) trazodone.

Section I: Self-Assessment Questions 37


252 255
A psychiatrist repeatedly and increasingly fantasizes W h i c h of the following signs or symptoms alone
about a sexual relationship with a patient in psy- would be sufficient to meet criterion A for the active
chotherapy w h o m the psychiatrist finds v e r y attrac- phase of schizophrenia?
tive. T h e psychiatrist is considering the possibility that
(A) Bizarre delusions
the prohibition of sex with patients m a y not apply in
(B) Catatonic behavior
this case because of some extenuating circumstances.
(C) Incoherent speech
W h i c h of the following options w o u l d be the most eth-
(D) Negative symptoms
ical behavior on the part of the psychiatrist?
(E) Tactile hallucinations
(A) Keep a diary of the sexual fantasies in order to contain them.
(B) Increase the frequency of therapy sessions in order to
moke the best use of the intensity of the transference that
is developing. 256
(C) Transfer the patient's care to another psychiatrist.
A 32-year-old w o m a n w a s unexpectedly terminated
(D) Because there are important psychodynamic therapeutic
from her job. T w o months later she presents tearfully
implications for the patient, shore the fantasies with the
with depressed mood and occasional feelings of
patient if the benefits seem to outweigh the risks.
hopelessness; she still feels stressed by the loss of her
job. She has no prior history of depression, is not sui-
cidal, has not had changes in appetite, weight, sleep,
or energy level, and still gets pleasure from family a n d
253 1
hobbies. W h i c h of the following diagnoses would be
W h i c h of the following statements about a defen- the most appropriate?
dant's competency to stand trial for a criminal offense
(A) Major depressive disorder
i s N O T correct?
(B) Bipolar II disorder
(A) The defendant must be able to remember what he or she (C) Bereavement
was doing at the time of the offense. (D) Adjustment disorder
(B) The defendant must be able to communicate with attorneys. (E) Dysthymic disorder
(C) The defenilrnt must be able to understand basic courtroom
procedure.
(D) The defendant must be able to understand the nature of
various possible pleas and their consequences. 257
W h i c h of the following has been a p p r o v e d by the
F D A for the treatment of alcohol dependence?

254 (A) Buprenorphine


(B) Levo-alpha-acetylmethadol (LAAM)
A 34-year-old Puerto Rican w o m a n presents in distress
(C) Naloxone
at the outpatient clinic. She reports that her grandfa-
(D) Naltrexone
ther recently died, and since then she has been
afflicted by several bouts of otaque de nervios. She
has these spells only w h e n she is upset. A detailed his-
tory should be obtained to distinguish ataque from
w h i c h other axis I diagnosis? 258
A psychiatrist maintains private therapy progress notes,
(A) Bipolar disorder
in addition to medical record notes, that contain
(B) Histrionic personality disorder
extremely sensitive clinical information. This practice is:
(C) Obsessive-compulsive disorder
(D) Panic disorder (A) an acceptable means of enhancing patient confidentiality.
(E) Schizophrenia (B) on acceptable way of preventing court-mandated access to
sensitive clinical information.
(C) not acceptable, because all clinical material should be
included in patients' medical records.
(D) not acceptable, because :ihe risks to the patient outweigh
the benefits.

FOCIJ^ Psvchiatrv Review: 400 Self-Assessment Questions


259 262
W h i c h of the following psychosocial treatments is Blocking craving for opiates with subsequent reduc-
most likely to be effective in the treatment of obsessive- tion irt associated drug use generally requires w h i c h
compulsive disorder? of the following daily doses of methadone?

(A) Cognitive therapy (A) 5 mg


(B) Supportive psychotherapy (8}10mg
(C) Interpersonal therapy . (C) 20 mg
(D) Behavioral therapy ( , (D) 40 mg
1
(E) Group therapy ' (E) 80 mg

260 263
A 7-year-old b o y presents to a clinic on referral from W h i c h of the foil owing is the most important consid-
the school with a number of behavior problems, includ- eration for the treatment plan w h e n performing an ini-
ing impaired attention, hyperactivity, and impulsivity. tial evaluation of a patient with borderline personality
His parents have described him as a "whirling dervish" disorder in suicidal crisis?
for years. At a g e five, he w a s evaluated by his primary
(A) Safety
care physician and started on methylphenidate, which
(B) Goals
produced significant improvement in his behavior.
(0 Type
However, he then developed jerky, irregular muscle
movements around the eyes and mouth that persisted (D) Frame
w h e n he w a s off the medication. The medication that (E) Outcome
could address all of his symptoms is:

(A) clonidine.
(B) d,l-amphetamine. 264
(C) haloperidol.
The best documented treatment for posttraumatic
(D) magnesium pemoline. •
stress disorder (PTSD) precipitated by a violent rape
(E) pimozide.
includes:

(A) event recall.


(B) martial arts instruction.
261 (C) prosecution of the rapist.
(0) cognitive-based therapy.
A 16-year-old girl has been blinking her eyes a n d
clearing her throat on an intermittent basis for years.
She has no control of the symptoms a n d has never
been free of them for more than a few d a y s , and they
cause significant problems. W h a t medication may be
265
helpful for treating this problem? A 40-year-old female comes to the mental health cen-
ter for the first time. After a thorough assessment, she
(A) Pimozide
is told that the best treatment would be a course of
(B) Nortriptyline
brief psychotherapy. She looks concerned and asks if
((] Paroxetine
she can be transferred to a doctor of her o w n race.
(0) Lorazepam
The appropriate step to take w o u l d be to:
(E) Methylphenidate
(A) attempt to convince her that any doctor is capable.
(8) explore why she feels this is necessary.
(C) grant her request and transfer her.
(0) help her find another clinic that will suit her.
(E) switch to medicotion management only.

Seccior. I: Self-Assessment Questions 39


266 269
DSM-IV-TR cultural formulation for a patient from a cul- W h i c h of the following best describes a characteristic
ture different than the psychictrist's requires: of the assertive community treatment ( A C T ) model for
management of schizophrenia?
(A) Q HISTORY OF THE PATIENT'S EDUCATION AND OCCUPATIONAL TRAINING.

( B ) INDEPENDENT INFORMATION FROM A CULTURAL CONSULTANT. (A) CLINIC-BASED SERVICES

(C) AN UNDERSTANDING OF THE NEUROBIOLOGY OF THE PATIENFS DISORDER. (B) FOCUS ON SYMPTOM RESOLUTION

( D ) AN UNDERSTANDING OF THE EFFECT OF THE PSYCHIATRIST'S OWN CUL- (C) HOSPITAL-BASED SERVICES

TURE ON TREATMENT VARIABLES. ( D ) PSYCHIATRIST-LED TREATMENT TEAM

( E ) USE OF AN INTERPRETER FROM OR ASSIMILATED IN THE PATIENT'S (E) 24-HOUR AVAILABILITY OF SERVICES

CULTURE.

270
267 T h e strong association between physical illness and
A 20-year-old male college student presents in the suicide has been demonstrated for which of the fol-
e m e r g e n c y department with confusion and agitation. lowing conditions?
He is distracted and talks in a rambling manner.
(A) AMYOTROPHIC LATERAL SCLEROSIS
During the interview, he reports seeing an angel w h o
( B ) BLINDNESS
is telling him about his mission. His roommate states
(C) EPILEPSY
that the student has been having problems for months,
( D ) HYPERTENSION
with worsening g r a d e s , not sleeping, and withdrawal
(E) DIABETES MELLITUS
from friends. In establishing a diagnosis and prepar-
ing to initiate treatment, the most appropriate labora-
t o r y test to obtain at this point w o u l d be:

( A ) A COMPLETE BLOOD COUNT, INCLUDING A PLATELET COUNT. 271


( B ) AN ELECTROCARDIOGRAM.
A 32-year-old man sees his primary care physician
(C) HEPATIC FUNCTION TESTS.
because of a recurrent productive c o u g h . The physi-
( D ) THYROID FUNCTION TESTS.
cian recommends blood w o r k a n d a chest X-ray.
(E) A TOXICOLOGY SCREEN.
W h e n the patient enters the phlebotomy suite, his
heart begins to race, he perspires, and his muscles
tense. W h e n he sits in the phlebotomy chair and a
tourniquet is applied, his symptoms w o r s e n . In addi-
268 tion, he becomes short of breath, begins to hyperven-
A 60-year-old w o m a n presents with daytime fatigue, tilate, a n d feels numbness and tingling in his hands
morning h e a d a c h e , and poor memory. Findings from and feet a n d around his mouth. W h e n the phle-
her physical examination a n d blood studies are all botomist uncaps the needle, the patient passes out.
within normal limits, a n d she reports that her mood is He awakens shortly after an ammonia capsule is bro-
normal. On further questioning she reports that her ken under his nose. He apologizes for his behavior
husband sleeps in a separate room because of her and says, "I a l w a y s get this w a y w h e n I see a nee-
snoring a n d thrashing. T h e most effective treatment for dle." This presentation is most consistent with:
this condition is: . . "7-1
(A) AGORAPHOBIA.

(A) FLUOXETINE. ( B ) GENERALIZED ANXIETY DISORDER.

( B ) CONTINUOUS POSITIVE AIRWAY PRESSURE. (C) PANIC DISORDER.

(C) LORAZEPAM. ( D ) SOCIAL PHOBIA.

( D ) METHYLPHENIDATE. (E) SPECIFIC PHOBIA.

( E ) RELAXATION THERAPY.

40 FOCUS PCYAJATRY Review: 400 Self-Assessment Questions


272 275
W h i c h of the following symptoms is significantly more W h i c h of the following therapies has the best evi-
likely to be associated with posttraumatic stress disor- dence »for effectiveness in the treatment of posttrau-
der (PTSD) than with normal bereavement? matic stress disorder?

(A) Initial shock (A) Present-centered group therapy


(B) Depressive symptoms (B) Psychological debriefings
(C) Numbing (C) Singfe-session techniques
(D) Avoidance of reminders t , (D) Cognitive behavior therapy
(E) Sleep disturbance 1 1
(E) Trauma-focused group therapy

273 276
A patient with schizophrenia is in the midst of a severe Linkage analysis can be defined as:
exacerbation but refuses treatment. The patient is able
(A) a test to identify which of several genes in a chromosomal
to paraphrase what the psychiatrist has said about the
region is involved in the disorder in question.
diagnosis, the prognosis, and the reasons for the pro-
(B) a test to determine the chromosomal region where a disor-
posed treatment with medications. W h i c h of the fol-
der resides by searching for co-segregation of a genetic
lowing statements by the patient is the clearest example
marker with the disorder locus.
of an impaired ability to "appreciate or understand"?
(C) a study that requires the cause of the disorder to be a com-
(A) "I have tried all those antipsychotics before. None of them mon risk variant.
work that well for me so why try again." (D) an analysis that is not sensitive to a genetic model.
(B) "Your office is bugged, but the reason why I do not want to
take the medication is that I am really afraid of gaining
more weight."
(C) "The space aliens living in my stomach would be injured if 277
I took those pills."
W h i c h of the following is true regarding adolescents
(D) "I am a Christian Scientist and I do not believe that I have
with attention deficit hyperactivity disorder ( A D H D )
a disease."
w h o are treated with methylphenidate?

(A) Significantly reduced risk of substance abuse in later life


(B) Higher level of all substance abuse in adulthood
274 (C) Increased alcohol abuse in adulthood
(D) Increased cannabis abuse in adulthood
A 42-year-old w o m a n with generalized anxiety disor-
der has responded favorably to 60 m g / d a y of Buspi-
rone. To avoid substantially increasing the blood level i
of the medication a n d producing side effects, you cau-
tion her to avoid regular consumption of which of the
278
following beverages? Elements of an individual's ability to make decisions
about undergoing treatment or participating in
(A) Apple juice
research include ell of the following E X C E P T :
(B) Coffee
[Q Grapefruit juice (A) understanding the information provided.
(D) Milk (B) reasoning with the information or weighing options.
(E) Red wine (C) repeating the outlined risks and benefits without prompting.
(D) appreciating the significance of the information for the
individual's own situation.

Section 1: Self-Assessment Questions


286 290
A patient has not responded to phenelzine after 10 C o m p a r e d with Caucasian Americans, African Amer-
weeks of treatment, a n d a switch to fluoxetine is icans are more likely to receive a diagnosis of:
planned. W h a t is the recommended minimum interval
between stopping phenelzine a n d starting fluoxetine?
(A) bipolar disorder, depressed.
(B) bipolar disorder, manic
( A ] l week (C) major depression.
(B) 2 weeks (D) schizophrenia.
(C) 4 weeks , (E) substance-induced psychosis.
(D) 6 weeks •
(E) 8 weeks

29V
287 T h e clinical sign that best differentiates delirium from
dementia is:
The single most effective treatment for major depres-
sion in elderly patients is: (A) agitation.
(B) confusion.
(A) bupropion.
(C) fluctuating consciousness.
(B) citalopram.
(D) poor attention span.
(0 EOT.
(E) psychosis.
(D) nortriptyline.
(E) venlafaxine.

292
288
Gabapentin has F D A approval as an indication for
A 30-year-old athletic man presents for evaluation of which of the following?
several syncopal episodes over the past month. He
has been treated for hypertension during the past (A) Postmenopausal hot flashes
y e a r and has responded nicely to 50 m g / d a y of (B) Posttraumatic stress disorder (PTSD)
metoprolol XR and 25 m g / d a y of hydrochloro- (C) Postherpetic neuralgia
thiazide. Three months a g o his primary care physi- (D) Cocaine dependence
cian started him on 20 m g / d a y of fluoxetine and
0.5 m g / d a y of lorazepam t.i.d. for mixed anxiety
and depression. On examination the patient seems
mildly anxious and demonstrates orthostatic hypoten- 293
sion. His E C G is unremarkable except for mild sinus A patient with a first episode of a nonpsychotic major
bradycardia. W h a t is the most likely explanation? depression has responded well to the acute phase
medication treatment. W h a t is the typical duration of
(A) Transient ischemic attacks
the continuation phase?
(B) Fluoxetine^—metoprolol interaction
(C) Overdiuresis (A) 3 months
(D) Benzodiazepine intoxication (B) 4 to 9 months
(E) Psychogenic syncope (C) 10 to 15 months
(D) 2 years
(E) Lifelong

289
Attention deficit hyperactivity disorder (ADHD)
appears to be most strongly associated with prenatal
exposure to:

(A) caffeine.
(B) lithium.
(C) nicotine.
(D) SSRIs.
(E) valproic acid.

Section 1: Self-Assessrr.e.n: Questions 43


279 282
A school guidance counselor refers a 5-year-old girl Symptoms of obsessive-compulsive disorder respond
w h o will not speak. The girl has been enrolled in best to w h i c h of the following tricyclic antidepressants?
school for 3 months. During this time, she has been
(A) Imipramine
noted to make hand gestures or nod in response to
(B) Amitriptyline
her teacher or peers. The guidance counselor has
(C) Doxepin
been meeting with the girl regularly, a n d recently the
(D) Clomipramine
child has begun to whisper. H o w e v e r , she will not use
(E) Desipramine
a normal voice. The girl's parents report that the child
has no problems speaking at home. T h e girl plays
with her peers, makes appropriate e y e contact w h e n
spoken to, seems interested in others, and has no
unusual movements. There have been no delays or
283
abnormalities in development. As an adult, this child A 23-year-old w o m a n presented with a 2-week history
is at high risk of developing: of difficulty sleeping, hearing voices, and problems
with thinking. She w a s fearful a n d suspicious, a n d
(A) major depressive disorder.
talked about evil alien forces out in the w o r l d . Some
(B) obsessive-compulsive disorder.
of her relatives have h a d "nervous b r e a k d o w n s "
(C) posttraumatic stress disorder.
requiring hospitalization. Further evaluation revealed
(D) schizophrenia.
that the w o m a n had been raped about 3 weeks ear-
(E) social phobia.
lier. H o w e v e r , she has no recollection of the event.
O n e w e e k after initial presentation, her symptoms
have disappeared and she has returned to normal
functioning. T h e most likely diagnosis at this time is:
280
(A) acute stress disorder.
A 10-year-old boy is brought for consultation for "bed-
(B) brief psychotic disorder.
wetting." His parents report that he b e g a n using the
(C) schizoaffective disorder.
toilet and staying dry during the d a y w h e n he w a s
(D) posttraumatic stress disorder.
3 years old. However, he has never consistently been
(E) schizophreniform disorder.
able to control his bladder during sleep. Physical
examination and laboratory studies have demon-
strated no abnormalities. His father reports that he
also wet the bed as a child but stopped w h e n he w a s
about 12 years old. The intervention that is most likely
284
to have long-term effectiveness with this boy is: W h i c h of the following medications is most likely to
be associated with polycystic o v a r y syndrome?
(A) hypnotheropy.
(B) low-dose tricyclic antidepressants. (A) Carbamazepine
(C) oral desmopressin. (B) Gabapentin
(D) psychotherapy. (C) Lithium
(E) urine alarm (bell and pad). (D) Topiramate
(E) Valproate

281
A 58-year-old man has a history of ingesting 1 to
285
2 pints of v o d k a on a daily basis over the, past In the National Institute of Mental Health's Epidemi-
20 y e a r s . He presents to the e m e r g e n c y department ologic Catchment A r e a study, the ethnic differences in
offer a minor motor vehicle accident a n d appears dis- the 1-month prevalence of mental health disorders
o r g a n i z e d . A computerized t o m o g r a p h y scan of his d r o p p e d after w h i c h of the. following factors w a s con-
h e a d is most likely to show which of the following? trolled for?

(A) Acoustic neuroma (A) Age


(B) Caudate calcification . (B) .Education
(C) Cerebellar degeneration (C) Gender
(D) Frontal lobe tumor (D) Literacy rate
(E) Prolactinoma * (E) Socioeconomic status

42 FOCUS Psychiatry Review: 400 Self Assessment Questions


294 297
A 40-year-old w o m a n consults a psychiatrist with a Currently, the efficacy of a psychotherapy for treat-
chief complaint of anxiety, insomnia with nightmares, ment of a particular disorder is best judged by:
loss of appetite, and chest pain. Tearfully, the patient
(A) cohort study.
reports that 2 weeks a g o her husband left her for
(B) individual case outcomes.
another w o m a n . The husband told the patient, "I need
(C) number needed to treat to number needed to harm ratio.
someone more adventuresome." She suspected that
(D) relative risk reduction measure.
her husband w c s having an affair, but she w a s unpre-
(E) systematic review of controlled studies.
p a r e d for his leaving. She avoids walking by his
office in their home because w h e n she sees his litter,
still on the desk, she feels chest pain. She reports fear
of being alone. She continually d a y d r e a m s about
their life together. She can " b a r e l y function" in her job
298
as a hospital administrator. T h e most likely prelimi- A 25-year-old w o m a n is diagnosed with bipolar I dis-
n a r y diagnosis is: order. She has a previous history of several suicide
attempts. Of the following medications, which would
(A) acute stress disorder.
be the most likely to decrease her risk for suicide if
(B) odjustment disorder with anxiety.
administered on a long-term basis?
(C) pathological bereavement.
(D) posttraumatic stress disorder. (A) Carbamazepine
(E) social phobia. (B) Lamotrigine
(CJ Lithium
(D) Risperidone
(E) Verapamil
295
A 74-year-old man falls on an ice patch and bumps
his h e a d . During the next 4 w e e k s , his wife notices
that he seems more forgetful a n d that at night he is dis-
299
oriented. He also develops a persistent headache. A 29-year-old n a n has severe panic attacks cued by
W h i c h of the following diagnoses is most likely to be public speaking. He has developed marked avoid-
causing this presentation? ance of such situations, which has greatly compro-
mised his career development. W h i c h of the following
(A) Cerebellar tumor
is the most appropriate diagnosis?
(B) MultMnfard dementia
(C) Occipital tumor (A) Agoraphobia without panic disorder
(D) Subdural hematoma (B) Acute stress disorder
(E) Wernicke's encephalopathy (C) Panic disorder with agoraphobia
(D) Social phobia
(E) Specific phobia

296
W h a t proportion of people with dysthymic disorder
experience an episode of major depression in their
300
lifetime? A patient w h o s e depression has responded well to an
SSRI n o w reports symptoms of erectile dysfunction
(A) 5%—10%
associated with the SSRI antidepressant therapy. This
(B) 20%-30%
dysfunction has persisted for more than a month. T h e
(C) 40%-50%
best initial a p p r o a c h w o u l d be to:
(D) 70°/o-80%
(E) 100% (A) add bupropion. . i (

1
(B) take a drug holiday.
(C)'reduce the dose of the antidepressant.
(D) switch to a different SSRI.
(E) continue treatment until the patient develops tolerance to
the side effect.

AA
294 297
A 40-year-old w o m a n consults a psychiatrist with a Currently, the efficacy of a psychotherapy for treat-
chief complaint of anxiety, insomnia with nightmares, ment of a particular disorder is best judged by:
loss of appetite, and chest pain. Tearfully, the patient
(A) cohort study.
reports that 2 weeks a g o her husband left her for
(B) individual case outcomes.
another w o m a n . The husband told the patient, "I need
(C) number needed to treat to number needed to harm ratio.
someone more adventuresome." She suspected that
(D) relative risk reduction measure.
her husband w c s having an affair, but she w a s unpre-
(E) systematic review of controlled studies.
p a r e d for his leaving. She avoids walking by his
office in their home because w h e n she sees his litter,
still on the desk, she feels chest pain. She reports fear
of being alone. She continually d a y d r e a m s about
their life together. She can " b a r e l y function" in her job
298
as a hospital administrator. T h e most likely prelimi- A 25-year-old w o m a n is diagnosed with bipolar I dis-
n a r y diagnosis is: order. She has a previous history of several suicide
attempts. Of the following medications, which would
(A) acute stress disorder.
be the most likely to decrease her risk for suicide if
(B) adjustment disorder with anxiety.
administered on a long-term basis?
(C) pathological bereavement.
(D) posttraumatic stress disorder. (A) Carbamazepine
(E) social phobia. (B) Lamotrigine
(C) Lithium
(D) Risperidone
(E) Verapamil
295
A 74-year-old man falls on an ice patch and bumps
his h e a d . During the next 4 w e e k s , his wife notices
that he seems more forgetful a n d that at night he is dis-
299
oriented. He also develops a persistent headache. A 29-year-old r
- .ian has severe panic attacks cued by
W h i c h of the following diagnoses is most likely to be public speaking. He has developed marked avoid-
causing this presentation? ance of such situations, which has greatly compro-
mised his career development. W h i c h of the following
(A) Cerebellar tumor
is tf-e most appropriate diagnosis?
(B) Multi-inforct dementia
(C) Occipital tumor (A) Agoraphobia without panic disorder
(D) Subdural hematoma (B) Acute stress disorder
(E) Wernicke's encepholopothy (C) Panic disorder with agoraphobia
(D) Social phobia
(E) Specific phobia

296
W h a t proportion of people with dysthymic disorder
experience an episode of major depression in their
300
lifetime? A patient w h o s e depression has responded well to an
SSRI n o w reports symptoms of erectile dysfunction
(A) 5%—10%
associated with the SSRI antidepressant therapy. This
(B) 20%-30%
dysfunction has persisted for more than a month. T h e
(C) 40%-50%
best initial a p p r o a c h w o u l d be to:
(D) 70%-80%
(E) 100% (A) add bupropion. . <(

1
(B) take o drug holiday.
(C)'reduce the dose of the antidepressant.
(D) switch too different SSRI.
(E) continue treatment until the patient develops tolerance to
the side effect.

AA
301 305
A patient with a 10-year history of alcohol depen- In order to determine the genomic location of a sus-
dence requests outpatient detoxification. In determin- ceptibility gene for panic disorder, w h i c h of the fol-
ing w h e t h e r outpatient detoxification is an lowing approaches w o u l d be most appropriate?
appropriate treatment setting for this patient, the most
important variable is: (A) Family risk studies
(B) Genetic epidemiology
(A) length of history of alcohol dependence. (C) Gene finding
(B) support of spouse or significant other. (D) Molecular genetics
1 1
(C) type of alcohol consumed. (E) Twin studies
(0) prior history of delirium tremens.

306
302
During treatment, a female patient reports sexual
During the sexual history, a married 35-year-old male encounters with a prior therapist in a state that man-
reveals that he considers himself to be " o n the d o w n dates the reporting of sexual abuse by therapists. In
low." Regarding his sexual orientation a n d partners, the interest of preserving the confidentiality of the doc-
he would most likely consider himself to be: tor-patient relationship, w h i c h of the following is the
best response of the therapist?
(A) bisexual, and has sex equally with men and women.
(B) heterosexual, and exclusively has sex with women. (A) Refer the patient to another physician for consultation,
(C) heterosexual, but also secretly has sex with men. specifically for the role of advocacy.
(D) homosexual, but also has sex with women. (B) Request court immunity from the statute to protect the doc-
(E) homosexual, and exclusively has sex with men. tor-patient relationship.
(C) Convince the patient to report the matter herself.
(D) Explore the a legation with the patient to determine
whether it actually occurred.
303
In order for a patient to meet the diagnostic criteria
for substance a b u s e , w h i c h of the following must be
307
present?
W h i c h of the following medications has been shown to
(A) Physiologic tolerance to the substance be most effective in reducing suicidal behaviors in
(B) Physiologic withdrawal from the substance patients with schizophrenia or schizoaffective disorder?
(C) Failure to attend to expected cultural role as a result of the
substance (A) Clozapine
(D) Positron emission tomography findings of mesolimbic tract (B) Haloperidol
1
hyperactivity (C) Lithium
(E) Family history of addiction (D) Olanzapine
(E) Ziprasidone

304
308
A m o n g patients with major depressive disorder,
w o m e n have w h i c h of the following characteristics W h i c h of the following diagnostic criteria most clearly
compared with men? distinguishes paranoid personality disorder from para-
noid schizophrenia, delusional disorder, and mood dis-
(A) Earlier age at onset order with psychotic features?
(B) Shorter episode duration
(C) Higher rates of comorbid drug abuse (A) Absence of positive psychotic symptoms
(D) Lower rates of comorbid generalized anxiety (B) Age at onset
(E) Fewer suicide attempts (C) Degree of impairment in interpersonal relationships
(D) Duration of symptoms
(E) Pervasive nature of symptoms

Section 1: Scif-Assessment Questions


309 312
A 19-year-old e x c h a n g e student from Malaysia is W h i c h of the following is most likely to be preserved
brought to the emergency department by his host par- in the early stages of frontotemporal dementia?
ents after he became violent at home a n d threatened
(A) Judgment
to kill them. The parents report that he seemed fine
(B) Personality
until they commented to him that he had left the faucet
(() Verbal output
running in the bathroom. Initially, he went to his room
(D) Visuospatial skills
a n d seemed sullen. He then b e g a n "ranting a n d rav-
(E) Sociability or social involvement
ing" about h o w he is not an irresponsible person,
accused the host parents of spying on him, threatened
them, threw objects about, a n d collapsed on the floor
in exhaustion. In the e m e r g e n c y department, the stu-
dent is calm and cooperative. Mental status examina-
313
tion is unremarkable. T h e student denies a n y recall of Of the following disorders, which has the greatest
the episode. This presentation is most consistent with genetic contribution or heritability?
w h i c h culture-bound syndrome?
(A) Major depressive disorder
(A) Amok (B) Alcoholism
(B) Dhat (C) Obsessive-compulsive disorder
(C) Koro (D) Schizophrenia
(D) Locura (E) Panic disorder
(E) Rootwork 1

314
310 _ _ _
A 25-year-old man collects women's bras a n d under-
W h a t is the most common comorbid condition in chil- pants from public laundries a n d uses the objects to
dren with autistic disorder? become sexually aroused. This description is most
consistent with w h i c h of the following OSM-IV-TR
(A) Attention deficit hyperactivity disorder
diagnoses?
(B) Major depression
(C) Mental retardation (A) Exhibitionism
(D) Schizophrenia (B) Fetishism
(E) Social phob'a (C) Frotteurism
(D) Sexual masochism
(E) Kleptomania

311
A 65-year-old patient is admitted to the surgical inpa-
tient service for a hernia repair. T h e family reported
315
that o v e r the past f e w months the patient has had A 24-year-old man comes for an evaluation because he
episodes of confusion. W h i l e on the w a r d , the patient cannot relax. He reports that he constantly is thinking
b e g a n to h a v e prominent visual hallucinations. T h e about whether his car will break d o w n , his bills will get
staff administered 1 mg of haloperidol orally. A sec- paid, and if his school performance is adequate. For
ond dose w a s g i v e n 3 hours later. Soon after receiv-' over a year, he often is tired, irritable, and on e d g e .
ing the second dose of haloperidol, the patient had a Upon reflection, the student is unable to identify a n y
severe extrapyramidal response. W h i c h of the, follow- aspect of his life that is going so well that it does not
ing is the most likely diagnosis? generate concern. The most likely diagnosis is:

(A) Delirium with preexisting dementia (A) depressive disorder, not, otherwise specified.
(B) Parkinson's dementia (B) generalized onxiet>' disorder.
(C) Lewy body dementia (C) obsessive-compulsive disorder.
(D) Major depressive disorder with psychosis ? (D) panic disorder.
(E) Alcohol withdrawal . (E) social phobia.

46 FOCUS Psychiatry Review: 400 Self-Assessment Questions


316 320
T h e side effect of pancreatitis is linked most closely to An actor has received repeated complaints from col-
which of the following? leagues about his behavior in professional situations.
He has just started rehearsals for a play. T h e prob-
(A) Divalproex lematic behavior consists of excessive demands for
(B) Oxcarbazepine special treatment a n d outbursts w h e n special treat-
(C) Lamotrigine ment is not g r a n t e d . He is diagnosed as having nar-
(D) Topiramate cissistic personality disorder. He has been in treatment
for several months; treatment has been going w e l l ,
and there have been fewer demands and outbursts at
w c r k . W h i c h of the following is the patient most likely
317 to da next?
W h i c h of the following is a technique of supportive
(A) Generalize this behavior to his home environment
dynamic psychotherapy?
(B) Demand new concessions from the play's director
(A) Transference interpretation (C) Show a new understanding of his behavior
(B) Promoting therapeutic regression (D) Continue to show appropriate behavior at work
(C) Extreme passivity of therapist (E) Discuss his feelings about the therapist
(D) Problem-solving focus
(E) Frequent genetic reconstruction

321
Of the following, w h i c h is the best definition of eth-
3]8 nicity? H u m a n groups that:
T h e most effective behavior therapy technique used in
(A) share a sociopolitical designation.
the treatment of compulsions of obsessive-compulsive
(B) share common values, beliefs, history, and customs.
disorder is:
(C) have common identities, ancestries, and histories.
(A) exposure and response prevention. (D) share distinct identifying phenotypic characteristics.
• (B) negative reinforcement. (E) are living together in the same location.
(C) positive reinforcement.
(D) punishment.
(E) systematic desensitization.
322
W h i c h of the following psychotherapeutic approaches
provides the primary framework for dialectical behav-
319 ior therapy for borderline personality disorder?
A man w h o is receiving cognitive behavior therapy
(A) Cognitive behavior therapy
for depression feels guilty for massive layoffs at his
(B) Interpersonal psychotherapy
workplace, even though he w a s not involved in the
(C) Psychodynamic psychotherapy
management decision. W h i c h of the following types
(D) Family systems therapy
of cognitive error is most consistent with this patient's
(E) Supportive psychotherapy
feeling?

(A) Arbitrary inference


(B) Absolutist thinking
(C) Catastrophic thinking 323
(D) Magnification and minimization The most common DSM-IV-TR axis II personality disor-
(E) Personalization der demonstrated among persons with substance use
disorders is:

(A) borderline personality disorder.


(B) narcissistic personality disorder.
(C) dependent personality disorder.
(D) antisocial personality disorder.

Section 1: Self-Assessment Questions


324 328
A 72-year-old w o m a n is hospitalized with findings of W h i l e reviewing the treatment plan for a patient with
dementia, ataxia, and macrocytic a n e m i a . T h e most methamphetamine dependence, the psychiatrist thinks
likely diagnosis is: about h o w best to help the patient progress from the
contemplation phase to the preparation phase. T h e
(A) dementia of the Alzheimer's type. psychiatrist's a p p r o a c h to treatment in this case is
(B) vascular dementia. based on the principles of:
(C) vitamin B deficiency.
12

(D) Huntington's disease. (A) 12-step focilitation therapy.


(E) pellagra. (B) cognitive behavior therapy (CBT).
(C) contingency management therapy.
(D) motivational enhancement therapy (MET).

325
A 20-year-old w o m a n describes a 6-month history of
329
frequent binge eating followed by self-induced vomit-
ing a n d laxative use to maintain normal b o d y weight. W h i c h of the following is the most effective treatment for
W h i c h of the following medications is FDA-approved catatonic features associated with a manic episode?
for her disorder?
(A) Lithium
(A) Bupropion ^ (B) Electroconvulsive therapy
(B) Crfalopram (C) Divalproex
(C) Escitalopram (D) Clozapine
(D) Fluoxetine
(E) Venlafaxine

330
The cornerstone of relapse prevention as a modality
326
of treatment for substance-dependent patients is:
A psychiatrist w h o is grieving from a recent sudden
(A) psychodynamic technique.
loss of a spouse shares those feelings with a psy-
(B) 12-step group attendance.
chotherapy patient. W h a t is the most ethical interpre-
(C) motivational enhancement.
tation of the psychiatrist's actions?
(D) skills training.
(A) It may be ethically problematic if the psychiatrist was
driven by personal needs rather than by serving the
patient's needs.
(B) It is always ethically unacceptable because a psychiatrist 331
should never reveal personal information to a patient.
W h i c h of the following is the most c o m m o n sexual dis-
(C) It is problematic to reveal any information other than the
order in men?
psychiatrist's professional training.
(D) It is not ethically problematic because sharing the psychia- (A) Hypoactive sexual desire disorder
• trist's authentic feelings with patients is therapeutic for the" (B) Male erectile disorder
patient. (C) Premature ejaculation
(D) Male orgasmic disorder
(E) Dyspareunia

327 ,
W h i c h of the following will cause the greatest increase
in serum lithium levels?

(A) Theophylline ,
(B) Ziprasidone
(C) Hydrochlorothiazide
(D) Celecoxib
332 334
An adult female patient consumes an a v e r a g e of 14 Because of an emergency, Mr. B's psychiatrist w a s 20
glasses of w i n e per week, never consuming more than minutes* late to the second interview. Mr. B makes an
four glasses on a n y one occasion. Based solely on this offhand a n d somewhat negative comment about
drinking pattern, her physician should do w h i c h of the "doctors being too busy these d a y s . " In all likelihood,
following? this is an example of:

(A) Refer her to an addiction specialist for further evaluation. (A) reaction formation.
(B) Recommend that she begjn attending AA meetings. (8) transference.
(C) Inform her that she is drinking at a safe level. (C) idealization.
(D) Recommend that she reduce her drinking by about 50%. (D) splitting.
(E) suppression.

The following four questions {333-336} form a serial 335


vignette.
On hearing the irritation in the patient's voice, the cli-
nician begins to explain in detail the reasons for his
tardiness and apologizes profusely. He assures the
patient that he will not be late for future meetings. This
333 is an example of:

Mr. B, a high school teacher in his mid-30s, w a s (A) denial.


recently separated from his wife and two children. An (B) regression.
intelligent and verbally facile man with a particular (C) countertransference.
talent in the arts, Mr. B w a s plagued by his conviction (D) deidealization.
that he w a s unacceptable to other people unless he (E) dissociation.
complied with their expectations and gratified their
needs. This w a s a pleasant, agreeable, and compli-
ant facade that hid his feelings of weakness and stu-
pidity. He constantly sought a p p r o v a l from his 336
superiors, but underneath he felt resentment and
In beginning a brief therapy with Mr. B, the most
rebelliousness about others' expecting him to accom-
important challenge for this psychiatrist is to:
modate to their needs and wishes.
(A) prescribe an antidepressant.
Mr. B's mother w a s an embittered, burdened w o m a n ,
(B) prescribe an antianxiety agent.
contemptuous of men and preoccupied with her o w n
(C) contact the patient's wife for additional history.
needs and interests. His father, while somewhat
(D) establish a therapeutic or working alliance.
approachable, had often been a w a y from home trying
(E) set clear limits on the patient's behavior.
to make a living to support the family. Mr. B remem-
bered his father as erratic and moody and given to
temper outbursts, w h i c h , he recalls, would lead to
beatings with a leather strap. T h e middle of three chil-
337
dren, the patient felt that his father favored his older
sister and that his mother favored his younger brother, W h i c h of the following psychiatric disorders is con-
a n d he saw himself as the neglected outsider. sidered to be predominantly culture specific?

W h a t is the most likely defense mechanism utilized by (A) Bulimia nervosa


this patient w h e n first meeting the psychiatrist? (B) Generalized anxiety disorder
(C) Major depressive disorder
(A) Regression
(D) Posttraumatic stress disorder
(B) Altruism
(E) Schizophrenia
(C) Undoing projection
(D) Intellecfualization rationalization
(E) Dissociation

Section 1: Self-Assessment Questions


338 341
A 73-year-old man with moderate congestive heart W h i c h of the following is N O T F D A - a p p r o v e d for the
failure and degenerative arthritis in his right knee vis- treatment of acute mania?
its his physician for a scheduled outpatient appoint-
(A) Carbamazepine
ment. Although his physical examination findings
(B) Gabapentin
from the previous visit are unchanged, the physician
(C) Divalproex
notes that the patient appears tired a n d less interac-
(D) Olanzapine
tive than usual. Concerned that the patient may be
(E) Risperidone
experiencing a major depressive episode, the physi-
cian wishes to gather more information. The presence
of w h i c h of the following would be most helpful in
making a diagnosis of major depressive disorder?
342
(A) Complaints of pain
T h e term "four D's of n e g l i g e n c e " - d u t y , dereliction,
(B) Decreased concentration
direct, a n d damages—refers to:
(C) Loss of appetite
(D) Poor energy (A) the questions a defendant physician will be asked at depo-
(E) Die wish to die sition.
(B) what a patient/plaintiff must prove to win a malpractice suit.
(C) the calculation of punitive versus compensatory damages.
(D) the level of care that would be expected of a reasonable
339 ] physician under similar circumstances.

A 46-year-old w o m a n presents to her primary care


physician with a 2-month history of l o w back pain,
dull headaches several times a w e e k , insomnia,
fatigue, and irritability. She has a l w a y s been healthy.
343
Findings from her physical examination are all within In a psychotherapy session, a patient reveals that he
normal limits, and a review of systems is noncontribu- has been having trouble obtaining an orgasm with his
tory. Routine laboratory tests such as c chemistry partner. He states that he has a l w a y s felt aroused
p a n e l , C B C , and thyroid function tests are oil normal. w h e n traveling to w o r k on a c r o w d e d bus, and he
T h e most likely diagnosis is: used to think that this enhanced his sexual life. He
never thought it w a s a problem, but n o w he thinks it
(A) major depressive disorder.
is interfering with his relationship. W h a t is the most
(B) generalized anxiety disorder.
likely diagnosis?
(C) pain disorder.
(D) hypochondriasis. (A) Exhibitionism
(E) somatization disorder. (B) Fetishism
(C) Frotteurism
(D) Pedophilia
(E) Voyeurism
340
A 29-year-old patient with borderline personality dis-
order is being seen in psychotherapy twice weekly.
T h e psychiatrist realizes that the patient is uncon-
344
sciously trying to coerce her into acting in a judg- W h i c h of the following comparisons regarding the
mental w a y . This phenomenon is best described as: incidence a n d prevalence of posttraumatic stress dis-
order (PTSD) is the most accurate?
(A) identification with the oggressor.
(B) projection. (A) Hie condition is more prevalent in men.
(C) projective identification. "(B) The presence of a psychiatric disorder does not predispose a
(D) regression. person to PTSD. 1

(E) splitting. (C) Older individuals have a higher prevalence than younger
individuals.
(D) Certain types of trauma are more likely to cause PTSD.

4
345 348
W h i c h of the following statements is correct about the A 15-year-old girl is brought in for an emergency eval-
concordance of schizophrenia in the twin of an indi- uation because she has been out all night and refuses
vidual with schizophrenia? to tell her parents w h e r e she has been. T h e parents
report that for several months the girl has been irritable
(A) 50% if twin is monozygotic ;
and oppositional with severe" mood swings. She has
(B) 75% if twin is monozygotic been leaviag home and school without permission. T h e
(C) Almost 100% if twin is monozygotic girl admits that she has been somewhat moody but
(D) 50% if twin is dizygotic insists that her parents are making a big deal about
1
(E) 75% if twin is dizygotic '» nothing. A preliminary diagnosis of bipolar disorder is
made. W h i c h of the following is the most common
comocbid condition with bipolar disorder?

346 (A) Conduct disorder


(B) Generalized anxiety disorder
A 33-year-old w o m a n with a diagnosis of borderline
(C) Oppositional defiant disorder
personality disorder w a s recently discharged from
(0) Posttraumatic stress disorder
medical service after an aspirin overdose. She des-
(E) Substance use disorder
cribes having had thoughts of suicide off and on since
early adolescence a n d has made two previous suicide
attempts. In addressing her suicidality in treatment,
which of the following approaches would be most
appropriate?
349
A 29-year-old w o m a n is admitted to the hospital with
(A) Partial hospitalization or brief inpatient hospitalization
acute herpes simplex encephalitis. W h i c h of the fol-
(B) Outpatient psychoanalysis
lowing is the most common residual deficit upon
(C) Gabapentin pharmacotherapy
recovery?
(D) Valproic acid pharmacotherapy
(A) Apraxia
(B) Aphasia
(C) Amnesia
347 (D) Ataxia
(E) Dysarthria
In addition to a stimulant trial for attention deficit
hyperactivity disorder symptoms, the parents of an 8-
year-old b o y ask w h a t other treatment would be most
helpful for managing his refusal to cooperate at
350
home. W h i c h of the following is the best recommen-
dation? W h i c h of the following is most effective for the psy-
chotherapeutic treatment of obsessive-compulsive dis-
(A) Biofeedback order?
(B) Behavior therapy
(C) Cognitive behavior therapy (A) Biofeedback
(D) Family therapy (B) Exposure and response prevention
(E) Psychodynamic psychotherapy (C) Psychodynamic psychotherapy
(D) Relaxation and visualization
(E) Interpersonal therapy

351
Rebound insomnia is most severe after abrupt with-
drawal of w h i c h of the following medications?

(A) Alprazolam
(B) Clonazepam
(C) Diazepam
(D) Chlordiazepoxide
(E) Quazepam

Section 1: Self-Assessment Questions


352 357
All of the following are symptom clusters of posttrau- Trichotillomania is a difficult symptom to treat with
matic stress disorder (PTSD) E X C E P T : either psychotherapy or medication. Emerging evi-
dence indicates that medication plus which of the fol-
(A) REEXPERIENCING.
lowing types of psychotherapy is effective?
( B ) AVOIDANCE/NUMBING.

( ( ) HYPERAROUSAL. (A) EXPOSURE

( D ) DEREALIZATION/DEPERSONALIZATION. (B) FLOODING

(C) HOBIT REVERSAL

( D ) INTERPERSONAL PSYCHOTHERAPY

(E) PSYCHODYNAMIC PSYCHOTHERAPY

353
In clinical or forensic evaluations w h e n financial com-
pensation or special benefits may be available, a psy-
chiatrist must consider the diagnosis of:
The following vignette applies to questions 358

(A) FACTITIOUS DISORDER. and 359.

( B ) MALINGERING.
A 19-year-old w o m a n presents to a clinic for treatment
(C) SOMATIZATION.
of chapped hands. She reports that for several months
( D ) HYPOCHONDRIASIS.
she has had "this notion in my h e a d " that there are
germs e v e r y w h e r e . At first she w a s h e d her hands more
frequently, but as the thoughts have become more
prominent, she now usually wears gloves and washes
354 her hands with diluted bleach several times a day. She
A v o i d a n c e symptoms in posttraumatic stress disorder says that if she does not complete her cleansing rituals,
(PTSD) include which of the following? she cannot stand the anxiety.

(A) HYPERVIGILANCE

(B) INTRUSIVE IMAGES OF THE EVENT

(C) SENSE OF RELIVING THE EVENT OR EXPERIENCE

( D ) DIFFICULTY RECALLING IMPORTANT ASPECTS OF THE EVENT 358


The most common comorbid condition with this dis-
order is:

355 (A) ALCOHOL ABUSE.

(B) GENERALIZED ANXIETY DISORDER.


W e i g h t gain is L E A S T likely to be a side effect of
(C) MAJOR DEPRESSIVE DISORDER.
which of the following?
( D ) SOCIAL PHOBIA.

(A) LITHIUM (E) SCHIZOPHRENIA.

(B) LAMOTRIGINE

(C) DIVALPROEX

( D ) OLANZAPINE

359
T h e structural brain abnormality that has been demon-
strated most consistently in this disorder is:
356
(A) ASYMMETRICAL SEPTAL NUCLEI.
In Erikson's epigenetic model, each life stage.has an
( B ) DECREASED SIZE OF THE CAUDATE.
identity crisis that must be n a v i g a t e d . Intimacy vs. iso-
(C) ENLARGED LATERAL VENTRICLES.
lation is the developmental crisis associated with:
( D ) HYPERTROPHY OF THE AMYGDALA.

(A) SCHOOL AGE. (E) SHRINKAGE OF THE HIPPOCAMPUS.

(B) ADOLESCENCE.

(C) YOUNG ODULTHOOD.

( D ) ADULTHOOD.

(E) OLD OGE.

52 FOCUS Psychiatry Review: 400 Self-Assessment Questions


360 364
In w h i c h of the following disorders has reduced v o l Heightened arousal in posttraumatic stress disorder
ume been observed in the prefrontal cortex? (PTSD) i s ^ s s o c i a f e d with an increase in which of the
following?
(A) ADHD

( B ) DELUSIONAL DISORDER (A) HEART RATE

( ( ) OBSESSIVE-COMPULSIVE DISORDER (B) CONSTRICTION OF PUPILS

( D ) PANIC DISORDER (C) WEIGHT'

(E) SCHIZOPHRENIA , ( D ) TIDAL VOLUME

361 365 '


The first step in the evaluation of a patient with male According to DSM-IV-TR, a mixed episode must meet
erectile disorder is to: diagnostic criteria for a manic episode and w h i c h of
the following?
(A) TAKE A GENETIC HISTORY.

(B) RULE OUT MEDICAL PROBLEMS AND SUBSTANCE USE. (A) PANIC ATTACKS

(C) REFER THE PATIENT TO A SEX THERAPIST. ( B ) RAPID CYCLING

( D ) CHALLENGE WITH A TEST DOSE OF A PDE-5 INHIBITOR. (C) BRIEF PSYCHOTIC EPISODE

(E) ORDER A SLEEP STUDY. ( D ) MAJOR DEPRESSIVE EPISODE

362 366
A 75-year-old w o m a n with Parkinson's disease devel- A psychiatrist is called to see a 78-year-old female
ops vivid dreams a n d night terrors. T h e most likely patient postoperatively on the surgical service w h o is
explanation for these symptoms is: said to be "manic." She is hardly sleeping, she is a g i -
tated and talking rapidly, and she believes she needs
(A) THE ONSET OF DEMENTIA.
to talk with the President of the United States. W h i c h of
( B ) A RAPID PROGRESSION OF PARKINSON'S DISEASE.
the following interventions is most likely to be effective?
(C) A NORMAL EFFECT OF AGING.

( D ) AN ANXIETY DISORDER. (A) TRANSFER TO A PSYCHIATRIC UNIT

(E) SIDE EFFECTS FROM CARBIDOPA-LEVODOPA. (B) DIVALPROEX SODIUM

(C) HALOPERIDOL

( D ) ECT

(E) A BENZODIAZEPINE

363 i
In which of the following therapies, w h i c h has been
studied for the treatment of patients with borderline
367
personality disorder, is mindfulness training a central
component? W h i c h of the following variables is most important to
take into account w h e n evaluating the score on a
(A) COGNITIVE BEHAVIOR THERAPY
Mini-Mental State E x a m (MMSE)?
(B) DYNAMIC PSYCHOTHERAPY

(C) DIALECTICAL BEHAVIOR THERAPY (A) EDUCATIONAL LEVEL

( D ) SHORT-TERM GROUP PSYCHOTHERAPY (B) GENDER

(E) INTERPERSONAL PSYCHOTHERAPY (C) HISTORY OF ALCOHOL USE

( D ) MEDICAL HISTORY

(E) PAST PSYCHIATRIC HISTORY

Section L: Self-Assessment Questions 53


368 371
T h e parents of a 5-year-old b o y bring their child to a C o m m o n side effects of selective serotonin reuptake
clinic with the complaint that he frequently awakens inhibitors include:
during the early part of the night screaming; he looks
(A) orthostatic hypotension and dry mouth.
terrified, his pupils are dilated, a n d he hyperventi-
(B) confusion and disorientation.
lates. He is also sweating, agitated, and confused,
(C) priapism and arrhythmia.
a n d he cannot be comforted. W h e n fully a w a k e n e d ,
(D) seizures ond hallucinations.
the child has no recall of the event. This presentation
(E) nausea and sexual dysfunction.
is most consistent with:

(A) narcolepsy.
(B) nightmare disorder.
(C) primary insomnia. 372
(D) sleep disordered breathing.
W h i c h of the following classes of medications is sup-
(E) sleep terror disorder.
ported by well-designed studies as the first-line phar-
macologic treatment of posttraumatic stress disorder
(PTSD)?

369 (A) Mood stabilizers


(B) Benzodiazepines
A 6-year-old girl is brought to a clinic because of
(C) Tricyclic antidepressants
unusual stereotyped hand w a s h i n g . Pregnancy, labor,
(D) Selective serotonin reuptake inhibitors (SSRIs)
a n d delivery w e r e unremarkable, as w e r e develop-
mental milestones until the a g e of 8 months, w h e n the
child seemed to lose interest in her social environ-
ment. Thereafter,, significant delays, in development
w e r e noted. She did not w a l k until 2 years of a g e a n d
373
has had no spoken l a n g u a g e . H e a d growth has stag- A patient in early recovery from opiate dependence
nated. Recently she has d e v e l o p e d breath-holding has been maintained on 40 m g / d a y of oral meth-
spells. Examination reveals a small, noncommunica- adone for the last month. W h i l e the patient has not
tive child w h o demonstrates truncal ataxia and non- been experiencing a n y withdrawal symptoms at that
purposeful hand movements. E E G is abnormal. This dose, the w e e k l y random urine drug tests begin show-
presentation is most consistent with: ing a resumption of heroin use. Pharmacologically, the
best change to make in medication would be to:
(A) Asperger's syndrome.
(B) autism. (A) increase the maintenance dose of methadone.
(C) childhood schizophrenia. (B) decrease the maintenance dose of methadone.
(0) mild mental retardation. (C) change the opiate agonist to levo-alpha-acetylmethadol
(E) Rett's disorder. (LAAM).
(D) augment with buprenorphine.

370
W h i c h of the following actions on the' part of a'psy-
374
chiatrist constitutes abandonment? A 45-year-old man w h o travels frequently finds that on
returning from his most recent trip to a distant city, he
(A) Failing to show up for a scheduled appointment with a patient
has had difficulty maintaining daytime alertness a n d
(B) Referring, with appropriate notification to the patient, an
falls asleep easily a n d at inappropriate times. W h i c h
extremely difficult patient to a colleague with more experi-
of the following is the most likely diagnosis?
ence in the treatment of the patient's disorder •
(C) Terminating the treating relationship when a patient threat- (A) Orcadian rhythm sleep disorder
ens to sue the psychiatrist (B) Dissociative fugue » 1

(D) Prematurely discharging a patient from the hospital (C) Dyssomnia


(D) Parasomnia
(E) Narcolepsy

54 FOCUS Psychiatry Review: 400 Self-Assessment Questions


375
C o m p a r e d with other dementias, the early presenta- An 18-year-old w o m a n is starting her freshman y e a r in
tion in Creutzfeldt-Jakob disease more often includes: college. She is living at home with her parents. On
campus, she hopes to make friends but usually stays to
(A) choreoathetosis.
herself, fearing that she will be rejected by her peers.
(B) dysarthria. W h e n called on in class, she.avoids e y e contact with
(C) extrapyramidal symptoms. the professor. Although she almost a l w a y s knows the
(D) frontal release signs. answer to questions asked by the professor, she e x p e -
(E) myoclonus. riences inordinate anxiety that she will make a mis-
take. |n private moments, she refers to herself as "the
big nobody." This presentation is most consistent with:

376 (A) avoidant personality disorder.


(B) dependent personality disorder.
A 27-year-old man has a 4-month history of persecu-
(C) paranoid personality disorder.
tory delusions about being spied on at w o r k by
(0) schizoid personality disorder.
coworkers. A p a r t from the delusions, he functions rea-
(E) schizotypal personality disorder.
sonably well, a n d there is no evidence of medical ill-
ness or substance abuse. T h e most likely diagnosis is:

(A) brief psychotic disorder.


(B) delusional disorder. 380
(C) major depression with psychotic features.
The oncology team is concerned because a patient
(D) schizophrenia, paranoid type.
from another culture acts resigned w h e n faced with a
(E) schizophreniform disorder.
diagnosis of terminal cancer. T h e consulting psychia-
trist points out that in the patient's culture illness a n d
death are part of the normal cycle of life. W h i c h of
the following best describes the use of culture in this
377 psychiatric formulation?
N a u s e a a n d other gastrointestinal side effects with
(A) Interpretive and explanatory tool
SSRIs a p p e a r to be related to w h i c h receptor subtype?
(B) Pathogenic and pathoplastic agent
(A) 5-HT receptor
2
(C) Diagnostic and nosologic factor
(B) DA-2 receptor (D) Therapeutic and protective element
(C) DA-4 receptor (E) Management and service instrument
(D) H receptor
2

381
378 According to bSM-IV-TR, w h i c h of the following char-
W h i c h of the following abilities is N O T directly rele- acterizes acute stress disorder (ASDj?
vant to a person's capacity to make medical decisions?
(A) Lasts a maximum of 8 weeks
(A) Communicate or evidence a choice (B) Does not involve symptoms of hyperarousal
(B) Understand the facts of the situation (C) Often occurs as a result of a minor threat
(C) Appreciate how the facts of a situation apply to oneself (D) Requires dissociative symptoms for a diagnosis
(D) Choose an option that reflects what most reasonable per-
sons in that situation would do

382
Olfactory hallucinations are most commonly associ-
ated with:

(A) grand mal seizures.


(B) hypoparathyroidism.
(() parietal tumor.
(D) partial complex seizures.
(E) psychotic depression.

Seccion 1: Self-Assessment Questions 55


383 386
A n e w psychologist in town approaches an estab- A middle-aged man consults a psychiatrist at the rec-
lished psychiatrist and proposes that the psychiatrist ommendation of his primary care physician because
refer therapy patients to the psychologist in return for he has been unable to recover from his deep grief
a small percentage of fees collected by the psycholo- and feelings of abandonment since his divorce 18
gist from treating those patients. This practice is: months a g o . He endorses many symptoms of major
depression and has w i t h d r a w n from the social activi-
(A) not occeptoble because it does not put the patients' inter- ties that he used to enjoy, but he is not suicidal. Of the
ests first. following things that this patient reports, which w o u l d
(81 not acceptable because psychiatrists should refer patients to be the most positive indicator that he w o u l d be able
psychiatrist therapists. to benefit from psychodynamic psychotherapy?
(C) acceptable because it provides incentives for all parties to
benefit. (A) He is very angry at his ex-wife.
(D) acceptable because the psychologist is fairly compensating (B) He has no family history of psychiatric illness.
the psychiatrist. (C) He has been a successful writer.
(D) He gets significant support from his two best friends.
(E) He is very religious.

384
In order for an individual to recover from PTSD after
387
interpersonal violence, which of the following processes
is likely to be most helpful? 1 Lorazepam may be a better choice of a benzodiazepine
than diazepam for an elderly patient because the:
(A) Go to court and see the perpetrator brought to justice.
(B) Wait for symptoms 1o subside with time. (A) volume of distribution decreases with age.
(C) Emotionally engage with the memory of the trauma. (B). hepatic oxidation is unaffected by age.
(D) Restore sleep with a benzodiazepine. (C) hepatic conjugation is unaffected by age.
(E) Obtain treatment with eye movement desensitization tech- (D) glomerular filtration rate is unaffected by age.
niques. (E) hepatic blood flow is unaffected by age.

385 388
A hospital risk manager speaks with y o u about devel- A patient w h o is an artist is severely depressed a n d
oping an educational seminar on suicide prevention has occasional passive suicidal thoughts. T h e patient
contracts for emergency department staff. As part of tells the psychiatrist that health insurance benefits
the seminar, which of the following w o u l d be a most have been discontinued and that the patient is no
appropriate point to emphasize? longer able to p a y the psychiatric bills. T h e psychia-
trist has decided not to provide free care to this
(A) A patient's willingness to enter into a suicide prevention patient. The psychiatrist can avoid abandoning this
contract indicates reodiness for discharge from an emer- patient b y :
gency setting.
(B) In emergency settings, suicide prevention contracts are D. . . (A) giving the patient a written, 30-day notice of termination
helpful method for reducing suicide risk but should not be and terminating the patient at the end of the 30-day peRIod.
used to determine readiness for discharge. (B) reducing the frequency of the patient's appointments to
(C) Using suicide prevention contracts in emergency settings is help make the patient's bill more affordable.
not recommended. (C) arranging to commission an artwork by the patient in lieu
(D) Suicide prevention controcts can be useful for assessing the of the professional fees.
physician-patient relationship with individuals who. are (D) continuing to see the patient until acute depression-related
intoxicated, agitated, or psychotic. crises are resolved and then discharging the patient to the
local state-funded community agency clinic.

56 FOCUS Psychiatry Review. 400 Self-Assessment Questions


389 392
A primary substance abuse prevention program is T h e family of a 40-year-old retired police officer
being developed for adolescent girls in a large, met- reports tfiat in the past y e a r he has been.increasingly
ropolitan school district i n ' t h e United States. T h e isolative, w i t h d r a w n , a n d b i z a r r e . He has accused his
school district is diverse, with youths from African, family of trying to poison him. He put tarps over the
A s i a n , C a u c a s i a n , Middle 'Eastern, a n d N a t i v e w i n d o w s in his house. He is disheveled and carries a
A m e r i c a n families. Based on epidemiologic studies, set of torn capers at all times. He has been observed
which ethnic g r o u p of adolescent girls is at greatest mumbling a n d talking to himself. He has no history of
risk of substance use? , > substance abuse or prior depressive episodes. W h i c h
of the following is the most likely diagnosis?
(A) African American
(3) Asian American (A) Bipolar disorder
(C) Caucasian (3)'Delusional disorder
(D) Middle Eastern (C) Dementia of the Alzheimer's type
(E) Native American (D) Major depression with psychotic features or schizoaffective
disorder
(E) Schizophrenia

390
Posttraumatic stress disorder (PTSD) is considered to
393
be chronic PTSD after:
Clinical signs of major depression m a y emerge for a
(A) 1 month.
patient during bereavement after a parent's death.
(B) 3 months.
According to DSM-IV-TR criteria, w h a t is the earliest
(C) 6 months.
time interval after the parent's death that this diagno-
(D) 1 year.
sis is generally made?
(E) 3 years.
(A) 1 month
(B) 2 months
(C) 3 months
39J (D) 6 months

A 9-year-old b o y is seen in the emergency department


after attempting to jump out of a moving vehicle. His
parents report that he has had a difficult time in the
394
past year. Previously he had done well in school, but
now he is struggling academically. He often says he Involuntary hospitalization of a patient with schizo-
does not want to go to school, "because I am so stupid phrenia w h o is hearing voices is justified in which of
and ugly." His teacher has contacted his parents and the followina situations?
informed them that he is falling asleep in class, seems
(A) The patient hears a voice that he cannot resist telling him
fatigued, has little to do with his peers, and often does
to kill himself.
not eat his lunch. The child used to play with friends in
(B) Third-party payer deems hospitalization appropriate and
the neighborhood, but for the past 2 months has kept
will pay.
to himself, playing alone in his room or just sitting and
(C) The patient appears dirty and disheveled.
looking out the window. A few days earlier, he
(D) The patient lacks insight info the nature of his illness.
informed his mother of what to do with his most impor-
tant belongings should he die, but she did not make
anything out of it. He has generally seemed v e r y
grouchy and "on edge." On questioning, he acknowl-
edges that he w a s hoping to be killed w h e n he tried to 395
jump out of the car. The most likely diagnosis is: Kidney stones are most likely to be a side effect of
which of the following?
(A) borderline personality disorder.
(B) major depressive disorder. (A) Gabapentin
(C) oppositional defiant disorder. (B) Lithium
(0) separation anxiety disorder. (C) Lamotrigine
(E) somatization disorder. (D) Topiramate

Section 1: Self-Assessment Questions 57


396 399
The C E O of a large c o m p a n y is fearful of speaking at W h i c h of the following is the most common extrapyra-
a large stockholders' meeting. His fear of public midal side effect of antipsychotic medication?
speaking has been a lifelong disability, but he does
(A) Akathisia
not have anxiety in other social settings. W h i c h of the
(B) Torticollis
following is the most reasonable agent to prescribe?
(C) Oculogyric crisis
(A) A benzodiazepine (D) Neuroleptic malignant syndrome
(B) A beta-blocker (E) Tardive myoclonus
{() Buspirone
(D) A serotonin norepinephrine reuptake inhibitor (SNRI)
(E) An SSRI
400
The best legal protection for a psychiatrist w h o is
accused of malpractice after a patient's suicide is:
397
(A) the documentation of the patient's risk factors for suicide
A 49-year-old man with schizophrenia taking an
recorded in the chart.
antipsychotic asks to change medication because of
(B) the patient's documented history of an axis II disorder.
intolerable side effects. He has had extrapyramidal
(C) a doctor-patient suicide prevention ("no-harm") contract.
side effects and has experienced a 24-pound weight
g a i n . His body mass index is n o w 32.4^ His family
(D) the patient's family having promised to supervise the
history is significant for obesity, diabetes, hypercho-
patient closely.
lesterolemia, hypertension, and sudden cardiac
death. Of the following medications, w h i c h w o u l d be
the next best one in the management of this patient?

(A) Aripiprazole
(B) Olanzapine
(C) Quetiapine
(D) Risperidone
(E) Ziprosidone

398
A 34-year-old man w h o is comatose, has myoclonic
twitching, and has a serum lithium level of 4.2
m E q / L should respond best to w h i c h of the following
treatments?

(A) Activated charcoal


(B) Hemodialysis
(C) Intravenous sodium chloride
(D) Osmotic diuresis
(E) Plasmapheresis

58 F O H IS Psvchiarrv Review 400 Self-Assessment Questions


Section 2: Answers and Explanations
Sachs GS, Baidassano CF, Truman CJ, Guille C: Comorbidity of atten-
(& tion deficit hyperactivity disorder with early- and late-onset bipo-
A clinician is considering combination therapy for lar disorder. Am J Psychiatry 2000; 157:466-468
treatment-resistant depression. W h i c h of the following
Geller B.^un K, Zimerman B, Luby J, Frazier J, Williams M: Complex
and rapid-cycling in bipolar children and adolescents: a prelimi-
combinations has the most potential for serious
nary study. J Affect Disorder 1995; 34:259-268 .
adverse reactions?
Reddy YCJ, Srinath S: Juvenile bipolar disorder. Acta Psychiatr Scand
(A) Bupropion and fluoxetine 2000;102:162-170
(B) Buspirone and nortriptyline Spencer T J , Biederman J, Wozniak J, Faraone SV, Wilens TE, Mick E:
(C) Paroxetine and desipramine ,
Parsing pediatric bipolar disorder from its associated comorbidity
with the disruptive behavior disorders. Biol Psychiatry 2001;
(D) Phenelzine and lithium carbonate
.49:1062-1070
(E) Venlafaxine and tranylcypromine
Geller B, Lupy J: Child and adolescent bipolar disorder, a review of
T h e correct response is option E: Venlafaxine and
the past 10 years. J Am Acad Child Adolesc Psychiatry 1997;
36:1168-1176
tranylcypromine

Serious adverse reactions, sometimes fatal, with fea-


tures resembling serotonin syndrome and neuroleptic
malignant syndrome have been reported w h e n ven-
lafaxine has been used with a monoamine oxidase
®
A 39-year-old actuary for an insurance c o m p a n y is
inhibitor. While some adverse interactions could offered a substantial promotion that will require her to
occur with the other combinations listed, none consti- move to another city. Her n e w office will be on the
tu te_co_n trq indicati on s. 23rd floor of a high-rise building. She informs her psy-
chiatrist that she is "terrified" of riding in an elevator
Hodgman MJ, Martin TG, Krenzelok EP: Serotonin syndrome due to and terrified of heights, but desperately wants the
venlafaxine and maintenance tranylcypromine tnerapy. Hum Exp new job. W h i c h of the interventions listed b e l o w is
Toxicol 1997; 16:14-17
most likely to be successful for her?
Diamond S, Pepper BJ, Diamond ML, Freitag FG, Urban GJ,
Erdemoglu AK: Serotonin syndrome induced by transitioning from (A) Cognitive therapy
phenelzine to venlafaxine: four patient reports. Neurology 1998; (B) Hypnotherapy
51:274-276
(C) Insight-oriented psychotherapy
(D) Selective serotonin reuptake inhibitors
(E) Systematic desensitization

2 The correct response is option E: Systematic desensiti-


zation
W h i c h of the following conditions is most commonly
comorbid with prepubertal bipolar disorder? This w o m a n is suffering from a specific p h o b i a .

(A) Attention deficit hyperactivity disorder (ADHD) Although all of the therapies listed have been found to
(B) Autistic disorder be at least of some use in the treatment of phobias, the
(C) Separation anxiety disorder method that has been most studied and found most
(D) Tourette's disorder effective is behavior therapy. T h e behavior therapy
techniques that have been employed with phobias
T h e correct response is option A: Attention deficit
include systematic desensitization (serial exposure to a
hyperactivity disorder ( A D H D )
predetermined list of anxiety-provoking stimuli g r a d e d
Studies of grep^jbejigJ^bipo[gr_disorder consistently find in a hierarchy from the least to the most frightening),
that attention deficit hyperactivity disorder [ADHD] is a imaginal flooding (intensive exposure to the phobic
common c ^ D o j t i d ^ c p j T a j t b n . For example, Geller et stimulus through imagery), and flooding (in vivo e x p o -
al. (1995) reported that about 90% of prepubertal sure to the actual phobic stimulus).
(and 30% of .adolescent) bipolar patients also had
Stein DJ, Hollander E (eds): American Psychiatric Publishing Textbook
A D H D . Other studies had similar findings, namely,
of Anxiety Disorders. Washington, DC, American Psychiatric
A D H D in 90% of children with mania and jn 57% of Publishing, 2002, p 350
adolescents with mania. These high proportions have
not been accepted universally, and further study has
been recommended (Reddy and Srinath, 2000). A
study in adults found a much earlier onset of bipolar
disorder in those with a history of childhood A D H D
(12.1 years vs. 20 years) than in those without A D H D .

Section 2: Answers and Explanations


4 v£2 .
During resettlement, a refugee takes on the vau l es andA 27-year-old mae l patient with an initial epsi ode of
attitudes of the new culture and does not retain his schizophrenia is treated with risperidone at an initial
original cultural values. Whc i h of the folowing best dose of 2 mg daily, and after 1 week of treatment he
describes this process? no longer expere i nces agitation. By the third week of
treatment, with gradual titration of risperidone to
(A) Integration 6 mg daily, his delusions and halucinations are sig-
(B) Assimilation nificantly improved. At week 4, he descrb i es some
(C) Separation trouble sleeping at night because of reste l ssness but
(D) Marginolizotion reports that he s i much e l ss fearful and no longer
hears voices. When seen for a schedue l d appoint-
The correct response is option B: Assimilation
ment at week 6, however, he is noticeably drooling
Assimilation entails makmg_contactsjvjhjhe_new_cuI- and is in constant motion, rocking back and forth and
ture wjth.ojtjetainina^one's original cultural values. fidgeting in his chair. The side effect of treatment that
During resettlement, there are a number of ways to he is most likely experiencing is:
adapt to the new culture. Integration is_retaining ones '
(A) akathisia. -
own cultural identity while maintaining contact with (B) neuroleptic malignant syndrome.
members of the newer culture. Separation s i man i- (C) restless leg syndrome.
taining the original cultural identity and not seekn ig (D) serotonin syndrome.
contact with the newer culture. Marginalization is (E) tardive dyskinesia.
sheddn i g_ones ' originaljdentity and cultural vau
l es
The correct response is option A: Akathisia
but not seekingcpntact with other cultural groups.
Akathisia consists of a subjective .feeling _crf restless-
Fullilove MT: Psychiatric implications ot displacement contributions
from the psychology of place. Am J Psychiatry 1996; ness along with resjless movements, u^yajlyj^hejegs
153:1516-1523 or feet, which may be nTs iaj JcenJpO
jT j aaJn
Lustig SL, Kia-Keating M, Knight WG, Geltman P, Ellis H, Kinzie JD,
v_ihjgsja_ajea^ Akathisia is generaly seen
Keane T, Saxe GN: Review of child and adolescent refugee mental
soon after the initiation of treatment, • but it may
health. J Am Acad Child Adolesc Psychiatry 2004; 43:24-36
become more prominent o s _ the dose ofontipsycho'ic
medication is increased. In this vignette, the onset of
reste
l ssness soon after the start of antipsychoticjreat-
©_ ment makes akathisja more likely then reste l ss leg
syndrome. Neuroleptic malignant syndrom." and sero-
Parasomna i s can be differentiated from dyssomna is
tonin syndrome would be unlikely causes of this pres-
because parasomna i s involve abnormalities in which
entation, as they are associated with rigidity and
of the folowing aspects of se l ep?
motcjMwtc^
(A) Amount of sleep
(B) Initiationpfjleep Hales RE, Yudofsky SC (eds): American Psychiatric Publishing
(C) Physiological systems that occur during sleep Textbook of Clinical Psychiatry, Fourth Edition. Washington, DC,
(D) Quality of sleep American Psychiatric Publishing, 2003, p 1087
American Psychiatric Association: Practice Guideline for the
(E) Timing of sleep Treatment of Patients With Schizophrenia, 2nd ed. Am J
The correct response is option C: Physiological sys- Psychiatry 2004; 161 (Feb suppI):1-56
tems that occur during se
l ep

The factors listed in the other options are affected in


dyssomna i s.

American Psychiatric Association: Diagnostic and Statistical Manual


of Mental Disorders, Fourth Edition, Text Revision (DSM-IV-TR).
Washington, DC, American Psychiatric Association, 2000, pp 598,
630-631

FOCUS Psvchiatrv Review. 400 Self-Assessment Questions


<f)
A patient takes a medication for bipolar I disorder A forensic psychiatric evaluation differs from a general
throughout p r e g n a n c y a n d d e l i v e r y . . T h e n e w b o r n is psychiatric evaluation in that a forensic evaluation:
noted to be cyanotic and in' respiratory distress. An
(A) typically includes a mental status examination.
echocardiogram reveals significant displacement of
(B) does not have a doctor-patient relationship.
t w o leaflets of the tricuspid v a l / e into the ventricle and
(Q requires a completed written report.
o large atrial septal defect consistent with Ebstein's
(D) requires the presence of a lawyer during the evaluation.
anomaly. Of the following medications, w h i c h w a s
the w o m a n most likely taking during her pregnancy?
The correct response is option B: Does not have a
doctor-patient relationship
(A) Carbamazepine
(B) Gabapentin A forensic evaluation essentialy indya^_ajgeneral
(C) Lithium
psychiatric evaluatjoriwithm tjs_co_r_text. Forensc
i eval-
(0) Topiramate
uato i ns are done for third parties_and not for a
(E) Valproate
"patient," and hence there is no doctor-patient rela-
The correct response is option C: Lithium tionship (therapeutic aliance). There is no requr iement
iri forensic evaluations for-the presence, ofanjgttorney
Lithium is the on|y_pjycJToacjiye, noj>anti'convulsant
or for a report. Forensc i evaluations are not confiden-
dnJaJhaMs^ fh q u g ht_ to_be_a ssoc^a^d_wltli_the _s pec i f i c
tial in the sam e sense as a general evaluation in that
birth defect r ^ ^ n ^ n o r n a l y . This defect is 2J)jimes
the information is typicaly transmitted to the thi.rd_pa.rty.
more common in children born to mothers taking lithium
than in the general population. Echocardiography and S
mion R:l T hea lw a n
d psycha i try, n
i T h eA m e cr
ian Psych a
i tr
c
i
fetal ultrasonography can be used after the 1_6th_week Pub silh
nig T exb
took of Cn il c
i al Psych a
i try, 4th ed. Ed e
tid by H ae
ls
R E,Y udos
fk
yS C . Wash n
igo tn, D C ,A m ec
ria
n Psycha i trc
i Press,
qf_pregnancy to check for the presence, of_caraja C
2004, p 1 618
abnormalities. Gutheli TG :Type s of wtn
i esses, n i T h
e Psycha i trs
it n
i Court: A
Survv ial Gudie. Wash nigotn, D C ,A me cr
ian Psych a
i tr
c
i Press,
The general risk of major birth defect appears to be 1998. Repn rie
td ni FO C US 2003; 1:385-388 (p 3 8 6 )
two to three times greater with lithium than in the gen-
eral population. W h i l e initial information about the
teratogenic risk of lithium treatment w a s derived from
biased retrospective reports, more recent epidemio-
logic data indicate that the_ tera^ogenic^xisk^ofJirst-
@
A 30-year-old man reports that he is unable to sleep
frLinesfer_lithium e x p o s u r e is [ower_jhg^previou_sly
and hears noises and voices at night even though he
suggesjed. The clinical management of w o m e n with lives alone. T h e symptoms started abruptly on the d a y
bipolar disorder w h o have childbearing potential preceding the, visit. During the interview, he repeat-
should be modified with this revised risk estimate. edly brushes off his arms, muttering about bugs. T h e
VaJpnDaJejsj^ information that w o u l d be most helpful in determining
tube defects in t h g j e t u s , a n d caj^ani^z^pjne_with initial interventions w o u l d be the history of:
craniofacial defects, f i n g j r n d M T y p o r j j ^ neural (A) family disorders.
0
tube_defects, a n j d j J e j / e J c j D n ^ Qpi^pj?!^ (B) medical problems.
and topiramate have not been systematicqllysjudied (C) psychiatric hospitalization.
in pregnant w o m e n . (D) recent stresses.

Sc
h az
tbe r
gA F ,C oel JO, OeBa s
tita C: M anu a l of Cn
il c
i al The correct response is option B: Medical problems
P sy
c h oph ar
m a
coolgy. Wash nigo
tn, D C,A m e c
r
ia n Psycha i trc
i
Pub s
ilhn
i g, 2005, pp 260-261,272,281,292 Psychotic symptoms m a y be due to a general medical
Co
h en LS, F e
ridm a n JM, Jeferson J W ,J oh
n so nE M ,W en
ie
rML:A
condition, m a y be medication induced, or m a y be
revaul ato
i n of rs
i k of n
i utero exposure to lithium. J A M A 1994; induced by substances of abuse. Medical reasons for
2 71
1:4 6
psychotic symptoms should be m l e d j w t , especially in

Sa
coc
k BJ, S adockV A (eds): Kap aln and Sadock s
' C omp r
ehe
nsv
ie
Tex b
to ok of Psycha
i try, 8th ed. Pha
lidep
l ha
i , Lp
i pn
i cot Wa
ilims &
Wk lin
i s, 2005, p 9
8 9

Section 2: Answers and Explanations 63


Practice Guideline for the Treatment of Patients With Major
10 Depressive Disorder, 2nd ed (2000), in American Psychiatric
T h e practice of obtaining informed consent from an Association Practice Guidelines for the Treatment of Psychiatric
Disorders, Compendium 2004. Washington, DC, APA, 2004,
individual prior to initiating a n y treatment fulfills
pp 464,498-505
w h i c h of the following ethical principles?

(A) Nonmoleficence
(B) Autonomy
(C) Justice 12
(D) Competence
W h i c h of the following antidepressants would be the
T h e correct response is option B: Autonomy best choice for a patient concerned about erectile dys-
function?
A u t o n o m y refers tojhe notion in medical ethics_of_.indi-
v i d u a [ sejffulg,or selfqovernanee to_ ma ke decisions. (A) Bupropion
Nonmoleficence embodies the ethical principle of (B) Fluoxetine
(C) Nortriptyline
a v o i d ing harm. Justice refers to fairness in j h e _ d is tri-
r c n (0) Imipramine
but]on__ p r_ _ a ppii ca t i o n _ _ o L psyc h lQ-tri?....l ^. >iQl§ t-
(E) Venlafaxine
C o m p e t e n c e is generally considered a legal determi-
nation o fa. p_eis5n!sjab^ certain decisions, T h e correct response is option A: Bupropion
including but not limited to treatment-related decisions
Most antidepressants other than bupropion have sig-
(e.g., competenc.e.to_execute,.a will is termea_^testa-
nificant rates of erectile dysfunction as well as other
rn_tary.capgcity"). Cojppj^e_ncj^
aspects of sexual dysfunction. Mirjgzapine^ha.slower
capacity is a necessary requirementjpr informed_con-
rotes o M e x y a l dysfunction than, the SSRIs.
sent but is_not sufficient for informed consent, which
has additional requirements (i.e., disclosure of rele- Labbate LA, Croft HA, Oleshansky MA: Antidepressant-related erectile
vant informajion a n d v.ojujtfajiness). dysfunction: management via avoidance, switching antidepres-
sants, antidotes, and adaptation. J Clin Psychiatry 2003; 64(suppi
Simon Rl: A Concise Guide to Psychiatry and Law for Clinicians, 3rd 10):11-19
ed. Washington, DC, American Psychiatric Publishing, "2001, Hales RE, Yudofsky SC (eds): The American Psychiatric Publishing
pp 63-65 Textbook of Clinical Psychiatry, 4th ed. Washington, DC, American
Beauchamp TL, Childress JF: Principles of Biomedical Ethics, 5th ed. Psychiatric Publishing, 2003, p 1058
New York, Oxford University Press, 2001, pp 77,114,189,226
Kaplan HI, Sadock BJ: Synopsis of Psychiatry: Behavioral
Sciences/Clinical Psychiatry, 9th ed. Baltimore, Lippincott Williams
& Wilkins, 2003, pp 1365-1258
_ 2

The National Comorbidity S u r v e y identified a number


of gender differences in exposure a n d in the develop-
ment of posttraumatic stress disorder (PTSD). C o m -
p a r e d with females, males h a v e :
W h i c h of the following p^_hothergpies has the best
documented effectiveness in the treatment of major (A) higher trauma exposure, and higher prevalence of PTSD.
depressive disorder? (B) lower trauma exposure, and lower prevalence of PTSD.
(C) higher trauma exposure, and lower prevalence of PTSD.
(A) Supportive
(D) lower trauma exposure, and higher prevalence of PTSD.
(B) Psychodynamic
(E) the some trauma exposure, and the same prevalence of PTSD.
(C) Interpersonal
(D) Psychoeducational The correct response is option C: Higher trauma e x p o -
(E) Family , sure, a n d lower prevalence of P T S D

T h e correct response is option C: lnjej£ej_onbl T h e National Comorbidity S u r v e y found that males


are more likely than females ..to_be e x p o s e d to trau-
I ntergerson a [_ond cognitive b e h a v i o r t h e r o p y are the
matic events (60% vs. 50%), while females a r e j n o r e
best documented psychotherapeutic treatments_for
likely than males to develop PTSD ( 1 2 % v s . 6%). This
m a j o r d e p r e s s i v e disorder. P s y c h o d y n a m i c psy-
finding may represent a gender difference in suscep-
chotherapy is usually used with patients w h o also
tibJlity_ib PTSD linked to biological, psychological, or
h a v e to w o r k on some other life goals. Supportive and
social factors, or it may b e , a direct function of the dif-
psychoeducational techniques c a n also be useful,
ferential in types of traumatic events to w h i c h men and
depending on the severity of the depression. *
w o m e n are e x p o s e d .

64
Yehuda R (ed): Treating Trauma Survivors With PTSD. Washington, alcohol per unit of body weight. This includes a woman's
DC: American Psychiatric Publishing, 2002, p 26
lower body water contenhejgtiye tojnenjgjcghpl is dis-
American Psychiatric Association: Practice Guideline for the
tributed in the total body .wafer, and w o m e n have Jess
Treatment of Patients With Acute Stress Disorder and
Posttraumatic Stress Disorder. Am J Psychiatry 2004; 161 (Nov water in^their body to dilute the alcohol); an.increased
suppl):20 ratio of jat-to-water content as w o m e n . a g e ; lower quan-
Kessler RC, Sonnega A, Bromet E, Hughes M, Nelson CB: Hh'es of alcohol dehydrogenasejn the gastric mucosa of
Posttraumatic stress disorder in the National Comorbidity Survey.
w o m e n compared with men; a tendency j o r women's
Arch Gen Psychiatry 1995; 52:1045-1060
bodies jo absorb more of the alcohol they drink than do
men's bodies; and variation in blood alcohol concentra-
tion, related to menstrual cycle.

Romans SE, Seeman MV: Women's Mental Health: A Life-Cycle


A cancer patient with significant nausea requires an Approach. Philadelphia, Lippincott Williams & Wilkins, 2006,
pp 182-183
antidepressant. W h i c h of the following medications
Cyr MG, McGarry KA: Alcohol use disorders in women: screening
w o u l d be the best choice?
methods and approaches to treatment. Postgrad Med 2002;
(A) Bupropion 112(6):31-32,39-40,43-47

(B) Duloxetine
(C) Mirtazopine
(D) Paroxetine
(E) Venlafaxine
W h i c h of the following situations best describes w h e n
The correct response is option C: M i r t a z a p i n e
weight considerations should determine hospitalization
W i t h j h e exception of mirtazapine, all of the drugs listed for anorexia nervosa in children and young adolescents?
have been shown in clinical trials to cause considerably
(A) Weight is less than 20% of recommended healthy body
more nausea than placebo. The reason that nausea is weight.
not a prominent side effect of mirtazapine is thought to (B) Weight is less than 25% of ideal body weight.
be its ^ j ^ ^ ^ ^ ^ S _ § Q _ ^ ? a n effect shared with (C) Weight is being rapidly lost and outpatient efforts are inef-
antinausea drugs such as ondansetron a n d granisetron. fective, regardless of actual weight.
(D) The family asks for hospitalization.
Nutt D: Mirtazapine: pharmacology in relation to adverse events. Acta
(E) Weight is fluctuating unpredictably over 2-3 months.
Psychiatr Scand 1997; 96(suppl 391):31-37
McManis PG.Talley NJ: Nausea and vomiting associated with selec- The correct response is option C: V ^ g J ] M s _ b j ] n g _ r a p -
tive serotonin reuptake inhibitors: incidence, mechanisms, and
Idly lost a n d outpatient efforts are ineffective, r e g a r d -
management CNS Drugs 1997; 8:394-401
l e s s ^ actual weight
Montgomery SA: Safety of mirtazapine: a review. Int Clin
Psychopharmacol 1995; 10(suppl 4):37-45
For patients w h o s e initial weight falls 25% b e l o w
e x j D e d e ^ w e i g h t ^ j p j y i z a t i p n ]s often necessary to
ensure adequate intake and to limit physical activity.
I n j / o u n q e r children_aj}d^adpiesc_ents h ospita I i za f.ion
152. s j T p j ^ ^ _ _ c o n s i d e r e d . . e v e n .earlier w h e n e v e r the
W h e n non-substance abusing men and w o m e n drink patient isJosing weight rapidly and before too much
the same amount of alcohol, the w o m e n are likely to weight is lost, since early intervention may avert rapid
have higher alcohol blood levels than the men. T h e best
physiological decline and loss of cortical white a n d
explanation for this is that compared with men, w o m e n :
g r a y matter. Generally, specialized..eating disorder
(A) have a larger volume of distribution. un.its_yield better outcomes than general psychiatric
(B) have lower excretion rates. un.its.becau.se of nursing expertise a n d effectively c o n -
(C) only metabolize by first-order kinetics. ducted protocols.
(D) metabolize less alcohol in the gut.
Yager J, Devlin MJ, Halmi KA, Herzog DB, Mitchell JE, Powers PS,
(E) are deficient in acefaldehyde dehydrogenase.
Zerbe KJ: Eating disorders. Focus 2005; 3:502-510
The correct response is option D: IHa^lizeil^ Practice Guideline for the Treatment of Patients With Eating
Disorders, 2nd ed (2000), in American Psychiatric Association
9S§fioF_n35£§23 Practice Guidelines for the Treatment of Psychiatric Disorders,
Alcohol metabolism, regardless of gender, is based on Compendium 2004. Washington, DC, APA, 2004
zero-order kinetics. However, a number of factors con-
tribute to higher blood alcohol concentrations in w o m e n
than in men after consumption of the same amount of

Section 2: Answers and Explanations 6^>


m 122
A 33-year-old m a n started twice-weekly psychody-
W h i c h of the following antipsychotic drugs is most
likely to be associated with hypejproloctinemia? namic p s y c h o t h e r a p y 6 months a g o with the goal of
exploring issues stemming from his distant relationship
(A) Aripiprazole with his father a n d his inability to form adequate men-
(B) Clozapine toring relationships in his w o r k as a research chemist.
(C) Olanzapine He reports an increasing preoccupation with his ther-
(D) Quetiapine apist's unwillingness to see him more frequently. T h e
(E) Risperidone patient has been speaking in therapy of his wish that
the therapist see him on Sunday. He believes that the
T h e correct response is option E: Risperidone
therapist refuses to h a v e extra sessions because he
Risperidone causes prolactin elevationsi t h a t a r e j i m i - prefers other patients. W h i c h of the following best
I q r t o those caused by h i g h - p o t e n c y d o p a m i n e antag- explains the patient's behavior?
onist antipsychotic medications. T h e other atypical
(A) Transference neurosis
antipsychotics cause minimal or no increase in pro-
(B) Delusional system
lactin levels.
(C) Obsessional diathesis
Maguire GA Prolactin elevation with antipsychotic medications: (D) Erotomania
mechanisms of action and clinical consequences. J Clin (E) Psychotic distortion
Psychiatry 2002; 63(suppl 4):55-62
Compton MT, Miller AH: Antipsychotic-induced hyperprolactinemia The correct response is option A: Transference neuro-
and sexual dysfunction. Psychopharmacol Bull 2^02; 36:143-164 sis
American Psychiatric Association: Practice Guideline for the
Treatment of Patients With Schizophrenia, 2nd ed. Am J O v e r 6 months of intensive therapy, the patient has
Psychiatry 2004; 161 (Feb suppl):1-56 developed a p e r v a s i v e transference that reenacts
aspects of his childhood relationship with his father.
This is an e x a m p l e of a c | a j ^ J r a j ] s j e j ^ ^
at the core of traditional long-term psychodynamic
m p j y c h o t h e r a pjes.

W h i c h of the following atypical ar '-psychotic drugs is


Olds DD: Psychotherapy, in Psychiatry. Edited by Cutler JL, Marcus
a D receptor partial agonist?
2
ER. Philadelphia, WB Saunders, 1999, pp 281-307

(A) Aripiprazole
. (B) Olanzapine
(C) Quetiapine
(D) Risperidone 20
(E) Ziprasidone W h i c h of the following disorders has the highest rela-
tive risk for first-degree relatives?
T h e correct response is option A: A r i p i p r a z o l e
(A) Alcoholism
A r i p i p r a z o l e is a partial agonist. A partial agonist is
(B) Anorexia
an a^pjT]sjjhat_cannol; maximallyjsctivate a receptor
(C) Bipolar disorder
regajdless_of the concentration of drug present. W h i l e
(D) Panic disorder
this feature of aripiprazole suggests a mechanism of (E) Somatization disorder
action that differs from other atypical antipsychotics,
there is no evidence to date that aripiprazole is a n y T h e correct response is option C: Bipolar disorder

more or less effective then the other drugs.


The relative risk for bjpolar d i s o r d e n s j ^ r o u n d 25. For
Shapiro DA, Renock S, Arrington E, Chiodo LA, Liu LX, Sibley DR, s c h i z o p h r e n i a , it is 1 8 ; P J ^ i c j i s o r d e r , 10; a n o r e x i a ,
Roth BL, Mailman R: Aripiprazole, a novel atypical antipsychotic 5; alcoholism, a r o u n d 7; a n d somatization, around 3.
drug with a unique and robust pharmacology. gjrelanyelrjs^ is defined as
Neuropsychopharmacology 2003; 28:1400-1411
t h e j j r o b a b i l i t y that a first^egree_£elative_of aj>atient
with qiLillnessjwJJLoLso d e v e l o p j h a t illness.

Knowles JA: Genetics, in The American Psychiatric Publishing ,


Textbook of Clinical Psychiatry, 4th ed. Edited by Hales RE,
Yudofsky SC. Washington, DC, American Psychiatric Publishing,
2003, pp 5-7

66 FOCUS Psychhrry Review: 400 $clf-A.w<<;ment Questions


Social rhythm therapy g r e w from a chronobiological
model of bipolar disorder. If modulates both biologi-
A 68-yearold man has a g r a n d mal seizure that is
cal and psychosocial factors to mitigate a patient's cir-
attributed to an abrupt hyponatremia, with a serum
cadidn and sleep-wake cycle vulnerabilities. O b j e c t
sodium concentration of 110 mmol/L. W h i c h of the
relations theory is psychodynamic theory b a s e d _ o n .
following medications is the most likely cause?
Melanie Klein's metapsychology. Psychoeducation
(A) Gabapentin entails offering the patient education about the
(B) lithium patient's disorder. Cognitive thergpy is a treatment
(C) Oxcarbazepine designed to help people learn to identify a n d monitor
(D) Topiramate \ .
negative w a y s of thinking a n d then alter this tendency
(E) Valproate
and think in a more positive manner.
The correct response is option C: O x c a r b a z e p i n e
Frank E, Swartz HA, Kupfer DJ: Interpersonal and social rhythm ther-
According to the o x c a r b a z e p i n e package insert, clin- apy: managing the chaos of bipolar disorder. Biol Psychiatry
2000; 48:593-604
ically significant hyponatremia (serum sodium,
Rapaport M, Hales D: Relapse prevention and bipolar disorder: a
< 125 mmol/L) developed in 2.5% of patients in con-
focus on bipolar depression. FOCUS 2003; 1:15-31 (p 21)
trolled studies of epilepsy. Several possible mecha-
nisms h a v e been proposed, but none are well
substantiated. Risk factors include o j d e j ^ a g e , high
doses or blood levels, low pretreatment sodium levels,
other drugs that cquse__hypona(remia a n d possibly
(__L
<
Rapid cycling in bipolar I or II disorder is associated
cigarette smoking.
with:

Valproate can cause elevated liver function test results (A) menopause.
a n d increased ammonia levels. There have been (B) antidepressant use.
some reports of hyponatremia with valproate, but this (C) cocaine abuse.
occurs rarely. Topiramate can cause a hyperchlor- (D) early onset.
emic, non-ion-gap metabolic acidosis (elevated chlo- (E) alcohol abuse.
ride level and reduced bicarbonate level). Lithium
The correct response is option B: Antidepressant use
may_ lead to_ djabetes insipidus^_ which _in turn can
c a u s e J i y p e r n q t r e m i a . G a b a p e n t i n is not associated Rapid cycling is associated with antidepressant use.
with a n y alterations in serum electrolytes.. Rapid cycling in bipolar disorder is defined as four or
more mood episodes in the previous 1.2 months. Rapid
Steinhoff BJ, Stall KD, Stodieck SR. Paulus W: Hyponatremia coma
cycling is not related to a n y phase of the menstrual
under oxcarbazepine therapy. Epilepsy Res 1992; 11:67-70
Van Amelsvoort T, Bakshi R, Devaux CB, Schwabe S: Hyponatremia cycle. It occurs in both pre- and postmenopausal
associated with carbamazepine and oxcarbazepine therapy: a women. On the other hand, w o m e n constitute 7 0 % to
review. Epilepsia 1994; 35:181-188 90% ofjhe patients affected .by.copid_cycIi.ng. The syn-
Sachdeo RC, Wasserstein A, Mesenbrink PJ, D'Souza J: Effects of drome can appear or disappear at a n y time during the
oxcarbazepine on sodium concentration and water handling. Ann
course of bipolar I or II disorder. By definition, ful>j
Neurol 2002;51:613-620
jjajjcejiE^
Trileptal (oxcarbazepine) prescribing information (package insert), 2005
American Psychiatric Association: Diagnostic and Statistical Manual
of Mental Disorders, 4th Edition, Text Revision (DSM-IV-TR).
Washington, DC, American Psychiatric Association, 2000,
@) pp 427-428
American Psychiatric Association: Practice Guideline for the
Social, rhythm therapy, w h i c h is designed specifically Treatment of Patients With Bipolar Disorder (Revision). Am J
for bipolar disorder, is based on which of the follow- Psychiatry 2002; 159(April suppl). Reprinted in FOCUS 2G03;
ing models? 1:64-110 (p 94)

(A) Psychoeducation
(B) Object relations and self psychology theory
(C) Orcadian regulation and interpersonal psychotherapy
(D) Cognitive therapy techniques to address social dysfunction
(E) Supportive psychotherapy

T h e correct response is option C: G r ^ I g F r e g u I a t p i


alidjnterpersbQdj " ^ y c h o t h e r a p y

S-ction 2: Answers and Explanations


26;
A 65-year-old w o m a n has a history of a left frontal A psychiatrist proposes to use an FDA-approved drug
lobe stroke. W h i c h of the following psychiatric symp- not previously used for the treatment of mania
toms is most commonly associated with a stroke in this because it has biochemical properties similar to
a r e a of the brain? known antimanic agents. T h e psychiatrist has also
read several articles describing open-label studies
(A) Panic suggesting efficacy of the drug. The patient in ques-
(B) Mania tion hcs not responded to any agent thus far. The psy-
{() Depression chiatrist must do which of the following?
(D) Obsessions
(E) Anxiety (A) Get an institutional review board approval, since what is
proposed is clinical research.
T h e correct response is option C: Depression
(B) Notify the FDA, since the drug is being used for o non-
Poststroke depression has been documented after FDA-approved purpose.
cerebrovascular accidents occurring in many areas in
(C) Obtain informed consent from the patient or from an
appropriate proxy agent.
the brain. H o w e v e r , anterior left hemisphere lesions,
(D) Wait until there is higher-quality data Jo support this use of
particularly large Jesions a n d in the.early_reco,yejy
the drug.
period, a p p e a r to c a r r y a j i i g h e x x j . s k j s f p o s t s t r o k e
depression. O t h e r factors, such as history of depres- The correct response is option C: Obtain informed con-
sion and d e g r e e of disability after stroke, a r e . a l s o sent from the patient or from an appropriate proxy agent
associated with poststroke depression. \
W h i l e obtaining explicit informed consent for any treat-
Sadock BJ, Sadock VA (eds): Kaplan and Sadock's Comprehensive ment is important, it is especially important w h e n the
Textbook of Psychiatry, 8th ed. Philadelphia, Lippincott Williams & treafmei]Lhas_noLbecome standard. W h e n all avail-
Wilkins, 2005, pp 349-357 able treatments have failed, a nonstandard approach
Whyte EM, Mulsant BH: Post stroke depression: epidemiology, patho-
basedjon. limited-evidence rnay_be all that j.s.ayajia.ble (
physiology, and biological treatment Biol Psychiatry 2002;
52:253-264 provided that proper precautions,_are taken. Unless the
psychiatrist's intent is to demonstrate generalizable
knowledge; a nonstandard treatment is usually not con-
sidered research. Physicians routinely and permissibly
use drugs for non-FDA-approved indications using their
best clinical judgment. W h i l e it is probably wise to wait
Genetic studies of obsessive-compulsive disorder have
revealed linkages to which of the following disorders? un*i] higher quality data" support the use of a nonstan-
dard treatment, it is not a "must."
(A) Alcohol dependence
(B) Schizophrenia Simon Rl: A Concise Guide to Psychiatry and Law for Clinicians, 3rd ed.
(C) Shared psychotic disorder Washington, DC, American Psychiatric Publishing, 2001, pp 91-98
(D) Somatoform disorder
(E) Tourette's syndrome
T h e correct response is option E: Tourette's syndrome

Genetic studies of patients with obsessive-compulsive


disorder h a v e revealed h1glfemil0atesT6^aepril?
f
SioH and ^ ] e t ^ i j p r d e r ^ a n d Jdurerteysyndronief.

Hales RE, Yudofsky SC (eds): The American Psychiatric Publishing


Textbook of Clinical Psychiatry, 4th ed. Washington, DC, American
Psychiatric Publishing, 2003, p 32

68 FOCUS Psychiatry Review: 400 Self-Assessment Questions


Garlow SJ, Purselle D, Heninger M: Ethnic differences in patterns of
27,
suicide across the life cycle. Am J Psychiatry 2005; 162:319-323
A 75-year-old retired physicist w h o is suffering from Hales RE, Yudofsky SC (eds): The American Psychiatric Publishing
metastatic cancer is referred to a psychiatrist by the Texttjook of Clinical Psychiatry, 4th ed. Washington, DC, American
primary care physician because the patient wants to Psychiatric Publishing, 2003, p 1458
die and has requested assistance in suicide. On eval-
uation, the psychiatrist finds that the patient's cogni-
tion is intact. T h e most appropriate next step for the
psychiatrist is to: ®_1
(A) be as persuasive as possible so that the patient accepts the A 15-year-old b o y is referred for psychiatric evalua-
cancer treatment. tion after taking an overdose of an over-the-counter
(B) find out whether there are areas of suffering that can be cold medication. The patient denies that this w a s a
suicide attempt. T h e patient acknowledges that he has
addressed by available palliative care measures.
been having difficulties for about a year, since the
(C) tell the referring physician that the patient can be given assis-
separation of his parents. He often feels a n g r y a n d
tance in suidde because the patient is a competent adult.
irritable, has difficulty sleeping, has little appetite, has
(D) tell the referring physidan that even though the patient's
lost weight, has little interest in his usual activities, a n d
cognition appears intact, the patient is probably incapacitated
often wishes he w a s d e a d . His grades have dropped
by virtue of the unreasonable choice that is being made.
to the point that he is failing his courses. O v e r the past
T h e correct response is option B: Find out whether year, he has been smoking 1-2 packs of cigarettes a
there gre_areqs of suffering _mat_cgn be addressed by day, drinking to the point of intoxication on the week-
available palliative ^ e j n e c s u r e s ends, and taking over-the-counter cold medication to
enhance the effects of the alcohol. His past psychiatric
W h i l e sometimes direct persuasion to accept a med- history is significant for attention deficit hyperactivity
ically indicated treatment m a y be acceptable, the disorder ( A D H D ) , for which he has a prescription for
main issue is to identify the sources of the patient's suf- a stimulant medication. He has not taken his medica-
fering. Most patients w h o request assisted suicide tion as prescribed. Instead, he hoards the medication
eventually c h a n g e their minds w h e n the sources of and then takes large quantifies to experience a
their suffering are better addressed. To recommend euphoric effect. W h i c h of the following medications
assistance in suicide at this stage of the patient's can- would be the most efficacious in addressing this
cer ignores the complexity of the situation. To use patient's symptom constellation?

incapacity as a reason for paternalism (option D) is


(A) Bupropion
also unacceptable. (B) Gtalopram
Block SD, Billings JA: Evaluating patient requests for euthanasia and
(C) Desipramine
assisted suicide in terminal illness: the role of the psychiatrist, in (D) Trazodone
End of Life Decisions: A Psychosocial Perspective. Edited by (E) Venlafaxine
Steinberg MO, Youngner SJ. Washington, DC, American
Psychiatric Press, 1998, pp 205-233 The correct response is option A: Bupropion
Bascom P, Tolle SW: Responding to requests for physician-assisted
Bupropion is effectiyejn,ji)ejregtmenj o j _ m g n y J y p e s
suicide. JAMA 2002; 288:91-98
oi_depressio.n. It is niejaj?pjjzed Jo^_g_number__pf
amphetamine-like products, which are effective in the
treatment of_A_DHD. Bupropion has also been demon-
strafed to decrease nicotine use. In adolescents w h o
have depression, nicotine dependence, substance
Of the following ethnic groups, w h i c h is at lowest risk
abuse, and A D H D , bupropion might be the first-line
of completed suicide?
treatment, as it has been shown to be effective in
(A) African Americans assisting with b o t h j m o j ^ j i e i ^
(B) Asian Americans the_£oj^yrnpJojn^^ Citalopram and other
(C) Caucasian Americans SSRIs as well as trazodone have not demonstrated
(D) Hispanic Americans these a d d e d benefits. Venlafaxine and desipramine
(E) Native Americans have been found to be effective in both childhood a n d
The correct response is option B: A s i a n Americans adult A D H D , but they lack the pote_ntiglp.f_bupropjon
f q _ a s j | s h ^ j j T i j D k j n g _ cessotion._
With regard to ethnicity, most studies have demon-
strated that Caucasjajis^re_ajjTJ Schatzberg AF, Cole JO, DeBattista C: Manual of Clinical
Psychopharmacology. Washington, DC, American Psychiatric
lowed in order by Native Americans, African
Publishing, 2005, pp 85-89
Americans, Hispanic Americans, and Asian Americans.

Section 2: Answers and Explanations


A psychiatrist attends a dinner lecture sponsored by a More severe and prolonged forms of conduct disorder
major pharmaceutical company, the maker of a newly are most often associated with w h i c h of the following
a p p r o v e d drug for major depression. The company's comorbid disorders?
representative approaches the psychiatrist after the
(A) Anxiety disorders
lecture a n d says, "I hope we can count on you to pre-
(B) Attention deficit hyperactivity disorder
scribe our medication. This is a great medication!"
T h e psychiatrist does not k n o w w h a t to say and later
(C) Depression
feels troubled by this encounter. W h i c h of the follow-
(D) Eoting disorder
ing statements reflects the psychiatrist's ethical obliga-
(E) Tic disorder
tion in this situation? The correct response is option B: Attention deficit
hyperactivity disorder
(A) The psychiatrist can accept dinners and "repay" the com-
pany with favorable prescribing practices if the psychiatrist More_severe and prolonged conduct disorder js.asso-
chooses to do so. ciated[with early onset, comorbid A D H D , a n d con-
(B) The psychiatrist should report the pharmaceutical represen- duct symptoms that are more frequent, numerous, a n d
tative's behavior to the local APA branch's ethics committee. varied.
(C) The psychiatrist should be aware that "strings attached"
industry-sponsored activities are unethical. Dulcan MK, Martini DR, Lake MB: Concise Guide to Child and
(D) The psychiatrist must repay the representative for the cost Adolescent Psychiatry, 3rd ed. Washington, DC, American
of the dinner, since there are apparent, though unstated, Psychiatric Publishing, 2003, p 49
ethical conflicts. \
T h e correct response is option C: The., psychiatrist
should be a w q r e j h a t "strings attached" industry-sppn-
sored activities a re u nethjca I
A 62-year-old man Js taking desipramine for depres-
T h e A m e r i c a n Medical Association C o d e of Mediccl sion. He presents with marked sedation, tachycardia,
Ethics states that " G i f t s _ j j } p u J d j o t _ b e _ a c ^ p J e j ^ f and postural hypotension about 10 days after the
strings a r e attached." In this question, the psychia- addition of a second antidepressant. W h i c h of the fol-
trist's attendance at a dinner w h o s e purpose was edu- lowing medications is most likely responsible^-
catjonajjyi^ if there are implicit or
(A) Venlafaxine
explicit strings attached. Most psychiatrists would not (B) Mirtazapine Vy*
consider the psychiatrist's behavior in this situation (C) Citalopram ^V* 0

unethical, although it w o u l d be unethical to promise to (D) Sertraline


prescribe a certain medication in return for drug com- (E) Fluoxetine
p a n y favors, such as free dinners or concert tickets.
The correct response is option E: Fluoxetine
Clearly, the blurring of the boundaries between edu-
cational activities and pharmaceutical company pro- Desipramine is a substrate for cytochrome P450 2D6.
motions continues to be a d j f f i c j j ] H s j y j J g ^ g j h e Of the drugs listed, only fluoxetine is a potent inhibitor
psychigtric community, and eihjcaj„pjjcjiiajrists may of this enzyme. In this patient, taking fluoxetine resulted
disagree about whether participation in a n y educa- in a marked increase in his blood levels of desipramine,
tional activities provided by pharmaceutical compa- which accounted for his new physical symptoms.
nies is ethical.
Greenblatt DJ, von Moltke LL, Harmatz JS, Shader Rl: Drug interac-
American Psychiatric Association: Ethics Primer of the American tions with newer antidepressants: role of human cytochromes
Psychiatric Association. Washington, DC, American Psychiatric P450. J Clin Psychiatry 1998; 59(suppl 15):19-27
Association, 2001, pp 48-49 • Preskorn SH, Alderman J, Chung M, Harrison W, Messig M, Harris S:
Chren MM, Landefeld CS, Murray TH: Doctors, drug companies, and Pharmacokinetics of desipramine coadministered with sertraline
gifts. JAMA 1989; 262:3448-3451 or fluoxetine. J Clin Psychopharmacol 1994; 14:90-98
American Medical Association Council on Ethical and Judicial Affairs: Schatzberg AF, Nemeroff CB (eds^Jhe American Psychiatric
Code of Medical Ethics, Current Opinions With Annotations, Publishing Textbook of Psybhopharmacology, 3rd ed, Washington,
2000-2001. Chicago, American Medical Association 2000 DC, American Psychiatric Publishing, 2004, pp 223-224

70 FOCUS Psychiatry Review: 400 Self-Assessment Questions


W h i c h of the following diseases associated with W h i c h of the following aspects of cognitive perform-
dementia characteristically has early changes in per- ance is most likely to decline in the course of normal
sonality a n d a late decline in memory? aging?

(A) HIV infection (A) Short-term memory


(B) Creurzfeldt-Jakob disease (B) Speed of performance
(C) Parkinson's disease (C) Store o'f knowledge
(D) Lewy body dementia , (D) Syntax
IE) Pick's disease '« ' (E) Vocabulary
The correct response is option E: Pick's disease The correct'response is option B: Speed of performance

Pick's disease specifically affects the frontal and tem- Speed of learning, p r o ^ ^ s m ^ j p e e d , ond speed of
p o r a l j o b e s , accounting for the early signs of person- performance of cognitive tasks tend to decline with
ality changes, loss of social skills, and emotional normal a g i n g . T h e other functions listed do not
blunting. Other features of dementia, such as memory decline with normol g g i n g , g n d g decline in a n y of
loss and a p r a x i a , come later. Specific diagnosis of them may be an indication for a thorough or formal
Pick's disease is usually made only on autopsy. T h e assessment for cognitive impairment.
medical illnesses Huntington's disease, Parkinson's
Spar JE, La Rue A: Concise Guide to Geriatric Psychiatry, 3rd ed.
disease, a n d H I V j n f e c t i o n precede those dementias.
Washington, DC, American Psychiatric Publishing, 2002, pp 25-26
Creutzfeldt-Jakob disease often has a clinical triad
associated with dementia, involuntary movement, a n d
periodic E E G activity. Lewy body dementia often pre-
sents first with hallucinations_and psychosis.

American Psychiatric Association: Diagnostic and Statistical Manual Disorders with significant psychiatric s ^ o t o m s that
of Mental Disorders, Fourth Edition, Text Revision (DSM-IV-TR). can be linked to a single g e n e include:
Washington, DC, American Psychiatric Association, 2000,
pp 148-151 (A) attention deficit hyperactivity disorder.
Practice Guideline for the Treatment of Patients With Alzheimer's (B) bipolar disorder.
Disease and Other Dementias of Late Life (1997), in American (C) fragile X syndrome.
Psychiatric Association Practice Guidelines for the Treatment of (D) major depression.
Psychiatric Disorders, Compendium 2004. Washington, DC, APA, (E) schizophrenia.
2004, pp 82-83
The correct response is option C: Fragile X syndrome

All of these disorders h a v e evidence of genetic trans-


mission, although only fragile X syndrome is due to a
@ single gene. I
W h i c h of the following features best distinguishes
Sadock BJ, Sadock VA (eds): Kaplan and Sadock's Comprehensive
anorexia nervosa from bulimia nervosa?
Textbook of Psychiatry, 8th ed. Philadelphia, Lippincott Williams &
(A) Amenorrhea Wilkins, 2005, pp 237-240
(B) Decreased body weight
(C) Calluses on the dorsum of the hand
(D) Dental enamel erosion
(E) Enlarged parotid glands
T h e correct response is option B: Decreased body
weight

Decreased b o d y weight is a defining feature of


c n o r e x i a nervosa, whereas persons with bulimia ner-
vosa typically have normal body weight. All of the
other features listed may be present in both disorders.

American Psychiatric Association: Diagnostic and Statistical Manual


of Mental Disorders, Fourth Edition, Text Revision (DSM-IV-TR).
Washington, DC, American Psychiatric Association, 2000,
pp 583-595

Section 2: Answers ind Explanations


(m)
T h e rule of confidentiality is w a i v e d in a psychiatrist- T h e highest percentage of persons with mental retar-
patient interaction w h e n the treatment or evaluation dation h a v e an intelligence quotient of:
includes:
(A) <20.
(A) o minor. (B) 20 to 35.
(B) o forensic consultation. (C) 35 to 50.
(C) on impaired physician. (D) 50 to 70.
(D) a patient who reveals a past felony. (E) 70 to 90.
T h e correct response is option B: A forensic consultation T h e correct response is option D: 50 to 70

Ethically, confidentiality is required in psychiatric treat- Up to 8 5 % of persons with mental retardation have an
ment. However, there are someinstgnces irrwhich con- IQ b e t w e e n 50 a n d 7 0 , w h i c h is mild mental retar-
fidentiality is w a i v e d . W h e n a psychiatrist is examining dation. Patients with an IQ b e t w e e n 70 a n d 90 are
a patient for forensic p_ujjpgs.es, the individual must be not considered mentally retarded.
informed that information collected during the examina-
Dulcan MK, Martini DR, Lake MB: Concise Guide to Child and
tion will be shared with the partyjhat engaged the psy- Adolescent Psychiatry, 3rd ed. Washington, DC, American
chiatrist, such as the patientjJawyeior.Jhe_cp.urt. In Psychiatric Publishing, 2003, p 180
addition, common situations that require a waiver of the
confidentiality rule include mjej_ejDortingj^
a n d , in some states, elder abuse and spouse abuse.

A l s o , in most states, the psychiatrist evaluating a


®A 4-year-old girl w h o has been cared for in seven dif-
potentjally violent patient is required t o w a r n a poten-
ferent foster homes since the a g e of 6 months, n o w
tial victim of violence. H o w e v e r , in most states, a psy-
exhibits excessive familiarity with strangers. Her cur-
chiatrist treating an impaired physician, unlike the
rent foster parents, with w h o m she has lived for the
physician's colleagues, is not required to report the
past 5 months, state that she does not seem to be par-
physician if the phys'?'an is a private patient of the ticularly close to them. T h e girl's biological mother is
psychiatrist. On the otfer h a n d , if the psychiatrist is reported to have used alcohol in a binge pattern dur-
treating the patient under the auspices of a state diver- ing her pregnancy. W h i c h of the following is the most
sion program, the relationship becomes more compli- likely diagnosis?
cated and depends on the arrangement with the
(A) Attention deficit hyperactivity disorder
specific state's diversion p r o g r a m .
(B) Fetol alcohol syndrome
A patient w h o reveals p^sj^arjajejjOjjjness_buJJsj]ot
(C) Oppositional defiant disorder
c
J J r x § D j y ^ j ^ g e r o j J i ^ o u ] c L n o t j i k e l y . meet_ require-
(D) Pervasive developmental disorder
(E) Reactive attachment disorder
m j n j s j o r a b r e a c j W confidentiality. Finally, a minor's
confidentiality w o u j d also be respected unlessjhere The correct response is option E: Reactive attachment
r r n
^l_9£L?.. I16 9i tsituation in which either the minor disorder
or another person w a s likely to be harmed. In sum-
The girl's indiscriminate sociability is behavior typical
mary, the rule of confidentiality is w a i v e d w h e n there
of reactive attachment disorder, disinhibited type.
is a threat of h a r m j o the patient or another person.
DSM-IV-TR also specifies an inhibited type of this disor-
Rosner R: Principles and Practice of Forensic Psychiatry, 2nd ed. der. Often children with this disorder have experienced
New York: Oxford University Press, 2003, pp 8-9,177 a series of caregivers or prolonged separation from a
caregiver in early childhood. Symptoms of "markedly
disturbed and developmentally inappropriate social
relatedness" must be present before a g e 5 years.

Sadock BJ, Sadock VA (eds):'Kaplan and Sadock's Comprehensive


Textbook of Psychiatry, 8th ed. Philadelphia, Lippincott Williams &
Wilkins, 2005, pp 3248-3252
American Psychiatric Association: Diagnostic and Statistical Manual
of Mental Disorders, Fourth Edition, Text Revision (DSM-IV-TR).
Washington, DC, American Psychiatric Association, 2000,
pp 127-130

11
|3| <g_
W h i c h of the following diseases associated with W h i c h of the following aspects of cognitive perform-
dementia characteristically has early changes in per- ance is most likely to decline in the course of normal
sonality and a late decline in memory? aging?

(A) HIV infection (A) Short-term memory


(B) Creirfzfelclt-Jcikob disease (B) Speed of performance
(C) Parkinson's disease (C) Store of knowledge
(D) Lewy body dementia , (D) Syntax
1
(E) Pick's disease (E) Vocabulary
T h e correct response is option E: Pick's disease The correctresponse is option B: Speed of performance

Pick's disease specifically affects the frontal and tem- Speed of learning, processing speed, and speed of
poral lobes, accounting for the early signs of person- performance of cognitive tasks tend to decline with
ality changes, loss of social skiljs, ond emotional ' normal a g i n g . T h e other functions listed do not
blunting. Other features of dementia, such as memory decline with normol g g i n g , gnd a decline in a n y of
loss and a p r a x i a , come later. Specific diagnosis of them may be an indication for a thorough or formal
Pick's disease is usually made only on autopsy. The assessment for cognitive impairment.
medical illnesses Huntington's disease, Parkinson's
Spar JE, La Rue A: Concise Guide to Geriatric Psychiatry, 3rd ed.
disease, and HlV_.infec.tipn precede those dementias.
Washington, DC, American Psychiatric Publishing, 2002, pp 25-26
Creutzfeldt-Jakob disease often has a clinical triad
associated with dementia, involuntary movement, and
periodic E E G activity, l e w y body dementia often pre-
sents first with hallucinations and psychosis.

American Psychiatric Association: Diagnostic and Statistical Manual Disorders with significant psychiatric symptoms that
of Mental Disorders, Fourth Edition, Text Revision (DSM-tV-TR). can be linked to a single g e n e include:
Washington, DC, American Psychiatric Association, 2000,
pp 148-151 (A) attention deficit hyperactivity disorder.
Practice Guideline for the Treatment of Patients With Alzheimer's (B) bipolar disorder.
Disease and Other Dementias of Late Life (1997), in American (C) fragile X syndrome.
Psychiatric Association Practice Guidelines for the Treatment of (D) major depression.
Psychiatric Disorders, Compendium 2004. Washington, DC, APA, (E) schizophrenia.
2004, pp 82-83
The correct response is option C: Fragile X syndrome

All of these disorders h a v e evidence of genetic trans-


mission, although only fragile X syndrome is due to a
single gene, i
W h i c h of the following features best distinguishes
anorexia nervosa from bulimia nervosa? Sadock BJ, Sadock VA (eds): Kaplan and Sadock's Comprehensive
Textbook of Psychiatry, 8th ed. Philadelphia, Lippincott Williams &
(A) Amenorrhea Wilkins, 2005, pp 237-240
(B) Decreased body weight
(C) Calluses on the dorsum of the hand
(D) Dental enamel erosion
(E) Enlarged parotid glands
The correct response is option B: Decreased body
weight

Decreased b o d y weight is a defining feature of


c n o r e x i a nervosa, whereas persons with bulimia ner-
vosa typically have normal body weight. All of the
other features listed may be present in both disorders.

American Psychiatric Association: Diagnostic and Statistical Manual


of Mental Disorders, Fourth Edition, Text Revision (DSM-IV-TR).
Washington, DC, American Psychiatric Association, 2000,
pp 583-595

Section 2: Answers and Explanations


38)
T h e rule of confidentiality is w a i v e d in a psychiatrist- The highest percentage of persons with mental retar-
patient interaction when the treatment or evaluation dation h a v e an intelligence quotient of:
includes:
(A) <20.
(A) o minor. (B) 20 to 35.
(B) o forensic consultation. (C) 35 to 50.
(C) an impaired physician. (D) 50 to 70.
(D) a patient who reveals a past felony. (E) 70 to 90.
T h e correct response is option B: A forensic consultation The correct response is option D: 50 to 70

Ethically, confidentiality is required in psychiatric treat- Up to 85% of persons with mental retardation have an
ment. However, there are some instances injvhich con- IQ between 50 a n d 7 0 , w h i c h is mild mental retar-
fidentiality ji_y_gjy__l W h e n a psychiatrist is examining dation. Patients with an IQ b e t w e e n 70 a n d 90 are
a patient for forensic pj_rjjp_ses, the individual must be not considered mentally retarded.
informed that information collected during the examina-
Dulcan MK, Martini DR, Lake MB: Concise Guide to Child and
tion will be shared with the partyjhat engaged the psy- Adolescent Psychiatry, 3rd ed. Washington, DC, American
chiatrist, such as the patient's lawyer. oc_the,court. In Psychiatric Publishing, 2003, p 180
addition, common situations that require a waiver of the
confidentiality rule include the reporting of child abuse
a n d , in some states, elder abuse and spouse abuse.

A l s o , in most states, the psychiatrist evaluating a


®
potentially violent patient is required to w a r n a poten- A 4-year-old girl w h o has been cared for in seven dif-
ferent foster homes since the a g e of 6 months, now
tial victim of violence. H o w e v e r , in most states, a psy-
exhibits excessive familiarity with strangers. Her cur-
chiatrist treating an impaired physician, unlike the
rent foster parents, with w h o m she has lived for the
physician's colleagues, is not required to report the
past 5 months, state that she does not seem to be par-
physician if the phys'T'an is a private patient of the ticularly close to them. T h e girl's biological mother is
psychiatrist. On the ofter h a n d , if the psychiatrist is reported to have used alcohol in a binge pattern dur-
treating the patient under the auspices of a state diver- ing her pregnancy. W h i c h of the following is the most
sion p r o g r a m , the relationship becomes more compli- likely diagnosis?
cated a n d depends on the arrangement with the
(A) Attention deficit hyperactivity disorder
specific state's diversion p r o g r a m .
(B) Fetal alcohol syndrome
A patient w h o reveals past dangerousness b u H s not
(C) Oppositional defiant disorder
c n
JJrX^ Jy_dqjjgejpjJij youJcL n o t j i k e l y . meeX.reqyire-
(D) Pervasive developmental disorder
)

(E) Reactive attachment disorder


mentsjor abregcjh_of confidentiality. Finally, a minor's
contidenjiality w o u j d also be respected unlessjhere T h e correct response is option E: Reactive attachment
^YSS-i^ILSn^rS^nt^iiyatiqn in w h i c h either the minor disorder
or another person w a s likely to be harmed. In sum-
T h e girl's indiscriminate sociability is behavior typical
mary, the rule of confidentiality is w a i v e d when there
of reactive attachment disorder, disinhibited type.
is a threat of harm to the patient or another person.
DSM-IV-TR also specifies an inhibited type of this disor-
Rosner R: Principles and Practice of Forensic Psychiatry, 2nd ed. der. Often children with this disorder have experienced
New York: Oxford University Press, 2003, pp 8-9,177 a series of caregivers or prolonged separation from a
caregiver in early childhood. Symptoms of "markedly
disturbed a n d developmentally inappropriate social
relatedness" must be present before a g e 5 years.

Sadock BJ, Sadock VA (eds):'Kaplan and Sadock's Comprehensive


Textbook of Psychiatry, 8th ed. Philadelphia, Lippincott Williams &
Wilkins, 2005, pp 3248-3252
American Psychiatric Association: Diagnostic and Statistical Manual
of Mental Disorders, Fourth Edition, Text Revision (DSM-IV-TR).
Washington, DC, American Psychiatric Association, 2000,
pp 127-130

11
American Psychiatric Association: Diagnostic and Statistical Manual
of Mental Disorders, 4th Edition, Text Revision (DSM-IV-TR).
An 8-year-old girl insists on keeping a rigid routine Washington, DC, American Psychiatric Association, 2C00,
w h e n dressing, will w e a r only certain clothes, insists pp 467-068
on recopying her h o m e w o r k if there are a n y mistakes, Kaplan HI, Sadock BJ: Synopsis of Psychiatry: Behavioral
and has temper tantrums w h e n the items on her desk Sciences/Clinical Psychiatry, 9th ed. Baltimore, Lippincott Williams .
& Wilkins, 2003, p 626
are m o v e d . During a discussion of the diagnosis'and
Shalev AY: What is posttraumatic stress disorder? J Clin Psychiatry
treatment options, her parents express reluctance to
2001;62(suppl17):4~10
use medication a n d w a n t to explore other options.
The first recommendation w o u l d be:

(A) cognitive behavior therapy.


(B) family therapy.
(C) interpersonal psychotherapy. W h i c h of the following medications is considered first-
(D) parent training. line monotherapy for posttraumatic stress disorder?
(E) supportive psychotherapy.
(A) Clonazepam
The correct response is option A: Cognitive behavior (B) Sertraline
therapy (C) Olanzapine
Cognitive behavior therapy has been demonstrated to
(D) Valproate
(E) Propranolol
benefit children with obsessive-compulsive disorder.
Uncontrolled trials of cognitive behavior therapy have The correct response is option B: Sertraline
shown excellent response in up to three-fourths of
SejIrcilineJsjDA^ PTSD.
p_ajdej]]sjregted. M a n y experts recommend cognitive
The other_ medications are less well established as
behavior therapy as the first-line a p p r o a c h for the
beneficial. C l o n a z e p a m , widely used as an anxi-
majority of children a n d adolescents with pbsessive-
olytic, has F D A approval p_nly for use as an anticon-
compujsj_/e dlisorder.
vulsant and frecfment for panic disorder, for which it
Sadock BJ, Sadock VA (eds): Kaplan and Sadock's Comprehensive is a second-line choice. O l a n z a p i n e has not been
Textbook of Psychiatry, 8th ed. Philadelphia, Lippincott Williams & reported as a treatment for PTSD. Propranolol has
Wilkins, 2005, p 3285
been tried experimentally to diminish the autonomic
arousal associated with the trauma. The rationale for
its use is that it will curtail the body's emotional/auto-

-® nomic response to the trauma by diminishing the


body's " m e m o r y " of the heightened emotional state.
W h i c h of the following are common hyperarousal The beta-blockers, theoretically, will prevent or at least
symptoms in posttraumatic stress disorder (PTSD)?
minimize the PTSD syndrome.
(A) Intense psychological distress at exposure to external cues Albucher RC, Liberzon I: Psychopharmacological treatment in PTSD:
resembling the trauma a critical review. J Psychiatr Res 2002; 36:355-367
(B) Difficulty falling or staying asleep Friedman MJ, Donnelly CL, Mellman TA: Pharmacotherapy for PTSD.
(C) Intrusive images of the event Psychiatr Ann 2003; 31:57-62
(D) Feelings of estrangement from others Stein DJ, Hollander E (eds): American Psychiatric Publishing Textbook
of Anxiety Disorders. Washington, DC, American Psychiatric
The correct response is option B: Difficulty falling or Publishing, 2002, p 397
staying asleep

Difficultyjailing or staying asleep is a symptom of


hypej^arousal. In the DSM-IV-TR, posttraumatic stress
disorder symptoms are clustered into three categories:
reexperiencing, avoidance a n d numbing, a n d hyper-
arousal. Options A a n d C a r e symptoms of reexperi-
encing the event. O p t i o n D falls into category C of the
DSM-IV-TR criteria for PTSD, that is, "Persistent avoid-
ance of stimuli associated with the trauma a n d numb-
ing of general responsiveness."

Section 2: Answers and Explanations


King SA: Pain disorders, in The American Psychiatric Publishing
43 Textbook of Clinical Psychiatry, 4th ed. Edited by Hales RE,
A 50-yecr-old w o m a n has a long history of difficulty Yudofsky SC. Washington, DC, American Psychiatric Publishing,
w i t h driving because she worries that she might hit a 2003, p 1038
c a r or a person accidentally. She also worries exces-
sively about her son getting hurt or attacked w h e n he
g o e s out. Her husband can often reassure her. W h i c h
of the following diagnoses is most appropriate? ;45)
Echolalia and echopraxia are most likely manifesta-
(A) Agoraphobia
tions of which of the following disorders?
[B] Delusional disorder
((] Generalized anxiety disorder (A) Hypochondriasis
(D) Obsessive-compulsive disorder (B) Bipolar disorder, mixed episode
(E) Panic disorder (C) Depression with catatonic features
T h e correct response is option C: G e n e r a l i z e d anxiety (D) Lewy body dementia
disorder (E) Frontotemporal dementia
T h e correct response is option C: Depression with
T h e worries that occur in generalized anxiety disorder
catatonic features

|urgncej. Obsessive-compulsive disorder involves a


^£h°l_^i _£Di?Sh?P^?^-? an
characterize catatonic
obsessions, which are intrusive unrealistic i d e a s that depression. A mood disorder with catatonic features
m a y be rjej;ognjze.d_as.being absurd <but cannot be must have two or more of the following features:
resisted. Panic disorder involves anxiety episodes "^olo^lclirMoEp^,.. .e^eitTeiggitafiBn: extreme"neg-
without specific content. Ag^oxaphojblainvolves, bejng ratmsrrf; JSICOIK^ and
i n^jh2ai]jDnji_fj^rn_j^ icji_esca pe_m ig ht_be.. d iff icu lt_or eTcfetqte?or ^ ^ g r a 3 _ i G l > ( A P A Practice Guideline).
embarrgssjng o j J o n v t a M i e l p may not be.available
Practice Guideline for the Treatment of Patients With Major
in the event of panic.
Depressive Disorder, 2nd ed (2000), in American Psychiatric
American Psychiatric Association: Diagnostic and Statistical Manual Association Practice Guidelines for the Treatment of Psychiatric
of Mental Disorders, Fourth Edition, Text Revision (DSM-IV-TR). Disorders, Compendium 2004. Washington, DC, APA, 2004, p 469
Washington, DC, American Psycnlatric Association, 2000,
pp 472^176

An adolescent female took an unknown drug at an all-


night dance party. She w a s brought to the e m e r g e n c y
A 40-year-old w o m a n with chronic headaches has department for evaluation of altered mental status and
u n d e r g o n e trials with several narcotic a n d nonnar- marked hyperthermia. W h i c h of the following w a s
cotic agents with variable success. Her' physician most likely the drug that w a s ingested?
elects to try her on a newer antidepressant medica-
tion. W h i c h of the following medications is most likely
(A) Ketamine
to be effective?
(B) Methylenedioxymethamphetamine (MDMA)
(C) Flunitrazepam
(A) Bupropion (D) Gomma-hydroxybutyrate (GHB)
(B) Mirtazapine (E)Phencyclidine(PCP)
(C) Nefazodone
T h e correct response is option B: Methylenedioxy-
(D) Sertraline
methamphetamine { M D M A )
(E) Venlafaxine
T h e correct response is option E: Venlafaxine All of these drugs are commonly k n o w n as "club
drugs" and are frequently used at all-night dance par-
jdjLejpromisjnq^as^t^oaj^ ties. M D M A has been reported in some instances to
qesicWarMefazodb'nel? /nirjazapine, or cause severe adverse effects, including altered mental
A l t h o u g h the precise mechanism of action for analge- status, convulsions, hypo- or hyperthermia, cardiovas-
sia is u n k n o w n , itjs^beljeyed thajagents that increase c^QrJDitability, he^LajpJoxicity, a n d death.
serojojjjrKmd norepinephrine are more effective than
those jhgt o n j y j n c r e a s e serotonin.

74 FOCUS Psvchisrrv Review: £00 Self-Assessment Questions


a d o p a m i n e enhancer that causes euphoria. nia, such as should
Higher doses j > [ G _ H B can make the user feel sleepy also be considered.
and may cause_Y_o.mili.pg, muscle spasms, and loss of
Schatzberg AF, Cole JO, DeBattista C: Manual of Clinical
consciousness. If m i x e d with alcohol, G H B can slow Psychopharmacology, 4th ed. Washington, DC, American
breathing to a dangerously low..rate, which..has Psychiatric Publishing, 2003, p 350
caused a number of deaths. ' Lee JW, Schwartz DL, Hallmayer J: Catatonia in a psychiatric inten-
sive care facility: incidence and response to benzodiazepines. Ann
Bie^^Bm^SncSeTamm^are related substances. Clin Psychiatry 2000; 12:89-96
They belong to a class of drugs called &i$scoafivfc
^anestherjci" vv^dTjiaye^^
cepHonJromjej}^ A t lpwej^dose^k^ojpinej:auses
ajregjnyjeeiingjir^^ ajTajy^a^prp- A 49-year-old w o m a n is referred for treatment of
duce numbness,in_rjie_exlreinities. Higher doses of ket- chronic, severe m a j o r d e p r e s s i p n . W h i c h of the fol-
amine may produce a hallucinogenic effect. lowing treatment approaches is most likely to be asso-
ciated with sustained improvement in her symptoms?
lium'trdzepom? a | F i ^ _ _ i 3 n _ _ g n ^ T S g n j 6 , also
k n o w n a s j o h y p r j o f (a trade name) or |rjwfi&J," has
(A) Antidepressant medication plus psychotherapy
(B) Psychotherapy alone
been characterized as the l i ^ 2 a p e ^ r y § ' ' Flu-
(C) Antidepressant medication alone
nitrazepam is prescribed in Latin A m e r i c a and Europe
(D) EG alone
as a shorHean .treatment.for.insomnia arid as a pre-
(E) ECT plus psychotherapy
anesthetic medication. Flunitrazepam can cause a
chemically induced amnesia and may cause The correct response is option A: ' ^ d e p r e s s a n t med-
decreased blood pressure, drowsiness, visual distur- ication plus psychotherapy

bances, dizziness, confusion, gastrointestinal distur- Since the publication in 2000 of the A P A Practice
bances, a n d urinary retention. Guideline for the Treatment of Patients W i t h M a j o r
McDowell DM: MDMA, kotamine, GHB, and the "club drug" scene, in Depression, additional studies n o w support combined
The American Psychiatric Publishing Textbook of Substance Abuse psychotherapy a n d antidepressant therapy for chronic
Treatment 3rd ed. Edited by Galanter M, Kleber HD. Washington, depression. T h e evidence for this approach in treating
DC, American Psychiatric Publishing, 2004, pp 321-331
mild or moderate depression is less compelling. W h i l e
E C T is highly effective, E C T alone usually does not
produce sustained improvement. There have been f e w
controlled studies of E C T plus psychotherapy.
MI .
A 23-year-old man w h o is hospitalized for psychosis Pampallcna S, Bollini P, Tibaldi G, Kupelnick B, Munizza C: Combined
pharmacotherapy and psychological treatment for depression: a
displays prominent, excessive, a n d purposeless motor
systematic review. Arch Gen Psychiatry 2004; 61:714-719
activity together with peculiar voluntary movements. Hegerl U, Plattner A, Moller HJ: Should combined pharmaco- and
On one occasion, he stands in the middle of the w a r d psychotherapy be offered to depressed patients? A qualitative
immobile and mute. He demonstrates w a x y flexibility. review of randomized clinical trials from the 1990s. Eur Arch
The appropriate medical intervention is: Psychiatry Clin Neurosci 2004; 254:99-107
Thase ME, Greenhouse JB, Frank E, Reynolds CF III, Pilkonis PA,
(A) benztropine. Hurley K, Grochocinski V, Kupfer DJ: Treatment of major depres-
[Bj clonidine. sion with psychotherapy or psychotherapy-pharmacotherapy
(() lorazepam. combinations. Arch Gen Psychiatry 1997; 54:1009-1015
(D) propranolol. De Jonghe F, Kool S, van Aalst G, Dekker J, Peen J: Combining psy-
(E) ziprasidone. chotherapy and antidepressants in the treatment of depression.
J Affect Disord 2001; 64(2-3):217-229
The correct response is option C: Loraze_pani Arnow BA, Constantino MJ: Effectiveness of psychotherapy and com-
bination treatment for chronic depression. J Clin Psychol 2003;
Lorazepam, by a variety of routes of administration, 59:893-905
imcjroves^atajonja^dramaticaIly, although temporar- Casacalenda N, Perry JC, Looper K: Remission in major depressive
ily. ffijS-^depressi^ and scES^fiSS&I(catatonic disorder: a comparison of pharmacotherapy, psychotherapy, and
control conditions. Am J Psychiatry 2002; 159:1354-1360
type) are the most frequently observed psychiatric dis-
Sega! Z, Vincent P, Levitt A: Efficacy of combined, sequential, and
orders that are associated with catatonia. Possible crossover psychotherapy and pharmacotherapy in improving out-
medical causes include rca ana comes in depression. J Psychiatry Neurosci 2002; 27:281-290
eTc^h"al6^"a"thy] Catatonia m a y also appear as an UK ECT Review Group: Efficacy and safety of electroconvulsive ther-
apy in depressive disorders: a systematic review and meta-analy-
adverse drug effect of a neuroleptic medication or
sis. Lancet 2003; 361(9360):799-808
phencyclidine^(PCPI7 Neurological causes of catato-

Section 2: Answers and Explanations 75


(ST)
In people with typical left-brain d o m i n a n c e , the ability A patient w h o is completely deaf arrives with an inter-
to interpret the emotional tone of speech is a function preter at the outpatient clinic for an evaluation of
of the: depressed m o o d . You w i s h to k n o w about the
patient's sleep quality. Of the following, which is the
(A) left premotor cortex (Brora's areo). most appropriate w a y to w o r k with the interpreter and
(B) right premotor cortex. the patient?
(C) left parietotemporal cortex (Wernicke's area).
(D) right parietotemporal cortex. (A) Ask the interpreter, "How is she sleeping?"
(E) anterior cingulate gyrus. (B) Ask the interpreter, "Please ask her how she is sleeping."
(C) Look at the patient and ask, "How are you sleeping?"
T h e correct response is option D: Right parietotempo- (D) Loudly enunciate "How are you sleeping?" to the patient.
ral cortex (E) Write out "How ore you sleeping?" and give it to the patient.
Just as Wernicke's area in thedpminant hemisphere is The correct response is option C: LooJc_ajJ}ej3atient
inyolv^d_in_understanding language, the correspon- and_asJcjJ[Hoy/i^ ?"
d]ncj_a_rea in the nondominant hemisphere interprets the
emotional tone of speech, or prosody. Analogous to O n e should always address the patient directly while

Broca's area in the left hemisphere, the right premotor speaking in a regular manner. It is the job of the inter-

cortex is involved in expressive l a n g u a g e production, preter to translate the words into sign language and vice

providing the "music" for the semantic content. versa. The, interpreter is not jo be addressedjjjrecjly.

Kaufman DM: Clinical Neurology for Psychiatrists. Philadelphia, WB Haskins BG: Serving deaf adult psychiatric inpatients. Psychiatr Serv
Saunders, 2001, p 175 2004; 55:439-441
Panksepp J: Affective Neuroscience: The Foundations of Human and Phelan M, Parkman S: How to work with an interpreter. BMJ 1995;
Animal Emotions. New York, Oxford University Press, 1998, p 334 311:555-557
Steinberg A: issues in providing mental health services to hearing-
impaired persons. Hosp Community Psychiatry 1991; 42:380-389

®
W h i c h of the following psychotherapies has the great-
est b o d y of evidence demonstrating efficacy for social
An internist consults a psychiatrist because of his frus-
phobia?
tration with an elderly patient w h o has a diagnosis of
(A) Insight-oriented psychotherapy hypochondriasis. Medical tests are negative, but the
(B) Interpersonal psychotherapy patient is unable to accept that he is not ill. T h e psy-
(C) Brief psychodynamic psychotherapy chiatrist confirms the diagnosis of hypochondriasis.
(D) Cognitive behavior psychotherapy W h i c h of the following is the best management strat-
(E) Supportive psychotherapy egy for a patient with hypochondriasis?

T h e correct response is option D: Cognitive, behavior (A) Refer the patient to a more psychologically minded internist
psychotherapy colleague.
(B) Have regularly scheduled appointments with limited reas-
T h e most effective commonly used treatment for social surance.
p h o b i a is based on cognitive behavior therapy prin- (C) See the patient as needed, but for a limited time.
ciples a n d techniques. Other theoretical approaches (D) Instruct the patient to call only for urgent matters.
h a v e been used, but little research has been done to (E) Refer the patient for psychotherapy.
establish their usefulness. The major problem in social
The correct response is option B: R e ^ j y J g j y ^ c h e j M e d
p h o b i a is njaj3tiye_ejfaJyjhlon. M e r e exposure to the
a pp^mt^ejTtsj^dW
social interaction does not produce anxiety reduction.
T h e individual with social phobia must alter dysfunc- The management of hypochondriasis is a challenge for
tionaj^belleis^arid,biased,perceptions. Therefore, cog- the internist. Regularly i scheduled appointments with
n iliy^Jnpjjtjpj;^ Olej^ejTtjpjvfor limited reassurance appears to be the _ management
treatment success. strategy of choice. A more psychologically minded
internist might facilitate dependency, w h i c h might result
Stein DJ, Hollander E (eds): American Psychiatric Publishing Textbook
of Anxiety Disorders. Washington, DC, American Psychiatric in more visits and greater preoccupation with the symp-
Publishing, 2002, pp 323-324,330-332 toms. The other approaches do not provide enough
structure to help the patient contain his anxiety.

76 FOCUS Psvchiarrv Review; 400 Self-Assessment Ou -"inns


American Psychiatric Association: Diagnostic and Statistical Manual
of Mental Disorders, Fourth Edition, Text Revision (DSM-IV-TR).
Washington, DC, American Psychiatric Association, 2000, A 45-year-old w o m a n with bipolar disorder w h o has
pp 504-507 been successfully maintained on lithium presents at
Wise MG, Rundell JR (eds): The American Psychiatric Publishing the clinic with the complaint of swelling in her ankles.
Textbook of Consultation-Liaison Psychiatry: Psychiatry in the Examination reveals 2+ pitting e d e m a . Her serum
Medically III, 2nd ed. Washington, DC; American Psychiatric'
lithium level is 0.8 m E q / L . T h e physician prescribes a
Publishing, 2002, pp 377-378
thiazide diuretic. Four d a y s later the patient presents
at the e m e r g e n c y department with confusion, a
coarse tremor in her extremities, a n d a t a x i a . H e r

® serum lithium level is n o w 2.6 m E q / L . Urinalysis


reveals a •slightly elevated specific gravity a n d an
A 29-year-old w o m a n presents for an initial evalua- absence of b l o o d , ketones, and protein. W h i c h of the
tion. She describes periods of mood lability and following best explains the patient's lithium toxicity?
unstable interpersonal relationships, particularly with
men. During periods of stress, she reports feeling (A) Acute nephrogenic diabetes insipidus
a n g r y a n d "empty" a n d sometimes scratches herself (B) Increased reabsorption in the proximal tubules
with sharp items. Sleep is often a problem, a n d alpra- (C) Decreased glomerular filtration rate
zolam has been helpful. In developing a treatment (D) Glomerulonephritis
plan, w h i c h of the following principles would be most (E) Tubulointerstitial nephropathy
appropriate? The correct response is option B: I n c j ^ g s ^ r e a b s o r p -

(A) Restrict pharmacotherapy to antidepressants and mood sta- tiojijnjhejjroxj^


bilizers. The patient has developed acjjtejjthjujj^^ after the
[BJ Treat with multiple classes of medications for potential administration of a thiazide diuretic. Lithium is excreted
future symptoms.
through the kidneys a n d is reabsorbed in the proximal
{() Target specific symptoms that are currently causing disrup-
tubules with sodium,and water. W h e n the body has a
tion.
sodium deficiency, such as occurs with the administra-
(D) Refuse to prescribe a benzodiazepine.
(E) Withhold medications if the patient engages in acting out tion of sodium-depleting diuretics, theJddneys_comp_en-
a f

behavior. sgje_byj^gbs^rbing_^ 0^3!°J}9J!yi!hj '


lithium. This,, a n d j h e [oss of fluid volume, results in ele-
The correct response is option C: Tajg ej_sp^cjfic_symp-
A
vated j e i u m lithium levels and toxicity. Lithium m a y
toms that are currently causing disruption
induce nephrogenic diabetes insipidus. H o w e v e r , this
Of the options listed, C is the most germone to the would cause production of large volumes of dilute urine.
management of borderline personality disorder. Paradoxically, the administration of a hSjazjde^diuretic
PajiejTJsj^ajjirjM^ taj3ejea[_a^p_roach. M a n y re^dy^ejjjnne^ojjtp_ut. Cumulative exposure to lithium
patients will need mulHpJe_clqs_Ses-of^medications to may result in a tujpjjlojntej^ and_a
target_diffej^ent domajns_ofjJifficujty (affective, behav- degejpjejnjf}^ however, this
ioral, a n d cognitive). Due to the heterogeneity of pre- is an insidious rather than an acute process and is unre-
sentation, however, clinicians, must be flexible jn their lated to the introduction of a thiazide diuretic. Finally,
pharrnacotherapeutic a p p r o a c h . lithium may cause a glomerulonephritis, resulting in a
nephrotic syndrome. This is characterized by the pres-
Practice Guideline for the Treatment of Patients With Borderline
ence of large quantities of protein in the urine.
Personality Disorder (2001), in American Psychiatric Association
Practice Guidelines for the Treatment of Psychiatric Disorders, Schatzberg AF, Cole JO, DeBattista C: Manual of Clinical
Compendium 2004. Washington, DC, APA, 2004', pp 755-757 Psychcpharmacology. Washington, DC, American Psychiatric
Publishing, 2005, pp 250,259-265
Andreasen NC, Black DW: Introductory Textbook of Psychiatry, 3rd
ed. Washington, DC, American Psychiatric Press, 2001

Section 2: Answers and Explanations


Crijnen AA, Achenbach TM, Verhulst FC: Problems reported by par-
(552 - ents of children in multiple cultures: the Child Behavior Checklist
A random community sample contains 100 individu- syndrome constructs. Am J Psychiatry 1999; 156:569-574
als w h o meet diagnostic criteria for borderline per-
sonality disorder. W h i c h of the following is the best
estimate of the gender ratio of the sample?

(A) 50% men and 50% women


(B) 40% men and 60% women Characteristic cognitive processes in persons with
obsessive-compulsive disorder include:
(C) 25% men and 75% women
(D) 10% men and 90% women (A) above average spatial recognition.
T h e correct response is option C: 25% men a n d 75% (B) better memory for pleasant events.
women (C) decreased capacity for selective attention.
(D) impaired reality testing.
Borderline personality disorder isjhiejnosj^mmoj^per- (E) normal confidence in one's own memory.
soj3gjity_disojde^ It is present in 10%
T h e correct response is option C: D j a ^ a s e d ^ c a p a c i t y
o£jndividuj2|s^
i^^^^^^g^lnooM^ f?Xi§lectjve_aJtej^
and 30%H50% of
In persons with obsessive-compulsive disorder, a
cljmcaJjDQpuJarjons with a personality disorder. It occurs
in an estimated 2% cj^jhe^e^^j^ c]e£rej3$^a^cajDC^ is hypoth-
Borderline personality disorder is diagnosed predomi- esized to be related to the difficulties in dismissing
najitjyjrrwomen, with an estimated female-to-male ratio obsessions. Persons with the disorder h a v e negative
of J3jJ_. The disorder is present in cultures around the beliefs about responsibility, m e m o r y biases for dis-
w o r l d . It is approximately five t[mes_mo£e_common turbing themes, a n d decreased confidence in their
anwig^firsj^l^ memory, a n d they m a y s h o w d e j i d j ^ J r x _ j p a t i a l
dispjdejJ}anJnjTje^ There is also a rec^gj}itJon.
greaJeHajTijIi^^ anti-
Hollander E, Simeon D: Anxiety disorders, in American Psychiatric
sodgJ^p^s^n^jityjjspjder, and mood disorders. Publishing Textbook of Clinical Psychiatry. Fourth Edition. Edited
Practice Guideline for the Treatment of Patients With Borderline by Hales RE, Yudofsky SC. Washington, DC .American Psychiatric
Personality Disorder (2001), in American Psychiatric Association Publishing, 2003, p 586
Practice Guidelines for the Treatment of Psychiatric Disorders,
Compendium 2004. Washington, DC, APA, 2004

@
In family studies of patients with schizophrenia, the
personality disorder that has been found to occur most
T h e Child Behavior Checklist is a commonly used frequently in first-degree relatives is:
instrument completed by parents about their children's
behaviors. In a study comparing the results from sub-
(A) borderline.
ject groups obtained from multiple cultures, girls
(B) histrionic.
scored higher than boys across all cultures on which
(C) paranoid.
behavior scale?
(D) schizoid.
(E) schizotypal.
(A) Aggression
T h e correct response is option E: Schizotypal
(B) Anxious/depressed
(C) Attention problems Although all cluster A personality disorders (paranoid,
(D) Delinquency schizoid, a n d schizotypal) are more c o m m o n in the
(E) Thought problems biological relatives of patients with schizophrenia
T h e correct response is option B: A n x i o u s / d e p r e s s e d than in control g r o u p s ; , t h e greatest correlation has

r
bejnjojjndjDeJ^
Acj^ss^aJJjtudie^^ 9i js_sj^ejLJ^ a
.!li5ShizoroJ}renia. T h e r e is increasing evidence, pri-
spjT^jc__compJa^ marily from twin studies, that genetJ£ja^tor^con-
while boys w e r e higher on attention problems, delin-
t r i b y j e t p p e r s o n a l i t y disorders. O t h e r evidence to
auejTt^behavior, and ajjgjj5Sj>ive__be^^ support a genetic link is the relationship between cer-
Therej^/asjTo_signjficant differ_encej^jween boys and tain axis I disorders and personality disorders.
girls on thought problems.
Sadock BJ, Sadock VA: Kaplan and Sadock's Synopsis of Psychiatry, T h e correct response is option C: Paroxetine
9th ed. Philadelphia, Lippincott Williams & Wilkins, 2003, p 800
This patient is presenting with the classic symptoms of
The following vignette applies to questions 59 and 60. panjc_dT|order. All of the medications listed h a v e
been used in the treatment of this condition. In gen-
A 25-year-old w o m a n presents to the e m e r g e n c y
eral, experience is showing t f j e ^ y ^ e j j p j i t y _ o M h e
department with the chief complaint, "I think I'm hav-
SJiRlsjDnd^d^
ing a heart attack." She reports that while g r o c e r y
monoamine b x i d a s e inhibitors, a n d tricyclic and tetra-
shopping she suddenly felt "scared to death." Her
heart w a s racing, she felt short of breath and d i z z y , cylic drugs in terms of effectiveness and tolerance of
a n d she w a s nauseated and broke out in a sweat. H e r adverse effects. T h e beta-adrenergic receptor antago-
fingers a n d hands and the area around her mouth felt nists have not been found to be particularly useful for
numb. T h e episode lasted about 10 minutes and dis- panic disorder.
sipated on its o w n . She managed to drive herself to
Stein DJ, Hollander E (eds): American Psychiatric Publishing Textbook
the emergency department. Physical examination a n d
of Anxiety Disorders. Washington, DC, American Psychiatric
laboratory studies, including a chest X-ray, blood
' Publishing, 2002, p 265
chemistries, cardiac e n z y m e s , and electrocardio- Stein DJ (ed): Clinical Manual of Anxiety Disorders. Washington, DC,
g r a m , are normal. American Psychiatric Publishing, 2004, pp 25-29

(§ @
A 38-year-old man with migraine headaches had suc-
In the lab, which of the following substances w o u l d be
cessfully obtained relief by taking codeine. Recently
most likely to induce an episode with these symptoms?
his physician started him on a trial of paroxetine for
(A) Carbon monoxide suspected depression. The patient notes improvement
(B) Sodium lactate in his symptoms of depression a n d n o w has head-
(C) Physostigmine aches less frequently, but w h e n he does have one, he
(D) Propranolol must take twice the amount of codeine for pain relief.
(E) Sodium pyruvate W h i c h of the following best describes this drug inter-
action?
T h e correct response is option B: Sodium lactate
(A) Cytochrome P450 enzymes: inhibition
T h e patient is exhibiting the classic signs and symptoms (B) Cytochrome P450 enzymes: induction ,
of panic disorder. W o m e n are two to three times more (C) Increased protein binding
likely to be affected than men; the mean age at pre- (D) Decreased absorption
sentation is about 25 years, and onset is typically (E) Increased excretion
acute. A number of panic-inducing substances (panico-
T h e correct response is option A: CyjpcJ¥qm_e_P450
gens) have been identified. Respiratory panicogens
enzymes: Inhibition
shift the acid-base balance. They include carbon diox-
ide, sodium lactate, and bicarbonate. Neurochemical Codeine's analgesic effect is a result of its metabolism
panicogens act through specific neurotransmitter sys- to morphine. This transformation is accomplished by a
tems. cytochrome P450 e n z y m e , C Y P 2 D 6 . If that e n z y m e is
inhibited—such as occurs with some drugs, including
Kaplan HI, Sadock BJ: Synopsis of Psychiatry. Philadelphia, Lippincott
paroxetine—thereby interfering with available sub-
Williams & Wilkins, 2003, p 262
strate (codeine) for transformation to the active
metabolite (morphine), the dose of codeine must be
increased a b o v e usual levels.

(j) Hales RE, Yudofsky SC (eds): The American Psychiatric Publishing


Textbook of Clinical Psychiatry, 4th ed. Washington, DC, American
T h e medication that is most likely to be effective in the
Psychiatric Publishing, 2003, pp 1117,1034,1057
long-term treatment of her condition with the best tol-
erance of side effects is:

(A) alprazolam.
(B) buspirone.
(C) paroxetine.
(D) propranolol.
(E) imipramine.

Section 2: Answers and Explanations


_3 Bl!
W h i c h of the following antidepressants is most likely A 35-year-old man presents with a 4-week history of
to be associated with substantial weight gain? low m o o d , crying spells, poor sleep with early morn-
ing awakening, poor appetite with a 12-pound
(A) Bupropion weight loss, a n d d j f f i c ^ t y j n j : o n ^ At
(B) Fluoxetine a g e 27 he had been hospitalized with an episode of
(() Sertraline mania, but shortly thereafter he decided not to con-
(D) Venlafaxine tinue in outpatient follow-up treatment. He has no
(E) Mirtazapine medical problems and takes no medications. As initial
pharmacotherapeutic treatment, which of the follow-
T h e correct response is option E: Mirtazapine
ing is most appropriate?
^^ejih3jncj^ase and w e ] g h [ _ g a i n ^ i a y e _ b e e n j n o r e
(A) Lamotrigine
consistently associated with mirtazapine than with the
(B) Nortriptyline
other listed antidepressants, perhaps because of its
(C) Sertraline
p_otent_H! antihistamine effect a_nd its. antagonism .of
(D) Valproate
t h e j e r o t o n i n 5 - H T .receptor.
2
(E) Venlafaxine
Fava M: Weight gain and antidepressants. J Clin Psychiatry 2000; T h e correct response is option A: Lamotngme
61(suppM1):37-41
Sussman N, Ginsberg DL: Weight effects of nefazodone, bupropion, A c c o r d i n g to the A P A Practice Guideline for the
mirtazapine, and venlafaxine: a review of available evidence. Treatment of Patients W i t h Bipolar Disorder (Revised),
Primary Psychiatry 2000; 7:33-34,47-48
thejirstjine. phqrmacological^ej]tment_ for_bjpolar
Schatzberg AF, Nemeroff CB (eds): The American Psychiatric
Publishing Textbook of Psychopharrnacology, 3rd ed, Washington, depression is the .initiation of either lithium or lamo-
DC, American Psychiatric Publishing, 2004, Tables 52-56, p 858 trigine. T h e treatment goals are the remission of the
symptoms of major depression and to avoidprecipita-
tion_of_a manic or hypomanic episode. Antidepressant
monotherapy is not recommended given the risk of
precipitating a switch into mania. Small studies have
Expert c o n s e n s u suggests that tbeJength of timejor a suggested that i n t i r p ^ r s o n a ^ j h e r a p y and cognitive
pharmacological trial in obsessive-compulsive .disor- behavior therapy may also be useful w h e n a d d e d to
der should be at least: p h a r m a c o t h e r a p y during depressive episodes in
patients with bipolar disorder.
IA| 3 weeks.
(B) 6 weeks. American Psychiatric Association: Practice Guideline for the
(C) 9 weeks. Treatment of Patients With Bipolar Disorder (Revision). Am J
(D) 12 weeks. Psychiatry 2002; 159(April suppl). Reprinted in FOCUS 2003;
1:64-110 (p 71)
T h e correct response is option D: 12 weeks ••

T h e latency for responses to medications is longer in


patients with obsessive-compulsive disorder than in
those with depression; response may take 10 to 1.2
weeks. There_isJ.ess.agreement.on what .is acceptable
qs an adequate,dose. Some fixed-dose trials suggest
that h i g j i e j j i o j e j j ^ ^ Trials of medica-
tions used for obsessive-compulsive disorder indicate
that a daily dose for 1 Q_ ]2 w e e b is optimal '(e.g.,
clomipramine,..150 mg; fluvoxgmine, 150 mg; fluoxe-
tln^jiQjng; sertraline, 150 mg; p a r o x e t i n e ^ 4 0 m g ) .

Stein DJ, Hollander E (eds): American Psychiatric Publishing Textbook


of Anxiety Disorders. Washington, DC, American Psychiatric
Publishing, 2002, pp 208-209

rr\r-i i c TI_..L: t>-..: M n C ' f A-,... r> :


The treatment of choice for interferon^psychiatric side
effects of depression, anxiety, and irritability is cur-
A patient with borderline personality disorder is in
rently a ^ejectiye^serofonin reuptake inhibitor such as
dialectical behavior therapy. She has left messages on
the therapist's voice-mail while he is on vacation sejlrdme Q r j m e ^ f j h ^ / ) ^ agents.
despite an agreement that she would not call him at all
during his vacation a n d w o u l d go to the emergency ^eponedfhepQTic^tox1cm7 The antipsychotics listed,
department if she became suicidal. T h e best approach risperidone pnd haloperidol, are not the first choice for
in dialectical behavior therapy is for the therapist to: symptoms of depression a n d anxiety. Finally, Jrjjz2

(A) explain that a treatment boundary has been violated and B5B a s t r o n
9 soporific, ISBBBBBBBBBSiBlS
therapy will have to end. tefeefcmedKxmc^onias^reqn
(B) wait for the patient to bring up the issue before discussing Wise MG, Rundell JR (eds): The American Psychiatric Publishing
the implications for therapy. Textbook of Consultation-Liaison Psychiatry: Psychiatry in the
(C) explain to the patient that the treatment plan will have to Medically III, 2nd ed. Washington, DC, American Psychiatric
Publishing, 2002, p 1065
change if she cannot keep the agreement.
(D) make an exception since there is a history of serious
attempts and safety is an issue.
T h e correct response is option C: Explain jo the
patient that the treatment p j a j v ^ l l ^ h a y e j o ^ c h a n g e if
An II-year-old girl is referred for an evaluation of
she cannot keep the agreement
school problems. H e r teachers and parents describe
Boundary issues a r e a significant aspect of treatment her as argumentative, hostile, disrespectful and diffi-
of patients with borderline personality disorder. cult. The girl often refuses jo listen, wjJ|_noLqbey instruc-
Therapists should be alert to the occurrence of bound- tions, does not do her w o r k , has temper tantrums, and
a r y violations a n d proactive in dealing with them — insists on having her o w n w a y . She has been this w a y
since preschool. T h e most likely diagnosis is:
both in terms of ascertaining their meaning and in
terms of restoring the boundaries to maintain the (A) antisocial personality disorder.
patient's safety a n d the effectiveness of therapy. (B) attention deficit hyperactivity disorder.
(C) conduct disorder.
Practice Guideline for the Treatment of Patients With Borderline
Personality Disorder (2001), in American Psychiatric Association
(D) intermittent explosive disorder.
Practice Guidelines for the Treatment of Psychiatric Disorders, (E) oppositional defiant disorder.
Compendium 2004. Washington, DC, APA, 2004, p 763
The correct response is option E: Opposifiono! defiant
disorder

The features of oppositional defiant disorder include a


recurrent, pattern of.negativistic, defiant, disobedient,

A patient being treated with interferon for hepatitis C and hostile behavior toward^ayfhqrity figures. Chil-
complains of depression, anxiety, and irritability. W h i c h dren with conduct disorder demonstrate a repetitive
of the following pharmacological agents has the most and persistent pattern of behavior in which the basic
evidence for efficacy in treating those symptoms? rights of others and major age-appropriate societal
norms or rules are violated. Oppositional behavior js
(A) Trazodone
n o t p a r t o f . the criteria of A D H D . These behaviors do
(B) Haloperidol
not meet the criteria for antisocial personality disor-
(C) Risperidone
der, w h i c h , moreover, cannot be diagnosed in an 11-
(D) Nefazodone
(E) Sertraline year-old. These behaviors also do not fit the criteria of
intermittent explosive disorder.
The correct response is option E: Sertraline
Loeber R: Oppositional defiant and conduct disorder: a review of the
last 10 years, part I. J Am Acad Child Adolesc Psychiatry 2000;
39:1468-1484
American Psychiatric Association: Diagnostic and Statistical Manual
of Mental Disorders, Fourth Edition, Text Revision (DSM-IV-TR).
Washington, DC, American Psychiatric Association, 2000,
pp 85-102,701-706,663-667

Section 2: Answers and Explanations


'68) m

Patients with end-stage renal disease w h o are on W h i c h of the following is the most likely symptom in
hemodialysis are most likely to present with which of cocaine intoxication?
the following psychiatric symptoms?
(A) Paranoid delusions
(A) Major depression (B) Hypotension
(B) Delirium (C) Bradycardia
(C) Psychosis (D) Depersonalization
(D) Panic attacks The correct response is option A: Paranoid delusions
(E) Generalized anxiety
Cocaine intoxication can produce hypjjjension,
T h e correct response is option A: Major depression
tachycardia, seizures, paranoid delusions, a n d ^delir-
W h i l e various psychiatric symptoms can occur in ium. Depersonalization is more commonly associated
hemodialysis patients, depressioni is the most prevalent. with hallucinogen intoxication.

Rouchell AM, Pounds R.Tierney JG: Depression, in The American Mack AH, Frances RJ: Substance-related disorders. FOCUS 2003;
Psychiatric Publishing Textbook ot Consultation-Liaison 1:125-146 (p 129)'
Psychiatry: Psychiatry in the Medically III, 2nd ed. Edited by Wise American Psychiatric Association: Practice Guideline for the
MG, Rundell JR. Washington, DC, American Psychiatric Publishing, Treatment of Patients With Substance Use Disorders: Alcohol,
2002,pp 313-314 Cocaine, Opioids. Am J Psychiatry 1995; 152(Nov suppl)

A 27-year-old w o m a n has h a d five hospitalizations A consultation-liaison psychiatrist, on arriving on the


o v e r the 3-year period since she w a s initially diag- internal medicine hospital unit, learns that the patient's
nosed with schizophrenia. On each occasion, recur- nurse requested the consultation and that the attending
rent psychotic symptoms have been associated with internist, does not want the consultation. Of t h e l o l -
treatment nonadherence. Which of the following lowing, the best action for the psychiatrist would be to:
strategies is supported by the greatest b o d y of
research evidence as the most likely to improve med- (A) talk briefly with the nurse about why he or she considered
ication adherence for this patient? the consultation important.
(B) apologize to the attending internist and leave the unit.
(A) Ccrnitive-motivational interventions (C) talk with the nurse's supervisor about the correct way to
. (B) Insight-oriented psychotherapy request a consultation.
(C) Psychoeducational interventions (D) proceed with the consultation and make treatment recom-
(D) Family therapy mendations.
(E) Supportive group psychotherapy (E) ask to have a case conference about the patient with the
physician and nursing staff.
T h e correct response is option A: Cp^njtiye-motiva-
ti ojTdjntej^ve n t i on s The correct response is option A: TaIkfjefly_wijhj_he
n urse a b o j j t j j y h y j i e ^ o j ^
A review of 39 studies of psychosocial interventions
for improving medication adherence s h o w e d that p_ro-
impjDrjant
0
QIoillijhsLi^^!.?^ the qttitudinal a n d behavioral The liaison process includes c a j e j i n d i n g a n d foster-
9iJ2ecjs_oijaJ<i ng JJiedicatiojTS_ generally^ have__better infl the deyelopmentof greater psychiatric knowledge
outcomes. Psychoeducation alone a n d family therapy inn_)j}pjyc_iatric_medicqlj:are^pipviders, as opposed
alone w e r e ineffective. T h e r e is no evidence-based *2_^!I!}R'y_ providing parient carejrecojmmendations
data on insight-oriented psychotherapy. , wh^n_asked. The_discrepancy between the internist's
1
o n d thejiurse's perceptions suggests an opp^rtunity
Zygmunt A, Olfson M, Boyer CA, Mechanic D: Interventions to
fojMe^chmg. [ •
improve medication adherence in schizophrenia. Am J Psychiatry
2002;159:1653-1664
Strain J J : Liaison psychiatry, in The American Psychiatric Publishing
Mueser KT, Corrigan PW, Hilton DW, Tanzman B, Schatfb A, Gingerich
Textbook of Consultation-Liaison Psychiatry: Psychiatry in the
S, Essock SM, Tarrier N, Morey B, Vogel-Scibilia S, Herz Ml: Illness
Medically III, 2nd ed. Edited by Wise MG, Rundell JR. Washington,
management and recovery: a review of the research. Psychiatr
DC, American Psychiatric Publishing, 2002, pp 38-46
Serv 2002; 53:1272-1284. Reprinted in FOCUS 2004; 2:37-43
Dn^sjWjnaj^^^
thine brojichpdilatgrs such as theophylline a n d amino-
W h i c h of the following accurately describes the major
phylline. Because the kidney excretes lithium, a n y
quality that fundamentally distinguishes brief dynamic
medicationjhat^altersjenal funcjion_can^ajfecHih^m
psychotherapy from long-term d y n a m i c psychother-
a p y ? Brief therapy has: levels. T h i a z i d e diuretics reduce lithium clearance a n d
hence may increase lithium, levels. Certain non-
(A) no more than five sessions. sterojdd qnWnflammatpry medications, such as
(B) limited focus and goals. , ibuprofen, m a y increase lithium levels. Beta-blockers
(C) less demonstrated efficacy. , do not affect lithium levels. Angiotensin-converting
(D) no transference or countertransference phenomena. e n z y m e inhibitors may increaseJithium levels.
(E) fewer demands on the therapist.
Marangell LB, Silver JM, Goff DC, Yudofsky SC: Psychopharmacology
T h e correct response is option B: Umjtep^jo^us_and
and electroconvulsive therapy, in The American Psychiatric
goals_ Publishing Textbook of Clinical Psychiatry, 4th ed. Edited by Hales
RE, Yudofsky SC. Washington, DC, American Psychiatric
W h i l e brief therapy is intended to be shorter-term, it
Publishing, 2003, p 1109
can v a r y from one to 40 jessions, wjth^the_ayerage
d o s e to six. Brief therapy is not just a shorter version
of long-term therapy but is structured to address a spe-
cific, limited focus with_the least-radical intervention
and generally does not aim J o r character change.
A patient with an alcohol problem is ambivalent
O u t c o m e data have not demonstrated superior effi-
about starting acamprosate. The psychiatrist explores
cacy for time-unlimited therapy. Trans|erence_and
the patient's thoughts about the a d v a n t a g e s a n d dis-
countertransference o c c u r j n all therapies, although advantages of taking and not taking the medication,
they may be less emphasized in brief therapy. The attempting to tip the patient's decisional balance in
therapist requires at least equal psychotherapeutic f a y c ^ h a k i n g the medication. W h i c h of the following
skills as in classical long-term dynamic therapy and techniques is the physician using?
must be m u c h m p r e a c t i v e .
(A) Cognitive reframing
Levenson H, Bujtler SF, Powers TA, Beitman BO- Concise Guide to (B) Contingency management
Brief Dynamic and Interpersonal Therapy, 2nc ed. Washington, (C) Motivational enhancement
DC, American Psychiatric Publishing, 2002 (D) Pessimistic anticipation
(E) Rational emotion
T h e correct response is option C: Motivational
ejihan^cement

A 68-year-old man with bipolar I disorder has been Bo^Wtional"enhancenielfflthgapyyis a form of psy-
adequately maintained on lithium. His most recent chotherapy that has been shown to be effective in the
serum lithium level w a s 0.8 m E q / L . He has a variety treatment g f W q s t a n c e use disorders. I] uses directive,
of medical problems for which he takes several med- em path ic, patienNcentered techniques fQ__address
ications. He now presents with pressured speech, rac- ambivalence and d e n i a l
ing thoughts, increased energy, and little sleep. His
serum lithium level is 0.3 m E q / L . His wife reports that Mack AH, Franklin JE, Frances RJ: Substance use disorders, in The
American Psychiatric Press Textbook of Clinical Psychiatry. Edited
the patient has been adherent to his medication regi-
by Hales RE, Yudofsky SC. Washington, DC, American Psychiatric
men, but she began to notice a c h a n g e 2 weeks after
Publishing, 2003, p 353
his primary care physician started him on a n e w med-
Polcin DL, Galloway GP, Palmer J, Mains W: The case for high-dose
ication. W h a t w a s the most likely class of medication motivational enhancement therapy. Subst Use Misuse 2004;
added to his regimen? 39:331-343

(A) Angiolensin-converting enzyme inhibitors


(BJ Beta-blockers
(C) Nonsteroidal anti-inflammatory drugs
(D) Thiazide diuretics
(E) Xanthine bronchodilafors
The correct response is option E: Xanthine bron-
chodilafors

Section 2: Answers and Explanations 83


activities, with amnesia for some or all of one's past.
im ^_ Hyperarousal is described by persistent,sympJojns_of
W h i c h of the following differentiates Lewy body anxiety or increased arousal, including difficujryjalling
dementia from dementia of the Alzheimer's type? or_ staying asleep, irritability, difficulty concentrating,
(A) Apraxia hypervigilance, and exaggerated startle response.
(B) Choreiform movements
American Psychiatric Association: Diagnostic and Statistical Manual
(C) Executive dysfunction of Mental Disorders, Fourth Edition, Text Revision (DSM-IV-TR).
(D) Grodual progression of deficits Washington, DC, American Psychiatric Association, 2000,
(E) Recurrent visual hallucinations pp 464-468
Wise MG, Rundell JR (eds): Textbook of Consultation/Liaison
T h e correct response is option E: Recurrent visual hal-
Psychiatry, 2nd ed. Washington, DC, American Psychiatric
lucinations Publishing, 2002, pp 403-404

Of the core criteria that are part of the consensus cri-


teria for the diagnosis of dementia with Lewy bodies,
visual hallucinations (usually well-formed) are a par-
ticularly important finding in the differentiation. Other
c o r e criteria are fluctuaHcmjnj^ and Response prevention is a useful psychotherapeutic
spontaneous motor features of parkinsonism. In a technique for w h i c h of the following disorders?

prospective study that aimed to validate these core cri- (A) Generalized anxiety disorder
teria using neuropathology at autopsy, the sensitivity (B) Intermittent explosive disorder
a n d specificity of these clinical criteria w e r e 0 J J 3 a n d (C) Obsessive-compulsive disorder
respectively. (D) Pedophilia
(E) Schizophrenia
McKeith IG, Perry EK, Perry RH: Report of the second Dementia With
Lewy Body International Workshop: diagnosis and treatment. The correct response is option C: Obsessive-compul-
Neurology 1999; 53:902-905
sive disorder
McKe'rth IG, Ballard CG, Perry RH, Ince PG, O'Brien JT, Neill D, Lowery
K, Jaros E, Barber R, Thompson P, Swann A, Fairbaim AF, Perry Resrjonse prevention^techniques that decrease the fre-
EK: Prospective validation of consensus criteria for the diagnosis
quencyjsfjituals have been shown in several controlled
of dementia with Lewy bodies. Neurology 2000; 54:1050-1058
clinical trials to be usefuljn the treatment of patients with
obessiveoompujsiye disorder. T h e patient is prevented
from engaging in compulsive acts, such as hand wash-

xESS ing after exposure to situations that the patient consid-


ers contaminating. T h e pajienHs gra_duajly_exposed_to
T h e symptom of "flashbacks" is a manifestation of
fhe_fea,red_siruation a n d is helped, to. abstain from
w h i c h of the following psychological states?
engaging^in compulsive behavior after the exposure.
(A) Psychosis , The patient begins with the easiest situation and gradu-
(B) Fugue ally _moves toward more difficult, tasks.
(C) Hyperarousal
(D) Dissociation Hales RE, Yudofsky SC (eds): The American Psychiatric Publishing
Textbook of Clinical Psychiatry, 4th ed. Washington, DC, American
T h e correct response is option D: Dissociation Psychiatric Publishing, 2003, p 594

Fla^hbadcexj^ as^ dissocia-


tive states. Uncommonly, the individual suffering from wm
P T S D experiences flashbacks that can last from a few
A patient with alcoholism wants a psychiatrist to bill
seconds to h q u r s j ^ d a y s . During flashbacks, pdrts of the patient's insurance c o m p a n y under another diag-
the traumatic event are reexperienced and the patient nosis because the patient is afraid of the stigma
behaves as though the event w a s occurring at .mat attached to the diagnosis.,The psychiatrist should:
moment. There is no evidence of psychosis during flash-
backs associated with PTSD. Flashbacks can also result
(A) tell the patient that this would be lying and refuse to comply.
from the toxic effects of LSD a n d othe_hollucinogenic
(B) comply with the request because stigmas are inherently
unfair to patients.
ap ents. Flashbacks associated with hallucinogenic
i
(C) comply with the request provided the patient's fears are
. agents are. usually characterized by repeated psyche-
adequately addressed.
delic experiences, usually visual, a n d occur after the
(D) explore the reasons behind the request and explain why
drug use has stopped. Fugue states include sudden
this is something the psychiatrist is reluctant to do.
unexpected travel a w a y from one's home or customary
T h e correct response is option D: Explore the reasons T h e correct response is option D: Reticular activating
behind the request a n d explain w n y this is something system
the psychiatrist is rejuctant to> do
Tjiej^tjc^laj^a^^ is a collection of fibers
T h e issue of therapeutic benefit'to the patient must guide and nuclei that include the main m o n o a m m e j i u c l e j ,
the manner in which the psychiatrist works within the lim- extending from the r n ^ d y j j a _ p b l p n g a ^
its of ethics and the law. M d o ^ a j a l s e i n s ^ mus. Structures within the reticular acWwoWng system
is__bojJTjl[eg.q[^ndj^ thus options B a n d C are moduJaje_ajgusal, sleep-y^J<:e_cycles, a n d conscious
not appropriate. Option A considers only the letter of the activity. The a m y g d a l a is associated with fear, a n x i -
law without an overall consideration of how to incorpo- ety, and aggression. T h e hippocampus is associated
rate one's response into the therapeutic relationship. with memory a n d anxiety. T h e _ h y p o t h a l a m u s is
re\a\e6 to hormonal regulation, e a j i n ^ a n d _ d r i n k i n g .
Lo B: Avoiding deception and nondisclosure, in Resolving Ethical
T h e ventral striatum is associated with motivation.
Dilemmas: A Guide for Clinicians, 2nd ed. Philadelphia, Lippincott
Williams & Wilkins, 2000 (chap 6) Cummings JL, Trimble MR: Concise Guide to Neuropsychiatry and
Behavioral Neurology, 2nd ed. Washington, DC, American
Psychiatric Association, 2002, pp 28-32
Iffl Neylan TC, Reynolds CF, Kupfer DJ: Sleep disorders, in The American
Psychiatric Piess Textbook of Clinical Psychiatry. E6M by Hales
In a patient experiencing bereavement, which of the
RE, Yudofsky SC. Washington, DC, American Psychiatric
following suggests the diagnosis of major depression?
Publishing, 2003, p 976
(A) A poor appetite
(B) Initial insomnia
(C) A feeling of worthlessness
(0) Hallucinations of the deceased
(E) Sadness A 38-year-old patient provides a 12-year history of
The correct response is option C: A feeling of worth- obsessive concerns about dirt, germs, and contamina-
iessness tion and spends more than 3 hours a d a y with w a s h -
ing and cleaning rituals. W h i c h of the following w o u l d
T h e symptoms that would prompt one to consider a be preferred as an initial medication treatment?
diagnosis of major depressive episode include feelings
(A) Desipramine
of worthlessness and generalized guilt, not guilt about
(B) Duloxetine
"missed opportunities" with the deceased. T h e usual
(C) Paroxetine
signs of bereavement include feelings of sadness as (D) Phenelzine
we]l_as insomnia, poor appetite, and weight loss. The (E) Venlafaxine
bereaved patient often believes_ that__the__depressed
s r
The correct response is option C: Paroxetine
nL°°^_i _ i 9 rma
L Additional symptoms of major
depression include general_preqccupation with death The patient's history is consistent with a diagnosis of
(not wishes to die to join the deceased), psychomotor obessjye<pfnpulsive disorder. Selective serotonin
retardation, extended Junctional impairment, and hal- reuptake inhibitors (SSRIs) a r e ] h e j W e j r e ^ ^
lucinatory experience^other than about the deceased. m^njjoj^is^ondition, an<
^ fluoxetine, fluyoxamjne,

American Psychiatric Association: Diagnostic and Statistical Manual of paroxetine, a n d sertraline have FDA approval for this
Mental Disorders, Fourth Edition, Text Revision (DSM-IV-TR). Wash- indicajion. The_FrjAjias_also approved clomipramine
ington, DC, American Psychiatric Association, 2000, pp 740-741 fpr_pbsessive-compu!sive disorder, but this agent has a
more^adverse side effect profile than the SSIlls.

Jenicke MA: Obsessive-compulsive disorder. N Engl J Med 2004;


350:259-265
Soil Schruers K, Koning K, Luermans J, Haack MJ, Griez E: Obsessive-
W h i c h C N S structure is most responsible for arousal compulsive disorder: a critical review of therapeutic perspectives.
and sleep-wake cycles? Acta Psychiatr Scand 2005; 111 -.261-271

(A) Amygdala
(B) Hippocampus
(C) Hypothalamus
(D) Reticular activating system
(E) Ventral striatum

Section 2: Answers and Explanations 85


(822 83)
A 59-yeaj-ojd w o m a n is seen for an initial outpatient A psychiatrist decides that a patient with alcohol
psychiatric assessment. Her h u s b a n d says that dependence w o u l d benefit from regular laboratory
increasingly oyexjhe_p^sJ_2_years she has seemed monitoring. W h i c h of the following single tests would
Iessjjkejiej^usual_outgoingjelf. She has been increas- best provide information about h e a v y alcohol use
ingly a pathetic and uninterested in her usual activities, oyejjIiejDrecedr^^
a n d more recently she has behaved Inappropriately in
(A) Aspartate aminotransferase (AST)
social interactions, m a ^ g ^ u n j j s j j o j ^ o n i m and
(B) Carbohydrate-deficient transferrin (CDT)
rejyjrujgJ}orne_ wjth items that, do n o t b e l o n g to her.
(C) Exhaled ethanol concentration (e.g., Breathalyzer)
Recently, her husband has had to b e ^ i n j i e j p j n g j i e r
dre^ssjnjhe^morning, and he notes that she is occa-
(D) Mean corpuscular volume (MCV)
sionally inconJ|n^nJ_.oLurine. On mental status exami- The correct response is option B: Carbohydrate-defi-
nation, her affect js. blunted and KeL speech js sparse, cient transferrin (CDT)
although she does not report specific psychotic symp-
toms or changes in mood. She kngw.sjhe y e . a r a n d j h e C D T is more sensitive and specific than the other tests
se_asan but.not.the month or.date, and she has partic- listed. H o w e v e r , better results may be obtained with
ular difficulty in namjnq objects. MRI shows prominent combined tests (e.g., C D T and ga_mjpa-g[ujajny|trans-
fccmiajjajid^pmej^^ with relative spar- f e r a s e — G G T ] . Because of interpatient variability in
ing of other cortical regions. W h i c h of the following the C D T test result, the individual patient is best used
diagnoses is most likely in this patient? as_his or her o w n baseline for C D T .levels.

(A) Dementia of the Alzheimer's type Franklin JE, Leamon MH, Frances RJ: Substance-related disorders,
(B) Creutzfeldt-Jokob disease in The American Psychiatric Publishing Textbook of
(C) Dementia associated with Huntington's disease Consultation-Liaison Psychiatry, 2nd ed. Edited by Wise MG,
(D) Dementia associated with Parkinson's disease Rundell JR. Washington, DC, American Psychiatric Publishing, -
(E) Pick's d i s e a s e ' 2002, pp 417-454

T h e correct response is option E: Pick's disease

O f the diagnoses listed, o n | ) ^ j c k ^ j s e ^ s ^


thejronjdejn^^ It typically begins insid-
iousjy, with oj}set.at an earlier, age .than, dementia, of A patient with major depression shows no improve-
t h j ^ J z h e i m e j i s _ t y p e . Incontinence and.abnormalities ment after an adequate trial (in dose and duration) of
of_sp_eech gnd_la.nguage_pccur_relatively..eaHyjn the an antidepressant. T h e best next step is to:
illness course. Sympk)ms_of_iLo.ntaJ.|gbe_ dysjynctipn, (A) augment the antidepressant with thyroid hormone.
including apathy a n d s o d d l y inappropriate behav- (B) augment with lithium.
iors, q r e c p m m o n . Unlike dementia of the Alzheimer's (C) augment with both thyroid hormone ond lithium.
type, which is generally associated with more diffuse (D) switch to a different class of antidepressant.
atrophy, changes seen on MRI and single photon (E) conduct a "washout" by stopping all medication for 4
emission computed tomography in Pick's disease.are weeks, and then reassess.
more localized Jo the frontotemporal_ regions. W i t h
T h e correct response is option D: S w i t c h j c ^ a d f e r e n t
Huntington's disease and P a r k i n s o n ' s j j s e a s e , char-
class_ of cntidep_res_s_gnt
ade^tic^neurolog'ical. .findings are .prominent.
Neurological findings are also observed in If a patient shows not even a partial response despite
Creutzfeldt-Jakob disease, but the pJogressjon__pf full therapeutic doses of a particular antidepressant,
dementia is more fulminant.thani.with.Pick'sdisegse ajjgjn^njajionjsjio^recom Switching to a dif-
a n d MRI may;_SJTOW changesjnjba_sql g a n g l i a j n a d d i - ferent drug, e.itheLwithin a class (if there has not been
1
^i°JU°_^§Llbl9i9trophy. another_trial withi_ajhe_same class) or in a different
class is w a r r a n t e d . T h e .addition of psychotherapy is
Spar JE, La Rue A; Concise Guide to Geriatric Psychiatry, 3rd ed.
also an option. W a s h i n g out by suddenly stopping all
Washington, DC, American Psychiatric Publishing, 2002, p 198
medications will probably precipitate discontinuation
symptoms that will w o r s e n the patient's status.

Practice Guideline for the Treatment of Patents With Major Depressive


Disorder, 2nd ed (2000), in American Psychiatric Association
Practice Guidelines for the Treatment of Psychiatric Disorders,
Compendium 2004. Washington, DC, APA, 2004, p 455, Figure 3
Practice Guideline for the Treatment of Patients Wit!; eating
Disorders, 2nd ed (2000), in American Psychiatric Association
Of the following, w h i c h is the most common reason Practice Guidelines for the Treatment of Psychiatric Disorders,
psychiatrists are sued for malpractice? Compendium 2004. Washington, DC, APA 2004, p 702

(A) Sexual improprieties with patients


(B) Suicide
(C) Failure to obtain informed consent
(D) Tardive dyskinesia T h e use of wliich of the following has been associated
(E) Unnecessary commitment with hyperparathyroidism?

T h e correct response is option B: Suicide (A)' Lamotrigine


(B) Divalproex
Of the answer choices, s u i a d e j s j h g most common rea-
(C) lithium
son for malpractice,litigation against a psychiatrist.
(D) Topiramate
Documentation of a proper assessment with consultation
helps to provide a reasonable defense. Sexual impropri- , T h e correct response is option C: Lithium
eties are viewed most often as torts and are not usually
Uthiy^rHnaj^ and r\X9^i9919^Y'
covered by malpractice, because a law has not been
rojaW^_aje_jjnco^ buJ^/eJUsJgbjijheo^
broken. Failure to obtain informed consent, especially
e f f e d s a ^ o c i a f e d wjtJxJithiujTijherapy. Both hyperpla-
when prescribing a conventional antipsychotic that could
sia a n d - a d e n o m a s o f j h e parathyroid glands h a v e
produce tardive dyskinesia, can be a cause for litigation.
been described in association with lithium therapy.
The best protection against malpractice is a documented
i
comparison of risks versus benefits in the decision about McHenr/ CR, Lee K: Lithium therapy and disorders of the parathyroid
treatment and an indication that this comparison has glands. Endocrine Practice 1996; 2:103-109
been shared with the patient or, if the patient is incom- Abdullah H, Bliss R, Guinea Al, Deibridge L: Pathology and outcome
petent, wjth g member of the patient's family. of surgical treatment for lithium-associated hyperparathyroidism.
Br J Surg 1999; 86:91-93
Gutheil TG: Liability issues and liability prevention in suicide, in The Haden ST, Stoll AL, McCormick S, Scott J, Fuleihan G el-H: Alterations
Harvard Medical School Guide to Suicide Assessment and in parathyroid dynamics in lithium-treated subjects. J Clin
Intervention. Edited by Jacobs DG. San Francisco, Jossey-Bass, Endocrinol Metab 1997; 82:2844-2848
1999, pp 561-578, p 561
Simon Rl: A Concise Guide to Psychiatry and Law tor Clinicians, 3rd
ed. Washington, DC, American Psychiatric Publishing, 2001, p 143
® ,
Narcolepsy is characterized by which of the following
signs and symptoms?
A n o r e x i a nervosa is most commonly comorbid with
which of the following personality disorders?
(A) Daytime nonrefreshing sleep episodes
(B) Bouts of urinary incontinence
(A) Dependent (C) Early morning awakening
(B) Paranoid (D) Sleepwalking
(C) Schizotypal (E) Sudden episodes of muscle tone loss
(D) Obsessive-compulsive
The correct response is option E: Syjiderj^epispdes of
(E) Histrionic
muscle tone loss
The correct response is option D: Obsessive-compulsive
Cgtapjexy_ or sudden joss of_myscle tone, is__often
The association between personality disorders and broy^ht^orvby^.strong emotions^in-patients with nar-
other psychiatric disorders is important because of colepsy. The other three components of the classic tetrad
impljcationsjor treatment. A n o r e x i a nervosa has been of narcolepsy are bouts of sleep attacks (that are refresh-
demonstrated to be associated with obsessive-com- ing), sleep paralysis, and hypnagogic or hypnopompic
pulsive personality disorder. A n o r e x i a nervosa has hallucinations that are abnormal intrusions of REM sleep.
not been associated with dependent, paranoid,
American Psychiatric Association: Diagnostic and Statistical Manual of
schizotypal, or histrionic personality disorders. Mental Disorders, Fourth Edition, Text Revision (DSM-IV-TR). Wash-
ington, DC, American Psychiatric Association, 2CC0, pp 609-615
Wonderlich S, Mitchell JE: The role of personality in the onset of eat-
ing disorders and treatment implications. Psychiatr Clin North Am
2001;24:249-258

Section 2: Answers and Explanations


(|£2
T h e four major components of a psychodynamic v i e w A 48-year-old man with a medical history of gastro-
of personality disorders are a b j o j o g j c o J I y ^ s e d j e m - esophageal reflux disease (GERD) is referred for a
pjmment, a s e j ^ o j ^ j n t e j n ^ an psychiatric evaluation of his anxiety. For jhe_ past
ejidmingjejisj^o^elf, and: month, since the patient's initial evaluation a n d treat-
ment for G E R D , he complains of an increasing sense
(A) an assessment of reality testing. of unease, nervousness, restlessness, a n d inabjlity to
(B) o punitive superego. sji_gnd..read the paper. His medications include
(C) on intact ego ideal. 20-mg/-day of esomeprazole, 10_mg_pf jnetoclo-
(D) o specific constellation of defense mechanisms. pramide q.i.d., and 0.5 mg of l o r a z e p a m J j . d . ^ o i a H y
or as n e e d e d . He is v e r y concerned about his condi-
T h e correct response is option D: A specific constella-
tion because a sibling w h o had a similar problem
tion of defense mechanisms
died from esophageal carcinoma. O t h e r than being
T h e psychodynamic clinician views personality disor- noticeably fidgety, his mental status e x a m is unre-
ders as involving four major components: a_biologi- markable. W h a t is the most likely explanation?
cajly based temperament, a set of internalized object
(A) Development of generalized anxiety disorder
relations, an enduring sense of self, a n d a specific
(B) Adjustment disorder with anxious features
constellation of defense mechanisms. (C) Somatoform disorder not otherwise specified (i.e., "sympa-
Gabbard GO: Psychodynamic approaches to personality disorders. thy symptoms" with deceased sibling]
FOCUS 2005; 3:363-367 , (D) Akathisia from metoclopramide
(E) Benzodiazepine withdrawal
The correct response is option D: Akathisia from meto-
clopramide

The description and observation of th_ejnai)il[tyJo_be


T h e antidepressant duloxetine m a y simultaneously
improve mood a n d : still—that is, .motor restlessness—suggests,_akqthjsia
r a j h e r j h a n a _simple anxiety or. adjustment disorder.
(A) panic attacks. Metoclopramide, an aliphatic phenothtazme and a
(B) Tronic pain. cousin_of_chlorpromazine, is. the. mostJikely_culprit.
(C) flashbacks. B e n z o d i a z e p i n e withdrawal would be a second pos-
(D) psychotic symptoms.
sibility, especially if the patient took l o r a z e p a m three
(E) night terrors. r
times a d a y for 1 month and then stopped sev<. ol
"""he correct response is option B: Chromc_pain d a y s before the evaluation.

The antidepressant duloxetine is a serotonin/norepi- Stem TA, Fricchione GL, Cassem NH, Jellinek MS, Rosenbaum JF (eds):
nephrine reuptake blocker with ^dopamine. reuptake Massachusetts General Hospital Handbook of General Hospital
effects as.well. It has been shown in several 'studies to Psychiatry, 5th ed. St Louis, Mosby, 2004, chapter 13, p 260

have efficacy in major depression. Major depression


is frequently comorbid with chronic pain, often with-
ouLorganic cause. Duloxetine appears to improve
°othjjejDresjipn^ par- m

ticularly bgckqche and shpyjder pain. It is thought that A 30-year-old patient with no prior history of mental

descending norepinephrine and serotonin fibers from R health treatment presents with a major depressive
episode. W h i c h of the following elements w o u l d be the
the brain via the spinal cord serve to dampen periph-
most important in choosing a medication for treatment?
eral pain signals. IncnejJsjdjTprepi^^^^ 5-HT
"tone"_may thus simultaneously improve mood and (A) Co-occurring diagnosis of alcohol dependence in full sus-
comorbid pain. At this time, there are no studies to tained remission
support duloxetine's use in treating panic attacks, (B) Good antidepressant response lo fluoxetine in a first-
flashbacks, psychotic symptoms, or night terrors. degree relative
(C) History of a hypomanic episode
Schatzberg AF: Recent studies ot the biology and treatment of (D) Inactive hepatitis C infection
depression. FOCUS 2005; 3;14-24
[i\ Suicide attempt by aspirin overdose at age 16
T h e correct response is option C: History of a hypo-
4 manic episode
in deciding on pharmacotherapy of a major depres-
sive episode, it is most important to rule out a diaq-
Obsessive-compulsive disorder is hypothesized to
nosis of a bipolar disorder. Ini such patients, initiation
involve* a neural circuit connecting the_cortex a n d
of either lithium or lamotrigine w o u l d be a reasonable
striatum with the:
option. Particularly in more seriously depressed indi-
viduals, some clinicians initiate simultaneous' treat- (A) amygdala.
ment with lithium a n d anjintidepressgnt. In contrast to (B) hippocampus.
treatment of major depressive disorder, qntidepres- (C) hypothalamus.
sant monotherapy is not recommended for treating (D) mammillary body.

depression in patients with oipojar disorder. (E) thalamus.

The correct response is option E: Thalamus


Practice Guideline for the Treatment of Patients With Major
Depressive Disorder, 2nd ed (2000), in American Psychiatric
Brain imaging studies suggest that obsessive-compul-
Association Practice Guidelines for the Treatment of Psychiatric
Disorders, Compendium 2004. Washington, DC, APA, 2004, sive disorder involves abnormalities in a cortico-stri-
pp 477-478 atalthalamic circuit. A complementary model of
Practice Guideline for the Treatment of Patients With Bipolar Disorder, obsessive-compulsive disorder has emphasized that
2nd ed (2002), in American Psychiatric Association Practice the orbifqfronfal cortex plays a major role in the
Guidelines for the Treatment of Psychiatric Disorders, " w o r r y circuit." Data have indicated that hyperactivity
Compendium 2004. Washington, DC, APA, 2004, p 534
of the oj^itpjr^ntgUortex as well as the anterior cin-
gujajejiorjex a^mimshesjA'.ith Jreatment.

93; Stein DJ, Hollander E (eds): American Psychiatric Publishing Textbook


A 32-year-old w o m a n with bipolar I disorder has of Anxiety Disorders. Washington, DC, American Psychiatric
been adequately maintained on lamotrigine. Recently Publishing, 2002, pp 194-195
she has experienced an exacerbation of her manic
symptoms, a n d her physician elects to a d d a second
mood stabilizer. Instead of improving, the patient's
symptoms w o r s e n . Her serum lamotrigine levels are
nearly undetectable. W h a t w a s the most likely mood W h i c h of the following psychiatric disorders occurs
stabilizer that w a s added? most commonly as a comorbid disorder with a n o r e x i a
nervosa?
(A) Olanzapine
(B) Carbamazepine (A) Somatization disorder
(C) Valproate (B) Generalized anxiety disorder
(D) Topiramate (C) Major depressive disorder
(E) Lithium (D) Obsessive-compulsive disorder
(E) Social phobia
T h e correct response is option B: C a r b a m a z e p i n e
The correct Response is option C: Major_ depressive
Lamojrigine, vvhich_has _been approved for the treat-
disorder
rpjnt_of^bip_olar depression, i sjjietg bo I i z e d through .the
liver. C a r b a m a z e p i n e and oral contraceptives contain- A n o r e x i a nervosa is associated with depression in
ing ethynyl estradiol, which induce hepatic e n z y m e sys- 65%_of cases, sqcial_ phobia in 34% of cases, a n d
tems, can rapidly decrease lamotrigine levels. o b s ^ s i y ^ c o m p u l s i v e disorder in 26% of cases.
Valproate, which inhibits these e n z y m e s , could
Sadock BJ, Sadock VA: Kaplan and Sadock's Synopsis of Psychiatry,
markedly increase lamotrigine levels. O l a n z a p i n e , to- 9th ed. Philadelphia, Lippincott Williams & Wilkins, 2003, p 739
piramate, and lithium do not affect the hepatic enzyme
system involved in the metabolism of lamotrigine.

Marangell LB, Silver JM, Goff DC, Yudofsky SC: Psychopharmacology


and electroconvulsive therapy, in The American Psychiatric
Publishing Textbook of Clinical Psychiatry, 4th ed. Edited by Hales
RE, Yudofsky SC. Washington, DC, American Psychiatric
Publishing, 2003, p 1114
Schatzberg AF, Nemeroff CB (eds): The American Psychiatric
Publishing Textbook of Psychopharmacology, 3rd ed. Washington,
DC, American Psychiatric Publishing, 2004, p 624

Section 2: Answers and Explanations


96 V 97J
W h i c h of the following is the LEAST problematic for In the initial assessment, a psychiatrist is consulted by
the psychiatrist according to ethical principles? a lesbian couple seeking help for some problems in
their long-standing committed relationship. W h i c h of
l^ju- (A) A psychiatrist in a metropolitan area agrees to treat her the following is the best a p p r o a c h for the psychiatrist
financial adviser's child. to take in assessing the possibility of domestic vio-
(B) A psychiatrist in a remote area with no other psychiatrists is lence within the couple?
involved in a romantic relationship with a patient's adult
grandchild. (A) Ask about it only when material is presented that suggests
(C) A psychiatrist hires a current patient to perform clerical the problem.
work in the psychiatrist's office. (B) Ask routine questions about battering while taking the history.
(D) A psychiatrist convinces a patient who was sexually abused (C) Obtain information from collateral sources.
by a former clinician to file a suit against that former clini- (D) The topic need not be raised because domestic violence is
cian and serves as the forensic expert for the patient. low in lesbian couples.
(E) Wait until the therapy is well established before asking
T h e correct response is option B: A psychiatrist in a
remote area with no other psychiatrists is involved in a
about it.
romantic relationship with a patient's adult grandchild T h e correct response is option B: Ask_r.Qutine_gues-
tions about battering while_ta|ting the history
^ Psychiatrists have an obligation in general to avoid
^*f\9 roles that can compromise the primary fiduciary duty Domestic violence in general is underestimated, and it
;~jJLM they have to their patients as well as rojes that may is particularly likely to.be overlooked, in lesbian cou-
i ncrease the* potential for exploitation of vulnerable ples becguse_of j h e stereotype .that_batt^nng_|s_only
patients. In option B, although a romantic relationship anjpjfense^of men.against w o m e n . Couples often do
n u
with a patient's adult grandchild may create a prob- Pli^D9JL p_sppnta.neously.
lematic dual role, the psychiatrist does not have the
Cabaj RP, Stein TS: Textbook of Homosexuality and Mental Health.
option of referring the patient to another competent Washington, DC, American Psychiatric Press, 1996, pp 809-813
clinician a n d thus lacks one possible w a y of avoiding McClennen JC: Domestic violence between same-gender partners.
the dual role. In options A a n d ^ C , the psychiatrist is J Interpers Violence 2005; 20:149-154
entering into a v o i d a b l e roles that involve interests of Owen SS, Burke TW: An exploration of prevalence of domestic vio-
lence in same-sex relationships. Psychol Rep 2004; 95:129-132
the psychiatrist that could potentially conflict..with, the
interests of the patient. In option D, the psychiatrist
f e e d s to distinguish between treatment a n d advo-
c a c y , the latter m a y not serve the patient while cer-
tainly s j r y j n g j h e p j s r s p j a i o r j j r o f a s ^ or
convictions qfjh.e psychiatristjnjh is_case.

American Psychiatric Association: Opinions ot the Ethics Committee


on the Principles of Medical Ethics With Annotations Especially
Applicable to Psychiatry. Washington, DC, American Psychiatric
Association, 2001, pp 17-41 (section 2)
Simon Rl: A Concise Guide to Psychiatry and Law for Clinicians, 3rd ed.
Washington, DC, American Psychiatric Publishing, 2001, pp 26-28
98
A patient is being treated for a cat phobia. The thera- A consultation is requested for a 22-year-old man
pist encourages the patient to pass by a pet store that because of a gradual onset of behavioral symptoms
has cats in the window. From which of the following that include irritability, aggression, a n d personality
psychotherapy approaches does this strategy derive? change. Associated findings include mild jaundice,
dysarthria, a n d choreiform movements. T h e consulta-
(A) Cognitive behavior
tion-liaisorvpsychiatrisf also notices a golden-brown dis-
(B) Insight oriented
coloration of the cornea. T h e most likely diagnosis is:
(C) Interpersonal ,
(D) Short-term anxiety-regufoting (A) Huntington's disease.
(E) Supportive (B) Wilson's disease.
(Q Parkinson's disease.
T h e correct response is option A: Cognitive behavior
(D) progressive supranuclear palsy.
5pecific_pjwbras_.qre.iearsjaf JP_eciric_objects, situa- (E) adrenoleukodystrophy.
tions, or activities. T h e treatment of choice for specific
The correct response is option B: W i l s o n ' s disease
phobias is exposure, a type of cognitive behavior
therapy. T h e patient is encouraged to discus_s_the irra- Wilson^d isease, or hej^ojej^fia^g^ is
tjonajify of the phobia and.encouraged to expose him- an autosomal recessive disorder.of. c^pgejMriefaho-
or herself to the reared_object. Interpersonal psy- lism characterized by C N S a n d hepatic manifesta-
chotherapy focuses on current interpersonal problems tions. C o p p e r deposition in the c o r n e a results in the
in depressed nonbipolar, nonpsychotic individuals. telltale r_g.yser^le.ischerjjng.
Insight p s y c h o l h e r ^ y attemp]s.tp,make what is out of
Hefter H: Wilson's disease: review of pathophysiology, clinical fea-
awareness conscious, so that one can identify and tures, and drug treatment. CNS Drugs 1994; 2:26-39
work through patterns of behavior derived from child-
hood. Suppqrtiye_ psychotherapy emphasizes external
evejTts_ajTdJs^ directed t o w a r d helping patients return
to_their_preyjo^s bestJevel_ofJunctioning. Short-term ]01
anxiety-regulating p s y c h o t h e r a p y uses psychody- According to DSM-IV-TR, a patient with recurrent h y p o -
namic principles and techniques to effect change. manic episodes without intercurrent depressive fea-
tures would receive which of the following diagnoses?
Hales RE, Yudofsky SC (eds): The American Psychiatric Publishing .
Textbook of Clinical Psychiatry, 4tti ed. Washington, DC, American (A) Bipolar I disorder
Psychiatric Publish.^, 2003, pp 581,1177-1198 (B) Bipolar II disorder .
(C) Cyclothymic disorder
(D) Bipolar disorder, not otherwise specified

The correct response is option D: Bipolar disorder, not


otherwise specified
W h i c h of the following cognitive functions is most
likely to remain stable with normal aging? Recurrent hypomania in the absence of depressive
periods w o u l d be classified as bipolar disorder not
(A) Language syntax
otherwise specified. According to DSM-IV-TR, a diag-
(B) Recent memory
nosis of bipolar I disorder requires at least one manic
(C) Speed of information processing
(0) Topographic orientation or mixed episode; a diagnosis of bipolar II disorder
(E) Working memory requires recurrent major depressive episodes with
hypomanic episodes; and a diagnosis of cyclothymic
T h e correct response is option A: Language syntax_ disorder requires periods of h y p o m a n i c symptoms
Syntax, vocabulary, communication, and store of and periods of depressive symptoms.
knowledge fend_ to.remain. stable with normal aging, American Psychiatric Association: Diagnostic and Statistical Manual of
but the other functions listed tend to decline with a g e . Mental Disorders, Fourth Edition, Text Revision (DSM-IV-TR).
Washington, DC, American Psychiatric Association, 2000,
Spar JE, La Rue A: Concise Guide to Geriatric Psychiatry, 3rd ed.
pp 382-400
Washington, DC, American Psychiatric Publishing, 2002, pp 25-25

Section 2: Answers md Explanations


Ritonavir induces the enzyme that metabolizes
102
lorazepam and some other benzodiazepines
T h e ventral tegmentum, the nucleus accumbens, and ( o x a z e p a m and temazepam] that rely on glucuronyl
the prefrontal cortex are brain structures or regions
transferase activity for clearance. Some other benzo-
most involved in the neurobiology of:
diazepines (e.g., midazolam) are dependent on C Y P
(A) alcohol dependence. 3 A 4 for metabolism. Potent inhibitors of this C Y P iso-
(B) anorexia nervosa. form, such as protease inhibitors, can decrease clear-
(C) bipolar disorder. ance of these drugs and result in increased sedation.
(D) panic disorder. Therefore, lorazepam remains a g o o d clinical choice
(E) schizophrenia. for short-term use in patients w h o need treatment for

T h e correct response is option A: Alcohol depen- panic disorder w h o must also take ritonavir for treat-

dence ment of H I V infection.

Dopaminergic and glutaminergic circuits in the Although the other options listed cannot be absolutely
tegmentum, accumbens, and prefrontal cortex are excluded, they are not as likely as the effect of rito-
necessary in producing pleasure from drug use, in the navir. A l s o , modern combination therapy in compliant
development of addiction, a n d in the maintenance of patients tends to be quite effective in preventing sec-
drug craving, salience, a n d impaired control over o n d a r y infections or lesions of HIV.
use. T h e a m y g d a l a plays a more central role in anxi-
Practice Guideline for the Treatment of Patients With HIV/AIDS
ety disorders. The cnterior cingulate gyr us, the thala-
(2000), in American Psychiatric Association Practice Guidelines
mus, the cerebellum, and the temporal lobe regions for the Treatment of Psychiatric Disorders, Compendium 2004.
a r e involved in schizophrenia. The hypothalamus has Washington, DC, APA, 2004, p 200
been suggested as a site of dysfunction in a n o r e x i a . Hsu A, Frenneman GR, Bertz RJ: Ritonavir: clinical pharmacokinetics
and interactions with other anfi-HW agents. Clin Pharmacokinet
A w i d e range of structures and regions have been
1998; 35:275-291
studied in the neurobiology of bipolar disorder.
Fernandez F: Ten myths about HIV infection and AIDS. FOCUS 2005;
3:184-193
Hyman SE: Addiction: a disease of learning and memory. Am J
Psychiatry 2005;162:1414-1422
Miller LA, Taber KH, Gabbard GO, Hurley RA: Neural underpinnings of
fear and its modulation: implications for anxiety disorders.
J Neuropsychiatry Clin Neurosci 2005; 17:1-6 104
Hales RE, Yudofsky SC (eds): The American Psychiatric Publishing
Textbook of Clinical Psychiatry, 4th ed. Wo.nington, DC, American A 24-year-old man w h o lives with his parents is being
Psychiatric Publishing, 2003, pp 474,1005,401 _ treated for schizophrenia in a continuing d a y treat-
ment p r o g r a m . Since the onset of his illness at a g e
20, he has had three hospitalizations for recurrent
psychosis. He is currently on quetiapine 300 mg
103 , b.i.d., a n d his auditory hallucinations have resolved,
but he still has some concerns that a government con-
A 32-year-old man with panic disorder treated with spiracy m a y be operating and spying on him. Apart
l o r a z e p a m for several years begins combination ther- from his family and the d a y treatment p r o g r a m , he
a p y (which includes ritonavir) for H I V infection. T w o has few interactions with others a n d no outside inter-
w e e k s later, his panic attacks increase in frequency. ests. If family therapy w e r e instituted with this patient's
W h a t is the most likely explanation? parents, w h i c h of the following outcomes would be
most likely to be observed?
(A) An HIV-related brainstem lesion
(B) An HIV-related lung infection
(A) Improved employobility
(C) A direct side effect of one of his HIV medications
(B) Improved social functioning
(D) Ritonavir is decreasing blood lorazepam levels
(C) Reduced likelihood of psychotic relapse and ^hospitalization
(E) Failure to take lorazepam as directed
(D) Reduced number md severity of negative symptoms
T h e correct response is option D: Ritonavir is decreas- (E) Reduced number and severity of positive symptoms
ing blood l o r a z e p a m levels
T h e correct response is option C: Reduced likelihood
of psychotic relapse and rehospifalizafion

Like most of the psychosocial treatments for schizo-


phrenia, family therapy results in i m p r o v e d outcomes
in important but discrete areas. Although studies of
family interventions have used v a r y i n g approaches to
treatment, all effective family interventions include 106
education about the illness a n d its course, training in
C o m p a r e d with y o u n g e r adults, the elderly require
coping a n d problem-solving skills within the family,
lower dpses of lithium to achieve a g i v e n serum lithium
improved communication, a n d stress reduction. By
concentration because of:
teaching practical educative a n d behavioral methods
in highly structured programs that last 9 months to sev- (A) impaired hepatic metabolism..
eral y e a r s , these interventions are designed to elicit (B) more complete absorption.
family participation a n d collaboration in treatment (C) reduced fat storage.
planning, g o a l setting, and, service delivery. T h e y are (D) reduced renal excretion.
also intended to complement and encourage the use (E) reduced serum protein binding.
of other treatments, such as having the patient adhere T h e correct response is option D: Reduced renal
to a medication regimen, a n d to embed the psychia- excrefjon
trist's care within a multidisciplinary team approach to
the patient and family. Meta-analyses a n d systematic Lithium is a water-soluble element that is not metabo-

reviews of such family programs have consistently lized a n d has no meaningful protein binding. There is

shown reduced family burden and reductions in no evidence that drug absorption is more efficient in

relapse rates, which are typically halved by structured the elderly, a n d the slight decreases in absorptive abil-

family interventions compared with control treatments. ities with a d v a n c e d a g e are not thought to be clini-
cally meaningful. Lithium is excreted unchanged
Bustillo JR, Laurisllo J, Horan WP, Keith SJ: The psychosocial treat- almost entirely by the kidneys. Because there is a ten-
ment of schizophrenia: an update. Am J Psychiatry 2001; dency for the glomerular filtration rate to decrease
158:163-175
with a g e , excretion of lithium becomes less efficient.
American Psychiatric Association: Practice Guideline for the
Treatment of Patients With Schizophrenia, 2nd ed. Am J Sproule BA, Hardy BG, Shulman Kl: Differential pharmacokinetics of
Psychiatry 2004; 161 (Feb suppl):1-56 lithium in elderly patients. Drugs Aging 2000; 16:165-177
Jefferson JW: Genitourinary system effects of psychotropic drugs, in
Adverse Effects of Psychotropic Drugs. Edited by Kane JM,
Lieberman JA. New York, Guilford, 1992, pp 431-444
Iber FL, Murphy PA Connor ES: Age-related changes in the gastroin-
105 .
testinal system. Drugs Aging 1994; 5:34-48
Biological relatives of individuals with antisocial per- Gitlin M: Lithium and the kidney: an updated review. Drug Saf 1999;
sonality disorder have an increased risk of having anti- 20:231-233
social personality disorder a n d substance-related
disorders. These relatives, especially if they are femde,
are also at greater risk of:

(A) autism.
(B) narcissistic personality disorder.
(C) bipolar disorder.
(D) schizophrenia.
(E) somatization disorder.
The correct response is option E: Somatization disorder

Family members of individuals with antisocial person-


ality disorder have an increased risk of having soma-
tization disorder. This is especially true for females,
although the rate of this disorder is also higher among
male family members than in the general population.
There is no association between antisocial personality
disorder a n d autism, schizophrenia, narcissistic per-
sonality disorder, or bipolar disorder.

American Psychiatric Association: Diagnostic and Statistical Manual


of Mental Disorders, Fourth Edition, Text Revision (DSM-IV-TR).
Washington, DC, American Psychiatric Association, 2000, p 704

Scccion 2: Answers and Explanations


Although there may be elements of each of these
107
options contributing to the patient's lack of distress,
W h i c h of the following is the best description of the
the one term that best describes this phenomenon is
thercpist's empathy?
"ego-syntonic." Personality disorder symptoms are
(A) Envisioning what it would be like for the therapist to be in described as alloplastic (i.e., able to adapt to, and
the patient's situation alter, the external environment] and ego-syntonic (i.e.,
(B) Mirroring the patient's presentations of a vulnerable self acceptable to the e g o ) . Because individuals with per-
(C) Understanding the patient's inner experience from the sonality disorders do not find their behaviors distress-
patient's perspective ing, these individuals often seem uninterested in
(D) Maintaining an attitude of compossion ond sympathy treatment.
(E) Avoiding making the patient onxious or uncomfortable
Sadock BJ, Sadock VA: Kaplan and Sadock's Synopsis of Psychiatry,
T h e correct response is option C: Grasping the patient's 9th ed. Philadelphia, Lippincott Williams & Wilkins, 2003, p 800
inner experience from the patient's perspective

W h e n a therapist empathizes, he or she understands


the patient's feelings without getting involved in them.
109
T h e empathic response would be to imagine thinking
a n d feeling from the patient's level of insight. Empathy, An 18-year-old female patient w h o is being evaluated
a critical skill for psychotherapy, may be confused with for depression reveals that she w o r r i e s excessively
sympathy, or just being nice, or avoiding anything that about her weight. She states that she is unable to diet
and consumes large quantities of food about once a
the patient dislikes. A common source of error is the
month. She appears to have normal w e i g h t for her
"almost right" notion of imagining h o w one would feel
height. W h a t is the most likely diagnosis?
if one w e r e in the patient's shoes.
(A) Anorexia nervosa
Mohl PC, Warrick GD: Listening to the patient in Psychiatry. Edited by
Tasman A, Kay J, Lieberman JA. Philadelphia, WB Saunders,
(B) Body dysmorphic disorder
1997, p 9 (C) Bulimia nervosa
(D) Eating disorder not otherwise specified
(E) Factitious disorder
The correct response is option D: Eating disorder not
108 otherwise specified
A 39-year-old secretary must do everything meticu-
The ec-Kng disorder not otherwise specified category
lously. H e r w o r k a r e a is extremely neat and organ-
is for disorders of eating that do not fully meet the cri-
i z e d . H o w e v e r , she is not v e r y productive, because
she will restart a n y project if she makes an error. She teria for a specific eating disorder. This patient
typically works through lunch and rarely socializes appears to have normal w e i g h t a n d thus does not
with her coworkers. At home, she is in constant con- meet the criteria for a n o r e x i a nervosa. She describes
flict with her children about the tidiness of their rooms, binges, but they do not occur frequently e n o u g h to
the neatness of their schoolwork, a n d the need to be meet the criteria for bulimia n e r v o s a , w h i c h is on aver-
frugal. H e r children and coworkers tell her that her age at least twice a w e e k .
behaviors "drive them nuts." She does not believe she
has a problem a n d in fact thinks her habits represent American Psychiatric Association: Diagnostic and Statistical Manual
"strong moral values." W h i c h term best describes the of Mental Disorders, Fourth Edition, Text Revision (DSM-IV-TR).
Washington, DC, American Psychiatric Association, 2000,
w o m a n ' s lack of distress about her problems?
pp 583-595
(A) Ambivalence
(B) Denial
(C) Ego-syntonic
(D) La belle indifference
(E) Projection
T h e correct response is option C: Ego-syntonic

94 FOCUS Psychiatry Review: 400 Self-Assessment Questions


no 112
Patients with bulimia nervosa who engage in Early-onset A l z h e i m e r ' s dementia due to mutations in
b i n g e / p u r g e behaviors a r e at risk for which of the fol- the amyloid precursor protein genes, presenilin-1
lowing medical disorders? a n d presenilin-2, a r e transmitted by w h a t mode of
inheritance?
(A) Hyperkalemia . .
(B) Decreased serum amylase (A) Autosomal dominant
(C) Cardiomyopathy (B) Autosomal recessive
(D) Hypothyroidism (C) X-linked
(E) Osteopenia \ . (D) Trinucleotide repeat
(E) Polygenic
T h e correct response is option C: C a r d i o m y o p a t h y
The correct response is option A: Autosomal dominant
In patients with bulimia nervosa, cardiomyopathy as
a result of ipecac intoxication may occur a n d usually Three genes h a v e been associated with early-onset
results in death. Patients w h o binge and vomit may Alzheimer's dementia: the ^-amyloid precursor pro-
have parotid gland enlargement associated with ele- tein gene (APP) on chromosome 2 1 , the presenilin-1
vated serum amylase levels. T h e y are susceptible to (PS1) gene on chromosome 14, and the presenilin-2
hypokalemic alkalosis. Unlike patients with anorexia (PS2) gene on chromosome 1. All three missense
nervosa, those with bulimia do not have a high risk of mutations are autosomal dominantly inherited a n d
osteopenia. together account for only about 5% of all cases of
Alzheimer's dementia.
Hales RE, Yudofsky SC (eds): The American Psychiatric Publishing
Textbook of Clinical Psychiatry, 4th ed. Washington, DC, American Stern TA, Herman JB: Massachusetts General Hospital Psychiatry
Psychiatric Publishing, 2003, pp 1011-1012 Update and Board Preparation, 2nd ed. New York, McGraw-Hill,
Rushing JM, Jones LE, Carney CP: Bulimia nervosa: a primary care 2004, p 494
review. Prim Care Companion J Clin Psychiatry 2003; 5:217-224 Blazer DG, Steffens DC, Bausse EW (eds): The American Psychiatric
Publishing Textbook of Geriatric Psychiatry, 3rd ed. Washington,
DC, American Psychiatric Publishing, 2004, pp 110-111

ni
A 76-year-old w o m a n presents with weakness, fatigue,
somnolence, and depression. Her husband has also
noticed that there has been some cognitive slowing
and her voice is hoarse. W h i c h of the following
endocrine disorders is the most likely diagnosis?

(A) Cushing's disease


(B) Hyperparathyroidism
(0 Hypoparathyroidism
(D) Hypothyroidism
(E) Pheochromocytoma
The correct response is option D: Hypothyroidism

Hypothyroidism presents with fatigue, somnolence,


weakness, d r y skin, brittle hair, cold intolerance, and
hoarse speech. Depression is common, and cognitive
slowing can occur. Hypoparathyroidism a n d hyper-
parathyroidism can both result in anxiety, irritability,
or depression. Hyperparathyroidism is commonly
accompanied by weakness and a n o r e x i a , w h e r e a s
hypoparathyroidism has mainly neuromuscular signs
such as spasms, tetany, a n d hyperreflexia. Pheo-
chromocytoma results in palpitations, panic attacks,
headaches, and hypertension.

Sadock BJ, Sadock VA (eds): Kaplan and Sadock's Comprehensive


Textbook of Psychiatry, 8th ed. Philadelphia, Lippincott Williams &
Wilkins, 2005, pp 2154-2155

Section 2: Answers and Explanations


The correct response is option E: Phencydidine
113
A 27-year-old man has a long-standing history of The patient's presentation is consistent with phencydi-
marked discomfort in social situations a n d avoids dine (PCP) intoxication. Hallucinogens such as LSD
g r o u p discussions, parties, dating, a n d speaking at and psilocybin do not cause a t a x i a , dysarthria, and
meetings. He also has a history of binge alcohol use, nystagmus. W h i l e miosis is characteristic of heroin
particularly w h e n he has to engage in social activi- intoxication, hypertension a n d agitation are not.
ties. T h e class of medication preferred for treatment of Cannabis-induced psychosis is not characterized by
this patient would be:
miosis or the above-mentioned neurological findings.
(A) benzodiazepines. Moore DP, Jefferson JW: Substance use and related disorders, in
(B) beta-blockers. Handbook of Medical Psychiatry, 2nd ed. Philadelphia, Elsevier
(C) tricyclics. Mosby, 2004, pp 61-44
(D) second-generation antipsychotic. McCarron MM, Schulze BW, Thompson GA, Conder MC, Goetz WA:
(E) selective serotonin reuptake inhibitors. Acute phencydidine intoxication: incidence of clinical findings in
1,000 cases. Ann Emerg Med 1981; 10:237-242
T h e correct response is option E: Selective serotonin
reuptake inhibitors

T h e patient's history is consistent with a diagnosis of


social anxiety disorder, generalized type. W h i l e beta-
115
blockers may be useful prior to occasional public A 35-year-old nurse is admitted to the medical service
speaking events, they are not effective f o j generalized with numerous ecchymoses on her b o d y a n d a com-
social anxiety disorder. Selective serotonin reuptake plaint of tarry stools. H e r prothrombin time w a s 4 INR
inhibitors (SSRIs) are well-established, effective treat- (international normalized prothrombin ratio) units
(normal, 0 . 7 8 - 1 . 2 2 ) . Several days after admission
ments. Sertraline and paroxetine (as well as the non-
her prothrombin time w a s normal. A medical w o r k u p
• SSRI venlafaxine) are FDA-approved for the treatment
failed to identify the cause of her abnormal clotting
of social anxiety disorder. There have been some stud-
time. Her stool w a s w e a k l y positive for blood. Four
ies showing benefit with benzodiazepines, but these
days after admission, more ecchymoses a p p e a r e d
agents are not preferred for long-term use a n d none
and her prothrombin time w a s a g a i n elevated. The
a r e F D A - a p p r o v e d for this indication. Particularly in patient e x p r e s s e d c o n c e r n that she might have
light of this patient's history of binge drinking, benzo- leukemia a n d inquired if she w o u l d need a bone mar-
d i a z e p i n e use w o u l d be a relative contraindication. A r o w biopsy. On the fifth d a y of admission, a w a r f a r i n
v e r y small double-blind placebo-controlled study of pill w a s found beneath her b e d . T h e patieiv signed
o l a n z a p i n e had promising results, but additional stud- out of the hospital that evening. W h i c h of the follow-
ies are necessary. Tricyclics have not been shown to ing is the most likely diagnosis?
be of benefit for social anxiety disorder.
(A) Somatization disorder
Bianco C, Raza MS, Schneier FR, Liebowitz MR: The evidence-based (B) Malingering
pharmacological treatment of social anxiety disorder. Int J (C) Hypochondriasis
Neuropsychopharmacol 2003; 6:427-442 (D) Factitious disorder
Bamett SD, Kramer ML, Casat CD, Connor KM, Davidson JR: Efficacy (E) Body dysmorphic disorder
of olanzapine in social anxiety disorder: a pilot study.
J Psychopharmacol 2002; 16:365-368 The correct response is option D: Factitious disorder

Factitious disorder is the most likely diagnosis for this


114 patient because the negative w o r k u p a n d the discov-
e r y of the w a r f a r i n pill beneath the bed strongly sug-
T h e consultation-liaison psychiatrist is called to the
gest that her bleeding problems w e r e self-induced.
e m e r g e n c y department to evaluate a 17-year-old
Such patients h a v e a great need to be taken care of
patient w h o is highly agitated and floridly "psychotic
by physicians a n d e v e n undergo serious procedures,
with findings of ataxia, nystagmus, dysarthria, miosis,
and elevated blood pressure. Intoxication with which of such as a b o n e m a r r o w bibpsy. T h e diagnosis is often
the following substances best explains this presentation? elusive unless evidence is found that the illness w a s
self-inflicted. No clear motive for the behavior is pres-
(A) Heroin ent; the motive most often identified is a desire to
(B) Psilocybin assume the sick role. In malingering, by contrast, an
(C) Cannabis individual presents with d disability claim or an illness
(D) LSD .
motivated by a g o a l of getting out of prison, collect-
(E) Phencydidine
ing insurance m o n e y , or some other s e c o n d a r y g a i n .
Somafizaiion disorder is a polysystem disorder that is 117
characterized by a combination of pain, gastrointesti-
Cocaine-induced euphoria is most highly associated
n a l , s e x u a l , a n d p s y c h o n e u r o l o g i c a l symptoms.
with which of the following neurotransmitters?
Hypochondriasis is the chronic fear that one has a
serious illness. T h e anxiety may be generated by an (A) Serotonin
e x a g g e r a t i o n of an actual mild illness or concerns in (B) Dopamine
the absence of medical causes. Body dysmorphic dis- (C) Norepinephrine
order is characterized by a significant preoccupation (D) Gammfl-aminobutyric acid
with imagined or e x a g g e r a t e d physical defects. (E) Acetylcholine
The correct response is option B: Dopamine
American Psychiatric Association: Diagnostic and Statistical Manual
of Mental Disorders, Fourth Edition, Text Revision (DSM-IV-TR).
Cocaine is" known to inhibit dopamine reuptake a n d
Washington, DC, American Psychiatric Association, 2000,
DO 471-475 increase extracellular dopamine concentration. These
effects, which occur in the nucleus accumbens, a r e
considered to be related to cocaine-induced eupho-
ria. Cocaine also blocks the reuptake of norepineph-
116 rine and serotonin, although the behavioral effects a r e
mediated primarily by the dopaminergic system. T h e
Patients w h o suffer from depression after a myocar-
inhibitory neurotransmitter gamma-aminobutyric acid
dial infarction should be treated with which of the fol-
lowing antidepressants? also interacts with d o p a m i n e neurons in the nucleus
accumbens and the ventral tegmental a r e a . T h e bind-
(A) A monoamine oxidase inhibitor ing of cocaine to the dopamine transporter correlates
(B) Bupropion best with its behavioral potency.
(C) Trazodone
(D) A tricyclic antidepressant Withers NW, Pulvirenti L, Koob GF, Gillin JC: Cocaine abuse and
(E) An SSRI dependence. J Clin Psychopharmacology 1995; 15:63-78
Dackis CA, O'Brien CP: Cocaine dependence: a disease of the brain's
T h e correct response is option E: An SSRI reward centers. J Subst Abuse Treat 2001; 21:111—117

Depression occurs in approximately 20% of patients


w h o h a v e had a myocardial infarction. Moreover, the
mortality rate is much higher among posf-myocardial
infarction patients w h o h a v e depression. Several stud-
ies have established that the SSRIs constitute the safest
antidepressants to use in such circumstances. T h e y
h a v e little effect on conduction time and do not cause
orthostatic hypotension. Various tricyclic antidepres-
sants are not ideal because they may cause orthosta-
tic hypotension, anticholinergic side effects, and
effects on conduction. Bupropion has been implicated
in hypertension in some patients. T r a z o d o n e presents
problems of alpha-adrenergic blockade and postural
hypotension. Finally, the monoamine oxidase
inhibitors may cause a hypertensive response w h e n
certain foods are eaten, and they have orthostatic
hypotension as a side effect as well.

Wise MG, Rundell JR (eds): The American Psychiatric Publishing


Textbook of Consultation-Liaison Psychiatry: Psychiatry in the
Medically III, 2nd ed. Washington, DC, American Psychiatric
Publishing, 2002, pp 542-545
Shulman J, Muskin P, Shapiro PA: Psychiatry and cardiovascular dis-
ease. FOCUS 2005; 2:208-224

Section 2: Answers and Explanations


V18 120
W h i c h of the following is an example of an instru- W h i c h of the following features differentiates delirium
mental activity of daily living that becomes impaired from dementia of the Alzheimer's type?
in the mild to moderate stages of dementia?
(A) Acuity of onset and level of consciousness
(A) Ambulating (B) Level of consciousness and orientation
(B) Dressing (C) Acuity of onset and orientation
(C) Feeding oneself (D) Visual hallucinations and memory
(D) Remembering appointments (E) Memory and level of consciousness
(E) Toileting
The correct response is option A: Acuity of onset and
T h e correct response is option D: Remembering level of consciousness
appointments
Delirium is often confused with dementia or functional
Instrumental activities of daily living (lADLs) include psychiatric disorders in elderly patients. Clinical fea-
more complex daily tasks such as managing finances tures help in differentiating between delirium and
(e.g., writing checks), g r o c e r y shopping, preparing dementia. Patients with delirium have an acute onset
meals, keeping track of current events, remembering and exhibit fluctuation in the level of consciousness,
appointments, managing medications, using the cognition, and clinical symptoms, whereas patients
phone, and traveling (e.g., taking the bus). A person's with Alzheimer's dementia tend to have an insidious
ability to manage these activities independently gen- onset and an alert and stable level of consciousness,
erally becomes impaired in the mild *to moderate and cognitive and symptom fluctuation are infrequent.
stages of dementia. As dementia progresses, the more Orientation a n d memory are impaired in both disor-
basic activities of daily living (ADLs) become impaired, ders. Visual hallucinations are frequent in delirium
a n d most people need assistance with them. ADLs and occur only occasionally in Alzheimer's dementia.
include feeding, dressing, toileting, grooming, physi- E E G shows marked slowing in patients with delirium
cal ambulation and transferring, and bathing. and either normal or mild slowing in patients with
Alzheimer's dementia.
Coffey CE, Cummings JL American Psychiatric Press Textbook of
6eriatr c Neuropsychiatry, 2nd ed. Washington, DC, American
:

Cole MG: Delirium in elderly patients. Am J Geriatr Psychiatry 2004;


Psychiai.ic Press, 2000, p 26 2:1,7-21
Ajilore OA, Kumar A: Delirium and dementia. FOCUS 2004;
2:210-220

119
A 66-year-old patient w h o is being treated for bipolar
disorder presents comatose with a serum sodium con- 121
centration of 112 mmol/L. W h i c h of the following is W h i c h of the following sleep disorders is more com-
most likely to be the cause of the sodium imbalance? mon in males than females during childhood?

(A) Divalproex (A) Breathing-related sleep disorder


(B) Carbamazepine (B) Nightmare disorder
(C) b'thium (C) Primary insomnia
(D) Olanzapine (D) Sleep terror disorder
(E) Sleepwalking disorder
T h e correct response is option B: C a r b a m a z e p i n e
The correct response is option D: Sleep terror disorder
Hyponatremia can be an adverse reaction of carba-
m a z e p i n e . Hyponatremic coma has been attributed to O n l y option D is correct. T h e other disorders either
the drug. If lithium use is associated with dehydration, have no g e n d e r differentiation or are more common
hypernatremia m a y be a complication. in females. • •
i
i
Schatzberg AF, Nemeroff CB (eds): American Psychiatric Publishing
Textbook of Psychopharmacology, 3rd ed. Washington, DC, American Psychiatric Association; Diagnostic and Statistical Manual
American Psychiatric Publishing, 2004, pp 591-592 of Mental Disorders, Fourth Edition, Text Revision (DSM-IV-TR).
Washington, DC, American Psychiatric Association, 2000,
pp 599-604,615-622,631-644

98 n
F O C I ft -.--hiatrv Review: 400 Self-Assessment Questions
122 123
A physician elects to treat a depressed patient with Soon after E C T , a patient is most likely to have prob-
imipramine. Four days after the start of treatment, the lems with w h i c h of the following items on the Mini-
physician receives a call from the emergency depart- Mentaf State Examination?
ment reporting that the patient has fallen. The staff
report that the patient stood up quickly after being in
(A) Reporting the date
bed overnight, felt d i z z y , a n d then lost consciousness,
(B) Spelling "WORLD" backwards
falling to the floor. Examination reveals a pulse of 76
(C) Repeating "no ifs ands or btrts*
b p m ; blood pressure is 1 3 6 / 8 2 mm Hg lying and
(0} Following a three-step command
1
8 4 / 4 6 mm Hg standing'. An electrocardiogram is
(E) Writing a sentence
unremarkable. W h i c h of the following best explains The correct response is option A: Reporting the date
the patient's symptoms?
E C T <can cause a retrograde amnesia. W h i l e some
(A) a-Adrenergic receptor blockade anterograde m e m o r y impairment may be present, it
(B) Cholinergic receptor blockade can be difficult to separate from the impairments
(C) Histamine receptor blockade
brought on by depression itself. Following a treat-
(D) First-degree atrioventricular block
ment, postictal/postanesthesia confusion is often pres-
(E) Prolongation of the QTc interval
ent but generally resolves within several hours. Of the
T h e correct response is option A: a-Adrenergic recep- Mini-Mental State Examination components listed,
tor blockade reporting the date is the only one that potentially relies
on material learned in the hours prior to a treatment.
The tricyclic antidepressants block peripheral alpha-
adrenergic receptors, delaying the reflexive constric- Practice Guideline for the Treatment of Patients With Major Depres-
tion of peripheral blood vessels w h e n a patient goes sive Disorder, 2nd ed (2000), in American Psychiatric Association
from lying to standing, and through this mechanism Practice Guidelines for the Treatment of Psychiatric Disorders,
Compendium 2004. Washington, DC, APA, 2004, pp 495-496
induce orthostatic hypotension. In this particular
UK ECT Review Group: Efficacy and safety of electroconvulsive ther-
vignette, the patient has a period of loss of conscious- apy in depressive disorders: a systematic review and meta-analy-
ness on rising and objective evidence of orthostatic sis. Lancet 2003; 361(9360):799-808
hypotension in light of a normal E C G . The signs, symp- American Psychiatric Association: The Practice of Electroconvulsive
toms, a n d E C G suggest that the patient's fall is sec- Therapy: Recommendations for Treatment Training, and
Privileging: A Task Force Report of the American Psychiatric
o n d a r y to orthostatic hypotension. T h e tricyclic
Association, 2nd ed. Washington, DC, APA, 2001
antidepressants may cause a variety of side effects,
including 'anticholinergic, cardiovascular, and central
nervous systerrTeffects. Anticholinergic effects include
d r y mouth, constipation, urinary hesitancy, a n d 124
blurred vision. Antihistaminic effects include sedation
Imaging genetics is a form of:
and weight gain. Cardiovascular effects tend to be the
most worrisome. All tricyclics prolong cardiac conduc- (A) association study.
tion, much like quinidine or procainamide, and carry (B) double-blind study.
the risk of exacerbating existing conduction abnor- (C) linkage study.
malities, such as first-degree atrioventricular block. (D) randomized study.
The correct response is option A: Association study
Schatzberg AF, Cole JO, DeBattista C: Manual of Clinical
Psychopharmacology. Washington, DC, American Psychiatric
An association study looks for a statistically significant
Publishing, 2005, pp 102,114-115
link between two variables in comparison with a con-
Andreasen NC, Black DW: Introductory Textbook of Psychiatry, 3rd
ed. Washington, DC, American Psychiatric Press, 2001, p 725 trol. Imaging genetics uses neuroimaging m e t h o d s -
structural M R I , positron emission tomography (PET),
functional MRI (fMRI), a n d magnetic resonance spec-
troscopy (MRS)—to assess the impact of genetic vari-
ation on the human brain in order to find aspects of
brain function or structure that can be examined in
association with genetic variations across individuals.

Kempf K, Meyer-Lindenberg A: Imaging genetics and psychiatry.


FOCUS 2006; 4

Section 2: Answers and Explanations


The patient is experiencing a treatment-resistant
125 episode of depression that has accompanying suicidal
W h i c h of the following is the most appropriate indi-
ideation, somatic delusions, severe psychomotor retar-
cation for E C T in a patient with borderline personality
dation, and probable dehydration. These clinical fea-
disorder?
tures make this patient a candidate for ECT, which has
(A) Comorbid major depression been shown to be an effective treatment for patients
(B) Severe mood instability with severe major depression. E C T is typically recom-
(C) Poor response to valproate mended for depressed patients with severe symptoms,
(D) Noncompliance with medications including psychosis, marked suicidal intent, and refusal
(E) Recurrent transient psychotic episodes to eat. G i v e n this patient's history of nonresponse to, or

T h e correct response is option A: C o m o r b i d major inability to tolerate, multiple antidepressants during the

depression current episode, the likelihood that her depression will


respond to ECT is significantly greater than the likeli-
T h e goal of E C T in patients with borderline personal- hood that it will respond to an increased dose of ven-
ity disorder is to decrease depressive symptoms in lafaxine or to a change to a different antidepressant
individuals with a comorbid axis I m o o d disorder. (e.g., mirtazapine) or augmentation with an anticon-
Although E C T is not a recommended treatment for vulsant (e.g., lamotrigine).
borderline personality disorder per se, it can be use-
ful in treating comorbid major depression. T h e deci- Although an occult carcinoma might also explain this
sion to use E C T in this patient g r o u p should be guided patient's loss of weight, abdominal computerized
by the neurovegetative symptoms more M a n the psy- tomography has not shown any hepatic abnormality,
chological symptoms of depression, w h i c h are chron- so obtaining a liver scan is not likely to be informative.
ically present in m a n y persons with borderline
Sadock BJ, Sadock VA (eds): Kaplan and Sadock's Comprehensive
personality disorder. There is, unfortunately, little
Textbook of Psychiatry, 7th ed. Philadelphia, Lippincott Williams &
research specifically testing E C T for treatment of Wilkins, 2000, p 507
depression in borderline personality disorder. Practice Guideline for the Treatment of Patients With Major
Depressive Disorder, 2nd ed (2000), in American Psychiatric
Practice Guideline for the Treatment of Patients With Borderline Association Practice Guidelines for the Treatment of Psychiatric
Personality Disorder (2001), in American Psychic iric Association Disorders, Compendium 2004. Washington, DC, APA, 2004, p 461
Practice Guidelines for the Treatment of Psychiatric Disorders,
Compendium 2004. Washington, DC, APA, 2004, pp 817-818

127
A 29-year-old unmarried w o m a n is admitted to an
126 acute inpatient unit after police spotted her wandering
A 70-year-old w o m a n presents with a depression that along a busy highway gesturing and muttering to her-
has not responded to treatment with sertraline, parox- self. On admission, she w a s disheveled and bizarrely
etine, or escitalopram. She has said that she would clothed. Her speech was tangential, a n d she reported
like to die, and she has a history of an overdose in the auditory hallucinations commenting on her behavior
past 3 months. Although a b d o m i n a l computerized and telling her that "criminal elements" w e r e watching
t o m o g r a p h y shows no abnormalities, she is convinced her. She had recently been residing with her parents
that a hole in her liver is causing her to lose weight. and g a v e permission for staff to contact them. Her par-
Mental status examination is also significant for ents report that her first hospitalization w a s at a g e 25,
severe psychomotor retardation, a n d physical exami- just after she began working on her thesis for a Ph.D.
nation shows evidence of d e h y d r a t i o n . She is cur- in mathematics. She responded rapidly to treatment
rently being treated with 150 m g / d a y of venlafaxine. with risperidone 3 mg daily, and several months later,
W h i c h of the following recommendations is' most with the support of her adviser, she w a s able to resume
appropriate at the present time? w o r k on her thesis. O v e r the past 6 months, after she
decided to stop her medication, her symptoms have
(A) Increase the dose of venlafaxine returned. In responding.to the parents' questions about
(B) Recommend ECT her prognosis, which of the following factors would be
(C) Change to mirtazapine , the best predictor of a g o o d prognosis for this patient?
(D) Add lamotrigine
(A) Age at onset of illness-'
(E) Obtain a liver scan to assess for evidence of carcinoma
(B) Initial response to medication
T h e correct response is option B: R e c o m m e n d E C T (C) Marital status.
(D) Number and duration of remissions between psychotic episodes
(E) Premorbid cognitive functioning

100 F O n P s v c h i a t r v Review: 400 Self-Assessment Qursrio--


T h e correct response is option B: Initial response to T h e correct response is option B: Sertraline
medication
Of the options listed, initial treatment with SSRIs for
A better prognosis is indicated by late a g e at onset, symptoms of affective dysregulation in patients with
g o o d premorbid functioning, longer remission peri- borderline personality disorder has the most empirical
ods, and being married. T h e best predictor, however, support. Although the use of mood stabilizers is com-
is a g o o d initial response to medication. mon with patients with borderline personality disorder,
there are not enough data at this time to support their
Stem TA, Herman JB: Massachusetts General Hospital Psychiatry
Update and Board Preparation, 2nd ed. Boston, Massachusetts use as a first-line therapy for affective dysregulation.
General Hospital, p 101 \ •
Practice Guideline for the Treatment of Patients With Borderline
Quick reference for schizophrenia. FOCUS 2004; 2:13
Personality Disorder (2001), in American Psychiatric Association
Practice Guidelines for the Treatment of Psychiatric Disorders,
Compendium 2004. Washington, DC, APA 2004, pp 770-773

128
A patient with a history of "manic a n d major depres-
sive episodes" w h o has persistent delusions or halluci- 130
nations even w h e n prominent mood symptoms are W h i c h of the following schools of therapy has its base
absent, would have which of the following diagnoses? in the idea that family problems are due to structural
imbalances in family relationships a n d symptoms a r e
(A) Bipolar I disorder
communications?
(B) Delusional disorder, grandiose type
(C) Schizoaffective disorder (A) Cognitive behavior
(D) Schizophrenia, disorganized type (B) Insight oriented
T h e correct response is option C: Schizoaffective dis- (C) Psychoeducational
order
(D) Solution focused
(E) Strategic
T h e DSM-IV diagnostic criteria for schizoaffective dis-
The correct response is option E: Strategic
order, bipolar type, include at least one manic or
mixed episode concurrent with symptoms of schizo- • Structural/strategic/systemafic therapies a r e b a s e d
phrenia and with the persistence of delusions and hal- on the idea that the family has a problem because
lucinations for at least 2 weeks w h e n prominent mood there is a structural imbalance and that symptoms a r e
:
symptoms are no longer present. Just h a v n g schizo- communications to control relationships w h e n other
phrenic symptoms during a manic or mixed episode is strategies c a n n o t be used. T h e therapist's role
insufficient for a diagnosis of schizoaffective disorder, becomes one of highlighting problematic interaction
because manic or mixed episodes in bipolar I disor- patterns, encouraging substituting new behaviors to
der can be severe with psychotic features. interrupt feedback cycles, and applying indirect
strategies ^uch as reframing a n d paradoxical inter-
American Psychiatric Association: Diagnostic and Statistical Manual
of Mental Disorders, Fourth Edition, Text Revision (DSM-IV-TR). ventions to alter the frame of reference a n d allow n e w
Washington, DC, American Psychiatric Association, 2000, . choices.
pp 319-323,413-415
Hales RE, Yudofsky SC (eds): The American Psychiatric Publishing
Textbook of Clinical Psychiatry, 4th ed. Washington, DC, American
Psychiatric Publishing, 2003, pp 1381-1385
Sadock BJ, Sadcck VA (eds): Kaplan and Sadock's Synopsis of
129 Psychiatry: Behavioral Sciences/Clinical Psychiatry, 9th ed.
Philadelphia, Lippincott Williams & Wilkins, 2003, p 945
A patient with borderline personality disorder reports
Szapocznik J, Williams RA: Brief strategic family therapy: twenty-five
prominent lability, sensitivity to rejection, anger, out-
years of interplay among theory, research, and practice in adoles-
bursts, and " m o o d crashes." As an initial a p p r o a c h to cent behavior problems and drug abuse. Clin Child Fam Psychol
pharmacotherapy, w h i c h of the following v/ould be Rev 2000; 3:117-134
most appropriate?

(A) Gabapentin
(B) Sertraline
(C) Quetiapine
(D) Phenelzine
(E) Valproic acid

Section 2: Answers and Explanations


Treatment for Adolescents with Depression Study team: Fluoxetine,
13] cognitive-behavioral therapy, and their combination for adoles-
C o m p a r e d with depressed elderly individuals w h o cents with depression. JAMA 2004; 292:807-820
n o d a first episode of depression in y o u n g adulthood, Prozac (fluoxetine) prescribing information (package insert)
individuals with a first episode of depression in late
life are more likely to have:

(A) broin imaging findings suggesting dementia. 133


(B) comorbid personality disorder.
A 35-year-old man has a 10-year history of schizo-
(C) first-degree relatives with depression.
phrenia and poor adherence with outpatient treat-
(D) good response to treatment.
ment. He has been stabilized on 20 mg of olanzapine
(E) suicidal ideation.
in the hospital, a n d he has previously done well on
T h e correct response is option A: Brain imaging find- 10 mg of oral haloperidol. He has a g r e e d to switch
ings suggesting dementia to haloperidol decanoate injections once a month. He
is given an initial injection of 50 mg. W h i c h of the fol-
Research suggests that compared with older patients lowing is the most likely amount of time he will need
with early-onset depression, elderly patients with late- to continue taking the oral olanzapine?
life onset of depression are more likely to have
chronic medical illness, brain imaging findings sug- (A) Two days
gestive of dementia, and poorer response to treat- (B) Two weeks
ment. Patients with early onset of depression are more
(C) One month
likely to have first-degree relatives with degression.
(D) Three months
(E) One year
Coffey CE, Cummings JL: American Psychiatric Press Textbook of
Geriatric Neuropsychiatry, 2nd ed. Washington, DC, American The correct response is option D: Three months
Psychiatric Press, 2000, p 313
The haloperidol decanoate may require 3 to 6 months
to reach a steady state. H e n c e , long-acting injectable
first-generation antipsychotic medications are seldom

132 used alone during acute treatment, w h e n the psychia-


trist is adjusting the dose in accordance with thera-
A 10-year-old b o y has a well-documented episode of
peutic effects a n d side effects.
moderately severe non-bipolar major depression.
W h i c h of the following medications is FDA-approved Ereshefsky L, Mascarenas CA: Comparison of the effects of different
for use in this patient? mutes of antipsychotic administration on pharmacokinetics and
pharmacodynamics. J Clin Psychiatry 2003; 64(suppl 16):18-23
(A) Bupropion American Psychiatric Association: Practice Guideline for the
(B) Duloxetine Treatment of Patients With Schizophrenia, 2nd ed. Am J
(C) Fluoxetine Psychiatry 2004; 161 (Feb suppl):1-56
(D) Sertraline ,
(E) Venlafaxine
T h e correct response is option C: Fluoxetine
134
W h i l e other antidepressants may be of benefit for A 51-year-old w o m a n presents to her physician with
treating children and adolescents with major depres- the chief complaint of feeling depressed over the past
sive disorder, only fluoxetine has met F D A require- t month. She has no energy, is disinterested in her chil-
ments for use in pediatric patients. F D A approval w a s dren, a n d has lost 25 pounds. She is unable to fall
based on t w o 8-9-week, placebo-controlled clinical asleep until the early morning hours. She has begun
trials. M o r e recently, the Treatment of Adolescents to feel that she is unworthy of her family. W i t h the
onset of these symptoms, she is quite certain that she
with Depression Study (TADS) found fluoxetine alone
has d e v e l o p e d a d e g e n e r a t i v e n e r v e condition,
to be more effective than cognitive behavior therapy
although all investigations h a v e been negative. The
alone, although the combination a p p e a r e d to be most
most appropriate first step in treating this patient is to
beneficial.
start her o n :

Whrttington CJ, Kendall T, Fonagy P, Cottrell D, Cotgrove A,


(A) a serotonin reuptake inhibitor alone.
Boddington E: Selective serotonin reuptake inhibitors in childhood
depression: systematic review of published versus unpublished
(B) a serotonin-norepinephrine reuptake inhibitor alone.
data. Lancet 2004; 363:1341-1345 (C) a serotonin reuptake inhibitor and an antipsychotic
(D) a serotonin-norepinephrine reuptake inhibitor and a benzo-
diazepine.

TO7. m r T ' c Twv-;,r-v inn W f . A s w « m * n t n , v


T h e correct response is option C: A serotonin reuptake Breslau N:The epidemiology of posttraumatic stress disorder, what
inhibitor a n d an antipsychotic is the extent of the problem? J Clin Psychiatry 2001; 62(suppl
17):16-22
A b o u t 15% of individuals with a major depressive dis- Connor KM»Butterfield Ml: Posttraumatic stress disorder. FOCUS
order will d e v e l o p delusions. Since this w o m a n ' s 2003; 1:247-262

belief about having a degenerative nerve condition is


quite fixed a n d unsupported by medical evaluation, it
is p r o b a b l y delusional. In addition, her belief that she
136
is unworthy of her family may reach delusional pro-
portions. In the presence of psychotic features, major A patient with schizophrenia, paranoid type, a n d
depression generally requires combined treatment mefha'mphetamine dependence receives mental
with an antidepressant a n d an antipsychotic. health c a r e through a community mental health clinic
( C M H G ) . The patient has a p p e a r e d to clinically dete-
Sadock BJ, Sadock VA (eds): Kaplan and Sadock's Comprehensive riorate over a period of 6 weeks a n d is hospitalized
Textbook of Psychiatry, 7th ed. Philadelphia, Lippincott Williams & with a psychotic decompensation. A drug screen on
Wilkins, 2000, pp 1354,1379-1380 admission shows methamphefamine and ampheta-
Practice Guideline for the Treatment of Patients With Major mine in the patient's urine. After a 3-day hospital stay,
Depressive Disorder, 2nd ed (2000), in American Psychiatric
the patient is ready for discharge. T h e outpatient psy-
Association Practice Guidelines for the Treatment of Psychiatric
chiatrist should do which of the following?
Disorders, Compendium 2004. Washington, DC, APA, 2004, p 469
(A) Resume psychiatric care through the CMHC, deferring sub-
stance dependence treatment unless the patient resumes
methamphetamine use.
135 (B) Resume psychiatric care at the CMHC, with increased
W h i c h of the following is N O T a predisposing risk fac- emphasis on the provision of substance dependence treat-
tor for the development of posttraumatic stress disorder ment by the mental health team.
(PTSD) after a traumatic event? (C) Enroll the patient in a separate program specifically for
substance dependence and continue to provide psychiatric
(A) Recent life stressors care through the CMHC.
(B) Female gender (D) Enroll the patient in a separate program specifically for
(C) Internal locus of control substance dependence and resume psychiatric care a? the
(D) Past history of depression CMHC once a period of sobriety is achieved.

The correct response is option C: Internal locus of control The correct response is option B: Resume psychiatric
care at the C M H C , with increased emphasis on the
Of the choices listed, all but option C are established
provision of substance dependence treatment by the
personal vulnerability-related risk factors for the mental health team
development of PTSD following trauma. W h e n the
type of trauma is controlled for, w o m e n appear to be Integrated treatment, in w h i c h the same clinicians pro-
at higher risk of developing PTSD compared with vide both mental health a n d substance abuse treat-
men. In o n e nationwide survey, the highest current ment, has better outcomes than treatment that is split
(17.8%) a n d lifetime (38.5%) rates of PTSD w e r e in either in a parallel or sequential fashion.
w o m e n w h o had been exposed to physical assault or McHugo GJ, Drake RE, Teague GB, Xie H: Fidelity to assertive com-
rape. PTSD is more common in younger than in older munity treatment and client outcomes in the New Hampshire dual
individuals, probably because of the higher incidence disorders study. Psychiatr Sen/ 1999; 50:818-824
of physical violence and accidents in the younger Drake RE, Osher FC: Treating substance abuse in patients with
severe mental illness, in Innovative Approaches for Difficult-to-
population. Individuals w h o respond to the initial
Treat Populations. Edited by Henggeler SW, Santos AB.
trauma with high levels of anxiety (e.g., a panic Washington, DC, American Psychiatric Press, 1997, pp 191-210
attack), also h a v e a higher risk of developing PTSD
after trauma, as are those w h o perceive an external
(vs. internal) locus of control.

Kaplan HI, Sadock BJ: Synopsis of Psychiatry: Behavioral


Sciences/Clinical Psychiatry, 9th ed. Baltimore, Lippincott Williams
& Wilkins, 2003, p 624
Resnick HS, Kilpatrick DG, Dansky BS, Saunders BE, Best CL:
Prevalence of civilian trauma and posttraumatic stress disorder in
a representative national sample of women. J Consult Clin
Psychol 1993; 61:984-991

Section 2: Answers and Explanations 103


137 139
W h i c h of the following is the most accurate w a y to A patient with mild dementia of the Alzheimer's type
diagnose early-stage dementia of the Alzheimer's type? is brought in by his wife, w h o is also his primary care-
giver, for follow-up evaluation. She brings along a list
(A) Apolipoprofein E genotyping of his medications. T h e patient is taking donepezil,
(B) Brain MRI hydrochlorothiazide, a n d w a r f a r i n . The use of which
(C) History of stepwise memory decline of the following herbal or over-the-counter products by
(D) Neuropsychological testing this patient w o u l d be of the most concern?
(E) Patient self-report of memory difficulties
(A) Ginkgo biloba
T h e correct response is option D: Neuropsychological
(B) Ginseng
testing (C) Hawthorn
Of the methods listed, neuropsychological testing is
(D) Vitamin C
the most sensitive, even for v e r y mild deficits in early
(E) Vitamin E
dementia of the Alzheimer's type. History of stepwise The correct response is option A: G i n k g o biloba
m e m o r y decline is associated with vascular dementia.
Psychiatrists should routinely ask about the use of
Patient self-report of memory difficulties is notoriously
herbal a n d over-the-counter preparations, as these are
unreliable.
often presumed by patients a n d caregivers to be safe.
Salmon DP, Thomas RG, Pay MM, Booth A, Hofstetter CR, Thai LJ, Although g i n k g o biloba has been widely advertised
Katzman R: Alzheimer's disease can be accurately diagnosed in
as having cognitive-enhancing properties, the evi-
very mildly impaired individuals. Neurology 2002; 59:1022-1028
dence for its slowing the progression of dementia of
the Alzheimer's type is modest. G i n k g o biloba should
be a v o i d e d by patients taking warfarin, because

138 there have been case reports of intracranial bleeding


and hematoma.
A man reports that he avoids public urinals even
w h e n he has great urgency to urinate. This type of De Smet PA: Herbal remedies. N Engl J Med 2002; 347:2046-2056
chief complaint is most consistent with a diagnosis of: Jacobson SA, Pies RW, Greenblatt DJ: Handbook of Geriatric • -
Psychopharmacology. Washington, DC, American Psychiatric
(A) body dysmorphic disorder. Publishing, 2002, pp 379-380
(B) obsessive-compulsive disorder.
(C) panic disorder with agoraphobia.
(D) social phobia.
(E) posttraumatic stress disorder. 140
T h e correct response is option D: Social phobia W h a t diagnostic specifier w o u l d be most appropriate
for a depressed patient w h o complains of a sense of
T h e inability to urinate in a public toilet is one of many
leaden paralysis a n d difficulty being around other
performance-related anxieties of people with social people but is able to enjoy himself w h e n g o o d things
p h o b i a . Eating, writing, telephoning, a n d especially happen?
speaking in public are all behaviors during which per-
sons with social phobia fear being scrutinized or (A) With atypical features
humiliated. T h e other options listed could be reasons
(B) With catatonic features
for avoiding a public toilet—for e x a m p l e , contamina- -
(C) With melancholic features
(D) With psychotic features
tion fears (obsessive-compulsive disorder), concern
that the penis is abnormal (body dysmorphic disor- The correct response is option A: W i t h atypical fea-
der), and prior sexual trauma in a public toilet (post- tures
traumatic stress d i s o r d e r ) - b u t are clinically unlikely.

Stein DJ, Hollander E (eds): American Psychiatric Publishing Textbook


of Anxiety Disorders. Washington, DC, American Psychiatric
Publishing, 2002, pp 289-290 f
Depression with atypical features includes symptoms The same dose that w a s effective during the acute a n d
of mood reactivity combined with some symptoms of continuation phases of treatment should be used during
increased sleep, appetite, weight, rejection sensitivity, the maintenance phase. In this instance, the initial dose
phobic symptoms, a n d severe fatigue. Depression of sertraline w a s associated with side effects, but a
with melancholic features includes a loss of pleasure therapeutic response w a s maintained with a lower dose
in almost all activities or a loss of enjoyment of usually of medication. W h i l e psychotherapy is often indicated,
pleasurable activities combined with some symptoms its use does not justify reducing or stopping medication.
of depression that a r e worse in the morning, early
Practice Guideline for the Treatment of Patients With Major
morning a w a k e n i n g , weight loss, inappropriate guilt,
Depressive Disorder, 2nd ed (2000), in American Psychiatric
and either psychomotor retardation or agitation. Association Practice Guidelines for the Treatment of Psychiatric
Depression with catatonic features includes at least Disorders, Compendium 2004. Washington, DC, APA, 2004, p 456
t w o of the following: motor immobility, excessive
motor activity, extreme negativism, peculiar voluntary
movements, echolalia, or a p r a x i a . Depression with
psychotic features refers to the presence of delusions 142
or hallucinations. W h i c h of the following antipsychotic drugs has the
greatest effect on prolonging the QT interval on the
American Psychiatric Association: Diagnostic and Statistical Manual
electrocardiogram?
of Mental Disorders, Fourth Edition, Text Revision (DSM-IV-TR).
Washington, DC, American Psychiatric Association, 2000, (A) Aripiprazole
pp 420-422
(8) Ha operidol
Practice Guideline for the Treatment of Patients With Major
(C) Olanzapine
Depressive Disorder, 2nd ed (2000), in American Psychiatric
Association Practice Guidelines for the Treatment of Psychiatric
(D) Thioridazine
Disorders, Compendium 2004. Washington, 0C, APA 2004, p 469 (E) Ziprasidone
The correct response is option D: T h i o r i d a z i n e

Thioridazine prolongs the QT interval in a dose a n d


141 blood level dependent fashion, and has a greater
effect on the QT interval than most other antipsychotic
A 34-year-old w o m a n with two previous episodes of
major depressive disorder is treated with 100 m g / d a y drugs. In a comparative study, ziprasidone caused a
of sertraline. Her most recent episode occurred 9 modest QT prolongation, but it w a s considerably less
months a g o . At that time sertraline w a s initiated, with than that caused by thioridazine.
the dose titrated up to 150 m g / d a y over a 2-week
Taylor DM: Antipsychotics and QT prolongation. Acta Psychiatr Scand
period. Her symptoms remitted after 6 weeks, but she
2003; 107:85-95
experienced significant sexual side effects that Harrigan EP, Miceli J J , Anziano R, Watsky E, Reeves KR, Cutler NR,
resolved with a decrease in the sertraline dose to Sramek J, Shiovitz T, Middle M: A randomized evaluation of the
100 m g / d a y . O v e r the past 7 months she has effects of six antipsychotic agents on QTc, in the absence and
remained free of depressive symptoms and n o w presence of Metabolic inhibition. J Clin Psychopharmacol 2004;
inquires about decreasing her dose of medication. 24:62-69
W h i c h of the following recommendations is most Yap YG, Camm AJ: Drug induced QT prolongation and torsades de
pointes. Heart 2003; 89:1363-1372
appropriate?
Witchel HJ, Hancox JC, Nutt DJ: Psychotropic drugs, cardiac arrhyth-
(A) Continue sertraline at 100 mg/day. mia, and sudden death. J Clin Psychopharmacol 2003; 23:58-77
(B) Decrease sertraline to 50 mg/day and continue at that dose. Glassman AH, Bigger JT Jr: Antipsychotic drugs: prolonged QTc inter-
val, torsade de pointes, and sudden death. Am J Psychiatry 2001;
(C) Discontinue sertraline after tapering the dose. 158:1774-1782
(D) Initiate psychotherapy and then decrease sertraline to
50 mg/day.
(E) Initiate psychotherapy and then gradually discontinue ser-
traline.
T h e correct response is option A: Continue sertraline
at 100 m g / d a y

Section 2: Answers and Explanations 105


Lamotrigine is metabolized outside the cytochrome
143
P450 system. It undergoes phase II conjugation
T h e therapeutic benefit of acamprosate is best estab-
mainly to an inactive 2-N-glucuronide. Nonetheless,
lished for which of the following conditions?
its pharmacokinetics can be altered by other drugs.
(A) Alcohol dependence E n z y m e inducers such as c a r b a m a z e p i n e , phenytoin,
(B) Barbiturate dependence and oral contraceptives lower blood levels of lamo-
(C) Cocaine withdrawal trigine. Lithium has no effect, but valproate inhibits
(D) Heroin addiction lamotrigine metabolism and doubles its blood level.
(E) Methamphetamine abuse Dosage adjustment is especially important to mini-
mize the risk of rash.
T h e correct response is option A: A l c o h o l depen-
dence
Jefferson JW: Lamotrigine in psychiatry: pharmacology and thera-
peutics. CNS Spectr 2005; 10:224-232
Acamprosate is F D A - a p p r o v e d "for the maintenance
Sabers A, Buchhott JM, Uldall P, Hansen EL Lamotrigine plasma levels
of abstinence from alcohol in patients with alcohol reduced by oral contraceptives. Epilepsy Res 2001; 47:151-154
dependence w h o are abstinent at treatment initiation" Practice Guideline for the Treatment of Patients With Bipolar Disorder,
( C a m p r a l p a c k a g e insert). Its efficacy w a s established 2nd ed (2002), in American Psychiatric Association Practice
in three double-blind placebo-controlled studies rang- Guidelines for the Treatment of Psychiatric Disorders,
Compendium 2004. Washington, DC, APA, 2004, pp 572-573.
ing in duration from 90 to 360 d a y s . " A c a m p r o s a t e
Reprinted in FOCUS 2003; 1:64-110 (pp 91-92)
m a y not be effective in patients w h o are actively
drinking at the start of treatment, or in patients w h o
abuse other substances in addition to ^alcohol" ( F D A
talk paper). Currently no clinical trials of acamprosate 145
for other psychiatric conditions are under w a y .
Of the following, which is the most important factor in
Safe R: Unhealthy alcohol use. N Engl J Med 2005; 352:595-607 determining whether a patient is suited to brief psy-
Bouza C, Angeles M, Munoz A, Amate JM: Efficacy and safety of nal- c h o d y n a m i c psychotherapy?
trexone and acamprosate in the treatment of alcohol dependence:
a systematic review. Addiction 2004; 99:811-828 (A) Ability to recognize and discuss feelings
Campral (acamprosate) prescribing information (package insert) (B) Existence of multiple conflicts
A talk paper: FDA approves new drug for treatment of alcoholism, (C) Compliance with medication
1
July 29,2004. Available at
(D) Lack of meaningful relationships
http7/vvvv'w.fda.gov/bbs/topics/ANSWERS/2004/ANS01302.html
(E) Failure of long-term psychodynamic psychotherapy
Mack AH, Frances RJ: Substance-related disorders. FOCUS 2003;
1:125-146 (p 139) The correct response is option A: Ability to recognize
a n d discuss feelings

Patient selection for brief psychodynamic psychother-


144 a p y requires that the patient has a focal conflict, is able
to think in feeling terms, is highly motivated, has had at
A 25-year-old w o m a n with bipolar disorder is about
least one meaningful relationship, and has had a good
to be started on lamotrigine for maintenance therapy.
She should receive one-half of the usual starting dose response to a trial interpretation. Complex or deep-
if she is taking w h i c h of the following medications? seated issues require more time. The patient w h o
reports having had at least one meaningful relationship
(A) Carbamazepine with another person during his or her life will have bet-
(B) Lithium ter object relations and be better able to tolerate the dif-
(C) Oral contraceptive
ficult feelings that can be stirred up by psychotherapy.
(D) Phenytoin
(E) Valproate Ursano RJ, Sonnenberg SM, Lazar SG: Concise Guide to
Psychodynamic Psychotherapy, 3rd ed. Washington, DC, American
T h e correct response is option E: Valproate
. Psychiatric Publishing, 2004, p 179

T06 rnnic n _ . r > . . . .;,.„.. CICArt^monr n«wmnt


146 148
C o m p a r e d with men, w o m e n with schizophrenia tak- Hypertension is most associated with which of the fol-
ing equivalent doses per weight of antipsychotics are lowing^edications?
less likely to h a v e which of the following?
(A) Bupropion
(A) Acute dystonia (B) Fiuvoxamine
(B) Drug-drug interactions (C) Mirtazapine
(C) Lower serum drug levels (D) Paroxetine
(D) Sedation (E) Venlafaxine
T h e correct response is option C: Lower serum drug T h e correct response is option E: Venlafaxine
levels
Venlafaxine is the antidepressant most associated with
Studies indicate that w o m e n have higher serum levels a sustained hypertension (systolic blood pressure
of antipsychotic medications compared with men. greater than 90 mm Hg a n d greater than 10 mm Hg
W o m e n are more likely to be taking other drugs, such a b o v e baseline for three consecutive visits). It tends to
as contraceptives, and hence are at greater risk of occur most frequently with doses a b o v e 300 mg but
drug-drug interactions. On equivalent doses, w o m e n can occur at a n y dose. There is also a v e n l a f a x i n e
appear to h a v e more acute dystonia. Sedation withdrawal syndrome that consists mainly of gastroin-
appears to occur equally in men and w o m e n . testinal and neurological symptoms. T h e other antide-
pressants listed do not have sustained or significant
Seeman MV: Gender differences in the prescribing of antipsychotic
drugs. Am J Psychiatry 2004; 161:1324-1333. Reprinted in hypertensive effects.
FOCUS 2006; 4:115-124
Sadock BJ, Sadock VA (eds): Kaplan and Sadock's Comprehensive
Textbook of Psychiatry, 7th ed. Philadelphia, Lippincott Williams &
Wilkins, 2000, p 2430

147
A 22-year-old man presents at the emergency depart-
ment with agitated, guarded behavior, paranoid delu- 14?
sional thoughts, and a 7-month history consistent with A patient reports regularly taking a drug bought on
a diagnosis of schizophrenia, paranoid type. Under- the street. Its effect is pleasurable, but it sometimes
standing the man's cultural background w o u l d be most causes nausea, restlessness, and teeth grinding. T h e
helpful for:' drug is most likely:

(A) choosing his acute and maintenance medications. (A) methamphetamine.


(B) determining the cause of his disorder. (B) flunitrazepam (Rohypnol).
(C) determining safety issues and the need for hospitalization. (C) methylenedioxymethamphetamine (MDMA).
(D) understanding the content of the delusions and hallucinations. (D) cocaine.
4
T h e correct response is option D: Understanding the
The correct response is option C: Methylenedioxy-
content of the delusions and hallucinations
methamphetamine ( M D M A )
Culture can influence the content of hallucinations a n d Methylenedioxymethamphetamine (MDMA, or
delusions. Schizophrenia is thought to occur at similar "Ecstasy") both releases and inhibits the reuptake of
prevalence rates w o r l d w i d e . The other options listed serotonin. Although M D M A is a synthetic ampheta-
are generally decided on the basis of the patient's mine derivative, its effects are quite different from
psychiatric status. those of its parent drug. Flunitrazepam, best k n o w n as

Sadock BJ, Sadock VA (eds): Kaplan and Sadock's Comprehensive Rohypnol, is a rapid-acting b e n z o d i a z e p i n e that is
Textbook of Psychiatry, 8th ed. Philadelphia, Lippincott WIHiams & available only illegally in the United States.
Wilkins, 2005, pp 616-617
Morgan MJ: Ecstasy (M0MA): a review of its possible persistent psy-
chological effects. Psychopharmacology 2000; 152:230-248
McDowell DM: MDMA, ketamine, GHB, and the "club drug" scene, in
The American Psychiatric Press Textbook of Substance Abuse
Treatment 3rd ed. Edited by Galanter M, Kleber HD. Washington,
DC, American Psychiatric Publishing, 2004 pp 321-333

Seccion 2: Answers and Explanations


Tseng WS, Streltzer J: Cultural Competence in Clinical Practice.
150 Washington, DC, American Psychiatric Publishing, 2004,
Indications of caffeine w i t h d r a w a l : pp169-170
De Leon J, Armstrong SC, Cozza KL: Clinical guidelines for psychia-
(A) ore evident following an average intake of 50 mg/day. trists for the use of pharmacogenetic testing for CYP450 2D6 and
(B) include symptoms that typically last 3 weeks. CYP450 2C19. Psychosomatics 2006; 47:75-85
(C) include a flushed face and diuresis.
(D) indude heodoche.
T h e correct response is option D: Include headache
152
Caffeine w i t h d r a w a l effects c a n be seen at doses as The preferred initial pharmacological treatment for a
l o w as T O O m g / d a y . W i t h d r a w a l effects may include 26-year-old w o m a n w h o w a s violently assaulted 3
h e a d a c h e , fatigue, drowsiness, nausea, and vomit- years a g o a n d suffers from recurrent nightmares
ing. W i t h d r a w a l headaches can cause functional about the event, hypervigilance, difficulty concentrat-
impairment. Flushing of the face a n d diuresis are ing, a n d constricted range of affect would be:
characteristics of caffeine intoxication. Caffeine with-
(A) alprazolam.
d r a w a l effects usually begin within 12 hours, peak
(B) clomipramine.
a r o u n d 24 to 48 hours, and last up to 1 week.
(C) clonidine.
Strain EC, Griffiths RR: Caffeine use disorders, in Psychiatry, 2nd ed. (D) valproate.
Edited by Tasman A, Kay J, Lieberman JA Philadelphia, WB (E) sertraline.
Saunders, 1997, pp 779-794 ,
The correct response is option E: Sertraline

According to the A P A Practice Guideline for the


Treatment of Acute Stress Disorder and Posttraumatic
151 Stress Disorder, "Evidence from, several large ran-
A 23-year-old college student from Ethiopia is admit- d o m i z e d , double-blind controlled trials suggests that
ted to the hospital for a psychotic disorder. His symp- SSRIs are first-line medication treatment for both men
toms include paranoid ideation, hallucinations, and and w o m e n with P T S D . " Sertraline a n d paroxetine
disorganized thinking. He is.-. initially started on are the only drugs a p p r o v e d by the F D A for this indi-
0.5 mg of risperidone twice daily, w h i c h is rapidly cation. T h e successful use of clonidine a n d valproate
titrated to 4 m g / d a y . Despite this treatment, there is
has been restricted to small, open-label studies.
no improvement in his symptoms. No detectable
-
A l p r a z o l a m w a s demonstrated in a controlled trial to
plasma levels of the. parent c o m p o u n d are noted.
be ineffective in the treatment of posttraumatic stress
. W h a t is the most likely explanation for these results?
disorder.
(A) Binding of the drug to fatty tissue
(B) First-pass effect American Psychiatric Association: Practice Guideline for the
Treatment of Acute Stress Disorder and Posttraumatic Stress
(C) Impaired absorption of the drug Disorder. Am J Psychiatry 2004; 161 (Nov suppl)
(D) Increased excretion Schemed FB, Marmar CR, Neylan TC: Current concepts in pharma-
(E) Ultrarapid metabolism of the drug cotherapy for posttraumatic stress disorder. Psychiatr Serv 2004;
55:519-531
T h e correct response is option E: Ultrarapid metabo-
Hidalgo RB, Davidson JRT: Diagnostic and pharmacologic aspects of
lism of the drug
posttraumatic stress disorder. Psychiatr Ann 2004; 34:837-841

T h e metabolism of risperidone is contingent on the


activity of the C Y P 2 D 6 e n z y m e , w h i c h in turn is genet-
ically determined. N e a r l y one-third of N o r t h Africans
a n d Middle Easterners have multiple alleles for this
e n z y m e a n d consequently are ultrarapid metabolizers
of medications using the C Y P 2 D 6 e n z y m e . Treatment
a p p r o a c h e s may include the addition of medications
such as paroxetine, w h i c h inhibits C Y P 2 D 6 , thus
allowing for therapeutic dosages of risperidone.
A p p r o x i m a t e l y 7% of Caucasians a n d 25% of East
•Asians lack this e n z y m e and are p o o r metabolizers of
risperidone.

TOO
153 Schatzberg AF, Nemeroff CB (eds): The American Psychiatric
Publishing Textbook of Psychopharmacology, 3rd ed. Washington,
In the initial assessment of a depressed patient, what DC, American Psychiatric Publishing, 2004, pp 598-599
is the most critical decision that the psychiatrist must
make?
155
(A) Type of psychotherapy
(B) Choice of medication Screening for hepatitis C [HO/] infection is L E A S T
important inpatients with:
(C) Level of care
(0) Medical workup , (A) methamphetamine dependence.
(E) Involvement of family • (B). marijuana dependence.
T h e correct response is option C: Level of care (C) heroin dependence.
(D) history of blood transfusion (before 1992).
Patients w h o present to the psychiatrist must be (E) hemodialysis.
assessed for suicidality, which will often determine the
The correct response is option B: Marijuana depen-
need for hospitalization. T h e initial determination of
dence
the safety of the patient is paramount.
Injection drug use is more common among patients
Sadock BJ, Sadock VA (eds): Kaplan and Sadock's Comprehensive
who abuse stimulants and opioids than among mari-
Textbook of Psychiatry, 7th ed. Philadelphia, Lippincott Williams &
Wilkins, 2000, pp 662-663,2038-2040 juana abusers. T h e most well established risk factors
for H C V a r e intravenous drug use, blood or blood
product transfusions prior to 1992, a n d hemodialysis.
154 Screening for patients with one or more of these risk
A 27-year-old patient announces that she is pregnant factors has been recommended by C D C .
despite having taken an oral contraceptive for 4
Rifai MA, Rosenstein DL Hepatitis C and Psychiatry. FOCUS 3:194-195
years. W h i c h of the following medications might Hagan H, Des Jarlais DC: HiV and HCV infection among injecting drug
account for the failure of her oral contraceptive? users. Mt Sinai J Med 2000; 67:423-428

(A) Lithium
(B) Divalproex
(C) Carbamazepine
(D) Lamotrigine 156
(E) Gabapentin W h i c h of the following is best characterized as a
degenerative dementia?
The correct response is option C: C a r b n r n a z e p i n e
(A) Systemic lupus erythematosus
C a r b a m a z e p i n e as well as o x c a r b a z e p i n e are induc-
(B) Korsakoff's syndrome
ers of the cytochrome P450 system and therefore can
(C) Parkinson's disease
lower serum concentrations of estrogen in birth control (D) HIV disease
pills. Gabapentin has few drug interactions, and it (E) Cerebrovascular accident
has been shown to have no effect on ethinyl estradiol
The correct response is option C: Parkinson's disease
levels. C a r b a m a z e p i n e and o x c a r b a z e p i n e , by con-
trast, reduce ethinyl estradiol levels by about 50% Degenerative dementias, such as Parkinson's disease
and topiramate by 18% to 25%. Breakthrough bleed- and Huntington's disease, are distinguished from non-
ing a n d an increased risk of p r e g n a n c y are possible degenerative dementias by intrinsic genetic processes
complications associated with a reduced ethinyl estra- that lead to neuron destruction rather than complica-
diol level. Lamotrigine does not have pharmacokinetic tions from systemic medical conditions or insults.
interactions that could potentially lead to decreased
Plizka SR: Neuroscience for the Mental Health Clinician. New York,
oral contraceptive efficacy. H o w e v e r , in a series of
Guilford, 2003, p 257
seven cases, oral contraceptives w e r e found to
Levenson JL: American Psychiatric Publishing Textbook of
decrease lamotrigine levels by a mean of 49%. Psychosomatic Medicine, Table 7-3, Disorders that may produce
dementia syndromes. Washington, DC, American Psychiatric
Wilbur K, Ensom MHH: Pharmacokinetic drug interactions between Publishing, 2005, p 133
oral contraceptives and second-generation anticonvulsants. Clin
Pharmacokinet 2000; 38:355-365
Sabers A, Buchholt JM, Uldall P, Hansen EL: Lamotrigine plasma lev-
els reduced by oral contraceptives. Epilepsy Res 2001;
47:151-154

Section 2: Answers and Explanations


157 159
In a urine test for phencydidine, a false positive test W h i c h of the following disorders has been s h o w n to
result can occur if a person has ingested: have genetic or familial links?

(A) dextromethorphan. (A) Autistic disorder


(B) ibuprofen. (B) Dissociative omnesia
(C) tonic water. (C) Factitious disorder
(D) phenylephrine. (D) Hypoactive sexual desire disorder
(E) diphenhydramine. (E) Pyromonia
The correct response is option A: Dextromethorphan T h e correct response is option A: Autistic disorder

Dextromethorphan can cause a false positive test Data from twin studies suggests that a substantial portion
result for phencydidine. Ibuprofen can cause a fclse of the liability to autism is genetic in origin. The other dis-
positive test result for marijuana metabolites. Tonic orders have not been shown to have genetic risk.
water can cause a false positive test result for opiates.
Popper CW, Gammon, GD, West SA, Bailey CE: Disorders usually first
Phenylephrine can cause a false positive test result for diagnosed in infancy, childhood, or adolescence, in The American
amphetamines. Diphenhydramine does not affect Psychiatric Publishing Textbook of Clinical Psychiatry, 4th ed.
urine drug tests. Edited by Hales RE, Yudofsky SC. Washington, DC, American
Psychiatric Publishing, 2003, pp 894-895
Tests tor drugs of abuse. Medical Letter on Drugs and Therapeutics
2002, vol 44 (W1137A), August 19 ' }

160
158 A 45-year-old man and his m o n o z y g o t i c . t w i n have
been diagnosed as having the same personality disor-
A 48-year-old man is admitted to the hospital with der. W h i c h of the following diagnoses is most likely?
cholecystitis, a n d after diagnosis he consents to and
undergoes a' cholecystectomy. On the third hospital (A) Histrionic personality disorder
d a y he becomes a n g r y at the nursing staff and wishes (B) Obsessive-compulsive personality disorder
to leave the hospital against medical advice. In asses- (C) Narcissistic personality disorder
sing this patient's capacity to refuse further medical (D) Antisocial personality disorder
care, which of the following questions w o u l d be most v
(E) Avoidant personality disorder
useful for the psychiatrist to ask?
The correct response is option D: Antisocial personal-
(A) Have you discussed with your family your decision to ity disorder
leave?
Antisocial personality disorder has the highest esti-
(B) What is the danger of your going home at this time?
(C) Have you been troubled by depression? mated heritability of the personality disorders listed
(D) Are you able to name all of your medications? (approximately 60%-70%). Borderline personality dis-
(E) When did you first become ill, and do you remember your order, in particular the emotional dysregulation char-
symptoms? acteristic of this disorder, also has a strong heritability.

The correct response is option B: W h a t is the danger Fu Q, Heath AC, Bucholz KX, Nelson E, Goldberg J, Lyons MJ, True
of your going home at this time? WR, Jacob T, Tsuang MT, Eisen SA: Shared genetic risk of major
depression, alcohol dependence, and marijuana dependence:
Central to the patient's ability to make health care contribution of antisocial personality disorder in men. Arch Gen
decisions is the understanding of the risks and bene- Psychiatry 2002; 59:1125-1132
Lieb K, Zanarini MC, Schmahl C, Linehan MM, Bohus M: Borderline
fits of one choice over another. Certainly the other
personality disorder. Lancet 2004; 364:453-461
questions are helpful in gaining this understanding,
but they are not the key concern. In addition, by d a y
three of hospitalization, concern about the presence
of a delirium, withdrawal syndrome, or otfjer meta-
bolic syndrome is raised. The patient had been agree-
able for all treatments before this time, having already
agreed to the surgery a n d postoperative treatments.

Appelbaum PS, Grisso T: Assessing patients' capacities to consent to


treatment N Engl J Med 1988; 319:1635-1668

T TQ
161 163
W h i c h of the following medication classes is the pre- W h i c h of the following will double the blood level of
ferred treatment for obsessive-compulsive disorder? lamotrigine?

(A) Atypical antipsychotic (A) Carbomazepine


(B) Anticonvulsants (B) Divalproex
(C) Mood stabilizers (C) Phenyloin
(D) SSRIs (D) Phenobttrbital
(E) Benzodiazepines ,
i The correct response is option B: Divalproex
T h e correct response is option D: SSRIs
Valproate (divalproex) reduces the clearance of lamo-
N u m e r o u s double-blind, placebo-controlled studies trigine by about 50%, whereas the other three drugs
have found serotonin reuptake inhibiting antidepres- listed actually increase lamotrigine clearance a n d
sants to be effective in the treatment of obsessive-com- reduce lamotrigine blood levels by 4 0 % to 50%. T h e
pulsive disorder (clomipramine a n d SSRIs). Several 'valproate/lamotrigine interaction is important, because
antidepressants h a v e been a p p r o v e d by the Food and unless dosage adjustments are made, the risk of serious
Drug Administration for this indication. rash will be increased.

Stein DJ: Obsessive-compulsive disorder. Lancet 2002; 360:397-405 Benedetti MS: Enzyme induction and inhibition by new antiepileptic
Park LT, Jefferson JW, Greist JG: Obsessive-compulsive disorder: dnjgs: a review of human studies. Fundam Clin Pharmacol 2000;
treatment options. CNS Drugs 1997; 7:187-202 14:305-319
Hollander E, Simeon D: Concise Guide to Anxiety Disorders. Kannen AM, Frey M: Adding valproate to lamotrigine: a study of their
Washington, DC, American Psychiatric Publishing, 2003, p 170 pharmacokinetic interaction. Neurology 2000; 55:588-591
Riva R, AJbani F, Contin M, Baruzzi A Pharmacokinetic interactions
between antiepileptic drugs: clinical considerations. Clin
Pharmacokinet 1996; 31:470-493

162
W h i c h of the following interventions is the best first step .
in the management of agitation in the elderly patient?
164
(A) Haloperidol, 5 mg twice a day, whatever the cause of the Buspirone has been found to be most consistently
agitation effective in the treatment of which of the following
(B) Physical restraints anxiety disorders?
(C) Evaluation of the patient's surroundings and daily schedule
(D) Diazepam, 5 mg every 6 hours, or until the patient is asleep (A) Generalized anxiety disorder
(E) Seclusion until the behavior ceases (B) Obsessive-compulsive disorder
(C) Panic disorder with agoraphobia
T h e correct response is option C: Evaluation of the (D) Panic disorder without agoraphobia
patient's surroundings a n d daily schedule (E) Social phobia
The initial assessment of a geriatric patient with agi- The correct response is option A: G e n e r a l i z e d anxiety
tation should begin with a thorough psychiatric and disorder
medical assessment, including initial laboratory
Although trials have been conducted on the use of
assessments a n d collateral information from the
buspirone in the treatment of all the anxiety disorders,
patient's caregiver; a diurnal a n d longitudinal record
efficacy in multiple trials has been shown only for g e n -
of the behavior; an accounting of a n y changes in
eralized anxiety disorder.
environment or routine; an assessment of functional
ability; and a schedule of daily activity. All prescrip- Davidson JR: Pharmacotherapy of social phobia. Acta Psychiatr
tion and nonprescription medications should be Scand Suppl 2003;417:65-71
Hollander E, Simeon D: Anxiety disorders, in American Psychiatric
r e v i e w e d . T h e patient should also be evaluated for
Publishing Textbook of Clinical Psychiatry, 4th ed. Edited by Hales
trauma or pain. RE, Yudofsky SC. Washington, DC, American Psychiatric
Publishing, 2003, pp 543-630
Cheong JA: An evidence-based approach to the management of agita-
tion in the geriatric patient. FOCUS 2004; 2:197-205 (pp 198-203)

Section 2: Answers and Explanations III


avoided in patients with moderate to severe hepatic
165
dysfunction, peripheral neuropathies, pregnancy,
W h i c h of the following parental transmissions of the
renal failure, or cardiac disease.
fragile X trinucleotide repeat is most likely to result in
an affected child? American Psychiatric Association: Practice Guideline for the
Treatment of Patients With Substance Use Disorders: Alcohol,
(A) Mother-to-daughter Cocaine, Opioids. Am J Psychiatry 1995; 152(Nov suppl)
(B) Mother-to-son
(C) Father-to-daughter
(D) father-to-son
(E) Ho parental gender difference 167
The correct response is option B: Mother-to-son W h i c h of the following signs and symptoms is more
likely to occur in females than in males at first presen-
The basic inheritance pattern of an X-linked disease is
tation of psychosis?
mother to son. Although females can inherit the g e n e ,
they are less likely to be affected because they usually (A) Amotivation
have a normal X chromosome as w e l l . As fragile X (B) Cognitive impairment
(C) Dysphoric mood state
syndrome is an X-linked disease, fathers cannot pass
(D) Paranoid ideation
it on to their sons.
(E) Social isolation
Sadock BJ, Sadock VA (eds): Kaplan and Sadock's Comprehensive
Textbook of Psychiatry, 8th ed. Philadelphia, Lippincott Williams & The correct response is option C: Dysphoric mood state
Wilkins, 2005, p 2283
At first presentation of psychosis, it has been shown that
males are more likely than females to show cognitive
deficits and females are more likely than males to show
depression. These sex differences are most prominent in
166
the preadolescent and adolescent periods. Affective
W h i c h of the following is N O T a relative contraindi-
symptoms, particularly depression, are associated with
cation for the use of disulfiram as an adjunct in the
a better prognosis for schizophrenia and psychotic dis-
treatment of alcohol dependence?
orders in general and may explain the more favorable
(A) Impulsive behavior prognosis for w o m e n with schizophrenia.
(B) Psychotic symptoms
(C) Suicidal thoughts Romans SE, Seeman MV: Women's Merfcl Health: A Life-Cycle
Approach. Philadelphia, Lippincott Williams & Wilkins, 2006, p 196
(D) A family history of alcoholism
BurtVK, Hendrick VC: Gender Issues in the treaiment of mental ill-
(E) Impaired judgment
ness, in Clinical Manual of Women's Mental Health. Washington,
T h e correct response is option D: A family history of DC, American Psychiatric Publishing, 2005, p 148. Reprinted in
FOCUS 2006; 4:66-80
alcoholism
Quick reference for schizophrenia. FOCUS 2004; 2:13-16
Patients with impulsive, psychotic symptoms a n d sui-
cidal thoughts a r e p o o r candidates for disulfiram
treatment, given the adverse effects of drinking alco-
hol while taking disulfiram. Disulfiram inhibits the
activity of the e n z y m e a l d e h y d e d e h y d r o g e n a s e ,
which metabolizes acetaldehyde, a major metabolite
of alcohol. If alcohol is consumed in the presence of
disulfiram, toxicity secondary to acetaldehyde, accu-
mulation m a y occur. ,

Patients w h o are intelligent, motivated, a n d not impul-


sive and w h o s e drinking is often triggered by unan-
ticipated internal or external cues that increase
alcohol craving a r e the best candidates for disulfiram
treatment. Poor candidates include patients w h o are
•impulsive, have poor judgment, or a r e suffering from
a comorbid psychiatric illness (e.g., schizophrenia,
major depression) w h o s e severity makes them unreli-
able or self-destructive. Disulfiram treatment should be
168 T h e patient needs an assessment of her capacity to
make this decision. A relatively young w o m a n , she is
A 45-year-old w o m a n complains of blurred vision,
likely fearful a n d struggling with this n e w diagnosis.
ocular pain, and headache and w a s . n o t e d to have
Her family would be able to help explain some of her
increased intraocular pressure'. W h i c h of the recently
started medications is the most likely cause? long-standing health beliefs as well as to support her
through this difficult time. Even though this is a serious
(A) Lamotrigine condition, without evaluating her capacity and
(B) Oxcarbazepine engaging the family, there a r e no grounds for treating
(C) liagabine , her against her will at this time.
1
(D) Topiramate '
(E) Valproate Schouten R: Legal aspects of consultation, in Massachusetts General
Hospital Handbook of General Hospital Psychiatry, 5th ed. Edited
T h e correct response is option D: Topiramate by Stem TA, Fricchione GL, Cassem NH, Jellinek MS, Rosenbaum
JF. St Louis, Mosby, 2004, pp 349-362
Topiramate is the only drug of those listed whose use
has been associated with acute secondary angle-clo-
sure glaucoma. Symptoms typically begin early in the
course of treatment and include blurred vision, ocular 170
pain, a n d headache. If recognized early, full recovery
W h i c h of the following treatments is most effective for
is likely after drug discontinuation.
patients with bulimia nervosa and major depressive
Fraunfelder FW, Fraunfelder FT, Keates EU:Topiramate-associated disorder?
acute, bilateral secondary angle-closure glaucoma.
Ophthalmology 2004;111:109-111 (A) Cognitive behavior therapy
Topamax (topiramate) prescribing information (package insert) (B) fluoxetine
Thambi L, Kapcala LP, Chambers W, Nourjah P, Beitz J, Chen M, Lu S: (C) Imipramine
Topiramate-associated secondary angle-closure glaucoma: a case (D) Bupropion
series. Arch Ophthalmol 2002; 120:1108 (E) Combined cognitive behavior therapy and fluoxetine
The correct response is option E: C o m b i n e d cognitive
behavior therapy a n d fluoxetine

169 Cognitive behavior psychotherapy is the psychosocial


A married 50-year-old w o m a n is admitted to the hos- intervention for which there is the most evidence of
pital with an acute myocardial infarction. It is recom- efficacy. A high percentage of treatment-seeking
mended that she have a cardiac catheterization with a patients with eating disorders report a lifetime history
possible procedure based on the findings. T h e patient of unipolar depression. Depression can impair a
refuses. W h i c h of the following would be a reason- patient's ability to become meaningfully involved in
able approach to the patient at this time?
psychotherapy a n d may dictate the need for medica-
(A) Discharge the patient to home so that she can make up her tion treatment, for the mood symptoms from the begin-
mind. ning of treatment.
(B) Call for a family meeting with her husband and adult chil-
The only medication a p p r o v e d by the Food and Drug
dren to discuss the options.
(C) Tell her that there is no guarantee that she wouldn't die if Administration for bulimia nervosa is fluoxetine.
she leaves the hospital without treatment. Imipramine has also demonstrated efficacy. Bupro-
(D) Consider treatment for depression since she does not seem pion has been associated with seizures in purging
to want to live. bulimic patients, so its use is not recommended.
(E) Recommend treatment against her will because of the seri-
Practice Guideline for the Treatment cf Patients With Eating
ousness of the condition. Disorders, 2nd ed (2000), in American Psychiatric Association
Practice Guidelines for the Treatment of Psychiatric Disorders,
T h e correct response is option B: Call for a family
Compendium 2004. Washington, DC, APA, 2004, p 717
meeting with her husband and adult children to dis-
Schatzberg AF, Nemeroff CB (eds): The American Psychiatric
cuss the options
Publishing Textbook of Psychopharmacology, 3rd ed, Washington,
DC, American Psychiatric Publishing, 2004, pp 1032-1035
Kotler LA, Devlin MJ, Davies M, Walsh BT: An open trial of fluoxetine
for adolescents with bulimia nervosa. J Child Adolesc
Psychopharmacol 2003;13:329-335

Section 2: Answers and Explanations


171 173
W h i c h of the following describes the American The a m y g d a l a is most specifically involved in which of
A c a d e m y of Pediatrics statement regarding maternal the following brain functions?
lithium use during breast-feeding?
(A) Determining social behavior
(A) Associated with significant side effects in some nursing (B) Emotional coding of sensory cues
infants; use with caution (C) Generating normal sleep patterns
(B) Unknown effects on nursing infants; may be of concern (D) Recalling previously learned material
(C) Absolutely contraindicated (E) Signaling reward by exogenous substances
(D) Usually compatible The correct response is option B: Emotional coding of
T h e correct response is option A: Associated with sig- sensory cues
nificant side effects in some nursing infants; use with
The amygdala plays an important role in emotional func-
caution
tioning. The hippocampus is involved in recalling previ-
T h e A m e r i c a n A c a d e m y of Pediatrics Committee on ously learned material. T h e frontal cortex is more
Drugs has stated that lithium has been associated with involved in determining social behavior. The thalamus is
significant effects on some nursing infants and should more involved in generating normal sleep patterns. The
be given to nursing mothers with caution. Lithium con- nucleus accumbens a n d the ventral tegmentum are more
centrations in breast milk are approximately 40% of involved in signaling r e w a r d by exogenous substances.
maternal serum concentrations, and infant serum con-
Fudge JL, Emiliano AB: The extended amygdala and the dopamine
centrations may be similar to or lower man milk lev-
system: another piece of the dopamine puzzle. J Neuropsychiatry
els. Nonetheless, there is only one convincing case of Clin Neurosci 2003;15:306-316
lithium intoxication in a breast-feeding infant, and it Mah LW, Arnold MC, Grafman J: Deficits in social knowledge follow-
w a s associated with an intercurrent infection. M a n y ing damage to ventromedial prefrontal cortex. J Neuropsychiatry
experts do not v i e w this contraindication as absolute Clin Neurosci 2005; 17:66-74
Taber KH, Wen C, Khan A, Hurley RA:The limbic thalamus.
but rather recognize the need to balance benefits and
J Neuropsychiatry Clin Neurosci 2004; 16:127-132
risks on an individual basis. Tekin S, Cummings JL: Frontal-subcortical neuronal circuits and clin-
:
ical neuropsychiatry: an update. J Psychosom Res 2002; <3:647
Burt VK, Suri R, Altshuler L, Stowe Z, Hendrick VC, Muntean E:The
use of psychotropic medications during breast-feeding. Am J
Psychiatry 2001; 158:1001-1009
American Academy of Pediatrics: The transfer of drugs and other
chemicals into human milk. American Academy of Pediatrics
committee on drug policy statement Pediatrics 2001;
108:776-789. Available at
http^/aappolicy.aappublications.org/cgi/content/full/
pediatrics; 108/3/776.

172
Treatment with which of the following cytokines has
been linked to suicidal behavior?

(A) Erythropoietin
(B) Granulocyte colony-stimulating factor
(C) Interferon-a
(D) lnterleukin-1 receptor agonist
(E) Anti-tumor-necrosis-factor antibodies
T h e correct response is option C: Interferon-a

Interferon-a has been associated with fatigue, depres-


sion, cognitive impairment, psychosis, a n d suicidal
ideation. T h e other medications listed h a v e a negligi-
' ble or undetermined effect on mental state.

Kronfol Z, Remick DG: Cytokines and the brain: implications for clini-
cal psychiatry. Am J Psychiatry 2000; 157:683-694
Joe GW, Simpson DD, Danserau DF, Rowan-Szal GA: Relationships
174
between counseling rapport and drug abuse treatment outcomes.
A 22-year-old w o m a n wants to take an antidepressant PsychiatrServ 2001;52:1223-1229
for treatment of her major depression but is concerned Gottheil E,^einstein SP, Sterling RC, Lundy A, Serota RD: A random-
about possible sexual side effects. W h i c h of the fol- ized controlled study of the effectiveness of intensive outpatient
lowing medications is the best choice for her? treatment for cocaine dependence. Psychiatr Serv 1998;
49:782-787
(A) Bupropion Brown RA, Monti PM, Myers MG, Martin RA, Rrvinus T, Dubreuil ME,
(B) Clomipramine Rohsenow BJ: Depression among cocaine abusers in treatment
relation to cocaine and alcohol use and treatment outcomes. Am
(C) Escitalopram ,
J Psychiatry 1998; 155:220-225
(D) Sertraline '« -
Ritsher JB, Moos RH, Finney JW: Relationship of treatment orienta-
(E) Venlafaxine
tion and continuing care to remission among substance abuse
patients. Psychiatr Sen/ 2002; 53:595-601
The correct response is option A: Bupropion

In placebcn:ontrolled clinical trials bupropion has


been shown to be associated with far less orgasm dys-
function (similar to placebo] than fluoxetine, escitalo- 176
pram, and sertraline. In general, the agents that Weight gain is most likely to occur with which of the
increase serotonergic tone tend to be more likely to following antipsychotic drugs?
cause sexual dysfunction in both men and w o m e n . A
(A) Aripiprazole
large primary care survey also found more sexual dys-
(B) Clozapine
function with venlafaxine than with bupropion. Sexual
(C) Haloperidol
dysfunction is a common side effect of clomipramine.
(D) Ziprasidone
Croft H, Settle E Jr, HouserT, Batey SR, Donahue RM, Ascher JA: A (E) Risperidone
placebo-controlled comparison of the antidepressant efficacy and
The correct response is option B: C l o z a p i n e
effects on sexual functioning of sustained-release bupropion and
sertraline. Clin Ther 1999; 21:643-658
Studies of the various antipsychotic drugs have found
Clayton AH, Pradko JF, Croft HA, Montano CB, Leadbetter RA, Bolden-
weight gain to be most likely to occur with clozapine
Watson C, Bass Kl, Donahue RM, Jamerson BD, Metz A:
Prevalence of sexual dysfunction among newer antidepressants. and next most c o m m o n l y with o l a n z a p i n e . It is impor-
J Clin Psychiatry 2002; 63:357-366 tant to realize that not e v e r y o n e w h o takes those
Clayton A, Wightman D, Modell JG, Horrigan J, Richard NE: Effects in drugs gains w e i g h t a n d that weight g a i n can compli-
MOD on sexual functioning of bupropion XL, escitalopram, and
cate the use of other antipsychotic drugs as w e l l .
placebo in depressed patients, in 2005 Annual Meeting New
. Research Program and Abstracts. Arlington, Va, American O l a n z a p i n e a n d c l o z a p i n e are listed as having a
t

Psychiatric Association, 2003, number 821 comparable risk of w e i g h t g a i n in the summary table
of the schizophrenia practice guideline, a n d the dif-
ference between them, if any, is minor.

Allison DB, Mentire JL, Heo M, Chandler LP, Cappelleri JC, Infante
175 MC, Weiden PJ: Antipsychob'c-induced weight gain: a comprehen-
W h i c h of the following factors is U N R E L A T E D to a sive research synthesis. Am J Psychiatry 1999; 156:1686-1696
positive treatment outcome for cocaine dependence? American Psychiatric Association: Practice Guideline for the
Treatment of Patients With Schizophrenia, 2nd ed. Am J
(A) Counseling rapport Psychiatry 2004; 161(Feb suppl):1-56
(B) Treatment retention Zimmermann U, KrausT, Himmerich H, Schuld A, PollmacherT:
(C) Patient choice of program type Epidemiology, implications, and mechanisms underlying drug-
induced weight gain in psychiatric patients. J Psychiatr Res 2003;
(0) Comorbid depressive symptoms
37:193-220
T h e correct response is option C: Patient choice of
program type

A patient's choice of one program compared with


another does not seem to have a significant impact on
treatment outcome. Considerable research has been
done in recent years to try to uncover useful correla-
tions between treatment outcomes and patient char-
acteristics, program characteristics, and interaction
variables.

Section 2: Answers and Explanations 110


177 178
A 16-year-old female patient is admitted to the hospi- Despite intensive psychosocial treatment for alcohol
tal for gastric rupture. On interview, she reports that in dependence, a patient continues to drink alcohol. T h e
the past 2 months, whenever she has felt anxious, psychiatrist decides to recommend adjunctive med-
angry, or sad, she has eaten unusually large quantities ication. W h i c h of the following medications w o u l d
of cookies, candy, crackers, "or whatever I can get my N O T b e a n acceptable treatment?
hands on like I'm a maniac, totally out of control." She
reports that after gorging, she feels better emotionally
(A) Disulfiram
but is afraid that she will "get fat." She g a g s herself to
(B) Bromocriptine
induce vomiting, and recently she has begun drinking
(C) Naltrexone
ipecac as an emetic after such episodes. This morning,
(D) Ondansetron
after an episode of self-induced vomiting, she devel-
(E) Acamprosate
o p e d severe abdominal pain and w a s brought to the The correct response is option B: Bromocriptine
emergency department by her parents. On physical
examination, her height is 5'6" and her weight is Bromocriptine has been tested for the treatment of
120 pounds. T h e psychotherapy that has been found cocaine dependence with inconsistent results. The
to be most effective in treating her eating disorder is: other listed medications are used in the treatment of
alcohol dependence. Use of ondansetron in treating
(A) cognitive behavior therapy.
alcohol dependence is off-label, and this agent is
(B) family therapy.
used less commonly than the other three.
(C) group therapy. .
(D) psychodynamically oriented individual therapy. Johnson BA, Roache JD, Javors MA, DiClemente CC, Cloninger CR,
Prihoda T J , Bordnick PS, Ait-Daoud N, Hensler J: Ondansetron for
T h e correct response is option A: Cognitive behavior
reduction of drinking among biologically predisposed alcoholic
therapy patients: a randomized controlled trial. JAMA 2000; 284:963-971
Schuckit MA, Tapert S: Alcohol, in The American Psychiatric
T h e characteristics of this teenage girl's eating disorder
Publishing Textbook of Substance Abuse Treatment, 3rd ed. Edited
include frenzied binges in response to a dysphoric mood by Galanter M, Kleber HD. Washington, DC, American Psychiatric
state; compensatory purging out of fear of becoming fat; Publishing, 2004, p 162
and normal weight. This D'esentation is most consistent
with bulimia nervosa. More than 35 controlled studies
have found cognitive behavior therapy (CBT) to be the
most effective treatment for bulimia nervosa, and CBT is ]79
n o w considered the first-line treatment for this disorder. The anxiety disorder that includes a dissociation-like
Some empirical studies have demonstrated the efficacy phenomenon in its criteria is:
of family and group therapy and the use of selective
serotonin reuptake inhibitors in the treatment of bulimia.
(A) generalized anxiety disorder.
There is no research evidence to support the use of psy-
(B) obsessive-compulsive disorder.
(C) panic disorder.
chodynamically oriented individual therapy.
(D) posttraumatic stress disorder.
Lewis M (ed): Child and Adolescent Psychiatry: A Comprehensive (E) social phobia.
Textbook, 3rd ed. Philadelphia, Lippincott Williams & Wilkins,
The correct response is option D: Posttraumatic stress
2002,p 697
Mehler PS: Bulimia nervosa. N Engl J Med 2003; 349:875-881 disorder

Intrusive episodes of recall account for one of the three


major symptom complexes in PTSD. The others are
avoidance with numbing a n d increased arousal. An
integral part of the intrusive symptoms are flashbacks
that are thought to be dissociative experiences.
Dissociation may occur-in other anxiety disorders, such
as panic disorder a n d generalized anxiety disorder,
but it is not a regular component of those syndromes.

Stein DJ, Hollander E (eds): American Psychiatric Publishing Textbook


of Anxiety Disorders. Washington, DC, American Psychiatric
Publishing, 2002, pp 362-363,404
180 182
Patients with w h i c h of the following personality disor- A psychiatrist is treating an 8-year-old child of a
ders would be expected to benefit most from adjunc- divorced j i n g l e parent w h o is the child's custodial par-
tive pharmacotherapy? ent. T h e noncustodial parent wishes to be informed of
the child's source of problems and progress of treat-
(A) Borderline
ment. T h e psychiatrist should-share clinical informa-
(B) Schizoid tion with the^noncustodial parent:
(C) Antisocial
(D) Obsessive-compulsive (A) without consent of the custodial parent or the child.
(E) Dependent \ ] (B) only with the informed consent of the custodial parent and
the child.
The correct response is option A: Borderline
(C) only with informed consent of the custodial parent.
Targeting specific symptoms with medications rather (D) only with the informed consent of the child.

than disorders per se currently appears to be the most T h e correct response is option C: O n l y with informed
effective a p p r o a c h . Pharmacotherapy appears to be -consent of the custodial parent
most helpful for borderline personality disorder, given
that medications can be effective for problems with In order to share a n y clinical information with a third

affective dysfunction, impulsive-behavioral issues, and party, including the noncustodial parent, the psychia-

cognitive-perceptual problems. There is little evidence trist needs the informed consent of the custodial par-

for the utility of medications for treating the major symp- ent. Although it m a y be clinically advisable to involve

toms of antisocial personality disorder. Obsessive<om- the child in the process, it is not required, since an 8-

pulsive personality disorder a n d dependent personality year-old cannot g i v e legally valid informed consent.

disorder are treated with psychotherapy. American Academy of Child and Adolescent Psychiatry Code of
Ethics, Principles X and XVII. Available at
Phillips KA, Yen S, Gunderson JG: Personality disorders, in The http://'wvvw.aacap.org/galleries/AboutUs/'Code()fEthics.PDF
American Psychiatric Publishing Textbook of Psychiatry, 4th ed.
Edited by Hales RE, Yudofsky SC. Washington, DC, American
Psychiatric Publishing, 2003, pp 810-825

183
Cerebral ventricular enlargement, one of the most
181 consistent structural brain findings in patients with
schizophrenia, is most closely associated with:
W h e n assessing a patient's suitability for short-term
psychodynamic psychotherapy, of the following fac- (A) prominent negative symptoms.
tors, w h i c h is the most important? (B) rapid onset of the disorder.
(C) improved response rates to atypical antipsychotics.
(A) The DSM-IV-TR diagnosis
(D) retained memory- and language-processing capabilities.
(B) Family psychiatric history
(E) increased risk of developing tardive dyskinesia.
(C) Level of education
(D) An identifiable focus T h e correct response is option A: Prominent negative
(E) Need for psychoactive medication symptoms

The correct response is option D: An identifiable focus Prominent negative symptoms are associated with cere-

W h i l e different models for short-term dynamic therapy bral ventricular enlargement. The phenomena listed in

stress different selection criteria, there is general options B through D are more associated with nonen-

agreement that traditional diagnostic categories or larged ventricles in patients with schizophrenia. O t h e r

patient characteristics a r e less important than the abil- factors have been more directly associated with the risk

ity of therapist a n d patient to a g r e e on and maintain of tardive dyskinesia than structural abnormalities.

a defined focus for the treatment. Ho B, Black D, Andreasen N: Schizophrenia and other psychotic disor-
ders, in The American Psychiatric Publishing Textbook of Clinical
Hollender MH, Ford CV: Dynamic Psychotherapy: An Introductory
Psychiatry, 4th ed. Edited by Hales RE, Yudofsky SC. Washington,
Approach. Washington, DC, American Psychiatric Press, 1990,
DC, American Psychiatric Publishing, 2003, pp 405-407
pp 135-136
Malhotra AX, Murphy GM Jr, Kennedy JL: Pharmacogenetics of psy-
chotropic drug response. Am J Psychiatry 2004; 161:780-796

Section 2: Answers and Explanations


184 186
A patient with heroin dependence purchases a drug Treatments shown to be effective for smoking cessa-
on the street. The patient feels a mild opiate high but tion include all of the following EXCEPT:
then, despite continued injection of a sizable volume
(A) bupropion.
of drug, feels opiate withdrawal coming on. The drug
(B) brief advice intervention.
injected is most likely:
(C) 12-step programs.
(A) buprenorphine. (D) nicotine replacement therapy.
(B) heroin. The correct response is option C: 12-step programs
(C) methadone.
(D) naloxone. Twelve-step programs have not demonstrated effec-
tiveness in smoking cessation. All nicotine replace-
T h e correct response is option A: Buprenorphine
ment therapies including g u m , patch, nasal spray,
Buprenorphine is a partial opiate agonist or a mixed and inhaler, have been found to be effective for smok-
agonist-antagonist. Rapidly injecting high doses of a ing cessation. Brief advice techniques in w h i c h a per-
partial agonist in a highly dependent patient w h o has sonal benefit of cessation is identified and discussed
recently used a full agonist (e.g., heroin] can produce in less than 10 minutes increase smoking cessation
mild withdrawal symptoms. As the dose is increased, rates from 5% to 10%. Pharmacotherapy with bupro-
a drug effect ceiling is reached, both with respect to pion has been shown to be effective.
a n y drug-induced euphoria a n d with drug-induced
Rore MC, Bailey WC, Cohen SJ, et al: Treating Tobacco Use and
respiratory depression. As the dose continues to rise,
Dependence. Clinical practice guideline. Rockville, MD, US Dept of
the opiate antagonist effects become more predomi- Health and Human Services, June 2000. Available at
nant. N a l o x o n e is a full antagonist a n d would pro- http7/www.surgeongeneral.govAobaccoAreating_tobacco_use.pdf.
d u c e an immediate full withdrawal syndrome.

Stine SM: Opiate dependence and current treatments, in New treat-


ments for chemical addictions. Edited by McCance-Katz EF, Kosten
TR. Washington, DC, American Psychiatric Press, 1998, pp 75-111 187
O'Brien CP, Kampman KM: Opioids: Antagonists and partial agonists, in ,- Based on the mental status examination, the psychia-
The American Psychiatric Publishing Textbook of Substance Abuse
" -trist believes that a patient is delirious. The examination
Treatment, 3rd ed. Edited by Galanter M, Kleber HD. Washington,
reveals disorientation, changing levels of con-
DC, American Psychiatric Publishing, 2004, pp 275,305-320
sciousness, and visual illusions. W h i c h of the following
tests has the greatest evidence supporting its use in con-
firming a diagnosis of delirium?

185 (A) Positron emission tomography


W h i c h of the following would be the most important (B) Magnetic resonance imoging
consideration w h e n evaluating an individual for a (C) Computerized tomography
personality disorder? (D) EEG

(A) Culture The correct response is option D: E E G


(B) Intelligence In most delirious patients, the E E G demonstrates slow-
(C) Gender
ing and may be helpful in confirming the diagnosis.
(D) Socioeconomic status
During alcohol w i t h d r a w a l , the E E G m a y show an
(E) Education
' increased frequency. N o n e of the other options listed
T h e correct response is option A: Culture has value in the differential diagnosis.

Personality disorders are defined as an enduring pat- Wise MG, Rundell JR (eds): The American Psychiatric Publishing
tern of inner experience and behavior that deviates Textbook of Consultation-Liaison Psychiatry: Psychiatry in the
markedly from the expectations of the individual's' cul- Medically III, 2nd ed. Washington, DC, American Psychiatric
1
Publishing, 2002, p 263
ture. In general, intelligence, socioeconomic status,
gender, a n d education have not been determined to be
helpful in making a diagnosis of a personality disorder.

Cloninger CR, Svrakic DM: Personality disorders, in Kaplan and


Sadock's Comprehensive Textbook of Psychiatry, 7th ed. Edited by
Sadock BJ, Sadock VA Philadelphia, Lippincott Williams 8
Wilkins, 2000, pp 1738-1741

1
TI8 FOCI IS Psvchiatrv Review 4 0 0 Self-Assessment Question ;
188 190
T h e diagnosis of shared psychotic disorder is most A managed care organization ( M C O ] is refusing to
commonly found in which of the following groups? p a y for additional treatment days for a patient in an
inpatient psychiatric facility. The attending psychiatrist
(A) Couple relationships believes that the additional treatment d a y s m a y be
(B) Groups larger than two people . needed to ensure the patient's-safety. W h i c h of the fol-
(C) Groups of men, rather than women lowing statements is correct regarding this situation?
(D) Family blood relations
(E) Children and adolescents (A) The psychiatrist is legally responsible to abide by the MCO's
i decision.
i
(B) The psychiatrist is responsible for making provisions for
T h e correct response is option'A: C o u p l e relationships
continuity of needed care even if additional days are not
Shared psychotic disorder is more common in couples
covered by the MCO.
but is occasionally seen in groups. It often involves
(C) As long as the psychiatrist documents that the MCO will not
n o n b i z a r r e delusions and occurs more often in pay, the psychiatrist may discharge the patient.
w o m e n than men. It has a low r e c o v e r y rate. (D) The psychiatrist may inform the patient of his or her right
American Psychiatric Association: Diagnostic and Statistical Manual to appeal the MCO's decision only if there are no "gag
of Mental Disorders, Fourth Edition, Text Revision (DSM-IV-TR). clauses" that limit what the psychiatrist is allowed to say.
Washington, DC, American Psychiatric Association, 2000,
pp 332-334 The correct response is option B: T h e psychiatrist is
Sadock BJ, Sadock VA (eds): Kaplan and Sadock's Comprehensive responsible for making provisions for continuity of
Textbook of Psychiatry, 8th ed. Philadelphia, Lippincott Williams & needed care even if additional days a r e not c o v e r e d
Wilkins, 2005, p 1530 by the M C O

Even if a m a n a g e d care organization refuses to pay,


once a psychiatrist determines that the patient needs
189 further treatment, especially if the issue of safety is still
a concern, the psychiatrist is ethically responsible for
A colleague w h o is a cardiac surgeon asks for a psy-
providing care or arranging for an acceptable alter-
chiatrist's help in raising funds for a n e w wing of the
local hospital. The cardiac surgeon asks the psychia- native method of care. G a g clauses that limit physi-
trist to solicit patients for charitable contributions. The cian disclosure to patients are unethical.
psychiatrist's ethical response should be to agree to
Simon Rl: A Concise Guide to Psychiatry and Law for Clinicians, 3rd ed.
solicit funds from: Washington, DC, American Psychiatric Publishing, 2001, pp 37-38
Amei^an Psychiatric Association: Principles of Medical Ethics With
(A) only former patients, because there is no longer a doctor- Annotations Especially Applicable to Psychiatry. Washington, DC,
patient relationship. American Psychiatric Association, 2001 (annotation addendum 1)
(B) only wealthy patients who have the means to contribute.
(C) no patients, because of the nature of the psychiatrist-
patient relationship.
(D) both current and former patients, since patients can make
autonomous decisions.
The correct response is option C: No patients, because
of the nature of the psychiatrist-patient relationship

W h i l e the psychiatrist could be more articulate in


describing the pertinent difference between psychia-
try a n d cardiac surgery in regard to b o u n d a r y issues,
a direct request by the treating psychiatrist can risk
exploitation of the patient. The first option ignores the
unique features of the psychiatrist-patient relationship.
The remaining options raise only partial truths and
ignore counterbalancing considerations.

American Psychiatric Association: Opinions of the Ethics Committee


on the Principles of Medical Ethics With Annotations Especially
Applicable to Psychiatry. Washington, DC, American Psychiatric
Association, 2001 (section 1-Q, p 9)

Section 2: Answers and Explanations 119


T h e correct response is option D: Interpersonal psy-
191 chotherapy
T h e parents of a 14-year-old b o y bring him to a clinic
because he has been refusing to go to school. In ele- Interpersonal psychotherapy is a time-limited therapy
mentary a n d middle school, he w a s in a special edu- that focuses on relationships and interpersonal inter-
cation class for mildly mentally retarded students. actions to effect change in symptoms and behavior.
W h e n he has been at school, he ruminates about O n e aspect of interpersonal psychotherapy is that the
"something really b a d " happening to his mother or patient is formally given permission to be in the "sick
father. Recently, he has been a w a k e n i n g with night- role," in that their feelings are framed in terms of a
mares that his parents h a v e been killed. His parents medical illness. T h e therapy focuses on the present
h a v e had to stay with their son in order for him to get
and on real-life change more than on altering endur-
back to sleep. His medical history is significant for
ing aspects of the personality. Transference and
strabismus and scoliosis. Physical examination reveals
genetic dream interpretations are a v o i d e d .
a long face with prominent ears and jaw, a high arched
palate, hyperextensible finger joints, macroorchidism, Hales RE, Yudofsky SC (eds): The American Psychiatric Publishing
a n d flat feet. This boy's overall presentation is most con- Textbook of Clinical Psychiatry, 4th ed. Washington, DC, American
sistent with: Psychiatric Publishing, 2003, pp 1208-1209
Sadock BJ, Sadock VA (eds): Synopsis of Psychiatry: Behavioral
(A) Angelmon syndrome. Sciences/Clinical Psychiatry, 9th ed. Philadelphia, Lippincott
(B) fragile X syndrome. Williams & Wilkins, 2003, pp 933-935
(C) Prader-Willi syndrome. Markowitz JC: The clinical conduct of interpersonal psychotherapy.
(D) Sturge-Weber syndrome. FOCUS 2006;4:179-184 (180)
(E) Williams syndrome. 1
T h e correct response is option B: Fragile X syndrome

This boy's history and physical presentation are clas- 193


sic for fragile X syndrome. Fragile X syndrome is the A 29-year-old w o m a n presents to the emergency
most common inherited cause of mental retardation. department complaining of migraine headache. A re-
A n x i e t y disorders are more common in fragile X syn- v i e w of her medical file reveals one brief admission
drome than any other form of mental retardation. The for a transient psychotic episod? a n d depression
k e y components of this vignette a r e a mentally within the past 3 y e a r s . She is noted to be dressed in
retarded b o y with specific physical stigmata w h o has o d d clothing. She insists that she is clairvoyant and
telepathic. H e r speech is noted to be metaphorical,
d e v e l o p e d an anxiety disorder. This youth is suffering
o v e r e l a b o r a t e , a n d stereotyped. She says she has no
from separation anxiety disorder. Although the typical
close friends or confidante _ ther than her mother and
a g e at onset is at the start of elementary school, sep-
father, a n d that this has been the case since she w a s
aration anxiety disorder has a bimodal pattern of
a teenager. She is not particularly bothered about her
presentation, with a second peak emerging in ado- lack of companionship because she has fears of
lescence and typically associated with the .transition being h a r m e d in relationships. H e r presentation is
into high school. most consistent with which of the following personal-
ity disorders?
Dulcan MK, Martini DR, Lake MB: Concise Guide to Child and
Adolescent Psychiatry, 3rd ed. Washington, DC, American (A) Avoidant
Psychiatric Publishing, 2003, pp 63-71,183-185
(B) Histrionic
Lewis M (ed): Child and Adolescent Psychiatry: A Comprehensive
Textbook, 3rd ed. Philadelphia, Lippincott Williams & Wilkins,
(C) Paranoid
2002, pp 14-20,824-825
(D) Schizoid
(E) Schizotypal
T h e correct response is option E: Schizotypal

192 ' This vignette includes several features of each of the


a b o v e personality disorders. Patients with paranoid
W h i c h of the following therapies explicitly gives the
personality disorder also fear being harmed by others.
patient permission to be in the sick role?
It is similar to avoidant personality disorder in the lack
(A) Brief psychotherapy of close relationships, but the patient's indifference
(B) Cognitive behavior therapy about' this rules out this diagnosis. She is overly dra-
(C) Insight-oriented therapy matic in her presentation, w h i c h is suggestive of a
(D) Interpersonal psychotherapy
histrionic personality disorder, w h i c h could include
(E) Rational-emotional therapy
options D or E. H o w e v e r , the oddity of her other symp-

120 P O O JS Psvchiatrv Review: 400 Self-Assessment Ow-' n<


roms suggests that this is unlikely. She seems to best fit 195
into a cluster A personality disorder marked by oddity
A 30-year-old man with schizophrenia has made sev-
and eccentricity. Schizotypal personality disorder is
eral significant suicide attempts o v e r the past 10
distinguished from schizoid personality disorder by the
years in response to auditory c o m m a n d hallucina-
presence of odd beliefs or magical thinking. Also, her
tions. W h i c h of the following has been shown in stud-
comorbid diagnoses of a transient psychotic episode
ies to be most likely to reduce his risk for further
a n d depression a r e consistent with the diagnosis. suicidal behaviors?

Sadock BJ, Sadock VA (eds): Kaplan and Sadock's Comprehensive


r
Textbook of Psychiatry, 7th ed. Philadelphia, Lippincott Williams & (A) Aripiprazole
Wilkins, 2000, pp 1743-1744 (B) Clozapine
American Psychiatric Association: Diagnostic and Statistical Manual (C) lithium
of Mental Disorders, Fourth Edition, Text Revision (DSM-IV-TR). (D) Olanzapine
Washington, DC, American Psychiatric Association, 2000, p 701 (E) Risperidone
The correct response is option B: C l o z a p i n e
i
T h e 2-year International Suicide Prevention Trial
194 (InterSePT) found that suicidal behavior w a s signifi-
W h i c h of the following w o u l d be most appropriate as cantly less c o m m o n in patients treated with clozapine
initial pharmacotherapy for a patient with borderline than in those treated with o l a n z a p i n e in the high-risk
personality disorder w h o is exhibiting impulsivity and population studied. (The number of completed sui-
behavioral dysconfrol? cides w a s low for both patient groups: three for those
treated with c l o z a p i n e , a n d five for those treated with
(A) Sertraline
olanzapine.) W h i l e a substantial literature supports
(B) Clozapine
an antisuicide effect for lithium, the studies w e r e not
(C) Haloperidol
(D) Naltrexone with schizophrenia patients.
(E) Alprazolam Meltzer HY, AJpns L, Green Al, AJtamura AC, Anand R, Bertoldi A,
Bourgeois M, Chouinard G, Islam MZ, Kane J, Krishnan R,
T h e correct response is option A: Sertraline
Lindenmayer JP, Potkin S; International Suicide Prevention Trial
Study Group: Clozapine treatment for suicidality in schizophrenia:
Various medications have been shown to be effective
International Suicide Prevention Trial (InterSePT). Arch Gen
for specific symptoms or behavior patterns in patients Psychiatry 2003; 60:82-91
with borderline personality disorder. SSRIs are con- Meltzer HY: Suicide in schizophrenia, clozapine, and adoption of evi-
sidered the treatment of choice for impulsive, disin- denct-iased medicine. J Clin Psychiatry 2005; 66:530-533
hibited behavior in this patient group. Of the options Tondo L, Baltiessarini RJ, Hennen J, Floris G, Silvetti F, Tohen M:
Lithium treatment and risk of suicidal behavior in bipolar disorder
listed, initial treatment with an SSRI for the symptoms
patients. J Clin Psychiatry 1998; 59:405-414
of impulsivity a n d behavioral dysconfrol in patients
with borderline personality disorder has the most
empirical support. i

Use of antipsychotics is common for patients with bor-


derline personality disorder, but given the potential
side effects, these agents should not be used as a first-
line therapy for impulsivity and behavioral dysconfrol.
O n l y preliminary support is available to support the
use of naltrexone for this cluster of symptoms in bor-
derline personality disorder. Benzodiazepine treat-
ment may be associated with an increase in impulsivity
in patients with borderline personality disorder.

Practice Guideline for the Treatment of Patients With Borderline


Personality Disorder (2001), in American Psychiatric Association
Practice Guidelines for the Treatment of Psychiatric Disorders,
Compendium 2004. Washington, DC, APA, pp 773-775

Section 2: Answers and Explanations


The greatest challenge to gene therapy for the treat-
196
ment of psychiatric disorders is finding vectors to trans-
A 23-yecr-old patient with chronic schizophrenia com- fer genes into the nervous system. Neurons are fragile,
plains of a milky discharge from her nipples. M e d -
and so are some of the more difficult genes on which
ication-induced antagonism of w h i c h of the following
to perform g e n e therapy. Viral vectors tend to infect
receptors is responsible?
only a subset of the cells around them. Potential target
(A) Acetylcholine genes are still being delineated. It will be many years
(B) Dopamine before gene therapy will be clinically applicable.
(C) GABA
Sapolsky RM: Gene therapy for psychiatric disorders. Am J
(D) Norepinephrine
Psychiatry 2003;160:208-220
(E) Serotonin
T h e correct response is option B: Dopamine

T h e patient has galactorrhea, w h i c h is probably due


198
to antipsychotic drug-induced hyperprolactinemia.
This is more common with the older, conventional W h i c h of the following agents w o u l d be most appro-
priate for a geriatric patient w h o has Parkinson's dis-
antipsychotics a n d with risperidone among the newer
ease a n d agitation?
atypicals. D receptor stimulation by dopamine has
2

an inhibiting effect on prolactin secretion, and drug- (A) Risperidone


induced blockade of this receptor will lead to an (B) Diazepam
increased release of prolactin. Various explanations (C) Quetiapine
h a v e been proposed as to w h y atypical antipsychotics (D) Haloperidol
v a r y considerably with regard to their effect (or lack (E) Lithium
of effect) on prolactin.
T h e correct response is option C: Quetiapine •

Haddad PM, Wieck A: Antipsychotic-induced hyperprolactinaemia: If nonpharmacological interventions are ineffective, a


mechanisms, clinical features, and management Drugs 2004;
trial of an atypical antipsychotic such as quetiapine or
64:2291-2314
Miller KK: Management of hyperprolactinemia in patients receiving olanzapine may be initiated. Atypicol antipsychotics
• antipsychotics. CMS Spectr 2004; 9(suppl 7):28-32 that have been studied with the geriatric population
Practice Guideline for the Treatment of Patients With Schizophrenia, are clozapine, risperidone, o l a n z a p i n e , and quetia-
2nd ed (2004), in American Psychiatric Association Practice pine. A patient with parkinsonism may not be able to
Guidelines for the Treatment of Psychiatric Disorders,
tolerate even the minimal extrapyramidal side effects
Compendium 2004. Washington, DC, APA, pp 348-351
of risperidone.

Cheong JA: An evidence-based approach to the management of agi-


tation in the geriatric patient. FOCUS 2004; 2:197-205
197 , (pp 200-203)

W h i c h of the following statements is most accurate


regarding the current status of g e n e therapy for the
clinical treatment of psychiatric disorders?

(A) Gene theropy will be clinically applicable within the next


2 years.
(B) Finding vedors to transfer genes into the nervous system is
a challenge.
(C) Neurons are among the easiest cells into which to insert
new genes.
(D) Target genes for gene therapy have been dearly defined.
(E) Viral vedors quickly spread novel genes throughout the
nervous system.
T h e correct response is option B: Finding vectors to
transfer genes into the nervous system is a challenge

ill
:
Grof P, Alda M, Grof E, ; ox D, Cameron P: The challenge of predicting
199
response to stabilising lithium treatment the importance of
A 32-year-old man is brought to the emergency depart- patient selection. Br J Psychiatry 1993; 163(suppl 21):16—19
ment by his family, w h o notes that he has been spend- Jefferson JW, Goodnick PJ (eds): Predictors of Treatment Response
ing a lot of time sitting motionless in his room and in Mood Disorders. Washington, DC, American Psychiatric Press,
1996, pp 95-117
appears to be losing weight. In the past, he had been
fearful that family members w e r e poisoning his food,
but his parents state that he has not expressed those
concerns recently. On examination, he is disheveled
and poorly g r o o m e d , a n d he sits quietly in his chair 201
except for intermittent grimacing. He has minimally A female patient reveals during a psychotherapy session
spontaneous speech but will occasionally repeat the that she .does not enjoy sexual intercourse. She states
last few words of a question posed by the interviewer. that she is aroused by her partner but has sharp pains
His affect is generally restricted in range, a n d he does throughout intercourse. She cannot relax and enjoy sex
not answer questions about his mood, hallucinations, and has begun to avoid sex because of the anticipation
delusions, and suicidal or homicidal ideation. W h i c h • of the pain. W h a t is the most likely diagnosis?
of the following subtypes of schizophrenia w o u l d best
describe this patient's current presentation? (A) Dyspareunia
(B) Female orgasmic disorder
(A) Catatonic (C) Sexual masochism
(B) Disorganized (D) Sexual sadism
(C) Paranoid (E) Sexual aversion disorder
(D) Residual
(E) Undifferentiated The correct response is option A: Dyspareunia

The correct response is option A: Catatonic Sexual pain disorders are not a common chief com-
plaint in mental health settings. H o w e v e r , during psy-
This patient has had persecutory delusions during pre-
chotherapy a psychiatrist may become a w a r e of the
vious episodes of illness and is n o w di sorgan i z e d in
symptoms a n d should be able to recognize them. This
his a p p e a r a n c e . H o w e v e r , he is also exhibiting
is a classic description of dyspareunia.
motoric immobility, mutism, echolalia, and grimacing,
making his current presentation most consistent with American Psychiatric Association: Diagnostic and Statistical Manuar:
of Mental Disorders, Fourth Edition, Text Revision (DSM-IV-TR). •
the catatonic subtype of schizophrenia.
Washington, DC, American Psychiatric Association, 2000,
American Psychiatric Association: Diagnostic and Statistical Manual pp 554-556
of Mental Disorders, Fourth Edition, Text Revision (DSM-IV-TR). Hales RE, Yudofsky SC (eds): The American PsychiaMc Publishing
Washington, DC, American Psychiatric Association, 2000, Textbook of Clinical Psychiatry, 4th ed. Washington, DO, American
pp 313-317 Psychiatric Publishing, 2003, p 757

i
200
W h i c h of the following is most predictive of a favor-
able response to lithium in bipolar disorder?

(A) Comorbid substance abuse


(B) Depression-mania-euthymia course
(C) Euphoric mania
(D) Psychotic features
T h e correct response is option C: Euphoric mania

A g o o d clinical response to lithium therapy is pre-


dicted if the patient has the following features:
euphoric mania, nonrapid cycling, full inferepisode
remission, no comorbidity, lack of psychotic features,
few lifetime episodes, and a mania-depression-
euthymia clinical course.

Bowden CL: Efficacy of lithium in mania and maintenar.ee therapy of


bipolar disorder. J Clin Psychiatry 2000; 61(suppl 9):35-40

Section 2: Answers and Explanations


also brain growth, structure, and function. As a result,
202
children exposed to alcohol have an unusually high
A 25-year-old w o m a n presents with severe anxiety prevalence of intellectual impairment and disruptive
after finding out that her biological mother was
behavior disorders. This young boy exhibits evidence of
recently d i a g n o s e d with Huntington's disease. There
several psychiatric disorders in association with specific
is no family history of the disease on her father's side.
physical stigmata. His history suggests the presence of
She wishes to k n o w if she is affected. T h e probability
oppositional defiant disorder ( O D D ) , Attention deficit
that she is affected is:
hyperactivity disorder ( A D H D ) , and overall impairment in
(A) 0%. intellectual functioning leading to deficits in academic
(B) 25%. functioning. In utero exposure to marijuana has been
(C) 50%. linked to mild problems with attention and impulsivify but
(Dl 75%. has not been found to permanently affect intellectual func-
(E) 100%. tioning or cause craniofacial abnormalities. Cocaine
T h e correct response is option C: 50% may cause a relative stcte of hypoxia in fetuses that are
small for gestational age and have a small head circum-
Huntington's disease is inherited by an autosomal
ference. Some studies have shown attentional problems
dominant transmission. W i t h one affected parent and
in children exposed to cocaine in utero, but no specific
o n e unaffected parent, by chance, one could expect
physical abnormalities have been demonstrated.
5 0 % of offspring would be affected a n d 50% unaf-
Prenatal opiate exposure reduces birth weight and head
fected, but each individual child has a 5 0 % chance of
circumference. However, studies have found no differ-
developing the disease.
ences in early childhood between children w h o were
Sadock BJ, Sadock VA (eds): Kaplan and Sadock's Comprehensive exposed to opiates in utero and those w h o w e r e not. In
Textbook of Psychiatry, 8th ed. Philadelphia, Lippincott Williams & utero effects of nicotine have been linked to A D H D and
Wilkins, 2005, p 237 growth retardation, but no other sequelae.
Stern TA, Herman JB: Massachusetts General Hospital Psychiatry
Update and Board Preparation, 2nd ed. New York, McGraw-Hill,
Lewis M (ed): Child and Adolescent Psychiatry: A Comprehensive
2004, p 499 •
Textbook, 3rd ed. Philadelphia, Lippincott Williams & Wilkins,
2002, pp 449-453

203
A 9-ysar-old b o y is referred for evaluation because he 204
is having! ."temper tantrums" in school. He cannot sit W h i c h of the following is the most accurate statement
still, constantly disrupts the class, runs out in the hall regarding psychotherapy for posttraumatic stress dis-
without permission and refuses to obey directives from order (PTSD)?
the teacher. He frequently fights with his peers, and if
he does not get w h a t he wants, he yells, screams, (A) The therapist should be as nondirective as possible for the
throws objects, a n d flails about on the floor. Edu- psychotherapy to be effective.
cational testing reveals borderline intellectual function- (B) Multiple modalities of psychotherapy have proven effective
ing and significant delays in reading, writing, spelling, for PTSD.
a n d mathematics. On physical examination, the boy is (C) Psychotherapy must be combined with pharmacotherapy to
noted to be in the fifth percentile for head circumfer- be effective.
ence. He has short palpebral fissures, a thin upper lip, (D) Cognitive behavioral therapy (CBT) is of little value for
a n d a smooth philtrum. The b o y w a s most likely patients with PTSD.
exposed to which of the following drugs in utero?
The correct response is option B: Multiple modalities
(A) Alcohol of psychotherapy have proven effective for PTSD
(B) Cocaine
In meta-analyses of controlled trials of psychological
(C) Marijuana
(D) Nicotine treatments of PTSD, multiple forms of psychotherapy,

(E) Opiates including exposure therapy, cognitive behavioral ther-


apy, a n d psychodynamic therapy, have been shown
T h e correct response is option A: A l c o h o l to be effective.

T h e terafological effects of prenatal alcohol exposure Adhead G: Psychological therapies for post-traumatic stress disorder.
have been well studied and are described by fetal alco- Br J Psychiatry 2000; 177:144-148
hol syndrome. Alcohol is a direct neuroteratogen that
affects not only fetal facial morphology and growth but

If A
Sherman J J : Effects of psychotherapeutic treatments for PTSD: a
207
meta-analysis of controlled clinical trials. J Trauma Stress 1998;
11:413-435 A 32-year-old w o m a n develops anorgasmia while tak-
Davidson JRT: Effective management strategies for posttraumatic ing paroxetine. Switching to which of the following
stress disorder. FOCUS 2003; 1:239-243 (p 241) medications is most likely to resolve this problem?

(A) Gtalopram
(B) Venlafaxine
205 (C) Sertraline
(D) Bupropion
A husband and wife present fpr treatment because the
(E), Fluoxetine
wife is concerned. Her husband recently told her that
he believes he w a s b o r n a w o m a n . He states that he The correct response is option D: Bupropion
has always felt this w a y but can't fight it a n y m o r e . He
has started w e a r i n g dresses around the house after he A large survey of primary care clinics found that the
arrives home from w o r k at the end of the d a y . He says lowest risk of sexual dysfunction w a s with bupropion.
that he loves his wife and kids but that he needs to be ' Double-blind placebo-controlled studies found substan-
h a p p y as well. W h a t is the most likely diagnosis? tially more orgasm dysfunction with sertraline a n d with
fluoxetine than with bupropion.
(A) Exhibitionism
(S) Gender identity disorder Clayton AH, Pradko JF, Croft HA, Mcntano CB, Leadbetter RA, Bolden-
(C) Sexual arousal disorder Watson C, Bass Kl, Donahue RM, Jamerson BD, Metz A:
(D) Transvestic fetishism Prevalence of sexual dysfunction among newer antidepressants.
J Clin Psychiatry 2002; 63:357-366
(E) Voyeurism
Croft H, Settle E Jr, Houser T, Batey SR, Donahue RM, Ascher JA: A
T h e correct response is option B: G e n d e r identity dis- placebo-controlled comparison of the antidepressant efficacy and
effects on sexual functioning of sustained-release bupropion and
order
sertraline. ClinTher 1999; 21:643-658
This is a complicated disorder, but the scenario Coleman CC, King BR, Bolden-Watson C, Book MJ, Segraves RT,
Richard N, Ascher J, Batey S, Jamerson B, Metz A: A placebo-
describes someone w h o has been struggling with gen-
controlled comparison of the effects on sexual functioning of
der identity disorder despite functioning in culturally' bupropion sustained released and fluoxetine. Clin Ther 2001;
expected roles for a prolonged period. 23:1040-1058
Schatzberg AF, Nemeroff CB (eds): The American Psychiatric
American Psychiatric Association: Diagnostic and Statistical Manual
Publishing Textbook of Psychopharmacology, 3rd ed, Washington,
of Mental Disorders, Fourth Edition, Text Revision (DSM-IV-TR).
DC, American Psychiatric Publishing, 2004, Tables 52-57, p 859
Washington, DC, American Psychiatric Association, 2000,
pp 576-582 '•
Hales RE, Yudofsky SC (eds): Trie American Psychiatric Publishing
Textbook of Clinical Psychiatry, 4th ed. Washington, DC, American
Psychiatric Publishing, 2003, p 745

206
W h i c h of the following describes the pharmacokinet-
ics of children y o u n g e r than 12 years old?

(A) Children have a smaller volume of distribution than adults.


(B) Children have more efficient renal function than adults.
(C) Children metabolize through hepatic pathways more slowly
than adults.
(D) Children absorb medications more slowly than adults.
T h e correct response is option B: Children h a v e more
efficient renal function than adults

Children have more efficient renal elimination than


adults and therefore will clear drugs using this path-
w a y more quickly.

Martin A, Scahill L, Charney D, Leackman J (eds): Pediatric psy-


chopharmacology principles and practice. New York, Oxford
University Press, 2003, pp 48-50

Section 2: Answers and Explanations 125


Essential for the diagnosis of neuroleptic malignant
208
syndrome in a patient on antipsychotic medication
A 4-yearold boy is brought to the clinic by his parents are rigidity a n d elevated temperature. T w o or more of
with the chief complaint that "he keeps having night- the following symptoms are also required: diaphore-
mares." His parents report that for the past month, dur- sis, tachycardia, elevated or labile blood pressure,
ing the first one-third of the night, the b o y awakens from d y s p h a g i a , incontinence, tremor, changes in the level
his sleep with a startled scream. W h e n they enter the of consciousness ranging from confusion to coma,
room, they find that he has broken out in a sweat, is dif- mutism, leukocytosis, laboratory evidence of muscle
ficult to a w a k e n , and looks "scared to death." The next injury (e.g., elevated creatine kinase).
morning he has no recall of the event. These episodes
are most likely occurring during which stage of sleep? Sadock BJ, Sadock VA (eds): Kaplan and Sadock's Comprehensive
Textbook of Psychiatry, 8th ed. Philadelphia, Lippincott Williams &
(A) REM Wilkins, 2005, p 2714
(B) Stage 0—non-REM
(C) Stage 1 —non-REM
(D) Stage 2—non-REM
(E) Stage 3 or 4—non-REM 210
T h e correct response is option E: Stage 3 or 4 — n o n - A previously well 24-year-old w o m a n presented with a
REM 4-week history of progressively worsening expansive
irritable m o o d , pressured speech, racing thoughts,
Sleep terror disorder is a parasomnia. It occurs in grandiosity, a n d disfractibility. More recently she heard
d e e p non-REM (i.e., stages 3 or 4) sleepi This stage the voice of G o d proclaiming her to be a special mes-
of sleep occurs predominantly in the first third of the senger. W h i c h of the following is the most likely diag-
night. Although many parents assume that a child in nosis?
this state is having nightmares, in fact he has sleep ter-
(A). Brief psychotic disorder without marked stressor
ror disorder. According to DSM-IV-TR, the awakenings
(B) Bipolar disorder with psychotic features
from nightmares generally occur during R E M sleep.
(C) Schizoaffective disorder, bipolar type
Sadock BJ, Sadock VA: Kaplan and Sadock's Synopsis of Psychiatry, (D) Schizophrenia, catatonic subtype
9th ed. Philadelphia, Lippincott Williams & Wilkins, 2003. p 775 (E) Schizophreniform disorder
American Psychiatric Association: Diagnostic and Statistical Manual
of Mental Disorders, Fourth Edition, Text Revision (DSM-IV-TR). The correct response is option B: Bipolar disorder
Washington, DC, American Psychiatric Association, 2000, with psychotic features
pp 634-639
The patient meets DSM-IV-TR criteria for a manic
episode with mood-congruent auditory hallucinations.
The psychotic symptoms occurring only in the context
209 of a manic episode make the other diagnoses unlikely.

A 25-year-old male with a history of schizophrenia is American Psychiatric Association: Diagnostic and Statistical Manual of
hospitalized and treated with haloperidol and benz- Mental Disorders, Fourth Edition, Text Revision (DSM-IV-TR). Wash-
tropine. T h e patient becomes distressed, has a tem- ington, DC, American Psychiatric Association, 2000, pp 357-362,
413-415
perature of 103°F and has labile blood pressure.
Physical examination reveals hypertonicity, diaphore-
sis, a n d tachycardia. Laboratory studies reveal a cre-
atine kinase of 55,000 IU/L. W h a t is the most likely
diagnosis?
m
(A) Anticholinergic syndrome According to the principles of dialectical behavior
(B)CNS infection therapy, the core deficit in borderline personality dis-
(C) Malignant hyperthermia order is in:
(D) Neuroleptic malignant syndrome (A) regulation of affect. , ,(

(E) Serotonin syndrome (B) capacity for attachment.


(C) object constancy.
T h e correct response is option D: Neuroleptic malig-
(D) self-integration.
nant syndrome
(E) impulsive aggression. .

T h e correct response is option A: Regulation of affect

Dialectical b e h a v i o r therapy is based on the theory


that borderline symptoms primarily reflect dysfunction
of the emotion regulation system. Cognitive behavior 213
therapies v i e w the problem as cognitive distortions,
A patient with schizophrenia begins treatment with
a n d behavior management views the issue as learned
clozapine. The baseline white b l o o d , cell count
behavior. Empirical studies suggest that child abuse, 3
( W B C ] is 8100 ( n o r m a l = 4 5 0 0 - 1 1 , 0 0 0 / m m ] . T h e
incest, and early trauma may play a large role in
absolute neutrophil count (ANC) is 6200 (nor-'
development of borderline 'personality disorder. 3
m a l = 1 5 0 0 - 8 0 0 0 / m m ] . T h e tests remain normal in
Options B, C, a n d D are related to psychological for- weekly monitoring. After 3 months, the patient has
mulations of the disorder, a n d option E relates more to had significant clinical improvement, but the W B C
the biological concepts of .serotonergic dysfunction. drops to 3 2 0 0 , the A N C drops to 2 1 0 0 , a n d imma-
Early psychodynamic formulations postulated a lack ture cell forms are present on peripheral blood smear.
of object constancy a n d splitting of self a n d objects Repeat tests s h o w a W B C o f 3 1 0 0 , a n A N C o f
into "all g o o d " or "all b a d " as core problems in bor- 1900, a n d no immature cell forms. T h e physical
(

derline personality disorder. examination is normal, with no fever, sore throat, or


other sign of infection. W h a t would be the best next
Phillips KA, Yen S, Gunderson JG: Personality disorders, in The . step in the management of this patient?
American Psychiatric Publishing Textbook of Clinical Psychiatry,
4th ed. Edited by Hales RE, Yudofsky SC. Washington, DC, (A) Continue current dosage of clozapine and begin twice-
American Psychiatric Publishing, 2003, pp 810-825 weekly monitoring of the WBC and differential.
Moeller FG, Barratt ES, Dougherty DM, Schmitz JM, Swann AC: (B) Immediately and permanently discontinue clozapine.
Psychiatric aspects of impulsivity. Am J Psychiatry 2001; (C) Interrupt clozapine therapy until the WBC is normal, and
158:1783-1793 then resume treatment.
McMain S, Korman LM, Dimeff L: Dialectical behavior therapy and
(D) Reduce the dose of clozapine and begin weekly monitoring
the treatment of emotion dysregulation. J Clin Psychol 2001;
57:183-196
of the WBC and differential.
Cloninger CR, Svrakic DM: Personality disorders, in Kaplan and (E) Routinely monitor the WBC and differential unless the
Sadock's Comprehensive Textbook of Psychiatry, 7th ed. Edited by patient develops signs and symptoms of infection.
Sadock BJ, Sadock VA. Philadelphia, Lippincott Williams &
The correct response is option A: Continue current
Wilkins, 2000, p 1757
dosage of clozapine a n d begin twice-weekly monitor-
ing of the W B C a n d differential

A prorocol has been established for monitoring the


2]2 hematologic effects of clozapine. In this vignette, the
According to the A m e r i c a n Psychiatric Association patient's W B C a n d A N C have dropped from baseline,
guidelines, which of the following is true regarding c but the patient has demonstrated an excellent response
psychiatrist engaging in a sexual relationship with a .. to medication a n d has no signs or symptoms of infec-
former patient? tion. H o w e v e r , the drop in W B C a n d A N C are not
considered large enough to disrupt treatment. G i v e n
(A) Acceptable provided at least 2 years have passed since the
termination of the doctor-patient relationship the clinical response in a treatment-resistant patient, it

(B) Acceptable provided at least 5 years have passed since the is recommended that the patient be continued on the
termination of the doctor-patient relationship dose of clozapine that is effective and for the clinician
(C) Acceptable provided the former patient initiates the rela- to monitor the W B C a n d differential more frequently.
tionship and it is clear to both parties that no exploitation is For greater decreases in the W B C or A N C , it may be
taking place necessary to interrupt clozapine treatment temporarily
(D) Unethical no matter how long it has been since the termi- until these values return to safer levels or to immedi-
nation of the doctor-patient relationship ately and permanently discontinue clozapine treatment

T h e correct response is option D: Unethical no matter if there are concomitant signs of infection.
h o w long it has been since the termination of the doc-
Schatzberg AF, Cole JO, DeBattista C: Manual of Clinical
tor-patient relationship Psychopharmacology. Washington, DC, American Psychiatric
Publishing, 2005, p 186
W h i l e the issue of sexual relationships with former
Alvir JMJ, Lieberman JA, Safferman AZ, Schwimmer JL, Schaaf JA:
patients is not without controversy, psychiatrists should Clozapine-induced agranulocytosis. N Engl J Med 1993;
be a w a r e of the current position of the American 329:162-167
Psychiatric Association, which forbids sex with former
patients.

American Psychiatric Association: Principles of Medical Ethics With


Annotations Especially Applicable To Psychiatry. Washington, DC,
American Psychiatric Association, 2001 (section 2, annotation 1)

Section 2: Answers and Explanations 12/


Foa EB, Davidson JRT, Frances A: The Expert Consensus Guideline
214 Series: Treatment of Posttraumatic Stress Disorder. J Clin
A c c o r d i n g to DSM-IV-TR, which personality disorder Psychiatry 1999; 60(suppl 16): 1-76 (p 12)

cannot be diagnosed in children a n d adolescents?

(A) Paranoid
(B) Dependent 216
(C) Schizotypal
A 54-year-old w o m a n is hospitalized with hyperther-
(D) Borderline
mia, myoclonus, delirium, and autonomic instability.
(E) Antisocial
W h i c h of the following medication combinations would
T h e correct response is option E: Antisocial be most likely to cause this clinical presentation?

Antisocial personality disorder cannot be diagnosed in (A) Bupropion and venlafaxine


individuals under the age of 18 y e a r s . T h e other per- (B) Desipramine and escitalopram
sonality disorders can be diagnosed if the maladaptive (C) Duloxetine and fluoxetine
personality traits are pervasive, persistent, and unlikely (D) Paroxetine and phenelzine
to be limited to a particular developmental stage or (E) Sertraline and buspirone
episode of an axis I disorder. H o w e v e r , traits of a per- T h e correct response is option D: Paroxetine and
sonality disorder that appear in childhood frequently phenelzine
change in adult life". To diagnose a personality disorder
in an individual under age 18, the features must have T h e patient's symptoms are consistent with a serotonin

been present for at least 1 year. T h e o n e exception to syndrome. Monocmine oxidase inhibitors, such as

this is antisocial personality disorder, which cannot be phenelzine, combined with serotonergic antidepres-

diagnosed in individuals under a g e 18. This is because sants pose a g r a v e risk; hence, such combinations are

until that a g e , the behaviors associated, with antisocial contraindicated.

personality disorder are better explained by conduct Lane R, Baldwin D: Selective serotonin reuptake inhibitor-induced
disorder, a diagnosis of childhood a n d adolescence. serotonin syndrome: review. J Clin Psychopharmacol 1997;
17:208-221
Cloninger CR, Svrakic DM: Personality disorders, in Kaplan and Beasley CM Jr, Masica DN, Heilicc-pstein JH, Wheadon. DE, Zerbe RL
Sadock's Comprehensive Textbook of Psychiatry, 7th ed. Edited by Possible monoamine oxidase inhibitor-serotonin uptake inhibitor
Sadock BJ, Sadock VA. Philadelphia, Lippincott Williams & interaction: fluoxetine clinical data and preclinical findings. J Clin
Wilkins, 2000, pp 1739-1741 Psychopharmacol 1993; 13:312-320
American Psychiatric Association: Diagnostic and Statistical ManuaJ
of Mental Disorders, Fourth Edition, Text Revision (DSM-IV-TR).
Washington, DC, American Psychiatric Association, 2000, p 687

217
In addition to lithium, which of the following is rec-
215 • ommended as a first-line monotherapy for bipolar I
disorder, depressed mood, in the revised A P A Practice
T h e first-line treatment of choice (determined by expert
Guideline for the Treatment of Patients W i t h Bipolar
consensus) for acute posttraumatic stress disorder
Disorder (2002)?
(PTSD) milder severity is:
(A) Lamotrigine
(A) low-dose venlafaxine.
(B) Divalproex
(B) psychotherapy.
(C) Gabapentin
(C) combination of a mood stabilizer and psychotherapy.
(D) Bupropion
(D) any selective serotonin reuptake inhibitor (SSRI).
T h e correct response is option A: Lamotrigine
T h e correct response is option B: Psychotherapy
T h e Practice Guideline recommends the initiation of
T h e expert panel felt that for milder-severity acute
treatment of bipolar depression with lithium or lamo-
P T S D , psychotherapy first w a s the treatment of choice,
trigine; it further states that monotherapy with conven-
although the preferred first-line treatment for chronic
tional antidepressants is not recommended " g i v e n the
PTSD or for more severe acute P T S D is either psy-
risk of precipitating a switch into m a n i a . " A large dou-
chotherapy first or combined medication and psy-
ble-blind monotherapy study of bipolar I depression
chotherapy. • This recommendation holds true for
found lamotrigine to be more effective than placebo
children, adolescents, adults, and geriatric pojients.
on most outcome measures. There h a v e been no pub-
lished controlled studies of divalproex or gabapentin.
American Psychiatric Association: Practice Guideline for the 219
Treatment of Patients With Bipolar Disorder (Revision). Am J
Psychiatry 2002; 159(April suppl). Reprinted in FOCUS 2003; A 55-year-old man presents with depressed mood,
1:64-110 (p 65) poor concentration, poor appetite, feelings of worth-
Calabrese JR, Bowden CL, Sachs GS,.Ascher JA, Monaghan E, Rudd lessness, a n d insomnia 4 weeks after alcohol cessa-
GD (Lamictal 602 Study Group): A double-blind placebo-controlled
tion. T h e r e is no history of mania. W h i c h of the
study of lamotrigine monotherapy in outpatients with bipolar I
following is the best next step?
disorder. J Clin Psychiatry 1999; 60:79-88
(A) Begirran antidepressant.
(B) Begin a sleep aid.
(C) Begin an anticonvulsant.
218 (D) Begin to phase-advance sleep onset.
A 15-year-old African American male high school
(E) Waif 7-10 days, then reassess.
freshman is referred fo a psychiatrist because of The correct response is option A: Begin an antide-
increasing oppositional behavior at school. In middle pressant
school he w a s an honor roll student, played soccer,
and was on student council, all of which he continued The patient has symptoms of a major depressive
in his first 9 weeks of high school. On the weekends, episode that h a v e persisted for 2 weeks. Rather than
he volunteers at a local Boys and Girls Club and plays addressing insomnia as a symptom in isolation, it is
the keyboard at his church. After a couple of sessions, preferable to begin treatment for the depressive dis-
he finally admits that he needed to " p r o v e myself to order. Previous investigations have suggested waiting
my boys because they said I w a s 'acting white'."
30 days after onset of abstinence before making a
W h i c h of the following is the most likely reason for his
diagnosis of a mood disorder. H o w e v e r , recent data
peers' denigration?
suggest that persistence of mood symptoms 2 w e e k s
(A) Being on student council after cessation of drinking merits treatment.
(B) Doing volunteer work
Nunes EV, Levin FR: Treatment of depression in patients with alcohol
(C) Having honor roll grades or ether drug dependence. JAMA 2004; 291:1887-1896
(D) Playing soccer Brady KT, Malcolm RJ: Substance use disorders and co-occurring
(E) Playing the keyboard axis I psychiatric disorders, in The American Psychiatric
Publishing Textbook of Substance Abuse Treatment, 3rd ed. Edited
The correct response is option C: Having honor roll
by Galanter M, Kleber HD. Washington, DC, American Psychiatric
grades Publishing, 2004, pp 529-538

A subset of African American culture that particularly


affects male adolescents devalues academic perfor-
mance and emphasizes aggressive and "street"
220
behavior. The clinician should be a w a r e of this phe-
nomenon in the African A m e r i c a n community and A 16-year-old girl with depression has suicidal ide-
take this into account when assessing a student whose ation. W h i c h of the following characteristics is the
grades and behavior c h a n g e . most strongfy associated with a greater risk of c o m -
pleted suicide?
Day-Vines Nl, Patton JM, Baytops JL: Counseling African American
adolescents: the impact of race, culture, and middle class status. (A) Limited cognitive abilities
Professional School Counseling 2003; 7:40-51 (B) Perfedionist characteristics
Day-Vines NL, Day-Hairston BO: Culturally congruent strategies for (C) Previous suicide attempt
addressing the behavioral needs of urban African American mala (D) Strong religious beliefs
adolescents. Professional School Counseling 2005; 8:236-243 (E) Superficial cutting of forearms
The correct response is option C: Previous suicide
attempt

A previous suicide attempt is the most potent predic-


tor of suicide in girls.

American Academy of Child and Adolescent Psychiatry: Practice


Parameters for the Assessment and Treatment of Children and
Adolescents With Suicidal Behavior. J Am Acad Child Adolesc
Psychiatry 2001; 40(suppl 7):26S-32S

Section 2: Answers and Explanations


Yudofsky SC, Hales RE (eds): The American Psychiatric Publishing
22] Textbook of Neuropsychiatry and Clinical Neurosciences, 4th ed.
A psychiatrist routinely receives free golf outings, con- Washington, DC, American Psychiatric Publishing, 2002,
cert tickets, a n d dinners as gifts from a local pharma- pp 326-329
Stern TA, Herman JB: Massachusetts General Hospital Psychiatry
ceutical representative. Which of the following
Update and Board Preparation, 2nd ed. New York, McGraw-Hill,
statements most adequately describes the ethics of this
2004, p 493
practice? Kendler KS, Eaves U (eds): Psychiatric Genetics. Review of
Psychiatry, vol 24. Washington, DC, American Psychiatric
(A) It is ethical if no single gift is worth more than S250.
Publishing, 2005, p 55
(B) Self-monitoring and self-regulation are the most effective
ways of minimizing harm from conflicts of interest.
(C) There is no evidence that pharmaceutical company market-
ing to physicians influences physicians' behavior.
(D) This is a conflict of interest for the psychiatrist.
223
W h i c h of the following is the most common psychiatric
T h e correct response is option D: This is a conflict of
disturbance among adolescents w h o die by suicide?
interest for the psychiatrist
(A) Schizophrenia
Self-monitoring a n d self-regulation actions by the psy-
(B) Depressive disorders
chiatrist a r e important but usually are not seen as suf-
(C) Antisocial behavior/conduct disorder
ficient to prevent abuse due to conflicts of interest. (D) Anxiety disorders
Pharmaceutical company marketing to physicians (E) Alcohol dependence
affects physicians' behavior. 1
The correct response is option B: Depressive disorders
American Medical Association: Council on Ethical and Judicial Affairs
of the American Medical Association, 2000-2001, Opinion 8.061 A p p r o x i m a t e l y one-half to two-thirds of adolescent sui-
Wazana A: Physicians and the pharmaceutical industry: is a gift ever cide victims have a depressive disorder, with the odds
just a gift? JAMA 2000; 283:373-380
ratio for increased suicide risk in those with an affec-
Thompson DF: Understanding financial conflicts of interest. N Engl J
tive disorder ranging, in various studies, from 11 to
Med 1993; 329:573-576
27. Substance use and abuse are highly comorbid,
particularly in male suicido- completers, and conduct
disorder has been reported in about one-third of male
222 suicide victims. Few adolescent suicides are related to
schizophrenia.
T h e process of g e n e mapping, performed to deter-
mine whether or not a particular allele occurs more Gould MS, Greenberg T, voting DM, Shaffer D: Youth suicide risk and
frequently than by chance in affected individuals, is preventive interventions: a review of the past 10 years. J Am
k n o w n as w h i c h type of study? Acad Child Adolesc Psychiatry 2003; 42:386-405

(A) Twin
(B) Linkage
(C) Association
224
(D) Family
(E) Segregation analysis W h i c h of the following is the best medication treat-
ment for premature ejaculation?
T h e correct response is option C: Association
(A) Bupropion
Association studies can examine whether a particular (B) Lorazepam
allele occurs more frequently than by chance by com- (C) Paroxetine
paring affected a n d unaffected individuals. Twin, and (D) Risperidone
family studies are not gene-mapping studies, Linkage (E) Trazodone
. studies, a type of gene-mapping study, examine
The correct response is option C: Paroxetine
w h e t h e r t w o or more genetic loci a r e co-inherited
>

m o r e often than expected by c h a n c e . A segregation Premature ejaculation is the persistent or recurrent


analysis is used to determine mode of inheritance onset of o r g a s m and ejaculation with minimal sexual
(dominant, recessive, etc.). stimulation before, on, or shortly after penetration and
before the person wishes it. Typically it is a problem
•Sadock BJ, Sadock VA (eds): Kaplan and Sadock's Comprehensive
Textbook of Psychiatry, 8th ed. Philadelphia, Lippincott Williams & in y o u n g m e n , w h o eventually develop behavioral
Wilkins, 2005, pp 256-257 * strategies to delay ejaculation. H o w e v e r , some men
never d e v e l o p the ability or lose if because of
decreased frequency of sexual activity or perfor- 227
mance anxiety or as a component of erectile dysfunc-
W h i c h of the following is the most common side effect
tion. It can be a problem in recovering substance
of chcjinesterase inhibitors?
abusers w h o h a v e relied on the substances to delay
ejaculation. SSRIs h a v e been s h o w n to be a g o o d (A) Anorexia
treatment for premature ejaculation, with paroxetine (B) Muscle cramps
being the medication that delays ejaculation the most. (C) Nausea
(D) Somnolence
American Psychiatric Association: Diagnostic and Statistical Manual (E) Syncope
of Mental Disorders, Fourth Edition.'Text Revision (DSM-IV-TR).
Washington, DC, American Psychiatric Association, 2000, T h e correct response is option C: N a u s e a
pp 552-554
Schatzberg AF, Nemeroff CB (eds): The American Psychiatric N a u s e a , reported in 11 % - 4 7 % of patients, is the
Publishing Textbook of Psychopharmacology, 3rd ed, Washington, most common a d v e r s e effect of the cholinesterase
DC, American Psychiatric Publishing, 2004, pp 238,272
inhibitors (donepezil, rivastigmine, and galantamine).
Vomiting is the next most c o m m o n side effect,
reported in 10%-31% of patients. Diarrhea w a s
reported in 5 % - 1 9 % of patients, and anorexia in
225
4%-17%. Other, less frequent side effects include
The highest rates of posttraumatic stress disorder insomnia, muscle cramps, syncope, fatigue, abnormal
(PTSD) have been reported to be induced b y : dreams, incontinence, a n d b r a d y c a r d i a .

(A) combat. Cummings JL: Use of cholinesterase inhibitors in clinical practice:


(B) sexual assault. evidence-based recommendations. Am J Geriatr Psychiatry 2003;
(C) natural disasters. 11:131-145
(D) motor vehicle accidents.
T h e correct response is option B: Sexual assault

Assaultive violence, including sexual assault, pro- 228


duces the highest rates of PTSD, compared with other There is accumulating evidence suggesting that all of
precipitating traumas (i.e., combat, natural disasters, the following psychotherapies are beneficial in bipo-
a n d motor vehicle accidents). lar I disorder EXCEPT:

Breslau N, Kessler RC, Chilcoat HD, Schultz LR, Davis GC, Andreski P: (A) interpersonal and social rhythm therapy.
Trauma and posttraumatic stress disorder in the community: the (B) cognitive behavioral therapy."
1996 Detroit Area"Survey of Trauma. Arch Gen Psychiatry 1998; (C) family therapy.
55:626-632
(D) psychoanalysis.
The correct response is option D: Psychoanalysis

There is peer-reviewed evidence suggesting that inter-


226
personal a n d social rhythm therapy, family therapy,
W h i c h of the following laboratory test results is ele- and cognitive behavioral therapy m a y decrease
vated in some patients with anorexia nervosa? cycling a n d decrease the severity of bipolar I disor-
(A) Amylase der. There a r e no controlled studies demonstrating
(B) Magnesium that psychoanalysis decreases the frequency or sever-
iC] Phosphate ity of episodes of mania or depression.
(D) Potassium
Rapaport MH, Hales D: Relapse prevention and bipolar disorder: a
(E) Zinc focus on bipolar depression. FOCUS 2003; 1:15-31 (p 21)
The correct response is option A: Amylase Miklowitz DJ, Simoneau TL, George EL, Richards JA, Kalbag A,
Sachs-Ericsson N, Suddath R: Family-focused treatment of bipolar
Values for the other tests are often decreased in patients disorder: 1 -year effects of a psychoeducational program in con-
junction with pharmacotherapy. Biol Psychiatry 2000;
with anorexia nervosa.
48:582-592
American Psychiatric Association: Diagnostic and Statistical Manual Frank E, Novick D: Progress in the psychotherapy of mood disorders:
of Mental Disorders, Fourth Edition, Text Revision (DSM-IV-TR). studies from the Western Psychiatric Institute and Clinic.
Washington, DC, American Psychiatric Association, 2000, p 586 Epidemiol Psichiatr Soc 2001; 120:245-252

Section 2: Answers and Explanations


229 23]
A 45-year-old patient with heroin dependence is A psychiatric referral is requested to evaluate a 25-
admitted to the infectious disease service for intra- year-old w o m a n v / h o wishes to undergo a second
venous antibiotic treatment of bacterial endocarditis. rhinoplasty because, she states, "the first one left my
An H I V test is negative. There is no other past psychi- nose too big." In tears, the patient states that her dis-
atric history. Opiate withdrawal is adequately con- comfort about the a p p e a r a n c e of her nose prevents
trolled with oral methadone. On hospital d a y 3, the her from having an active social life. She pleads with
patient becomes acutely anxious, has moderate tachy- the psychiatrist to render an opinion that will permit
c a r d i a , a n d asks to be discharged from the hospital. the surgery. T h e patient does not a p p e a r psychotic.
A low-grade fever develops, but blood cultures are She does not express a n y other obsessional thoughts.
negative a n d a complete blood count shows no signif- In the psychiatrist's opinion, the patient's nose is
icant increase or shift in leukocytes. T h e most likely unremarkable. W h i c h of the following disorders is the
explanation for the change in the patient's condition is: most likely dicgnosis for this patient?

(A) an occult infection. (A) Delusional disorder, somatic type


(B) alcohol or sedative-hypnotic withdrawal. (B) Obsessive-compulsive disorder
(C) an undiagnosed anxiety disorder. (C) Body dysmorphic disorder
(D) a medication reaction, most likely to the antibiotic. (D) Hypochondriasis
(E) Somatization disorder
T h e correct response is option B: Alcohol or sedative-
hypnotic w i t h d r a w a l The correct response is option C: Body dysmorphic
1 disorder
T h e patient's tachycardia and symptoms of anxiety are
This patient demonstrates the diagnostic criteria for
consistent with the time frame for alcohol or sedative-
body dysmorphic disorder. She is preoccupied with a
hypnotic w i t h d r a w a l . Multiple drug use is common in
perceived defect in her a p p e a r a n c e , a n d this concern
patients with substance dependence. Rather than
affects her adaptation socially. On the basis of the
intentionally concealing polydrug use, a patient may
information g i v e n , there are no signs or symptoms of
be much more focused on a drug-ofchoice to the point
a delusional disorder or an obsessive-compulsive dis-
that abuse of.other substances is not a c k n o w l e d g e d .
order. She does not believe she has a serious illness,
American Psychiatric Association: Practice Guideline for the
which usually is associated with hypochondriasis, and
Treatment of Patients With Substance Use Disorders: Alcohol,
Cocaine, Opioids. Am J Psychiatry 1995; 152(Nov suppl) she does not have the list of physical complaints that
define somatization disorder.

American Psychiatric Association: Diagnostic and Statistical Manual


of Mental Disorders, Fourth Edition, Text Revision (DSM-IV-TR).
230 Washington, DC, American Psychiatric Association, 2000,
pp 507-511
Long-term treatment with which of the following med-
ications has been demonstrated to reduce suicide risk
in bipolar disorder?

(A) Carbamazepine
(B) Divalproex
(C) Lithium
(D) Olanzapine
T h e correct response is option C: Lithium

Analyses of the results of many studies support a


marked reduction in suicide rates and suicide attempts
during long-term lithium treatment.

Tondo L, Hennen J, Baldessarini RJ: Lower suicide risk with long-


term lithium treatment in major affective illness: a meta-analysis.
Acta Psychiatr Scand 2001; 104:163-172
Baldessarini RJ, Tondo L, Hennen J: Treating the suicide patient with
bipolar disorder: reducing suicide risk with lithium. Ann NY Acad
• Sci 2001; 932:24-38
Jefferson JW: Bipolar disorders: a brief guide to diagnosis ancktreat-
ment FOCUS 2003; 1:7-14 (p 12)

T.l? CAH'C r>... i:.._.P,..:-,.. d o n c . ! f J , r»


232 234
A 50-year-old man is treated with several trials of sin- A 42-year-old morbidly obese man is seen for chronic
gle antidepressants. His unipolar depression has been fatigue> Findings on polysomnography indicate
only partially responsive. W h i c h of the following obstructive sleep a p n e a . If the sleep apnea is left
agents has the best evidence from randomized con- untreated over a prolonged period, which of the fol-'
trolled trials to support its use in augmenting his anti- lowing conditions is most likely to develop?
depressant?
(A) Cataplexy
(A) Bupropion t
(B) Catalepsy
(B) Buspirone \ < (() Pulmonary hypertension
(C) Lithium '(D) Obstructive pulmonary disease
(D) Methylphenidate (E) Sleejj paralysis
(E) Triiodothyronine (T )
3

The correct response is option C: Pulmonary hyper-


The correct response is option C: Lithium tension

Lithium is the best studied augmentation agent in the Long-standing sleep apnea is associated with
treatment of unipolar depression. Most studies have increased pulmonary blood pressure and eventually
been with lithium augmentation of tricyclic antide- increased systemic blood pressure as well. These
pressants. F e w e r data are available on the effective- changes may account for a considerable number of
ness of T or stimulants, although both are used.
3
cases in which the diagnosis is essential hypertension.

Dubovsky SL, Dubovsky AN: Concise Guide to Mood Disorders. Sadock BJ/SadockVA: Kaplan and Sadock's Synopsis of Psychiatry,
Washington, DC, American Psychiatric Publishing, 2002, p 225 9th ed. Philadelphia, Lippincott Williams & Wilkins, 2003, p 770
Marangell LB, Silver JM, Goff DC, Yudofsky SC: Psychopharmacology
and electroconvulsive therapy, in The American Psychiatric
Publishing Textbook of Clinical Psychiatry, 4th ed. Edited by Hales
RE, Yudofsky SC. Washington, DC, American Psychiatric
235
Publishing, 2003, pp 1070-1072
A patient in psychotherapy believes that her therapist
wants to help her, she characteristically trusts him with
v e r y private material, a n d she has at times expressed
233 her feeling that they have many things in common a n d
that in many w a y s she v i e w s him as a role model. This
A 65-year-old man seen in the emergency department
patient's alliance is best characterized as:
is agitated, tachycardic, hypertensive, and tremulous.
He sees fish swimming on the wall: "It's just like watch- (A) erotic.
ing television." T h e most likely diagnosis is: (B) idealized.
(C) positive.
(A) delirium.
(D) primitive.
(B) delusional disorder.
(E) mirroring.
(C) depression.
(D) obsessive-compulsive disorder. The correct response is option C: Positive
(E) schizophrenia.
It is important to recognize an " a v e r a g e expectable"
The correct response is option A: Delirium positive transference a n d not to confuse it with other
transference configurations. W h i l e positive transfer-
Visual hallucinations suggest the need to rule out an
ence has elements of idealization in it, this is still v e r y
organic cause related to a delirium. Rarely, patients
different from an idealized transference per se. In
with schizophrenia, mania, or depression m a y expe-
order for therapy to proceed most effectively, a posi-
rience visual hallucinations.
tive alliance needs to be in place. Erotic, idealized,
Sadock BJ, Sadock VA (eds): Kaplan and Sadock's Comprehensive primitive, a n d mirroring transference may all occur in
Textbook of Psychiatry, 8th ed. Philadelphia, Lippincott Williams & the therapy a n d be amenable to examination/analy-
Wilkins, 2005, pp 1061-1063
sis because of the positive alliance.

Beitman B0, Yue D: Learning Psychotherapy: A Time-Efficient,


Research-Based, and Outcome-Measured Psychotherapy Training
Program. New York, WW Norton, 1999

Section 2: Answers and Explanations


236 238
For w h i c h of the anxiety disorders does c l o n a z e p a m A 36-year-old female graduate student presents with
h a v e an F D A indication? atypical depression that has not responded to selec-
tive serotonin reuptake inhibitors (SSRIs). W h i c h of the
(A) Generalized anxiety disorder following medications has the most evidence for effi-
(B) Obsessive-compulsive disorder cacy in this situation?
(0 Panic disorder
(D) Posttraumatic stress disorder (A) Bupropion
(E) Social phobia (B) Phenelzine
(C) Valproic acid
T h e correct response is option C: Panic disorder (D) Trazodone
Although clonazepam is often used as a general anx-
(E) Imipramine
iolytic, its only F D A indications are seizure disorders The correct response is option B: Phenelzine
a n d panic disorder. For most clinicians c l o n a z e p a m
The available data indicate that monoamine oxidase
w o u l d be a second-choice agent after the SSRIs for
inhibitors are more effective than tricyclic antidepres-
treating panic disorder. C l o n a z e p a m , a longer-acting
sants for atypical depression. SSRIs are probably
b e n z o d i a z e p i n e , is especially useful because it can
between these two classes in efficacy. Despite w i d e -
be given less frequently a n d is associated with less
spread use in clinical practice, there are no data sup-
rebound anxiety.
porting the efficacy of t r a z o d o n e , bupropion, or a n y
Stein DJ, Hollander E (eds): American Psychiatric Publishing Textbook anticonvulsants in patients with atypical depression.
of Anxiety Disorders. Washington, DC, American Psychiatric
Publishing, 2002, pp 260-263 Dubovsky SL, Dubovsky AN: Concise Guide to Mood Disorders.
Washington, DC, American Psychiatric Publishing, 2002, p 221
Practice Guideline for the Treatment of Patients With Major
Depressive Disorder, 2nd ed (2000), in American Psychiatric
Association Practice Guidelines for the Treatment of Psychiatric
237
Disorders, Compendium 2004. Washington, DC, APA, 2004, p 469
A 22-year-old female presents with symptoms of
depression. She is a l w a y s thinking that' fhs worst will
occur in her relationships a n d employment, and she
feels powerless to alter or control these events. She is 239
seeking treatment with a therapist w h o provides cog-
"Guevodoces," which translates as "penis at 12," re-
nitive behavior therapy. W h c M j the most likely focus
fers to Dominican children with a female a p p e a r a n c e
of the therapy for this patient?
at birth, reared as girls, w h o at puberty develop male
(A) Anger turned inward secondary sexual characteristics and male-typical sex-
(B) Early deprivation ual urges and behaviors. T h e genetic condition they
(C) Difficulties in relationships have is called:
(D) Self-image
(E) Maladaptive thought patterns (A) androgen insensitivity syndrome.
(B) cloaca! extrophy.
T h e correct response is option E: M a l a d a p t i v e thought (C) congenital adrenal hyperplasia.
patterns (D) Klinefelter syndrome.
(E) 5-a reductase deficiency.
Depression in cognitive behavior therapy is defined as
resulting from distortions of thinking. These include a The correct response is option E: 5-a Reductase defi-
negative self-image and a tendency to experience the ciency
w o r l d as negative, demanding, a n d self-defeating.
5-a Reductase is the e n z y m e that converts testosterone
Depressed patients catastrophize, leaping from one
to dihydrotestosterone ( D H T ) , the hormone necessary
imagined worst-case scenario to the next. T h e patient
for fetal masculinization.,'As testosterone levels rise at
will expect failure, punishment, and continued hard-
puberty in these chromosomal males, predominant
ship. In order to provide relief from symptoms,.the ther-
maleness emerges.
apist will address issues of pessimistic anticipation,
w h i c h is an example of a maladaptive thought process. Panksepp J: Affective Neuroscience: The Foundations of Human and
Animal Emotions. New York, Oxford University Press, 1998, p 233
Sadock BJ, SadxkVA (eds): Kaplan and Sadock's Comprehensive Sadock BJ, Sadock VA (eds): Kaplan and Sadock's Comprehensive
Textbook of Psychiatry, 8th ed. Philadelphia, Lippincott Williams & Textbook of Psychiatry, 8th ed. Philadelphia, Lippincott Williams &
Wilkins, 2005, pp 1607-1609 Wilkins, 2005, pp 1990-1991
240 242
Rapid cycling is L E A S T likely to respond to: A small-town newspaper's reporter calls a psychiatrist
to get "a professional's opinion" on the publicized mis-
(A) divalproex. behavior of the district's elected representative, w h o
(B) carbamazepine. may or m a y not h a v e bipolar disorder. T h e reporter
(C) haloperidol. asks the psychiatrist, w h o has' never met the represen-
(D) lithium. tative, " W h y do y o u think the representative misbe-
haved, doctor?" T h e psychiatrist's ethical obligations
T h e correct response is option C: Haloperidol
i would lead to w h i c h of the following responses?
Haloperidol, a convenfionaf antipsychotic, has no
indication for the treatment of the rapid cycling sub-
(A) Inform the public about the representative's bipolar disor-
der, since this person is a public figure.
type of bipolar disorder. In the treatment of bipolar
(B) Cbmment on the representative's condition only if the rep-
disorder it is used as an adjunct to a mood stabilizer
resentative has not been a patient.
to treat psychotic symptoms. Data support the use of
(C) Comment on either childhood dynamic origins or brain
divalproex as the most effective treatment for rapid abnormality as the possible cause of the representative's
cycling. Lithium a n d c a r b a m a z e p i n e are much less problems, if public records contain information that is con-
effective for rapid cycling. sistent with such possibilities.
(D) Comment on the general nature of psychiatric illnesses.
Bowden CL: Valproate: drugs for the treatment of bipolar disorder, in
The American Psychiatric Publishing Textbook of T h e correct response is option D: Comment on the
Psychopharmacology, 3rd ed. Edited by Schatzberg AF, Nemeroff general nature of psychiatric illnesses
CB. Washington, DC, American Psychiatric Publishing, 2004,
pp 573-574 A psychiatrist m a y comment on the general nature of
American Psychiatric Association: Practice Guideline for the psychiatric illnesses. However, it is unethical for a psy-
Treatment of Patients With Bipolar Disorder (Revision). Am J
chiatrist to publicly render a professional opinion about
Psychiatry 2002; 159(April suppl). Reprinted in FOCUS 2003;
1:64-110 (pp 94-97) a person w h o is not the psychiatrist's o w n patient, w h o
has not been properly examined by the psychiatrist,
and w h o has not given proper informed consent to the
psychiatrist to disclose professional opinions.
241
American Psychiatric Association: Opinions of the Ethics Committee
During an office follow-up visit, a 19-year-old w o m a n on the Principles of Medical Ethics With Annotations Especially
with schizophrenia reports no improvement in her Applicable to Psychiatry. Washington, DC, American Psychiatric
symptoms despite being on an appropriate dose- of an Association, 2001 (section 7-B)
antipsychotic medication for 3 weeks. The most rea- Beauchamp TL, Childress JF: Principles of Biomedical Ethics, 5th ed.
New York, Oxford University Press, 2001
sonable initial a p p r o a c h would be to:

(A) add an anticonvulsant medication.


(B) add another antipsychotic medication.
(C) change to another antipsychotic medication.
(0) explore potential nonadherence.
(E) increase the dose of the patient's current medication.
The correct response is option D: Explore potential
nonadherence

Poor adherence with the antipsychotic medication


regimen is a common reason for lack of improvement.
Even w h e n adherence is not an issue, 2 to 4 weeks of
treatment may be needed to see an initial effect of
treatment, a n d 6 months or more may be needed to
obtain a full or optimal response to medication.

Sadock BJ, Sadock VA (eds): Kaplan and Sadock's Comprehensive


Textbook of Psychiatry, 8th ed. Philadelphia, Lippincott Williams &
Wilkins, 2005, p 1473
American Psychiatric Association: Practice Guideline for the
Treatment of Patients With Schizophrenia, 2nd ed. Am J
Psychiatry 2004; 161 (Feb suppl): 24-25

Section 2: Answers and Explanations 13


eral reasons. First, the gift may represent a financial
243
hardship for this patient. Second, the gift may signify
In psychodynamic psychotherapy, a b o u n d a r y cross-
more than a "token" of gratitude described by the
ing—unlike a b o u n d a r y violation—is:
patient (e.g., it may indicate the development of
(A) discussed with the patient. romantic transference, or may be more indicative of
(B) an exploitative break in the therapeutic frame. the patient's mood state). T h i r d , the psychiatrist's
(C) generally not examined in the therapy. acceptance of the gift could send the w r o n g signal to
(D) harmful to the therapy. the patient, that is, that gifts are an accepted and
(E) a repeated occurrence. even expected part of the psychiatrist's practice.

T h e correct response is option A: Discussed with the American Psychiatric Association: Ethics Primer of the American
patient Psychiatric Association. Washington, DC, American Psychiatric
Association, 2001 pp 46-47
A b o u n d a r y crossing is typically a benign and even
helpful break in the therapeutic frame, especially w h e n
it is discussed in the therapy a n d the countertransfer-
ence action leads to further exploration of the transfer- 245
ence. According to G a b b a r d , b o u n d a r y crossings
A consultation is requested for a 16-year-old male w h o
usually occur in isolation, they a r e minor, and they
has been in detention for the past 2 months on charges
ultimately do not ccuse harm; an e x a m p l e would be
of possession of cocaine. The detention center staff
allowing the patient to stay a f e w extra^ minutes at the
describe the youth as hyperactive, inattentive, impul-
end of a session. Boundary violations, in contradis- sive, and easily distracted. A review of his educational
tinction, are exploitative and are often not discussable. history indicates that the youth has been in special edu-
cation classes since the first grade because of attention
Gabbard GO: Long-Term Psychodynamic Psychotherapy: A Basic Text.
deficit hyperactivity disorder, and a mixed expressive-
«• Washington, DC, American Psychiatric Publishing, 2004, pp 49-53
receptive language disorder. On examination, there is
no evidence of a mood or anxiety disorder or current
substance abuse. W h i c h of the following medications
would be most appropriate for this patient?
244
A male psychiatrist has been conducting weekly psy- (A) Atomoxetine
chotherapy for the last 4 months with a female patient. (B) Clonidine
T h e patient has serious financial problems due to over- (C) Desipramine
spending. O n e day, the patient brings in a gift-wrapped (D) fed salts of amphetamines
b o x to the session and, handing the b o x to the psychi- (E) Pemoline
atrist, blurts out, "It's a $100 tie ... I couldn't help
The correct response is option A: Atomoxetine
myself, it just looked like something y o u ' d w e a r and
I'm so grateful for all of your help. Please accept it!" Atomoxetine is a recently a p p r o v e d nonstimulant
W h i c h of the following is an appropriate response for medication for the treatment of A D H D . T h e drug is a
the psychiatrist to give to this patient?
potent inhibitor of presynaptic norepinephrine trans-
(A) Accept the gift but donate it to charity without telling the patient. porters with minimal affinity for other receptors or
(B) Accept the gift but make it dear that the psychiatrist is uncom- transporters. T h e most common side effects associ-
fortable doing so, given the patient's financial difficulties. ated with it, w h i c h are generally mild, include seda-
(C) Acknowledge the patient's gratitude, discuss the implica- - tion, decreased appetite, nausea, vomiting, a n d
tions, but state that as a general policy the psychiatrist does dizziness. Because atomoxetine is not k n o w n to be
not accept gifts from patients. abusable, it is an excellent alternative for use with
(D) Decline the gift without further explanation. youths w h o h a v e a history of illicit substance use or
i
abuse a n d A D H D . M i x e d salts of amphetamines is a
T h e correct response is option C: A c k n o w l e d g e the
patient's gratitude, discuss the implications, but state Schedule II stimulantjwifh a potential for abuse a n d
that as a general policy the psychiatrist does not w o u l d not be a g o o d choice for this patient. The other
accept gifts from patients * three agents listed are considered second-line agents
for A D H D a n d should be tried only after a failed trial
W h i l e accepting small gifts from patients m a y be eth- of a first-line agent.
ically acceptable at times (when the gift is a genuine
token of appreciation a n d to decline it w o u l d harm Dulcan MK, Martini D. R, Lake MB: Concise Guide to Child and
the therapeutic alliance], in the scenario in rnis ques- Adolescent Psychiatry, 3rd ed. Washington, DC, American
Psychiatric Publishing, 2003, p 277
tion, accepting the tie is p r o b a b l y unethical for sev-
246 depressants but is most likely with the SSRIs. A m o n g
the answer choices for this question, none of the med-
A 78-year-old patient with major depressive disorder is
ications listed other than the SSRI a r e likely to be pre-
being treated with atorvastatin and metoprolol for car-
scribed for the treatment of p a r o x y s m a l anxiety.
diovascular disease. W h i c h of the following antide-
pressants is best used with these two other medications? Sadock BJ, Sadock VA (eds): Kaplan and Sadock's Synopsis of
Psychiatry, 9th ed. Philadelphia, Lippincott Williams & Wilkins,
(A) Bupropion 2003, p 109?
(B) Esc'rtalopram Schatzberg AF, Nemeroff CB (eds): The American Psychiatric
(C) Fluoxetine , Publishing Textbook of Psychopharmacology, 3rd ed, Washington,
(D) Nefazodone '« • DC, American Psychiatric Publishing, 2004, pp 239,331,343,
(E) Paroxetine 395-398,915-917

T h e correct response is option B: Escitalopram

Escitalopram has no clinically meaningful P450


inhibitory effects, although it can cause a minor
248
increase in metoprolol blood levels. Atorvastatin is a Mefhylphenidate has its greatest action on w h i c h of
substrate for cytochrome P450 3 A 4 . Inhibition of 3 A 4 the following neurotransmitter systems?
by nefazodone could substantially increase atorva-
(A) Acetylcholine
statin blood levels and increase the risk of side effects.
(B) Dopamine
Metoprolol is a substrate for C Y P 2 D 6 , an e n z y m e
(C) Gamma-aminobutyric acid
potently inhibited by bupropion, fluoxetine, and (D) Glutamafe
paroxetine. Side effects of this beta-blocker could (E) Serotonin
become problematic if there is a considerable
The correct response is option B: D o p a m i n e
increase in its blood level.

Methylphenidate's mechanism of action is believed to


Williams 0, Feeiy J: Pharmacokinetic-pharmacodynamic drug inter-
actions with HMG-CoA reductase inhibitors. Clin Pharmacokinet lie in dopamine presynaptic release a n d reuptake
2002;41:343-370 blockade.
Wuttke H, Rau T, Heide R, Bergmann K, Bohm M, Weil J, Werner D,
Eschenhagen T: Increased frequency of cytochrome P450 2D6 Le .vis M (ed): Child and Adolescent Psychiatry: A Comprehensive
poor metabolizers among patients with metoprolol-associated lextbook, 3rd ed. Philadelphia, Lippincott Williams & Wilkins,
adverse effects. Clin Pharmacol Ther 2002; 72:429-437 2002, p 959
Martin A, Scahill l_ Chamey D, Leackman J (eds): Pediatric psy-
DeVane CL, Nemeroff CB: 2002 Guide to Psychotropic Drug
chopharmacology principles and practice. New York, Oxford
Interactions. Primary Psychiatry 2002; 9:28-57 "" .
University Press, 2003, appendix .

247
249 {

A 23-year-old w o m a n presents to the clinic with a


W h i c h mental disorder is the most frequent cause of
chief complaint of having sexual problems. She
first-onset psychosis after a g e 60?
reports that she gets aroused and enjoys intercourse
but is unable to have an o r g a s m . She believes the (A) Dementia of the Alzheimer's type
problem started about a month a g o , w h e n her physi- (B) Bipolar disorder
cian prescribed a medication for her " a n x i e t y (C) Delusional disorder
attacks." The medication w a s most likely: (D) Major depression
(E) Very late onset schizophrenia
(A) bupropion.
(B) buspirone. The correct response is option A: Dementia of the
(C) citalopram. Alzheimer's type
(D) mirtazapine.
(E) trazodone. Several epidemiological studies h a v e established
dementia of the Alzheimer's type as the most common
The correct response is option C: Citalopram
cause of late-life psychosis. Late-onset schizophrenia

Sexual side effects are the most common adverse is uncommon. T h e other disorders listed can cause

effect of SSRIs, with an incidence between 50% and psychosis in late life but not as commonly as demen-

80%. The most common complaints are inhibited tia of the Alzheimer's type.

orgasm and decreased libido. Impaired ability to Spar JE, La Rue A Concise Guide to Geriatric Psychiatry, 3rd ed.
achieve an orgasm may occur with other types of anti- Washington, DC, American Psychiatric Publishing, 2002, pp 253-254

Section 2: Answers and Explanations 137


250 252
Psychotic features do N O T occur during which of the A psychiatrist repectedly a n d increasingly fantasizes
following? about a sexual relationship with a patient in psy-
chotherapy whom the psychiatrist finds v e r y attrac-
(A) Manic episode tive. T h e psychiatrist is considering the possibility that
(B) Mixed episode the prohibition of sex with patients may not apply in
(C) Hypomonic episode this case because of some extenuating circumstances.
(D) Major depressive episode W h i c h of the following options w o u l d be the most eth-
ical behavior on the part of the psychiatrist?
T h e correct response is option C: H y p o m a n i c episode

DSM-IV-TR criterion E for hypomanic episode includes


(A) Keep a diary of the sexual fantasies in order to contain them.
(B) Increase the frequency of therapy sessions in order to
". .. and there are no psychotic features." Psychotic fea-
make the best use of the intensity of the transference that
tures can be associated with manic, mixed, and major
is developing.
depressive episodes.
(C) Transfer the patient's care to another psychiatrist.
American Psychiatric Association: Diagnostic and Statistical Manual of (D) Because there ore important psychodynamic therapeutic
Mental Disorders, Fourth Edition, Text Revision (DSM-IV-TR). Wash- implications for the patient, share the fantasies with the
ington, DC, American Psychiatric Association, 2000, pp 365-358 patient if the benefits seem to outweigh the risks.
T h e correct response is option C: Transfer the patient's
care to another psychiatrist

25] l W h i l e termination of psychotherapy is a dramatic step,


T h e parents of a 7-year-old b o y express concern about it is the best choice among the four options in preserv-
his bed-wetting. The b o y seems well adjusted, and the ing the patient's welfare. All of the other options are
family has developed a nonstigmatizing system to care based on- the impaired judgment of the psychiatrist.
for his bed a n d personal hygiene. He has no medical Indulging in the fantasies (option A) and increasing
problems. After explaining the natural history of enure- interactions with the patient (option B) may gratify the
sis, the most reasonable initial a p p r o a c h w o u l d be to:
psychiatrist more than the patient and often precede
(A) start desmopressin. eventual boundary violation by the psychiatrist. Self-
(B) start imipramine. disclosure of any kind should be done with caution; a
(C) provide observation and follow-up. psychiatrist's disclosure of sexual feelings toward the
(D) order a bell and pad. patient often precedes sexual b o u n d a r y violations.
(E) start psychotherapy.
Simon Rl: A Concise Guide to Psychiatry and Law for Clinicians, 3rd
T h e correct response is option C: Provide observation ed. Washington, DC, American Psychiatric Publishing, 2001,
a n d follow-up pp 228-238
Hundert EM, Appelbaum PS: Boundaries in psychotherapy: model
T h e natural history of enuresis is that a substantial num- guidelines. Psychiatry 1995; 58:345-356
ber of children remit after a g e 7. G i v e n that the boy
a p p e a r s to be doing well, a reasonable approach is
to wait a n d see if the enuresis improves with a g e .

Sadock BJ, Sadock VA (eds): Kaplan and Sadock's Comprehensive


Textbook of Psychiatry, 8th ed. Philadelphia, Lippincott Williams &
Wilkins, 2005, p 3243
253 Symptoms of ataque de nervios include uncontrollable
shouting, attacks of crying, trembling, a sense of heat
W h i c h of the following statements about a defen-
in the chest that migrates to the h e a d , a n d verbal a n d
dant's competency to stand trial for a criminal offense
physical a g g r e s s i o n , some of w h i c h overlap with
i s N O T correct?
panic disorder. However, unlike panic disorder,
(A) The defendant must be able to remember what he or she afaques a r e often associated with a specific event a n d
was doing at the time of the offense. an absence of acute fear.
(B) The defendant must be able to communicate with attorneys.
(C) The defendant must be able to understand basic courtroom Lewis-Fernandez R, Garrido-Castillo P, Bennasar MC, Parrilla EM,
1 1 Laria AJ, Ma G, Petkova E: Dissociation, childhood trauma, and
procedure.
•ataque de nervios among Puerto Rican psychiatric outpatients.
(0) The defendant must be able to understand the nature of Am J Psychiatry 2002; 159:1603-1605
various possible pleas and their consequences. American Psychiatric Association: Diagnostic and Statistical Manual
T h e correct response is option A: T h e defendant must of Mental Disorders, Fourth Edition, Text Revision (DSM-IV-TR).
Appendix I: Outline for Cultural Formulation and Glossary of
be able to remember w h a t he or she w a s doing at the
Culture-Bound Syndromes. Washington, DC, American Psychiatric
time of the offense
Association, 2000, p 899

A m n e s i a for the time of the offense usually does not


interfere with the defendant's ability to consult with
the attomey(s) a n d construct a valid defense unless
255
the amnesia is part of an ongoing mental process
such as dementia. To be competent to stand trial, a W h i c h of the following signs or symptoms alone
;

defendant must have a "rational as well as factual" would be sufficient to meet criterion A for the active
understanding of the proceedings against the defen- phase of schizophrenia?

dant; be able to communicate with the artorney(s) (A) Bizarre delusions


and assist in the defense; have the capacity to under- (B) Catatonic behavior
stand the basics of court procedures; a n d have the (C) Incoherent speech
capacity to understand the charges and the possible (D) Negative symptoms
consequences of the charges. (E) Tactile hallucinations
Hales RE, Yudofsky SC (eds): The American Psychiatric Publishing The correct response is option A: Bizarre delusions
Textbook of Clinical Psychiatry, 4th ed. Washington, DC, American
Psychiatric Publishing, 2003, pp 1611-1612 In order to meet criterion A for schizophrenia, there
has to be at least 1 month of symptoms, or less if suc-
cessfully treated. The symptoms are t w o or more out
of five items: delusions, hallucinations, d i s o rg a n iz e d
254 speech, d i s o r g a n i z e d or catatonic behavior, and neg-
ative symptoms. O n e of the five items could be suffi-
A 34-year-old Puerto Rican w o m a n presents in distress
at the outpatient clinic. She reports that her grandfa- cient to fulfill criterion A if delusions are bizarre or if
ther recently died, a n d since then she has been there is a v o i c e running a commentary on the patient's
afflicted by several bouts of afaque de nervios. She behavior or thoughts, or two or more voices are con-
has these spells only w h e n she is upset. A detailed his- versing with e a c h other.
tory should be obtained to distinguish afaque from
American Psychiatric Association: Diagnostic and Statistical Manual
w h i c h other axis I diagnosis?
of Mental Disorders, Fourth Edition, Text Revision (DSM-IV-TR).
(A) Bipolar disorder Washington, DC, American Psychiatric Association, 2000,
pp 312-313
(B) Histrionic personality disorder
(C) Obsessive-compulsive disorder
(D) Panic disorder
(E) Schizophrenia
The correct response is option D: Panic disorder

Section 2: Answers and Explanations


256 258
A 32-year-old w o m a n w a s unexpectedly terminated A psychiatrist maintains private therapy progress notes,
from her job. T w o months later she presents tearfully in addition to medical record notes, that contain
with depressed mood and occasional feelings of extremely sensitive clinical information. This practice is:
hopelessness; she still feels stressed by the loss of her
(A) an acceptable means of enhancing patient confidentiality.
j o b . She has no prior history of depression, is not sui-
(B) on acceptable way of preventing court-mandated access to
cidal, has not had changes in appetite, weight, sleep,
sensitive clinical information.
or energy level, and still gets pleasure from family and
[() not acceptable, because all clinical material should be
hobbies. W h i c h of the following diagnoses would be
included in patients' medical records.
the most appropriate?
(D) not acceptable, because the risks to the patient outweigh
(A) Major depressive disorder the benefits.
(B) Bipolar II disorder
T h e correct response is option A: An acceptable
(C) Bereavement
means of enhancing patient confidentiality
(D) Adjustment disorder
(E) Dysthymic disorder Private progress notes c a n be a useful tool for pro-

T h e correct response is option D: Adjustment disorder tecting patient confidentiality, since there is value in
maintaining more detailed therapy notes for patient
H e r symptoms developed in response to an identifi- care as well as for teaching purposes. H o w e v e r , pri-
able stressor and appeared within 3 months of the vate progress notes may be accessed under a court
stressor. T h e symptoms do not meet criteria for a order. Medical record notes should be detailed
major depressive episode. Bereavement is a reaction enough to provide for continuity of patient care as
to the loss of a loved one. Dysthymic disorder has a well as for necessary administrative transactions (e.g.,
2-year duration criterion in adults. third-party payment for services] but should not con-
tain highly sensitive material.
American Psychiatric Association: Diagnostic and Statistical Manual
of Mental Disorders, Fourth Edition, Text Revision (DSM-IV-TR).
American Psychiatric Association, Committee on Confidentiality:
Washington, DC, American Psychiatric Association, 2000,
Guidelines on Confidentiality. Am J Psychiatry 1987;
pp 679-683
144:1522-1526
American Psychiatric Association: Principles of medical ethics with
annotations especially applicable to psychiatry. Washington, DC,
American Psychiatric Association, 2001 (section 4)
257 Gutheil TG, Appelbaum PS: Clinical Handbook of Psychiatry and the
c
Law, 3rd ed. Philadelphia, Lippincott Williams &Wiikto , 2000
W h i c h of the following has been approved by the
F D A for the treatment of alcohol dependence?

(A) Buprenorphine
(B) Levo-alpha-acetylmethadol (LAAM) 259
(C) Naloxone
W h i c h of the following psychosocial treatments is
(D) Naltrexone
most likely to be effective in the treatment of obsessive-
T h e correct response is option D: Naltrexone compulsive disorder?

(A) Cognitive therapy


In 1995 the F D A approved naltrexone, an opioid
(B) Supportive psychotherapy
antagonist, for the treatment of alcohol dependence. •
(C) Interpersonal therapy
Buprenorphine, L A A M , naloxone, and naltrexone are all
(D) Behavioral therapy
used in the treatment of opioid dependence or overdose.
(E) Group therapy
Schatzberg AF, Nemeroff CB (eds): The American Psychiatric
The correct response is option D: Behavioral therapy
Publishing Textbook of Psychopharmacology, 3rd ed. Washington,
DC, American Psychiatric Publishing, 2004, p 1011
Of the psychosocial .treatments listed, the data for
Schuckit MA, Taper! S: Alcohol, in The American Psychiatric
effectiveness a r e strongest for behavior treatment, a n d
Publishing Textbook of Substance Abuse Treatment^ 3rd ed. Edited
by Galanter M, Kleber HD. Washington, DC, American Psychiatric especially for treatment with a response-inhibition
Publishing, 2004, p 162 component. Cognitive, therapy is promising, and
group therapy is cost effective a n d helpful for some
patients. Interpersonal therapy is not c o m m o n l y indi-
cated for obsessive-compulsive disorder.
Stein DJ, Hollander E (eds): American Psychiatric Publishing Textbook Oulcan MK, Martini DR, Lake MB: Concise Guide to Child and
of Anxiety Disorders. Washington, DC, American Psychiatric Adolescent Psychiatry, 3rd ed. Washington, DC, American
Publishing, 2002, pp 221-230 Psychiatric Publishing, 2003, p 81
Van Noppen BL, Steketee G: Individual, group, and multifamily cogni-
tive-behavioral treatments, in Current Treatments of Obsessive-
Compulsive Disorder, 2nd ed. Edited by Pato MT, Zohar J.
Washington, DC, American Psychiatric Publishing, 2001. Reprinted
26] ;
in FOCUS 2004; 2:475-495 (p 475)
A 16-year-old girl has been blinking her eyes a n d
clearing her throat on an intermittent basis for y e a r s .
She has no control of the symptoms a n d has never
260 been free of them for more than a f e w days, a n d they
cause significant problems. W h a t medication m a y be
A 7-year-old boy presents to a clinic on referral from
helpful for treating this problem?
the school with a number of behavior problems, includ-
ing impaired attention, hyperactivity, and impulsivity. (A) Pimozide
His parents have described him as a "whirling dervish" (B) Nortriptyline
for years. At age five, he w a s evaluated by his primary (C) Paroxetine
care physician and started on methylphenidate, which (D) Lorazepam
produced significant improvement in his behavior. (E) Methylphenidate
However, he then developed jerky, irregular muscle
movements around the eyes and mouth that persisted The correct response is option A: Pimozide
w h e n he w a s off the medication. The medication that
T h e symptoms a r e those of Tourefte's disorder.
could address all of his symptoms is:
Pimozide is a frequently prescribed medication.
(A) clonidine. Pimozide is a p p r o v e d by the F D A for the treatment of
(8) dj-amphetamine. Tourefte's disorder. D receptor antagonists are thought
2

(C) haloperidol. to be effective because the dopamine system is hypoth-


(D) magnesium pemoline. esized to be involved in the pathogenesis of Tourefte's
(E) pimozide. disorder. Drugs that increase dopamine (such as
The correct response is option A: Clonidine methylphenidate] may make the tics worse.

This b o y has classic symptoms of attention deficit hyper- Sadock BJ, Sadock VA (eds): Kaplan and Sadock's Comprehensive
Textbook of Psychiatry, 8th ed. Philadelphia, Lippincott Williams &
activity disorder ( A D H D ) , including impaired attention,
Wilkins, 2005, p 3232
h/peractivity, and impulsivity in school and at home,
with onset before the age of 7 years. He has had a pos-
itive response to the stimulant medication methyl-
phenidate but has developed simple motor tics involving 262
the eyes and mouth. W i t h some youths, the tics go
Blocking craving for opiates with subsequent reduc-
a w a y when stimulant medication is discontinued, but for
tion in associated drug use generally requires w h i c h
this boy they have persisted. Therefore, treatment will
of the following daily doses of methadone?
need to target the symptoms of inattention, hyperactiv-
ity, impulsivity, and the motor disorder. Clonidine, an (A) 5 mg
alpha-2 noradrenergic agonist, has been used success- (B)10mg
fully to treat A D H D and tic disorders and thus is the one (C) 20 mg
single agent that would be most effective.
(0) 40 mg
(E) 80 mg
D,l-amphefamine and pemoline are both stimulants
The correct response is option E: 80 mg
that may be helpful in addressing the A D H D symp-
toms, but they will not help with the tic disorder, and Most patients require doses greater than 60 mg to
in fact they would probably mcke it worse. block craving for opiates and to reduce subsequent
Haloperidol a n d pimozide are both antipsychotic associated drug use. A dose of 40 to 60 m g / d a y of
medications that are useful in decreasing or eliminat- methadone is usually sufficient to block opioid with-
ing tic-like movements, but they will not have an impact drawal symptoms.
on the inattention, distractibilify, and hyperactivity.
American Psychiatric Association: Practice Guideline for the
Treatment of Patients With Substance Use Disorders: Alcohol,
Lewis M (ed): Child and Adolescent Psychiatry: A Comprehensive
Cocaine, Opioids. Am J Psychiatry 1995; 152(Nov suppl)
Textbook, 3rd ed. Philadelphia, Lippincott Williams & Wilkins,
2002, pp 952,956

Section. 2: Answers and Explanations


Hales RE, Yudofsky SC (eds): The American Psychiatric Publishing
263 Textbook of Clinical Psychiatry, 4th ed. Washington, DC, American
W h i c h of the following is the most important consid- Psychiatric Publishing, 2003, pp 606-607
eration for the treatment plan w h e n performing an ini- Hembree EA, Foa EB: Interventions for trauma-related emotional dis-
turbances in adult victims of crime. J Trauma Stress 2003;
tial evaluation of a patient with borderline personality
16:187-199
disorder in suicidcl crisis?
Connor KM, Butterfield Ml: Posttraumatic stress disorder. FOCUS
(A) Safety 2003;1:247-262 (p 256)
(B) Goals
(0 Type
(D) Frame
(E) Outcome 265
A 40-year-old female comes to the mental health cen-
T h e correct response is option A: Safety
ter for the first time. After a thorough assessment, she
Safety should be the priority. Suicidal ideation and is told that the best treatment would be a course of
suicide attempts are c o m m o n , and it is essential dur- brief psychotherapy. She looks concerned and asks if
ing an initial assessment to decide whether inpatient she can be transferred to a doctor of her o w n race.
treatment is necessary. T h e appropriate step to take would be to:

Practice Guideline for the Treatment of Patients With Borderline (A) attempt to convince her that any doctor is capable.
Personality Disorder (2001), in American Psychiatric Association (B) explore why she feels this is necessary.
Practice Guidelines for the Treatment of Psychiatric Disorders, (C) grant her request and transfer her.
Compendium 2004. Washington, DC, APA, 2004, p 753 (D) help her find another clinic that will suit her.
(E) switch to medication management only.
The correct response is option B: Explore w h y she
feels this is necessary
264 "
T h e best documented treatment for posttraumatic Although the patient m a y ultimately be transferred to
stress disorder (PTSD] precipitated by a violent rape another clinician, one should still make an attempt to
includes: find out the reasoning behind her request.

(A) event recall. Tseng W, Streltzer J: Culture and Psychotherapy. Washington, DC,
(B) martial arts instruction. American Psychiatric Publishing, 2001, pp 146-147
(C) prosecution otjhe rapist.
(D) cognitive-based ilaropy.
T h e correct response is option D: Cognitive-based
therapy

Published reports by Foa and colleagues demonstrate


that cognitive-based psychotherapies that facilitate de-
stigmafizafion and desensitization are particularly
effective for rape victims. Bringing the rapist to justice
and requiring the victim to t>e at trial often causes a
reliving of the traumatic experience a n d becomes an
emotionally painful event for the victim. There is con-
troversy about whether it is helpful or harmful to the
patient's recovery if the victim is encouraged to tell the
story in detail. The choice of approaches depends on
the needs of the victim. Martial arts training for some
victims can be helpful in preventing rape and perhaps
providing a victim with some sense of security and pro-
tection against a repeated experience. H o w e v e r , many
victims would be uncomfortable in having to consider
that the event could occur again with such certainty that
they must prepare for it by martial arts training.
266 T h e correct response is option E: A toxicology screen

DSM-IV-TR cultural formulation for a patient from a cul- Substance use is a c o m m o n cause of psychotic symp-
ture different than the psychiatrist's requires: toms ana* should be eliminated before diagnosis a n d
treatment of other psychiatric disorders.
(A) a history of the patient's education and occupational training.
(B) independent information from a - cultural consultant. • Sadock BJ, Sadock VA (eds): Kaplan and Sadock's Comprehensive
(Q an understanding of the neurobiology of the patient's disorder. Textbook of Psychiatry, 8th ed. Philadelphia, Lippincott Williams &
(D) an understanding of the effect of the psychiatrist's own cul- Wilkins, 2005, p 1124
ture on treatment variables. , American Psychiatric Association: Practice Guideline for the
(E) use of an interpreter from of-assimilated in the patient's , Treatment of Patients With Schizophrenia, 2nd ed. Am J
Psychiatry 2004; 161 (Feb suppl):1-56
culture.
T h e correct response is option D: An understanding of
the effect of the psychiatrist's o w n culture on treatment
variables 268
According to DSM-IV-TR, the cultural formulation pro- A 60-year-old w o m a n presents with daytime fatigue,
vides a systematic review of the individual's cultural morning h e a d a c h e , a n d p o o r memory. Findings from
background, the role of the cultural context in the expres- her physical examination a n d blood studies are all
within normal limits, and she reports that her mood is
sion and evaluation of symptoms and dysfunction, and
normal. On further questioning she reports that her
the effect that cultural differences may have on the rela-
husband sleeps in a separate room because of her
tionship between the individual and the clinician. O n e of
snoring arid thrashing. T h e most effective treatment for
the components of a cultural formulation is a summary of
this condition is:
the cultural elements of the relationship between the indi-
vidual and the clinician, such as differences in culture (A) fluoxetine.
and social status. This requires that the psychiatrist be (B) continuous positive airway pressure.
knowledgeable about his or her o w n culture. ., (C) lorazepam.
(D) methylphenidate.
Interpreters would be used only in cases w h e r e the psy- (E) relaxation therapy.
chiatrist and the patient w e r e not fluent in the same lan-
T h e correct response is option B: Continuous positive
g u a g e . The educational history per se is not a required a i r w a y pressure
component of the formulation. Since the neurobiology
of a number of psychiatric disorders remains to be elu- The patient likely has sleep a p n e a . T h e best treatment
cidated, this could not be a formulation component. is continuous positive a i r w a y pressure (CPAP). O t h e r
measures are also helpful, such as weight loss a n d
American Psychiatric Association: Diagnostic and Statistical Manual of
sleep-position training.
Mental Disorders, Fourth Edition, Text Revision (DSM-IV-TR) Appen-
dix I: Outline for Cultural Formulation and Glossary of Culture-Bound Hales RE, Yudofsky SC (eds): The American Psychiatric Publishing
Syndromes. Washington, DC, American Psychiatric Association, 2000 Textbook of Clinical Psychiatry, 4th ed. Washington, DC, American
Psychiatric Publishing, 2003, p 984
Krahn LE, Richardson JW: Sleep disorders, in The American
Psychiatric Publishing Textbook of Psychosomatic Medicine.
Edited by Levenson JL. Washington, DC, American Psychiatric
267
Publishing, 2005, pp 342-344
A 20-year-old male college student presents in the
emergency department with confusion and agitation.
He is distracted and talks in a rambling manner.
During the interview, he reports seeing an angel w h o
is telling him about his mission. His roommate states
that the student has been having problems for months,
with worsening grades, not sleeping, and w i t h d r a w a l
from friends. In establishing a diagnosis a n d prepar-
ing to initiate treatment, the most appropriate labora-
tory test to obtain at this point would be:

(A) a complete blood count, including a platelet count.


(B) an electrocardiogram.
(() hepatic function tests.
(D) thyroid function tests.
(E) a toxicology screen.

Section 2: Answers ind Explanations


269 . 271
W h i c h of the following best describes a characteristic A 32-year-old man sees his primary care physician
of the assertive community treatment ( A C T ) model for because of a recurrent productive c o u g h . T h e physi-
management of schizophrenia? cian recommends blood w o r k a n d a chest X-ray.
W h e n the patient enters the phlebotomy suite, his
(A) Clinic-based services heart begins to race, he perspires, and his muscles
(BJ Focus on symptom resolution tense. W h e n he sits in the phlebotomy chair and a
(C) Hospital-based services tourniquet is applied, his symptoms w o r s e n . In addi-
(D) Psychiatrist-led treatment team tion, he becomes short of breath, begins to hyperven-
(E) 24-hour availability of services tilate, a n d feels numbness and tingling in his hands
and feet and around his mouth. W h e n the phle-
T h e correct response is option E: 24-hour availability
botomist uncaps the needle, the patient passes out.
of services
He a w a k e n s shortly after an ammonia capsule is bro-
In the A C T model, some level of case management is ken under his nose. He apologizes for his behavior
typically available around the clock. T h e A C T model and says, "I a l w a y s get this w a y w h e n I see a nee-
dle." This presentation is most consistent with:
emphasizes a flexible, horizontally o r g a n i z e d treat-
ment team that delivers services in the community,
(A) agoraphobia.
w h e r e the client lives, rather than in a clinic setting.
(B) generalized anxiety disorder.
Symptom recurrence is handled by increasing the (C) panic disorder.
intensity of outpatient services rather than relying on (D) social phobia.
inpatient services. Improved community functioning, (E) specific phobia.
often based on treatment goals negotiated with the
The correct response is option E: Specific phobia
client, is the desired outcome, not necessarily the elim-
ination of the symptoms of the disorder. The essential feature of a specific phobia is marked
and persistent fear of a specific object or situation.
Phillips SD, Bums BJ, Edgar ER, Mueser KT, Linkins KW, Rosenheck
RA, Drake RE, McDonel Hen EC: Moving assertive community treat- Exposure to the phobic stimulus provokes an immedi-
ment into standard practice. Psychiatr Serv 2001; 52:771-779 ate anxiety response that may a p p e a r as panic.
Dixon L: Assertive community treatment: twenty-five years of gold. H o w e v e , ; . unlike in panic disorder, the anxiety
Psychiatr Serv 2000; 51:759-765
response in specific phobia is situationally b o u n d . In
this case, the phobia is related to the fear of needles.

American Psychiatric Association: Diagnostic and Statistical Manual


270 ' of Mental Disorders, Fourth Edition, Text Revision (DSM-IV-TR).
Washington, DC, American Psychiatric Association, 2000,
T h e strong association between physical illness and pp 443-450
suicide has been demonstrated for w h i c h of the fol-
lowing conditions?

(A) Amyotrophic lateral sclerosis


(B) Blindness 272
(C) Epilepsy W h i c h of the following symptoms is significantly more
(D) Hypertension likely to be associated with posttraumatic stress disor-
(E) Diabetes mellitus der (PTSD) than with normal bereavement?

T h e correct response is option C: Epilepsy (A) Initial shock


(B) Depressive symptoms
An association between seizure disorders a n d
(C) Numbing
increased suicide risk has been found in many stud-
(D) Avoidance of reminders
ies. Temporal lobe epilepsy in particular is"associated (E) Sleep disturbance
with a higher suicide risk, perhaps b e c a u s e ' o f its
1

impulsivity, presence of mood disorders, and psy- T h e correct response is option C: N u m b i n g

chosis. Studies h a v e not demonstrated qn increased


suicide risk with the other conditions listed.

Practice Guideline for the Assessment and Treatment of Patients With


Suicidal Behaviors (2003), in Practice Guidelines for the
Treatment of. Psychiatric Disorders, Compendium 2004*
Washington, DC, APA, 2004, pp 875-876
Restricted r a n g e of affect, often called numbing, is 274
more likely to be associated with PTSD than with nor-
A 42-year-old w o m a n with generalized anxiety disor-
mal bereavement. N u m b i n g , a decrease in general
der has responded favorably to 60 m g / d a y of buspi-
responsiveness, is a mechanism for avoiding stimuli
rone. To a v o i d substantially increasing the blood level
that may be associated with the trauma. Initial shock,
of the medication a n d producing side effects, y o u cau-
depressive symptoms, and avcidance of reminders of
tion her to a v o i d regular consumption of w h i c h of the
the traumatic event or loved one are common to both following beverages?
PTSD and to normal bereavement.
(A) Apple juice
American Psychiatric Association: Diagnostic and Statistical Manual (B) Coffee
of Mental Disorders, Fourth Edition, Text Revision (DSM-IV-TR). (C) Grapefruit juice
Washington, DC, American Psychiatric Association, 2000,
(D) Milk'
pp 463-468,740-741
Shalev AY: What is posttraumatic stress disorder? J Clin Psychiatry
(E) Red wine
2001;62(suppM7):4-10 T h e correct response is option C: Grapefruit juice

Grapefruit juice is a potent inhibitor of intestinal


cytochrome P450 3 A 4 . In addition, it inhibits P-glyco-
273 protein (a transmembrane efflux pump protein). Even
a single glass of grapefruit juice will have strong
A patient with schizophrenia is in the midst of a
severe exacerbation but refuses treatment. T h e patient inhibitory effects for as long as 24 hours on the
is able to paraphrase w h a t the psychiatrist has said metabolism of various drugs, including some calcium
about the diagnosis, the prognosis, and the reasons channel blockers, benzodiazepines, statins, a n d
for the proposed treatment with medications. W h i c h cyclosporine. Similar effects have been noted with
of the following statements by the patient is the clear- Seville (sour) o r a n g e juice but not with sweet (regular)
est example of an impaired ability to "appreciate or orange juice. In a study with healthy volunteers,
understand"? grapefruit juice caused a 4.3-fold increase in the peck
blood level of buspirone a n d a 9.2-fold increase in
(A) "I have tried all those antipsychotics before. Hone of them •
work that well for me so why try again." extent of absorption (area under the curve).
(B) "Your office is bugged, but the reason why I do not want to Lilja J J, Kivisto KT, Backman JT, Lamberg TS, Neuvonen PJ:
take the medication is that I am really afraid of gaining Grapefruit juice substantially increases plasma concentrations of
more weight." buspirone. Clin Pharmacol Ther 1998; 64:655-660
(C) "The space aliens living in my stomach wouLvbe injured if Malhotra S, Bailey DG, Paine MF, Watkins PB: Seville orange juice-
I took those pills." felodipine interaction: comparison with dilute grapefruit juice and
(D) "I am a Christian Scientist and I do not believe that I have involvement of furocoumarins. Clin Pharmacol Ther 2001;
69:14-23
a disease."
Marangell LB, Silver JM, Goff DC, Yudofsky SC: Psycho pharmacology
The correct response is option C: "The space aliens liv- and electroconvulsive therapy, in The American Psychiatric
ing in my stomach would be injured if I took those pills." Publishing TeMbook of Clinical Psychiatry, 4th ed. Edited by Hales
RE, Yudofsky SC. Washington, DC, American Psychiatric
In option C, the patient's delusion is the causal factor Publishing, 2003, table 24-24, Drug Interactions, p 1117
leading to the false belief. The appreciation standard Lown KS: Grapefruit juice increases felodipine oral availability in
humans by decreasing intestinal CYP3A protein expression. J Clin
focuses on whether the patient is able to apply facts
Invest 1997; 99:2545-2553
to his or her o w n medical situation. Often this means
that the patient has a contrary, false belief regarding
some medical fact. A delusional belief can impair
appreciation if it directly causes the patient to fail to
apply important facts to his or her o w n case. Thus, in
option B, the delusion cannot be said to directly cause
the false belief. Skepticism based on potentially real-
istic experiences or on established religious beliefs
also does not constitute delusional beliefs that directly
affect appreciation.

Grisso T, Appelbaum PS: Assessing Competence to Consent to


Treatment: A Guide for Physicians and Other Health Professionals.
New York, Oxford University Press, 1993

Section 2: Answers and Explanations


In linkage analysis, geneticists use genetic markers to
275
determine the chromosomal location of the gene for a
W h i c h of the following therapies has the best evi-
disorder. Options C and D are features of association
dence for effectiveness in the treatment of posttrau-
study. In comparison, linkage analysis allows many
matic stress disorder?
different rare mutations in the linked genes and
(A) Present-centered group therapy requires a genetic model.
(B) Psychological debriefings
Burmeister M: Genetics of psychiatric disorders: a primer. FOCUS
(C) Single-session techniques
2006; in press
(D) Cognitive behavior therapy
(E) Trauma-focused group therapy
T h e correct response is option D: C o g n i t i v e behavior
therapy 277
Evidence has shown a variety of psychotherapeutic W h i c h of the following is true regarding adolescents
with attention deficit hyperactivity disorder ( A D H D )
approaches to be highly efficacious in reducing the
w h o a r e treated, with methylphenidate?
symptoms of posttraumatic stress disorder. Exposure
and other cognitive behavior therapy approaches as (A) Significantly reduced risk of substance abuse in later life
well as eye movement desensitization a n d reprocessing (B) Higher level of all substance abuse in adulthood
(EMDR) have been shown to reduce symptoms. There is (C) Increased alcohol abuse in adulthood
far less evidence to support the use of pjesentoentered (D) Increased cannabis abuse in adulthood
or trauma-focused group therapies a n d no evidence to
T h e correct response is option A: Significantly
support the use of psychological debriefings or single-
reduced risk of substance abuse in later life
session techniques. Early supportive interventions, psy-
choeducation, and case management also appear to In a long-term study of pediatrically and psychiatrically
be helpful for acutely traumatized individuals and may referred A D H D and n o n - A D H D youth, pharmacother-
facilitate their entry into evidence-based psychothera- a p y for A D H D did not predict a greater risk of sub-
peutic and psychopharmacological treatments. stance use disorder. Subjects with A D H D w h o did not
receive pharmacologic treatment w e r e at a signifi-
Bradley R, Greene J, Russ E, Dutra L, Westen D: A multidimensional
cantly increased risk of substance use disorder, sug-
meta-analysis of psychotherapy for PTSD. Am J Psychiatry 2005;
gesting that pharmacotherapy may protect children
162:214-227
American Psychiatric Association: Practice Guideline for the Treatment with A D H D from this risk.
of Patients With Acute Stress Disorder and Posttraumatic Stress
Disorder. Am J Psychiatry 2004; 161 (Nov suppl) Biederman J, Wilens T, Mick E, Spencer T, Faraone SV:
Pharmacotherapy of attention-deficit/hyperactivity disorder
reduces risk for substance use disorder. Pediatrics 1999; 104:e20
Hales RE, Yudofsky SC (eds): The American Psychiatric Publishing
Textbook of Clinical Psychiatry, 4th ed. Washington, DC, American
276 Psychiatric Publishing, 2003, p 368

Linkage analysis can be defined as:

(A) a test to identify which of several genes in a chromosomal


region is involved in the disorder in question. 278
(B) a test to determine the chromosomal region where a disor-
Elements of an individual's ability to make decisions
der resides by searching for co-segregation of a genetic
about undergoing treatment or participating in
marker with the disorder locus.
research include all of the following EXCEPT:
(C) a study that requires the cause of the disorder to be a com-
mon risk variant. (A) understanding the information provided.
(D) an analysis that is not sensitive to a genetic rrfodel. (B) reasoning with the information or weighing options.
The correct response is option B: A test to determine
• (C) repeating the outlined .risks and benefits without prompting.
the chromosomal region w h e r e a disorder resides by
(D) appreciating the significance of the information for the
searching for co-segregation of a g e n e t i o m a r k e r with individual's own situation.
the disorder locus T h e correct response is option C: Repeating the out-
lined risks a n d benefits without prompting

4
An individual is not required to memorize or recite the Lewis M (ed): Child and Adolescent Psychiatry: A Compfehensive
risks a n d benefits of a recommended procedure with- Textbook, 3rd ed. Philadelphia, Lippincott Williams & Wilkins,
2002, pp 616-617
out prompting in order to be able to make a decision
Dulcan MK, Martini DR, Lake MB: Concise Guide to Child and
about treatment or research.,Options A, B, and D in Adolescent Psychiatry, 3rd ed. Washington, DC, American
this question are considered by most experts to be cru- Psychiatric Publishing, 2003, pp 91 -94
cial elements of decision-making capacity, along with
communication of a choice. T h e individual's choice
need not be in accord with w h a t the physician recom-
mends. O n l y a careful evaluation of capacity-related 280
abilities, in the context of a patient's illness or situation, A 10-year-old b o y is brought for consultation for "bed-
can lead to a determination of capacity status. wetting." .His parents report that he b e g a n using the
toilet a n d staying d r y during the d a y w h e n he w a s 3
Simon Rl: A Concise Guide to Psychiatry and Law for Clinicians, 3rd
years old. H o w e v e r , he has never consistently been
ed. Washington, DC, American Psychiatric Publishing, 2001,
able to control his bladder during sleep. Physical
pp 64-65
Grisso T, Appelbaum PS: Assessing Competence to Consent to examination and laboratory studies have demon-
Treatment: A Guide for Physicians and Other Health Professionals. strated no abnormalities. His father reports that he
New York, Oxford University Press, 1998 also w e t the bed as a child but stopped w h e n he w a s
about 12 years old. The intervention that is most likely
to have long-term effectiveness with this b o y is:

(A) hypnotherapy.
279
(B) low-dose tricyclic antidepressants.
A school guidance counselor refers a 5-year-old girl (C) oral desmopressin.
w h o will not speak. T h e girl has been enrolled in (D) psychotherapy.
school for 3 months. During this time, she has been (E) urine alarm (bell and pad).
noted to make hand gestures or nod in response to
T h e correct response is option E: Urine alarm (bell
her teacher or peers. T h e guidance counselor has
a n d pad)
been meeting with the girl regularly, and recently the
child has begun to whisper. H o w e v e r , she will not use
Primary enuresis is diagnosed w h e n a child has never
a normal voice. T h e girl's parents report that the child
attained bladder control? W i t h nocturnal enuresis,
has no problems speaking at home. T h e girl plays
there is usually a positive family history and no
with her peers, makes appropriate eye contact when
demonstrable physical abnormalities to explain the
spoken to, seems interested in others, and has no
bladder incontinence. Behaviorcl interventions such
unusual movements. There h a v e been no delays or
abnormalities in development. As an adult, this child as the urine alarm- -the "bell and p a d " — h a v e been
is at high risk of developing: demonstrated to be the most innocuous and to have
the greatest efficacy in permanently eliminating noc-
(A) major depressive disorder. turnal enuresis. Hypnotherapy has not been proven as
(B) obsessive-compulsive disorder. a reliable intervention for this disorder. Psychotherapy
((} posttraumatic stress disorder.
may be helpful in managing the emotional impact of
(D) schizophrenia.
bed-wetting, but does not help with continence.
(E) social phobia.
The correct response is option E: Social phobia T h e mechanism by which tricyclic antidepressants,
such as imipramine, are helpful in this disorder is.
This y o u n g girl is suffering from selective mutism. unknown. Desmopressin, an analogue of antidiuretic
Although the disorder is fairly rare (a prevalence of hormone, is available in a nasal spray and tablets.
less than 1% of children seen in mental health set- Desmopressin is as effective as the tricyclic antide-
tings), the most common manifestations are a refusal pressants in the treatment of primary enuresis and has
to speak in school a n d to adults outside of the home fewer side effects, but it is much more expensive. Both
despite speaking normally within the home environ- the tricyclics and desmopressin have limited, short-
ment. Although these children do not speak, they term efficacy in attaining bladder control but have not
a p p e a r to be interested in their surroundings, as evi- proven to be effective in long-term management.
denced by interaction, head nodding, gesturing, and
Lewis M (ed): Child and Adolescent Psychiatry: A Comprehensive
so o n . M a n y of these children are shy, anxious, and
Textbook, 3rd ed. Philadelphia, Lippincott Williams & Wilkins,
overly dependent. Recent studies have identified a
2002, pp 702-705
link between selective mutism in children and social
phobia in adulthood.

Section 2: Answers and Explanations


281 283
A 58-year-old man has a history of ingesting 1 to A 23-year-old w o m a n presented with a 2-week history
2 pints of vodka on a daily basis over the past 20 of difficulty sleeping, hearing voices, and problems
y e a r s . He presents to the emergency department after with thinking. She was fearful and suspicious, and
a minor motor vehicle accident and appears disor- talked about evil alien forces out in the w o r l d . Some
g a n i z e d . A computerized t o m o g r a p h y scan of his of her relatives have had "nervous breakdowns"
head is most likely to show w h i c h of the following? requiring hospitalization. Further evaluation revealed
that the w o m a n had been raped about 3 weeks ear-
(A) Acoustic neuroma lier. H o w e v e r , she has no recollection of the event.
(B) Caudate calcification O n e w e e k after initial presentation, her symptoms
(() Cerebellar degeneration have disappeared and she has returned to normal
(D) Frontal lobe tumor functioning. The most likely diagnosis at this time is:
(E) Prolactinoma
(A) acute stress disorder.
T h e correct response is option C: Cerebellar degener-
(B) brief psychotic disorder.
ation (C) schizoaffective disorder.
Chronic alcoholism is associated with several neuro-
(D) posttraumatic stress disorder.
logic findings: cerebellar anterior lobe degeneration,
(E) schizophreniform disorder.
retrobulbar optic neuropathy, Wernicke's encephalopa- The correct response is option B: Brief psychotic disorder
thy, and amnestic disorder (Korsakoff's syndrome).
G i v e n the recent trauma and short duration of symp-
Cerebellar degeneration is characterized by unsteadi-
toms, brief psychotic disorder is the most reasonable
ness of gait, problems with standing, a n d mild nystag-
diagnosis. Acute stress disorder and PTSD require the
mus. Cerebellar degeneration is probably caused by a
presence of avoidance, increased arousal and reex-
combination of the effects of ethanol and acetaldehyde
periencing the event. PTSD and schizophreniform dis-
along with vitamin deficiencies.
order both require the presence of symptoms for at
Sadock BJ, Sadock VA (eds): Kaplan and Sadock's Comprehensive least 4 weeks.
Textbook of Psychiatry, 8tfi ed. Philadelphia, Lippincott Williams &
Wilkins, 2005, p 1172 Sadock BJ, Sadock VA (eds): Kaplan and Sadock's Comprehensive
Mack AH, Franklin JE, Frances RJ: Substance use disorders, in The Textbook of Psychiatry, 8th ed. Philadelphia, Lippincol: Williams &
American Psychiatric Publishing Textbook of Clinical Psychiatry, Wilkins, 2005, pp 1520-1521
4th ed. Edited by Hales RE, Yudofsky SC. Washington, DC, American Psychiatric Association: Diagnostic and Statistical Manual
American Psychiatric Publishing, 2003, pp 329-330 of Mental Disorders, Fourth Edition, Text Revision (DSM-IV-TR).
Washington, DC, American Psychiatric Association, 2000,
pp 329-332

282
284
Symptoms of obsessive-compulsive disorder respond
W h i c h of the following medications is most likely to
best to w h i c h of the following tricyclic antidepressants?
be associated with polycystic o v a r y syndrome?
(A) Imipramine
(B) Amitriptyline (A) Carbamazepine
(C) Doxepin (B) Gabapentin
(D) Clomipramine (C) Lithium
(E) Desipramine (D) Topiromate
(E) Volproate
T h e correct response is option D: Clomipramine
The correct response is option E: Valproate
Clomipramine, which blocks the neuronal reuptake of
A m o n g w o m e n w h o started valproate before age 2 0 ,
serotonin, improves the symptoms of obsessive-compul-
80% have polycystic' ovaries. Since over 5 0 % of
sive disorder in a manner similar to the newer SSRIs.
w o m e n on valproate are also obese and because
Clomipramine w a s the first drug a p p r o v e d by the F D A
obesity is associated with polycystic o v a r y s y n d r o m e ,
for the treatment of obsessive-compulsive disorder.
it is unclear whether valproate's effects on the high
Stein DJ, Hollander E (eds): American Psychiatric Publishing Textbook rate of polycystic ovaries are a direct result of the
of Anxiety Disorders. Washington, DC, American Psychiatric drug or an indirect effect of contributing to obesity.
Publishing, 2002, p 207
Schatzberg AF, Cole JO, DeBattista C: Manual of Clinical
Psychopharmacology. Washington, DC, American Psychiatric
Publishing, 2005, p 271
285 Feighner JP, Boyer WF, Tyler Dl_ Neborsky RJ: Adverse consequences
of fluoxen'ne-MAOl combination therapy. J Clin Psychiatry 1990;
In the National Institute of Mental Health's Epidemi- 51:222-225
ologic Catchment A r e a study, the ethnic differences in Schatzberg AF, Nemeroff CB (eds): The American Psychiatric
the 1-month prevalence of mental health disorders Publishing Textbook of Psychopharmacology, 3rd ed, Washington,
dropped after w h i c h of the following factors w a s con- DC, American Psychiatric Publishing, 2004, p 309 (Table 18-4)
trolled for?

(A) Age
(B) Education 287
(C) Gender \ ]
The single most effective treatment for major depres-
(D) Literacy rate
sion in elderly patients is:
(E) Socioeconomic status
T h e correct response is option E: Socioeconomic status
(A) bupropion.
(B) citalopram.
A g e , socioeconomic status, and education have been (C) ECT.
confounders in various studies comparing mental ill- (D) nortriptyline.
ness prevalences among different races. H o w e v e r , the (E) venlafaxine.
Epidemiologic Catchment A r e a study specifically con- The correct response is option C: E C T
trolled for socioeconomic status.
Remission rates for E C T are 90% or higher among eld-
Sadock BJ, Sadock VA (eds): Kaplan and Sadock's Comprehensive
erly patients. E C T is especially indicated w h e n an eld-
Textbook of Psychiatry, 8th ed. Philadelphia, Lippincott Williams &
Wilkins, 2005, pp 2285-2286 erly patient is actively suicidal, anorexic, noncompliant
Regier DA, Farmer ME, Rae DS, Myers JK, Kramer M, Robins LN, with medication, or unable to tolerate medication.
George LK, Kamo M, Locke BZ: One-month prevalence of mental
Spar JE, La Rue A: Concise Guide to Geriatric Psychiatry, 3rd ed.
disorders in the United States and sociodemograohic characteris-
Washington, DC, American Psychiatric Publishing, 2002, p 143
tics: the Epidemiologic Catchment Area study. Acta Psychiatr
Scand 1993; 88:35^7

286
A patient has not responded to phenelzine after 10
weeks of treatment, and a switch to fluoxetine is
•planned. W h a t is the recommended minimum interval
between stopping phenelzine and starting fluoxetine?

(A) 1 week
(B) 2 weeks
(C) 4 weeks
(0) 6 weeks
(E) 8 weeks
The correct response is option B: 2 weeks

Phenelzine is an irreversible inhibitor of monoamine


oxidase. O n c e discontinued, a period of 2 weeks is
required for n e w e n z y m e s to be synthesized. Thus, the
switch from phenelzine to fluoxetine would require a
2-week washout period. Without a washout of suffi-
cient length, a fatal serotonin syndrome could occur.
By contrast, because fluoxetine has an extremely long
half-life, a washout of at least 5 weeks is advised if a
switch is to be made from a monoamine oxidase
inhibitor to fluoxetine.

Gadde KM, Krishman KR: Current status of monoamine oxidase


inhibitors in psychiatric practice. Esser.t Psychopharmacol 1997;
1:255-272

Section 2: Answers and Explanations


Linnet KM, Dalsgaard S, Obel C, Wisborg K, Henriksen TB, Rodriguez
288 A, Kotimaa A Moilanen I, Thomsen PH, Olsen J, Jarvelin MR:
A 30-year-old athletic man presents for evaluation of Maternal lifestyle factors in pregnancy risk of attention deficit
hyperactivity disorder and associated behaviors: review of the
several syncopal episodes over the past month. He
current evidence. Am J Psychiatry 2003; 160:1028-1040
has been treated for hypertension during the past
y e a r a n d has responded nicely to 50 m g / d a y of
metoprolol XR and 25 m g / d a y of hydrochloro-
thiazide. Three months a g o his p r i m a r y care physi-
cian started him on 20 m g / d a y of fluoxetine and 290
0.5 m g / d a y of lorazepam t.i.d. for mixed anxiety C o m p a r e d with Caucasian Americans, African Amer-
a n d depression. On examination the patient seems icans are more likely to receive a diagnosis of:
mildly anxious and demonstrates orthostatic hypoten-
sion. His E C G is unremarkable except for mild sinus (A) bipolar disorder, depressed.
b r a d y c a r d i a . W h a t is the most likely explanation? (B) bipolar disorder, manic.
(C) major depression.
(A) Transient ischemic attacks (D) schizophrenia.
(B) Fluoxetine—metoprolol interaction (E) substance-induced psychosis.
(C) Overdiuresis
(D) Benzodiazepine intoxication The correct response is option D: Schizophrenia
(E) Psychogenic syncope
Probably because of multiple factors, such as differ-
T h e correct response is option B: Fluoxetine-metopro- ential health care utilization patterns, differential
lol interaction 1 symptom presentations, mismatch of provider and
patient race, and bias, A f r i c a n Americans a r e more
Fluoxetine is a powerful inhibitor of the cytochrome
likely than Caucasians to receive a diagnosis of
P450 2D6 isoenzyme, which metabolizes beta-block-
schizophrenia. African A m e r i c a n s have been under-
ers, including metoprolol. This c a n lead to increased
diagnosed with bipolar a n d other mood disorders.
serum levels of the beta-blocker, causing orthostatic
hypotension, b r a d y c a r d i a , and often syncope. Snowden L: Bias in mental health assessment and intervention: the-
ory and evidence. Am J Public Health 2003; 93:239-243
Ereshetsky L: Drug interactions of antidepressants. Psychiatr Ann Sadock BJ, Sadock VA (eds): Kaplan and Sadock's Comprehensive
1996; 26:342-350 Textbook of Psychiatry, 8th ed. Philadelphia, Lippincott Williams &
Robinson MJ, Owen JA Psychopharmacology, in The American Wilkins, 2005, p 2284
Psychiatric Publishing Textbook of Psychosomatic Medicine. West JC, Herbeck DM, Bell CC, Colquitt WL, Duffy FF, Fitek DJ, Rae D,
Edited by Levenson JL. Washington, DC, American Psychiatric Stipec MR, Snowden L, Zarin DA, Narrow WE: Race/ethnicity among
Publishing, LW5, pp 872-874, Table 37-2, pp 876-879 psychiatric patients: variations in diagnostic and clinical character-
istics reported by practicing clinicians. FOCUS 2006; 4:48-56

289
Attention deficit hyperactivity disorder (ADHD)
291
appears to be most strongly associated with prenatal T h e clinical sign that best differentiates delirium from
exposure to: dementia is:

(A) caffeine. (A) agitation.


(B) lithium. (B) confusion.
(C) nicotine. (C) fluctuating consciousness.
(D) SSRIs. (D) poor attention span.
(E) valproic acid. (E) psychosis.
T h e correct response is option C: Nicotine The correct response is option C: Fluctuating con-
sciousness
Of the pharmacological agents a n d substances listed,
the only one for w h i c h prenatal e x p o s u r e has been Distinguishing deliriurri from dementia is a frequent
associated with A D H D in a number of case-control clinical challenge. Patients with dementia may exhibit
studies is nicotine. psychosis, agitation, poor attention, a n d confusion.
H o w e v e r , an altered or fluctuating level of conscious-
Sadock BJ, Sadock VA (eds): Kaplan and Sadock's Comprehensive
Textbook of Psychiatry, 8th ed. Philadelphia, Lippincott Williams & ness is the hallmark of delirium. Several methods for
Wilkins, 2005, p 3186 . identifying delirium a n d rating its severity h a v e been
described, including the Delirium Rating Scale a n d
the Confusion Assessment Method. These can assist T h e concct response is option B: 4 to 9 months
the clinician in diagnosing and following delirium.
Continuation treatment typically lasts 4 to 9 months
Spar JE, La Rue A: Concise Guide to Geriatric Psychiatry, 3rd ed. for patients with first-episode nonpsychofic depres-
Washington, DC, American Psychiatric Publishing,'2002, pp 220-226
sion. For those with psychotic depressions, follow-up
American Psychiatric Association: Diagnostic and Statistical Manual
studies 1 y e a r after acute phase treatment indicate a
of Mental Disorders, Fourth Edition, Text Revision (DSM-IV-TR).
Washington, DC, American Psychiatric Association, 2000, p 143 poorer prognosis than for those with nonpsychofic
Trepacz PT, Baker RW, Greenhouse J: A symptom rating scale for depression. JThus, continuation phase treatment for
delirium. Psychiatry Res 1988; 23:89-97 psychotic depressions should be longer.
Inouye SK, van Dyck CH, Alessi CA, Balkin S, Siegal AP, Horwitz Rl:
Clarifying confusion: the confusion assessment method: a new Sadock BJ, Sadock VA (eds): Kaplan and Sadock's Comprehensive
method for detection of delirium. Ann Intern Med 1990; 113:941-948 Textbook of Psychiatry, 8th ed. Philadelphia, Lippincott Williams &
Wilkins, 2005, p 1659

292 -294
Gabapentin has F D A approval as an indication for A 40-year-old w o m a n consults a psychiatrist with a
which of the following? chief complaint of anxiety, insomnia with nightmares,
loss of appetite, a n d chest pain. Tearfully, the patient
(A) Postmenopausal hot flashes
reports that 2 w e e k s a g o her husband left her for
(B) Posttraumatic stress disorder (PTSD)
another w o m a n . T h e husband fold the patient, "I need
(C) Postherpetic neuralgia
someone more adventuresome." She suspected that her
(D) Cocaine dependence
husband w a s having an affair, but she w a s unprepared
The correct response is option C: Postherpetic neuralgia for his leaving. She avoids walking by his office in their
home because w h e n she sees his litter, still on the desk,
G a b a p e n t i n is a p p r o v e d by the Food a n d Drug she feels chest pain. She reports fear of being alone.
Administration for adjunctive treatment of partial She continually daydreams about their life together.
epilepsy a n d management of postherpetic neuralgia. She can "barely function" in her job as a hospital
G a b a p e n t i n , which has a low level of toxicity a n d administrator. T h e most likely preliminary diagnosis is:
renal excretion, w a s originally indicated as an
(A) acute stress disorder.
adjunct antiepileptic medication. As suggesred by the
(B) adjustment disorder with anxiety.
other options in the question, the drug has been stud-
(C) pathological bereavement.
ied with some promise in the treatment of hot flashes,
(D) posttraumatic stress disorder.
cocaine addiction, and P T S D . ' , (E) social phobia.

Physician's Desk Reference. Montvale, NJ, Medical Economics The correct response is option B: Adjustment disorder
Company, 2003, pp 2563-2564
with anxiety
Jeffery S, Pepe J, Popovich L, Vrtagliano G: Gabapentin for hot
flashes in prostate cancer. Annals of Pharmacotherapy 2002; Adjustment disorder with anxiety is the best working
36:433-435
diagnosis. T h e patient's loss, clthough painful, is not
Myrick H, Henderson S, Brady K, Malcolm R: Gabapentin in the treat-
the kind of life-threatening event that acute stress and
ment of cocaine dependence: a case series. J Clinical Psychiatry
2001;62:19-23 posttraumatic stress disorders require for a diagnosis.
Hamner M, Brodrick P, Labbate L: Gabapentin in PTSD: a retrospec- The clinical material offers little evidence for pathologi-
tive clinical series of adjunctive therapy. Annals of Clinical cal bereavement, especially because the husband left
Psychiatry 2001;13:141-146
only 2 weeks a g o . Finally, not being "adventurous"
and dreading the idea of meeting new people are
inadequate data to consider social phobia at this point.
293
Stein DJ, Hollander E (eds): American Psychiatric Publishing Textbook
A patient with a first episode of a nonpsychofic major of Anxiety Disorders. Washington, DC, American Psychiatric
depression has responded well to the acute phase Publishing, 2002, pp 374-375
medication treatment. W h a t is the typical duration of
the continuation phase?

(A) 3 months
(B) 4 to 9 months
(C) 10 to 15 months
(D) 2 years
(E) Lifelong

Section 2: Answers and Explanations 151


295 297
A 74-year-old man falls on an ice patch and bumps Currently, the efficacy of a psychotherapy for treat-
his h e a d . During the next 4 weeks, his wife notices ment of a particular disorder is best judged b y :
that he seems more forgetful and that at night he is
(A) cohort study.
disoriented. He also develops a persistent headache.
(B) individual case outcomes.
W h i c h of the following diagnoses is most likely to be
(C) number needed to treat to number needed to harm ratio.
causing this presentation?
(D) relative risk reduction measure.
(A) Cerebellar tumor (E) systematic review of controlled studies.
(B) Murti-infard dementia
The correct response is option E: Systematic review of
(C) Occipital tumor
controlled studies
(D) Subdural hematoma
(E) Wernicke's encephalopathy Systematic reviews and meta-analysis along with ran-
domized controlled trials provide level 1 evidence for
T h e correct response is option D: Subdural hematoma
treatment strategies. Relative risk is the ratio of the
This may be a reversible dementia ccused by a incidence of the disease among persons exposed to
chronic subdural hematoma. It frequently follows head the risk factor to the incidence among those not
trauma, such as those that occur in falls, w h e n the e x p o s e d . The number needed to treat is the number of
veins of the subdural space are torn. The most com- patients w h o need to receive the experimental treat-
m o n symptoms of subdural hematoma are headache, ment in order to prevent one additional b a d outcome.
m e m o r y loss, apathy, inattention, and confusion. There The number needed to harm is the number of patients
c a n be associated hemiparesis, hemianopsia, and cra- w h o need to receive the experimental treatment in
nial nerve abnormalities, but these are less common. order to lead to an additional bad outcome.

Sadock BJ, Sadock VA (eds): Kaplan.and.Sadock's Comprehensive Sackett DL, Richardson WS, Rosenberg W, Haynes RB: Evidence- •
Textbook of Psychiatry, 8th ed, Philadelphia, Lippincott Williams & Based Medicine: How to Practice and Teach EBM. New York,
Wilkins, 2005, pp 368,482,1092 Churchill Livingstone, 1997, p 15

296 298
W h a t proportion of people with dysthymic disorder A 25-year-old w o m a n is diagnosed with bipolar I dis-
experience an episode of major depression in r'ieir order. She has a previous history of several suicide
lifetime? attempts. Of the following medications, w h i c h w o u l d
be the most likely to decrease her risk for suicide if
(A) 5%—10%
administered on a long-term basis?
(B) 20%-30%
(C) 407o-50°/o (A) Carbamazepine
(D) 70°/o-80% (B) Lamotrigine
(E) 100% (C) Lithium
(D) Risperidone
T h e correct response is option D: 7 0 % - 8 0 %
(E) Verapamil
Some 70%-80% of people with dysthymia have a life-
T h e correct response is option C: Lithium
time diagnosis of major depression, and many seek,
treatment w h e n they develop major depression super- Lithium has been s h o w n in naturalistic studies of bipo-
imposed on their dysthymia. In clinical settings, up to lar disorder to reduce the risk of suicidal behavior. No
75% of individuals w i t h . dysthymic disorder will other medication has been s h o w n to h a v e this effect.
develop major depressive disorder within '5 years.
Schatzberg AF, Nemeroff CB (eds): The American Psychiatric
Dubovsky SL, Dubovsky AN: Concise Guide to Mood Disorders. Publishing Textbook of Psycliopharmacology, 3rd ed. Washington,
Washington, DC, American Psychiatric Publishing, 2002, p 19 DC, American Psychiatric Publishing, 2004, p 551
American Psychiatric Association: Diagnostic and Statistical Manual Cipriani A Pretty H, Hawton K, Geddes JR: Lithium in the prevention
of Mental Disorders, Fourth Edition, Text Revision (DSM-IV-TR). of suicidal behavior and all-cause mortality in patients with mood
Washington, DC, American Psychiatric Association, 2000, p 378 disorders: a systematic review of randomized trials. Am J
Psychiatry 2005; 162:1805-1819
Jefferson JW: Bipolar disorders: a brief guide to diagnosis and treat-
ment FOCUS 2003; 1:7-14 (p 12)
299 300
A 29-year-old man has severe panic attacks cued by A patient w h o s e depression has responded well to an
public speaking. He has d e v e l o p e d .marked a v o i d - SSRI n o w reports symptoms of erectile dysfunction
ance of such situations, whidh has greatly compro- associated with the SSRI antidepressant therapy. This
mised his career development. W h i c h of the following dysfunction has persisted for more than a month. T h e
is the most appropriate diagnosis? best initial a p p r o a c h would be to:

(A) Agoraphobia without panic disorder (A) add bupropion.


(B) Acute stress disorder , (B) take a drug holiday.
(C) Panic disorder with agoraphobia - (C), reduce the dose of the antidepressant.
(0) Social phobia (0) switch to a different SSRI.
(E) Specific phobia (E) contirtue treatment until the patient develops tolerance to
the side effect.
T h e correct response is option D: Social phobia
T h e correct response is option C: Reduce the dose of
Social phobia is a marked a n d persistent fear of social
' the antidepressant
and performance situations in w h i c h embarrassment
may occur. Exposure to the social or performance situ- W h e n a decrease in libido or a sexual dysfunction
ation evokes an anxiety response, which may take the persists despite improvement in mood with an SSRI,
form of a panic attack. The situationally bound (cued) the best a p p r o a c h is to attempt to lower the dose to
panic attacks occurring only in the context of public see if the side effect will remit without loss of efficacy.
speaking are consistent with a diagnosis of Social This often works, but if the side effect persists, then
Phobia. According to DSM-IV-TR, panic attacks can switching to another antidepressant or adding an anti-
occur in the context of many mental disorders. A diag- dote drug should be considered.
nosis of panic disorder requires unexpected (uncued)
Labbate LA, Croft HA, Oleshansky MA Antidepressant-related erectile
panic attacks. Because the fear and avoidance are dysfunction: management via avoidance, switching antidepres-
restricted to a social situation, diagnosis of agorapho- sants, antidotes, and adaptation. J Clin Psychiatry 2003; 64(suppl
bia or specific phobia is not appropriate. Acute stress 10):11—19
Hales RE, Yudofsky SC (eds): The American Psychiatric Publishing
disorder requires exposure to an extreme trauma.
Textbook of Clinical Psychiatry, 4th ed. Washington, DC, American
American Psychiatric Association: Diagnostic and Statistical Manual Psychiatric Publishing, 2003, p 1053
of Mental Disorders, Fourth Edition, Text Revision (DSM-IV-TR).
Washington, DC, American Psychiatric Association, 2000,
pp 430-432,469
301
A patient with a 10-year history of alcohol depen-
dence requests outpatient detoxification. In determin-
ing whether outpatient detoxification is an
appropriate treatment setting for this patient, the most
important variable is:

(A) length of history of alcohol dependence.


(B) support of spouse or significant other.
(C) type of alcohol consumed.
(D) prior history of delirium tremens.
The correct response is option D: Prior history of delir-
ium tremens

Patients with a past history of a complicated with-


drawal syndrome, especially those with a history of
delirium tremens, would not be g o o d candidates for
an outpatient treatment setting. Some outpatient treat-
ment settings can accommodate outpatient detoxifica-
tion that includes frequent clinical assessment.

American Psychiatric Association: Practice Guideline for the


Treatment of Patients With Substance Use Disorders: Alcohol,
Cccaine, Opioids. Am J Psychiatry 1995; 152(Nov suppi)

Section 2: Answers and Explinacions


302 304
During the sexual history, a married 35-year-old male Among patients with major depressive disorder,
reveals that he considers himself to be " o n the d o w n w o m e n have which of the following characteristics
l o w . " Regarding his sexual orientation and partners, c o m p a r e d with men?
he w o u l d most likely consider himself to be:
(A) Earlier age at onset
(A) bisexual, and has sex equally with men and women. (B) Shorter episode duration
(B) heterosexual, and exclusively has sex with women. (C) Higher rotes of comorbid drug abuse
(C) heterosexual, but also secretly has sex with men. (D) Lower rates of comorbid generalized anxiety
(D) homosexual, but also has sex with women. (E) fewer suicide attempts
(E) homosexual, and exclusively has sex with men.
T h e correct response is option A: Earlier a g e at onset
T h e correct response is option C: Heterosexual, but
C o m p a r e d with depressed men, depressed w o m e n
also secretly has sex with men
h a v e an earlier a g e at onset, longer duration of ill-
" D o w n l o w " generally refers to something that is done ness, lower rates of comorbid drug use, higher rates
in secret. It has also been used to describe men w h o of comorbid anxiety, and more suicide attempts.
do not identify themselves as either g a y or bisexual
Marcus SM, Young EA, Kerber KB, Komstein S, Farabaugh AH,
but w h o h a v e sex with w o m e n a n d with men.
Mitchell J, Wisniewski SR, Balasubramani GK, Trivedi MH, Rush
Centers for Disease Control and Prevention: Men on the down low. AJ: Gender differences in depression: findings from the STAR'D
HIV/AIDS Prevention, Questions and Answers. Available at study. J Affect Disord 2005; 87:141-150
http://www.cdc.gov/hiv/PUBS/faq/Downlow.htm ]

305
303 In order to determine the genomic location of a sus-
In order for a patient to meet the diagnostic criteria for ceptibility gene for panic disorder, which of the fol-
substance abuse, which of the following must be present? lowing approaches would be most appropriate?

(A) Physiologic tolerance to the substance (A) Family risk studie-


(B) Physiologic withdrawal from the substance (B) Genetic epidemiology
(C) Pailure to attend to expected cultural role as a result of the (C) Gene finding
substance (D) Molecular genetics
(D) Positron emission tomography findings of mesolimbic tract (E) Twin studies
hyperactivity
T h e correct response is option C: G e n e finding
(E) family history of addiction
Current psychiatric genetics can be o r g a n i z e d into
T h e correct response is option C: Failure to attend to
four paradigms of inquiry: basic genetic epidemiol-
expected cultural role as a result of the substance
o g y , w h i c h is used to quantify the degree of familial
T h e essential feature of substance abuse is a mal- a g g r e g a t i o n a n d / o r heritability; a d v a n c e d genetic
adaptive pattern of substance use manifested by epidemiology, which explores the nature and mode of
recurrent a n d significant a d v e r s e consequences. action of genetic risk factors; gene finding, which
Unlike the criteria for dependence, the criteria for sub- attempts to determine the genomic location a n d iden-
stance abuse do not include tolerance, w i t h d r a w a l , or tity of susceptibility genes; and molecular genetics,
a pattern of compulsive use a n d instead include only w h i c h uses critical D N A variants to trace the biologi-
the harmful consequences of repeated use. cal p a t h w a y s from D N A to disorder.

Mack AH, Frances RJ: Substance-related disorders. FOCUS 2003"; Hales RE, Yudofsky SC (eds): The American Psychiatric Publishing
1:125-146 (p 121) Textbook of Clinical Psychiatry, 4th ed. Washington, DC, American
American Psychiatric Association: Diagnostic and Statistical Manual Psychiatric Publishing, 2003, pp, 3-15
of Mental Disorders, Fourth Edition, Text Revision (DSM-IV-TR). Kendler KS: Psychiatric genetics: a methodologic critique. Am J
Washington, DC, American Psychiatric Association, 2000, Psychiatry 2005; 162:3-11 .
pp 198-199 ,

154 FOCUS Psychiatry Review; 400 Self-Assessment Questions


306 307
During treatment, a female patient reports sexual W h i c h of the following medications has been shown to
encounters with a prior therapist in a state that man- be most* effective in reducing suicidal behaviors in
dates the reporting of sexual abuse by therapists. In patients with schizophrenia or schizoaffective disorder?
the interest of preserving the confidentiality of the doc-
tor-patient relationship, w h i c h of the following is the (A) Clozapine
best response of the therapist? (B) Haloperidol
(C) lithiuni
(A) Refer the patient to another physician for consultation, (0) Olanzapine
specifically for the role of advocacy. (E), Ziprasidone
(B) Request court immunity from the statute to protect the doc-
The correct response is option A: C l o z a p i n e
tor-patient relationship.
(C) Convince the patient to report the matter herself. In individuals with schizophrenia or schizoaffective
(D) Explore the a legation with the patient to determine disorder, several lines of evidence suggest that rates
whether it actually occurred.
of suicidal behaviors, including suicide, a r e dimin-
The correct response is option A: Refer the patient to ished by clozapine treatment. For e x a m p l e , analyses
another physician for consultation, specifically for the of data from the clozapine national registry show
role of a d v o c a c y decreased rates of suicide c o m p a r e d with expected
rates for individuals with schizophrenia. In addition,
Separating the roles of advocate (one w h o reports the
the International Suicide Prevention Trial (InterSePT)
incident] a n d therapist (one w h o treats the patient) is
compared clozapine and olanzapine in 9 8 0 patients
a useful solution in this situation. A therapist must com-
over a 2-year period and found that the c l o z a p i n e
ply with the state statute that requires reporting a sex-
group h a d substantially fewer suicide attempts a n d
ual abuse incident. H o w e v e r , such reporting may
fewer hospitalizations related to suicidality. Lithium
interfere with transference in that the patient m c y be
maintenance treatment is associated with a substan-
inhibited from revealing other issues that she may
tial decrease in rates of suicide a m o n g individuals
want to be confidential but w o u l d hesitate to discuss
with mood disorders, particularly bipolar disorder,
because she fears her doctor w o u l d have to report the
but if has not been adequately studied in terms of sui-
information. Either the first therapist w h o sees the
cidal behaviors in individuals with schizophrenia.
patient reports the incident a n d then refers the patient
to another therapist for treatment or the first therapist Meltzer HY, Alphs L, Green Al, Altamura AC, Anand R, Bertoldi A,
sends the patient to another psychiatrist for the role of Bourgeois M, Chouinard G, Islam MZ, Kane J, Krishnan R,
advocate to report the incident. T h e patient then Undenm.-'ver JP, Potkin S; International Suicide Prevention Trial
Study Group: Clozapine treatment for suicidality in schizophrenia:
returns to the first therapist for further treatment.
International Suicide Prevention Trial (InterSePT). Arch Gen
Psychiatry 2003; 60:82-91
Blcom JD, Nadelson CC, Notman MT: Physician Sexual Misconduct
Reid WH, Mason M, Hogan T: Suicide prevention effects associated
Washington, DC, American Psychiatric Press, 1999, pp 259-261
with clozapine therapy in schizophrenia and schizoaffective disor-
Simon Rl: Therapist-patient sex: maintaining treatment boundaries, in
der. Psychiatr'serv 1998; 49:1029-1033
Concise Guide to Psychiatry and Law for Clinicians, 3rd ed.
American Psychiatric Association: Practice Guideline for the
Washington, DC, American Psychiatric Publishing, 2001, p 342
Assessment and Treatment of Patients With Suicidal Behaviors.
Am J Psychiatry 2003; 160(Nov suppl):1-60

Section 2: Answers and Explanations 155


308 Culture-bound syndromes are culturally based signs
and symptoms of mental distress or maladaptive
W h i c h of the following diagnostic criteria most clearly
behavior that are prominent in folk belief a n d prac-
distinguishes paranoid personality disorder from para-
tice. A m o k is a dissociative episode characterized by
noid schizophrenia, delusional disorder, and mood
a period of brooding followed by an outburst of vio-
disorder with psychotic features?
lent, aggressive, or homicidal behaviors directed at
(A) Absence of positive psychotic symptoms persons or objects. The episode tends to be precipi-
(B) Age at onset tated by a perceived slight or insult a n d seems to be
(C) Degree of impairment in interpersonal relationships prevalent only among men. T h e episode is often
(D) Duration of symptoms accompanied by persecutory ideation, automatism,
(E) Pervasive nature of symptoms amnesia, exhaustion, and a return to premorbid state
T h e correct response is option A: A b s e n c e of positive after the episode. The original reports that used this
psychotic symptoms term w e r e from Malaysia.

Paranoid personality disorder is m a r k e d by pervasive Sadock BJ, Sadock VA (eds): Kaplan and Sadock's Synopsis of
distrust a n d suspiciousness of others. This may be Psychiatry: Behavioral Sciences/Clinical Psychiatry, 9th ed.
Philadelphia, Lippincott Williams & Wilkins, 2003, pp 529-530
present in paranoid schizophrenia, a delusional dis-
Griffith EE, Gonzalez CA, Blue HC: Introduction to cultural psychiatry,
order, or a mood disorder with psychotic features. The
in The American Psychiatric Publishing Textbook of Clinical
a g e at onset of symptoms, d e g r e e of impairment, Psychiatry, 4th ed. Edited by Hales RE, Yudofsky SC. Washington,
duration of symptoms, or pervasive nature of the DC, American Psychiatric Publishing, 2003, p 1567
symptoms may be of little help in differentiating para-
n o i d personality disorder from the other disorders
listed. H o w e v e r , in paranoid personality disorder,
positive psychotic symptoms should not be present, 310
w h e r e a s they are key diagnostic criteria for each of W h a t is the most common comorbid condition in chil-
the other disorders. dren with autistic disorder?

Sadock BJ, Sadock VA (eds): Kaplan and Sadock's Comprehensive (A) Attention deficit hyperactivity disorder
Textbook of Psychiatry, 7th ed. Philadelphia, Lippincott Williams & (B) Major depression
Wilkins, 2000, pp 1742-1743 (C) Mental retardation
(D) Schizophrenia
(E) Social phobia
309 . T h e correct response is option C: Mental retardation

A 19-year-old e x c h a n g e student from Malaysia is Approximately 80% of children with autistic disorder
brought to the emergency department by his host par- are mentally retarded. Psychotic symptoms exclude
ents after he became violent at home a n d threatened the diagnosis of autistic disorder. A D H D , major
to kill them. T h e parents report that he seemed fine
depressive disorder, and social phobia may occur but
until they commented to him that he h a d left the faucet
are less frequent than mental retardation.
running in the bathroom. Initially, he w e n t to his room
a n d seemed sullen. He then b e g a n "ranting and rav- Dulcan MK, Martini DR, Lake MB: Concise Guide to Child and
i n g " about h o w he is not an irresponsible person, Adolescent Psychiatry, 3rd ed. Washington, DC, American
accused the host parents of spying on him, threatened Psychiatric Publishing, 2003, p 190
them, threw objects about, and collapsed on the floor
in exhaustion. In the emergency department, the stu-
dent is calm a n d cooperative. Mental status examina-
tion is unremarkable. T h e student denies a n y recall of
the episode. This presentation is most consistent with
w h i c h culture-bound syndrome?

(A) Amok
(B) Dhat
(CJ Koro
(D) Locura
(E) Rootwork
T h e correct response is option A: A m o k *

156 FOCUS Psvchiatrv Review: 400 Self-Assessment Ouestions


31] Unlike in dementia of the Alzheimer's type, visuospatial
skills are usually preserved in the early to middle stages
A 65-year-old patient is admitted to the surgical inpa-
of frontotemporal dementia. C o m m o n early symptoms
tient service for a hernia repair. T h e family reported
of frontotemporal dementia include personality changes
that over the past few months the patient has had
(particularly social withdrawal and apathy), loss of
episodes of confusion. W h i l e on the w a r d , the patient
b e g a n to have prominent visual hallucinations. The judgment and insight, disinhibition, and changes in oral
staff administered 1 mg of haloperidol orally. A sec- behaviors. Compulsions a n d cravings are frequently
o n d dose w a s given 3 hours later. Soon after receiv- seen (e.g., carbohydrate cravings and hyperorality).
ing the second dose of haloperidol, the patient had a Verbal output m a y be diminished, a n d semantic
severe extrapyramidal response. W h i c h of the follow- anomid, a specific form of anomia, is common.
ing is the most likely diagnosis?
Coffey CE, Cummings JL: American Psychiatric Press Textbook of
(A) Delirium with preexisting dementia Geriatric Neuropsychiatry, 2nd ed. Washington, DC, American
(B) Parkinson's dementia Psychiatric Press, 2000, pp 518-519
(C) Lewy body dementia
(D) Major depressive disorder with psychosis
|E) Alcohol withdrawal
313
The correct response is option C: Lewy body dementia
Of the following disorders, which has the greatest
Lewy body dementia is characterized by the appear- genetic contribution or heritability?
ance of visual hallucinations and other psychotic symp-
(A) Majcr depressive disorder
toms early and sensitivity to extrapyramidal side effects
(B) Alcoholism
of antipsychotics. It also responds with remarkable sen-
(C) Obsessive-compulsive disorder
sitivity to antipsychotic medication. The disorder has
(D) Schizophrenia
some Parkinsonian features. These perhaps account for (E) Panic disorder
the sensitivity to antipsychotics. The course of this
dementia follows a rapid evolution. The combination of The correct response is option D: Schizophrenia

"memory problems," visual hallucinations, and sensi- The proportion of phenotypic differences among indi-
tivity to haloperidol make the choice of Lewy body viduals that c a n be attributed to genetic factors, of
dementia the most compelling option for this question. heritability, is 7 0 % to 8 9 % for schizophrenia, around

Practice Guideline for the Treatment of Patients With Alzheimer's 60% for obsessive-compulsive disorder, 4 0 % to 60%
Disease and Other Dementias of Late Life (1997), in American for alcohol d e p e n d e n c e , a n d 40% to 4 5 % for major
Psychiatric Association Practice Guidelines for the Treatment of depression a n d panic disorder.
Psychiatric Disorders, Compendium 2004. Washington, DC, APA,
2004,p 82 Knowles JA: Genetics, in The American Psychiatric Publishing
Textbook of Clinical Psychiatry, 4th ed. Edited by Hales RE,
Yudofsky SC. Washington, DC, American Psychiatric Publishing,
2003, pp 17-34
Sadock BJ, Sadock VA (eds): Kaplan and Sadock's Comprehensive
3j2 Textbook of Psychiatry, 8th ed. Philadelphia, Lippincott Williams &
W h i c h of the following is most likely to be preserved
in the early stages of frontotemporal dementia?

(A) Judgment
(B) Personality
(C) Verbal output
(D) Visuospatial skills
(E) Sociability or social involvement
The correct response is option D: Visuospatial skills
Sadock BJ, Sadock VA (eds): Kaplan and Sadock's Comprehensive
314 Textbook of Psychiatry, 8th ed. Philadelphia, Lippincott Williams &
A 25-year-old man collects women's bras a n d under- Wilkins, 2005, p 1778
pants from public laundries and uses the objects to American Psychiatric Association: Diagnostic and Statistical Manual
of Mental Disorders, Fourth Edition, Text Revision (DSM-IV-TR).
b e c o m e sexually aroused. This description is most
Washington, DC, American Psychiatric Association, 2000,
consistent with which of the following DSM-IV-TR diag-
pp 472-476
noses?

(A) Exhibitionism
(B) Fetishism
(C) Frotteurism 316
(D) Sexual masochism
The side effect of pancreatitis is linked most closely to
(E) Kleptomania
which of the following?
T h e correct response is option B: Fetishism
(A) Divalproex
Fetishism involves nonliving objects (other than articles (B) Oxcarbazepine
(C) Lamotrigine
of clothing used for cross-dressing or devices designed
(D) Topiramate
for tactile genital stimulation) that result in recurrent,
intense, sexually arousing fantasies, sexual urges, or The correct response is option A: Divalproex
behaviors involving the objects. Exhibitionism is the
In 2000, a black box warning about cases of life-
exposure of one's genitals to an unsuspecting stranger.
threatening pancreatitis w a s a d d e d to the valproate
Frotteurism is the rubbing of one's gerfitals against
package insert. Although routine monitoring of pan-
unsuspecting, nonconsenting persons. Sexual mas-
creatic function is not necessary, clinical manifesta-
ochism involves sexual fantasies, urges, or behcviors
tions consistent with pancreatitis should be promptly
involving the real cct of being humiliated, beaten,
and fully evaluated.
b o u n d , or otherwise made to suffer.
Asconape J J , Penny JK, Dreifuss FE, Riela A, Mirza W: Valproate-
American Psychiatric Association: Diagnostic and Statistical Manual
associated pancreatitis. Epilepsia 1993; 34:177-183
of Mental Disorders, Fourth Edition, Text Revision (DSM-IV-TR).
Pizzuti DJ: Dear health care professional. Abbott laboratories, July
Washington, DC, African Psychiatric Association, 2000,
2000
pp 569-570
McArthur KE: Review article: drug-induced pancreatitis. Aliment
Pharmacol Ther 1996; 10:23-38

315
A 24-year-old man comes for an evaluation because he 317
cannot relax. He reports that he constantly is thinking
W h i c h of the following is a technique of supportive
about whether his car will break d o w n , his bills will get
dynamic psychotherapy?
paid, and if his school performance is adequate. For
o v e r a year, he often is tired, irritable, and on edge. (A) Transference interpretation
U p o n reflection, the student is unable to identify any (B) Promoting therapeutic regression
aspect of his life that is going so well that it does not (C) Extreme passivity of therapist
generate concern. The most likely diagnosis is: (D) Problem-solving focus
(E) frequent genetic reconstruction
(A) depressive disorder not otherwise specified.
(B) generalized anxiety disorder. The correct response is option D: Problem-solving
(C) obsessive-compulsive disorder. focus
(D) panic disorder.
(E) social phobia. Problem solving is an important technique a n d goal of
supportive treatment that augments certain weak-
T h e correct response is option B: G e n e r a l i z e d anxiety
nesses or deficits in the parent's psychological func-
disorder
tioning and provides a sense of mastery. Transference,
G e n e r a l i z e d anxiety disorder is the diagnosi* that best although of great help in understanding a patient
explains the student's symptoms. A person with gener- receiving supportive psychotherapy, is rarely addres-
. alized anxiety disorder finds it difficult to control the sed because it can often promote disorganization from
w o r r y , often about e v e r y d a y routine life circumstances. intense feelings outside the patient's a w a r e n e s s . A l -
T h e w o r r y is associated with symptoms such as fatigue, though p r o m o t i n g ' a therapeutic regression is a hall-
difficulty concentrating, and sleep disturbance. mark of psychoanalysis, patients in supportive

158 FOCUS Psvchiatrv Review: 400 Self-Assessment Questions


psychotherapy often lack the strength (emotionally a n d Personclization (assuming personal causality) is the
cognitively) to recover from a therapeutic regression linking of external occurrences to oneself. Arbitrary
both within the hour a n d after its conclusion. Extreme inference is the process of coming to a conclusion
passivity can often raise a patient's level of anxiety to without adequate supporting evidence or despite con-
an unhelpful level a n d lead to discomfort a n d leaving tradictory evidence. Absolutist thinking is categorizing
treatment. T h e exploration of'potentially significant oneself into rigid dichotomies. Catastrophic thinking is
early life experiences is often accompanied by high predicting the worst possible outcome while ignoring
levels of anxiety a n d dysphoria, which can compro- more likely eventualities. Magnification and minimiza-
mise a patient's level of functioning. tion entail over- or undervaluing the significance of a
personal attribute, a life event, or a future possibility.
Kay J: The essentials of psychodynamic psychotherapy. FOCUS
2006; 4:167-172 (171) Hales RE, Yudofsky SC (eds): The American Psychiatric Publishing
Kay J, Kay RL Individual psychoanalytic psychotherapy, in Textbook of Clinical Psychiatry, 4th ed. Washington, DC, American
Psychiatry. Edited by Tasman A, Kay J, Lieberman JA. Psychiatric Publishing, 2003, p 1248
Philadelphia, WB Saunders, 1997, p 1384

320
318
An actor has received repeated complaints from col-
T h e most effective behavior therapy technique used in leagues about his behavior in professional situations.
the treatment of compulsions of obsessive-compulsive He has just started rehearsals for a play. The problem-
disorder is: atic behavior consists of excessive demands for special
treatment and outbursts w h e n special treatment is not
(A) exposure and response prevention.
granted. He is diagnosed as having narcissistic per-
(B) negative reinforcement.
sonality disorder. He has been in treatment for several
(C) positive reinforcement.
months; treatment has been going well, and there have
(D) punishment.
been fewer demands and outbursts at work. W h i c h of
(E) systematic desensitization.
the following is the patient most likely to do next?
The correct response is option A: Exposure a n d
response prevention
(A) Generalize this behavior to his home environment
(B) Demand new concessions from the play's director
T h e compulsions of obsessive-compulsive disorder (C) Show a new understanding of his behavior
may be treated by exposing the patient to stimuli that (D) Continue to show appropriate behavior at work
evoke obsessive w o r r i e s (exposure). Then the patient (E) Discuss his feelings about the therapist
is not allowed to respond to his/her worries by per- The correct response is option B: Demand n e w con-
forming compulsions (response prevention). cessions from the play's director
Stein DJ, Hollander E (eds): American Psychiatric Publishing Textbook
Patients with narcissistic personality disorder struggle
of Anxiety Disorders. Washington, DC, American Psychiatric
and often cannot tolerate feeling and doing better
Publishing, 2002, p 222
because that would imply that the therapist has
helped them. In psychodynamic therapy, patients with
narcissistic personality disorder find it difficult to
319 improve because of this. Improvement means that the
therapist c a n " g i v e " them help, that is, that the thera-
A man w h o is receiving cognitive behavior therapy for
pist has something the patient does not.
depression feels guilty for massive layoffs at his work-
place, even though he w a s not involved in the manage- Cloninger CR, Svrakic DM: Personalit/ disorders, in Kaplan and
ment decision. W h i c h of the following types of cognitive Sadock's Comprehensive Textbook cf Psychiatry, 7th ed. Edited by
error is most consistent with this patient's feeling? Sadock BJ, Sadock VA. Philadelphia, Lippincott Williams &
Wilkins, 2000, pp 1757-1758
(A) Arbitrary inference Gabcard GO: Psychodynamic Psychiatry in Clinical Practice, 4th ed.
(B) Absolutist thinking Washington, DC, American Psychiatric Publishing, 2005, pp 508-509
(C) Catastrophic thinking
(D) Magnification and minimization
( E ) Personalization
T h e correct response is option E: Personalization

Section 2: Answers and Explanations 159


321 323
Of the following, which is the best definition of eth- T h e most common DSM-IV axis II personality disorder
nicity? H u m a n groups that: demonstrated among persons with substance use dis-
orders is:
(A) share a sociopolitical designation.
(B) share common values, beliefs, history, and customs. (A) borderline personality disorder.
(C) hove common identities, ancestries, and histories. (B) narcissistic personality disorder.
(D) share distinct identifying phenotypic characteristics. (C) dependent personality disorder.
(E) ore living together in the same location. (D) antisocial personality disorder.
T h e correct response is option C: H a v e common iden- The correct response is option D: Antisocial personal-
tities, ancestries, and histories ity disorder

Cultural groups share common values, beliefs, history, In epidemiological studies, antisocial personality dis-
a n d customs, while ethnic groups have common iden- order has been found to be the most common axis II
tities, ancestries, a n d histories. Phenotypic character- personality disorder comorbid with substance use dis-
istics are often mistakenly used as indicators of race. orders.

Sadock BJ, Sadock VA (eds): Kaplan and Sadock's Synopsis of Regier DA, Farmer ME, Rae DS, Locke BZ, Keith SJ, Judd U,
Psychiatry: Behavioral Sciences/Clinical Psychiatry, 9th ed. Goodwin FK: Comorbidity of mental disorders with alcohol and
Philadelphia, Lippincott Williams & Wilkins, 2003, p 169 other drug abuse. Results from the Epidemiologic Catchment Area
Tseng WS, Streltzer J: Culture and Psychotherapy: A Guide to Clinical (ECA) Study. JAMA 1990; 264:2511-2518
Practice. Washington, DC, American Psychiatric Publishing, 2001, Skodol AE, Oldham JM, Gallaher PE: Axis II comorbidity of substance
pp 4-6 use disorders among patients referred for treatment of personal-
Hales RE, Yudofsky SC (eds): The American Psychiatric Publishing ity disorders. Am J Psychiatry 1999; 156:733-738
Textbook of Clinical Psychiatry, 4th ed. Washington, DC, American
Psychiatric Publishing, 2003, p 1552
Levine BH, Albucher RC: Patient management exercise for gender,
race, and culture. FOCUS 2006; 4:14-22 (p 20)
324
A 72-year-old w o m a n is hospitalized with findings of
dementia, ataxia, and macrocytic anemia. T h e most
322 likely diagnosis is:

W h i c h of the following psychotherapeutic approaches (A) dementia of the Alzheimer's type.


provides the primary framework for d i a l e d ; *.nl behav- (B) vascular dementia.
ior therapy for borderline personality disorder'? • (C) vitamin B deficiency.
12

(D) Huntington's disease.


(A) Cognitive behavior therapy (E) pellagra.
(B) Interpersonal psychotherapy
(C) Psychodynamic psychotherapy The correct response is option C: Vitamin B 1 2 deficiency
(D) Family systems therapy
(E) Supportive psychotherapy Vitamin B 1 2 deficiency can present with cerebral, spinal
cord, and peripheral nerve involvement as well as
T h e correct response is option A: Cognitive behavior
defective hematopoiesis in the form of a macrocytic or
therapy
megaloblastic anemia. Pernicious anemia is the most
Linehan's dialectical behavior therapy is a cognitive common cause. Pellegra is caused by a deficiency of
behavior t h e r a p y - b a s e d approach to borderline per- niacin (vitamin B ) and is characterized classically by
3

sonality disorder that includes a mix of individual ther- findings of dementia, diarrhea, and dermatitis.
a p y and intensive skills training. .
Moore DP, Jefferson JW: Vitamin B deficiency, in Handbook of
12

Linehan MM: Cognitive-Behavioral Treatment of Borderline' Medical Psychiatry, 2nd ed. Philadelphia, Elsevier Mosby, 2004,
Personality Disorder. New York, Guilford, 1993, pp 37-41 pp 401-403 , ,
Practice Guideline for the Treatment of Patients With Borderline
Personality Disorder (2001), in Practice Guidelines for the
Treatment of Psychiatric Disorders, Compendium 2004\
Washington, DC, APA, 2004, p 799

FOCUS Psvchiatrv Review: 400 SeJf-Ass^r-cnt Questions


325 T h e correct response is option A: It m a y be ethically
problematic if the psychiatrist w a s driven by personal
A 20-year-old w o m a n describes a 6-month history of
needs rather than by serving the patient's needs
frequent binge eating followed by self-induced vomit-
ing a n d laxative use to maintain normal b o d y weight. Self-disclosure in p s y c h o t h e r a p y need not a l w a y s be
W h i c h of t h e following medications is F D A - a p p r o v e d problematic. For instance, a patient has the right to
for her disorder? information that is relevant to judge the professional
(A) Bupropion qualifications of the psychiatrist. In the substance
(B) Citalopram (
abuse treatment a r e n a , self-disclosures are more
(C) Escitalopram '« • accepted. O c c a s i o n a l , well-thought-out disclosures for
(D) Fluoxetine therapeutic effect m a y also be acceptable in some
(E) Venlafaxine instances' H o w e v e r , w h e n a psychiatrist discloses
inforrhation out of personal need rather than for the
T h e correct response is option D: Fluoxetine
therapeutic needs of the patient, e v e n if it arises out of
T h e patient's history is consistent with a diagnosis of deeply felt emotions, the primacy of the patient's best
bulimia nervosa. T h e only F D A - a p p r o v e d medication interests is put in jeopardy.
for this disorder is fluoxetine, based on two 8-week
Gabbard GO: Psychodynamic Psychiatry in Clinical Practice: The
and one 16-week placebo-controlled studies. Flu-
DSM-IV edition. Washington, DC, American Psychiatric Press,
oxetine w a s also shown to be more effective than 1994, pp 104-106
placebo in a 1-year relapse prevention study. T h e F D A Hundert EM, Appelbaum PS: Boundaries in psychotherapy: model
approves a medication after evidence of its effective- guidelines. Psychiatry 1995; 58:345-356
ness from more than o n e double-blind, placebo-con-
trolled study. O n c e a drug has been F D A a p p r o v e d , it
can be used for the treatment of other illnesses.
327
Becker AE, Grinspoon SK, Klibanski A, Herzog DB: Eating disorders. N
W h i c h of the following will c a u s e the greatest
Engl J Med 1999; 340:1092-1098
Mehler PS: Bulimia nervosa. N Engl J Med 2003; 349:875-881
increase in serum lithium levels?
Romano SJ, Halmi KA, Sarkar NP, Koke SC, Lee JS: A placebo-con- (A) Theophylline
trolled study of fluoxetine in continued treatment of bulimia ner-
(B) Ziprasidone
vosa after successful acute fluoxetine treatment. Am J Psychiatry
2002;159:96-102
(C) Hydrochlorothiazide
Prozac (fluoxetine) prescribing information (package insert) (0) Celecoxib
Yager J, Devlin MJ, Halmi KA, Here; q DB, Mitchell JE, Powers PS,
Zerbe KJ: Eating disorders. FOCUS Z2Q5; 3:503-510 (pp
The correct response is option C: H y d r o c h l o r o t h i a z i d e
506-507)
T h i a z i d e diuretics are well established as consistently
American Psychatric Association: Practice Guideline for the
causing substantial increases in serum lithium levels,
Treatment of Patients With Eating Disorders, 3rd ed. Am J
Psychiatry 2006; 163(July suppl):1-54 sometimes causing lithium intoxication. T h e o p h y l l i n e ,
a xanthine diuretic, increases renal lithium clearance
and may lower serum lithium levels. Ziprasidone d i d
not change steady-state levels or renal clearance of
326 lithium (see p a c k a g e insert). In a study of healthy v o l -
A psychiatrist w h o is grieving from a recent sudden unteers, celecoxib increased serum lithium levels by a
loss of a spouse shares those feelings with a psy- mean of 17%, w h i c h does not exclude the possibility
chotherapy patient. W h a t is the most ethical interpre- of a more clinically meaningful interaction but makes
tation of the psychiatrist's actions? it seem unlikely.
(A) It may be ethically problematic if the psychiatrist was Finley PR, Warner MD, Peabody CA: Clinical relevance of drug inter-
driven by personal needs rather than by serving the actions with lithium. Clin Pharmacokinet 1995; 29:172-191
Apseloff G, Mullet D, Wilner KD, Anziano RJ, TensfeldtTG, Pelletier
patient's needs.
SM, Gerber N:The effects of ziprasidone on steady-state lithium
(B) It is always ethically unacceptable because a psychiatrist levels and renal clearance of lithium. Br J Clin Pharmacol 2000;
should never reveal personal information to a patient. 49(suppl 1):61S-64S
(C) It is problematic to reveal any information other than the
psychiatrist's professional training.
(D) It is not ethically problematic because sharing the psychia-
trist's authentic feelings with patients is therapeutic for the
patient.

Section 2: Answers and Explanations


Catatonic features may be present in as many as one-
328
third of patients during a manic episode. W h i l e the
W h i l e reviewing the treatment plan for a patient with
patient may be responsive to benzodiazepines, elec-
methamphetamine dependence, the psychiatrist thinks
troconvulsive therapy is believed to be the most effec-
about h o w best to help the patient progress from the
tive treatment for catatonia, "regardless of etiology."
contemplation phase to the preparation phase. The
psychiatrist's cpproach to treatment in this case is American Psychiatric Association: Practice Guideline for the
based on the principles of: Treatment of Patients With Bipolar Disorder (Revision). Am J
Psychiatry 2002; 159(April suppl). Reprinted in FOCUS 2003;
(A) 12-step facilitation therapy. 1:64-110 (p 73)
(B) cognitive behavior therapy (CBT).
(C) contingency management therapy.
(D) motivational enhancement therapy (MET).
T h e correct response is option D: Motivational
330
enhancement therapy (MET) The cornerstone of relapse prevention as a modality
of treatment for substance-dependent patients is:
Motivational enhancement therapy (MET) is based on
the concept that people move through predictable stages (A) psychodynamic technique.
in changing habitual behaviors. As originally described (B) 12-step group attendance.
by Prochaska and DiClemente, these stages are precon- (C) motivational enhancement.
templation, contemplation, preparation, action, and (D) skills training.
maintenance. The treatment goals and, to ^ome extent, The correct response is option D: Skills training
therapeutic technique v a r y according to stage.
Relapse prevention is a behavior therapy that com-
DiClemente CC, Velasquez MM: Motivational interviewing and the
bines skill training with cognitive intervention tech-
stages of changejn MotivatjonaMnterviewing..Edited by Miller..
WR, Rollnick S. New York, Guilford Press, 2002, pp 201-216 niques. In this approach, patients are taught
Nowinski J, Baker S, Carroll KM: Twelve-step Facilitation Therapy behavioral a n d cognitive skills such as changing
Manual. Project MATCH Monograph Series, vol 1. Rockville, MD, thoughts and beliefs, resisting social pressure, increas-
National Institute on Alcohol Abuse and Alcoholism, 1995. Available ing assertiveness, and improving interpersonal com-
at http://www.commed.uchc.edu/match/pubs/monograph.htm
i munication. Relaxation and stress management
Kadden R, Carroll KM, Donovan D, Cooney N, Monti P, Abrams D, Litt
techniques are also emphasized. T h e effectiveness of
M, Hester R: Cognitive-Behavioral Coping Skills Therapy Manual.
Project MATCH Monograph Series, vol 3. Rockville, MD, National these cognitive behavioral techniques appears to
Institute on Alcohol Abuse and Alcoholism, 1995. Avai^'cle at have longer-lasting benefits than some other treatment
http^/mw.commed.uchc.edu/match/pubs'monograph.m.?! modalities. T h e y are particularly effective w h e n a co-
Miller WR, Zweben A, DiClemente CC, Rychtarik RG: Motivational
occurring psychiatric disorder such as anxiety or
Enhancement Therapy Manual. Project MATCH Monograph Series,
depression is present.
vol 2. Rockvilie, MD, National Institute on Alcohol Abuse and
Alcoholism, 1994. Available at
Mariatt GA, Gordon J (eds): Relapse Prevention: Maintenance Strategies
http://wwv.'.commed.uchc.edu/match/pubs/monograph.htm
in the Treatment of Addictive Behaviors. New York, Guilford, 1985
Budney AJ, Higgins ST: A Community Reinforcement Approach: Treating
Gold PB, Brady KT: Evidence-based treatments for substance use
Cocaine Addiction. Therapy manuals for drug addiction, Manual 2.
disorders. FOCUS 2003; 1:115-122 (p 116)
Rockville, MD, National Institute on Drug Abuse, 1998. Available at
Carroll KM, Ball SA, Martino S: Cognitive, behavioral, and motiva-
http-y/wwvv\dmgabuse.gov/TB/Clinical/Clinicarroolbox.html
tional therapies, in The American Psychiatric Publishing Textbook
of Substance Abuse Treatment, 3rd ed. Edited by Galanter M,
Kleber HD. Washington, DC, American Psychiatric Publishing,
' 2004, pp 369-370
329
W h i c h of the following is the most effective treatment
for catatonic features associated with a, manic
episode? 331
W h i c h of the following i<» the most common sexual dis-
(A) Uthium
order in men?
(B) Electroconvulsive therapy
(C) Divalproex * (A) Hypoactive sexual desire disorder
(D) Oozapine (B) Male erectile disorder •
T h e correct response is option B: Electroconvulsive
(C) Premature ejaculation
therapy
(D) Male orgosmic disorder '
(E) Dyspareunia
4

162 FOCUS Psvrlv.ury Review: 400 Self-Assessment Questions


T h e correct response is option C: Premature ejaculation 333
Data from the National Health and Social Life Survey Mr. B, a high school teacher in his mid-30s, w a s
showed that almost one-third of men said they had recently separated from his wife a n d two children. An
recurring problems with ejaculating too early, making intelligent a n d verbally facile man with a particular
it the most common sexual disorder in men. Premature talent in the arts, Mr. B w a s plagued by his conviction '
that he w a s unacceptable to other people unless he
ejaculation is defined as persistent or recurrent ejacu-
complied with their expectations a n d gratified their
lation with minimal sexual stimulation or before, o n , or
needs. This w a s a pleasant, agreeable, and compli-
shortly after penetration a n d before the person wishes
ant facade that hid his feelings of weakness a n d stu-
it, resulting in marked distress or interpersonal difficulty.
pidity. He constantly sought a p p r o v a l from his
Laumann EO, PaikA, Rosen RC: Sexual dysfunction in the United superiors, but underneath he felt resentment a n d
States: prevalence and predictors. JAMA 1999; 281:537-544 rebelliousness about others' expecting him to accom-
modate to their needs a n d wishes.

Mr. B's mother w a s an embittered, burdened w o m a n ,


contemptuous of men and preoccupied with her o w n
332
needs and interests. His father, while somewhat
An adult female patient consumes an a v e r a g e of 14 a p p r o a c h a b l e , had often been a w a y from home trying
glasses of w i n e per w e e k , never consuming more than to make a living to support the family. Mr. B remem-
four glasses on a n y one occasion. Based solely on this bered his father as erratic and m o o d y and given to
drinking pattern, her physician should do which of the temper outbursts, which, he recalls, would lead to
following? beatings with a leather strap. T h e middle of three chil-
dren, the patient felt that his father favored his older
(A) Refer her to an addiction specialist for further evaluation.
sister a n d that his mother favored his younger brother,
(B) Recommend that she begin attending AA meetings.
and he s a w himself as the neglected outsider.
(C) Inform her that she is drinking at a safe level.
(D) Recommend that she reduce her drinking by about 50%. W h a t is the most likely defense mechanism utilized by
this patient w h e n first meeting the psychiatrist?
T h e correct response is option D: Recommend that she
reduce her drinking by about 50% (A) Regression
(B) Altruist
The physician should assess the patient for alcohol- (C) Undoing projection
related problems and definitely recommend that the (D) Intellectualization rationalization
patient decrease her drinking to safer levels. This level (El Dissociation
of alcohol consumption puts the patient at risk of alco-
The correct response is option D: Intellectualization
hol-related problems. N o n h a z a r d o u s drinking for
rationalization
w o m e n is seven standard drinks per week, with no
more than three per occasion. Without a history of All defense mechanisms, by definition, are outside of
more alcohol-related problems, outside referral is not the patient'^ awareness. T h e y function to w a r d off
necessary. T h e patient will likely respond well to brief anxiety a n a conflict. Rationalization is the use of
office-based intervention. seemingly logical explanations to make untenable
thoughts or feelings more acceptable. In the case of
Fleming MF, Mundt MP, French MT, Manwell LB, Stauffacher EA Barry
KL: Brief physician advice with problem drinkers: long-term efficacy Mr. B, his verbal facility and educational accomplish-
and benefit-cost analysis. Alcohol Clin Exp Res 2002; 26:36-43 ment make this a likely characteristic of his personal-
Fiellin DA, Reid MC, O'Connor PG: Outpatient management of patients ity. Regression refers to a partial return to earlier
with alcohol problems. Ann Intern Med 2000; 133:815-827 levels of functioning or adaptation in order to a v o i d
painful or conflicted feelings or thoughts.

The following four questions (333-336) form a Altruism is a higher-order defense mechanism.
serial vignette. Projection is considered a primitive defense mecha-
nism that is frequently found in patients with signifi-
cant suspiciousness and that consists of attribution of
conflicted feelings, wishes, or thoughts to another per-
son or g r o u p . Dissociation is characterized as the
splitting off of threatening thoughts or feelings.

Kay J: The essentials of psychodynamic psychotherapy. FOCUS


2006; 4:167-172 (169)

Section 2: Answers and Explanations


Countertransference is defined broadly as all of the
334
feelings of the therapist evoked by the patient within the
Because of an emergency, Mr. B's psychiatrist w a s 20
therapeutic relationship. Although the term originally
minutes late to the second interview. Mr. B makes an
referred to a process outside the therapist's awareness
offhand a n d somewhat negative comment about
and therefore an indication of conflict, it is now used
"doctors being too busy these d a y s . " In all likelihood,
frequently to describe all of the therapist's feelings and
this is an example of:
behavior stimulated by the patient. Denial is a primitive
(A) reaction formation. defense mechanism characterized by a refusal to
(B) transference. appreciate information about oneself or the other.
(C) idealization. Regression refers to a partial return to earlier levels of
(D) splitting. functioning or adaptation in order to avoid painful or
(E) suppression. conflicted feelings or thoughts. Deidealization mini-
T h e correct response is option B: Transference mizes the other. Dissociation is characterized as the
splitting off of threatening thoughts or feelings.
A current definition of transference is a combination
of a real current relationship and relationships from Kay J: The essentials of psychodynamic psychotherapy. FOCUS
the patient's past. Reaction formation is the transfor- 2006;4:167-172 (169)
Inderbitzin LB, Levy ST: Psychoanalytic theories, in Psychiatry. Edited
mation of an unwanted thought or feeling into its
by Tasman A, Kay J, Lieberman JA. Philadelphia, WB Saunders,
opposite. Splitting is the experiencing of others as
1997, pp 443-446
being all g o o d or all bad (i.e., idealization or deide-
alization). Suppression is not a defenses mechanism
since it is the conscious attempt to control unaccept-
able feelings and wishes. Transference is also outside 336
of the patient's awareness during the initial phase of
In beginning a brief therapy with Mr. B, the most
treatment. Mr. B is offended by the therapist's lateness
important challenge for this psychiatrist is to:
a n d expresses his disappointment by characterizing
all doctors as_ being unavailable. T h e r e is a likely (A) prescribe an antidepressant.
transference to the patient's father, w h o w a s unavail- (B) prescribe an antianxiety agent.
able to the patient during his formative y e a r s . (C) contact the patient's wife for odci'-ional history.
(D) establish a therapeutic or working alliance.
Kay J: The essentials of psychodynamic psychotherapy. FOCUS (E) set clear limits on the patient's behavior.
2006; 4:167-172 (169)
The correct responsv is option D: Establish a thera-
peutic or working alliance

It is true in all types of psychotherapy that the initial


335 task of treatment is to establish a therapeutic or work-
On hearing the irritation in the patient's v o i c e the cli-
r ing alliance, without w h i c h treatment is unlikely to
nician begins to explain in detail the reasons for his progress. This alliance represents the willingness of
tardiness a n d a p o l o g i z e s profusely. He assures the patient a n d therapist to w o r k collaboratively t o w a r d
patient that he will not be late for future meetings. This
the patient's understanding and changing problematic
is an example of:
feelings a n d behaviors.
(A) denial.
It w o u l d be premature to prescribe a n y medication at
(B) regression.
(C) countertransference. this point in the treatment relationship because the
(D) deidealization. nature and extent of the patient's symptoms remain
(E) dissociation. • unclear. Since Mr. B is seeking treatment, it w o u l d be
inappropriate to bring another person—his w i f e , from
• T h e correct response is option C: Countertransference
w h o m he is separated—into the treatment process.
Setting limits on behavicr is clearly inappropriate and
w o u l d undoubtedly establish an adversarial relation-
ship with the patient.

Kay J: The essentials of psychodynamic psychotherapy. FOCUS


2006; 4:167-172 (170)

164 FOCUS Psychiatry Review- 400 Self-Assessment Questions


337 339
W h i c h of the following psychiatric disorders is con- A 46-year-old w o m a n presents to her primary care
sidered to be predominantly culture specific? physician with a 2-month history of l o w back pain,
dull headaches several times a w e e k , insomnia,
(A) Bulimia nervosa fatigue, and irritability. She has a l w a y s been healthy.
(B) Generalized anxiety disorder Findings from her physical examination are all within
(C) Major depressive disorder normal lirru'ts, a n d a review of systems is noncontribu-
(0) Posttraumatic stress disorder tory. Routine laboratory tests such as a chemistry
(E) Schizophrenia , panel, C B C , and thyroid function tests are all normal.
T h e correct response is option A: Bulimia nervosa T h e 'most likely diagnosis is:

Bulimia nervosa is considered to be a culture-specific (A) major depressive disorder.


syndrome, occurring predominantly in females in
(B) generalized anxiety disorder.
industrialized cultures that value slimness.
(C) pain disorder.
(D) hypochondriasis.
Keel PK, Klump KL Are eating disorders culture-bound syndromes? (E) somatization disorder.
Implications for conceptualizing their etiology. Psychol Bull 2003;
129:747-769 The correct response is option A: Major depressive
Sadock BJ, Sadock VA (eds): Kaplan and Sadock's Comprehensive disorder
Textbook of Psychiatry, 8th ed. Philadelphia, Lippincott Williams &
Wilkins, 2005, p 2007 In the primary care setting, major depressive disorder
American Psychatric Association: Practice Guideline for the most commonly presents with multiple somatic symptoms,
Treatment of Patients With Eating Disorders, 3rd ed. Am J with back pain among the most frequent complaints.
Psychiatry 2006; 163(Ju!y suppl):1-54
Fava M: Somatic symptoms, depression, and anti-depressant treat-
ment. J Clin Psychiatry 2002; 63:305-307

338 340
A 73-year-old man with moderate congestive hear!
A 29-year-old patient with borderline personality dis-
failure and degenerative arthritis in his right knee vis-
order is being seen in psychotherapy twice weekly.
its his physician for a scheduled outpatient appoint-
The psychiatrist realizes that the patient is uncon-
ment. Although his physical examination findings
sciously trying to coerce her into acting in a judg-
from the previous visit are unchanged, the physician
mental w a y . This phenomenon is best described as:
notes that the patient appears tired and less interac-
tive than usual. Concerned that the patient m a y be (A) identification wfth the aggressor. "* •
experiencing a major depressive episode, the physi- (B) projection.
cian wishes to gather more information. T h e presence (C) projective identification.
of which of the following w o u l d be most helpful in (D) regression.
making a diagnosis of major depressive disorder? (E) splitting.
(A) Complaints of pain The correct response is option C: Projective identification
(B) Decreased concentration
(C) Loss of appetite Otto Kernberg described the defense mechanism of
(D) Poor energy projective identification as it occurs in patients with
(E) The wish to die borderline personality disorder. In this primitive
defense mechanism, intolerable aspects of the self are
The correct response is option E: T h e wish to die
projected onto another with the aim of inducing the
W h i l e anergia, anorexia, somatic complaints, and person to play the projected role, a n d the two act in
diminished concentration commonly accompany med- unison. It is important that therapists be a w a r e of the
ical illnesses in older patients, psychological symptoms, process and act neutrally toward such patients.
including suicidal ideation, decreased self-esteem, and Sadock BJ, Sadock VA: Kaplan and Sadock's Synopsis of Psychiatr/,
guilt, do not. These symptoms should suggest the diag- 9th ed. Philadelphia, Lippincott Williams & Wilkins, 2003, p 809
nosis of depression. Gabbard GO: Psychodynamic approaches to personality disorders.
FOCUS 2005; 3:363-367
Jacobson SA, Pies RW, Greenblart DJ: Handbook of Geriatric Stem TA, Fricchione GL, Cassem NH, Jeliinek MS, Rosenbaum JF
Psychopharmacology. Washington, DC, American Psychiatric (eds): Massachusetts General Hospital Handbook of General
Publishing, 2002, pp 112-113 Hospital Psychiatry, 5th ed. St Louis, Mosby, 2004, pp 642-643

Section 2: Answers and Explanations


Hales RE, Yudofsky SC (eds): The American Psychiatric Publishing
341 Textbook of Clinical Psychiatry, 4th ed. Washington, DC, American
W h i c h of the following is N O T FDA-approved for the Psychiatric Publishing, 2003, p 1586
treatment of acute mania? Simon Rl: The law and psychiatry. FOCUS 2003; 1:349-372 (p 350)

(A) Carbamazepine
(B) Gabapentin
(CI Divalproex 343
(D) Olanzapine
In a psychotherapy session, a patient reveals that he
(E) Risperidone
has been having trouble obtaining an orgasm with his
T h e correct response is option B: Gabapentin partner. He states that he has a l w a y s felt aroused
w h e n traveling to w o r k on a c r o w d e d bus, a n d he
G a b a p e n t i n has not been a p p r o v e d by the FDA for
used to think that this enhanced his sexual life. He
treating a n y aspect of bipolar disorder. Lithium, chlor- never thought it w a s a problem, but n o w he thinks it
p r o m a z i n e , and divalproex w e r e the first agents is interfering with his relationship. W h a t is the most
a p p r o v e d by the F D A for the treatment of acute likely diagnosis?
m a n i a . Since then, five newer antipsychotics—olan-
zapine, risperidone, quetiapine, ziprasidone, and
(A) Exhibitionism
(B) Fetishism
a r i p i p r a z o l e — h a v e been a p p r o v e d for acute mania.
(C) Frotteurism
T h e extended-release formulation of carbamazepine,
(D) Pedophilia
an anticonvulsant, has also been approved for acute
(E) Voyeurism
mania.
The correct response is option C: Frotteurism
Tohen M, Baker RW, Altshuler Li, Zarate CA, Suppes T, Ketter TA,
Milton DR, Risser R, Gilmore JA, Breier A Tollefson GA This scenario best describes the disorder of frotteurism
• Olanzapine versus divalproex in the treatment ot acute mania. Am according to DSM-IV-TR c r i t e r i a - s e x u a l arousal
J Psychiatry 2002;159:1011-1017
caused by rubbing up against a nonconsenting per-
Ketter TA: Treatment of acute mania in bipolar disorder, in Advances
in Treatment of Bipolar Disorder (Review of Psychiatry, vol 24). son.
Washington, DC, American Psychiatric Publishing, 2005
American Psychiatric Association: Diagnostic and Statistical Manual
of Mental Disorders, Fourth Edition, Text Revision (DSM-IV-TR).
Washington, DC, American Psychiatric Association, 2000, p 570
Hales RE, Yudofsky SC (eds): The American Psychiatric Publishing
,342 Textbook of Clinical Psychiatry, 4th ed. Washington, DC, America-
Psychiatric Publishing, 2003, p 758
ri'.fjerm "four D's of negligence"—duty, dereliction,
direct, a n d damages—refers to:

(A) the questions a defendant physician will be asked at depo-


sition. 344
(B) what a patient/plaintiff must prove to win a malpractice W h i c h of the following comparisons regarding the
suit. incidence and prevalence of posttraumatic stress dis-
(C) the calculation of punitive versus compensatory domoges. order (PTSD) is the most accurate?
(D) the level of core that would be expected of a reasonable
physician under similar circumstances. (A) Hie condition is more prevalent in men.
(B) Hie presence of a psychiatric disorder does not predispose a
The correct response is option B: What a
person to PTSD.
patient/plaintiff must prove to w i n a malpractice suit
(C) Older individuals have a higher prevalence than younger
T h e four D's of negligence refers to: duty, dereliction, individuals.
direct, a n d d a m a g e s . T h e physician o w e s a duty to
(D) Certain types of trauma are more likely to cause PTSD.
the patient. W h e n negligence occurs, there is a dere- T h e correct response is option D: Certain types of
liction of this duty, w h i c h directly results in damage to trauma are more likely jo cause PTSD
the patient. To w i n a malpractice suit, a patient/plain-
Extreme stressors (such as rape, torture, a n d combat)
tiff must p r o v e by a preponderance of the* evidence
significantly increase morbidity of P T S D . W h e n the
that the physician o w e d . a duty of care to the patient
type of trauma is controlled for, w o m e n a p p e a r to be
and that negligence (a dereliction of this duty)
at higher risk of developing P T S D c o m p a r e d with
occurred, w h i c h directly resulted in damage to the
men. In o n e nationwide survey, the highest current
patient. O p t i o n D refers to the standard of c a r i ,
(17.8%) a n d lifetime (38.5%) rates of PTSD w e r e in

P O P T ! ^ Pcvrkorrv Pi-virw 400 ^j-lf-Awwmrnr Onrcrinnc


w o m e n w h o h a d been exposed to physical assault or 346
rape. PTSD is more common in y o u n g e r than in older
A 33-year-old w o m a n with a diagnosis of borderline
individuals, p r o b a b l y because of the higher incidence
personality disorder w a s recently discharged from
of physical violence a n d accidents in the y o u n g e r
medical service after an aspirin overdose. She des-
population. Individuals w h o respond to the initial
cribes having h a d thoughts of suicide off and on since
trauma with high levels of anxiety (e.g., a panic early adolescence a n d has m a d e t w o previous suicide
attack), also h a v e a higher risk of developing PTSD attempts, lo addressing her suicidality in treatment,
after trauma, as are those w h o perceive an external which of the following approaches w o u l d be most
(vs. internal) locus of control. J appropriate?

Hales RE, Yudofsky SC (eds): The American Psychiatric Publishing (A) Partial hospitalization or brief inpatient hospitalization
Textbook of Clinical Psychiatry, 4th ed. Washington, DC, American (B) Outpatient psychoanalysis
Psychiatric Publishing, 2003, p 578 (C) 'Gabapentin pharmacotherapy
Breslau N, Kessler RC, Chilcoat HD, Schultz LR, Davis GC, Andreski P:
(D) Valproic acid pharmacotherapy
Trauma and posttraumatic stress disorder in the community: the
1996 Detroit Area Survey of Trauma Arch Gen Psychiatry 1998; The correct response is option A: Partial hospitaliza-
55:626-632 tion or brief inpatient hospitalization
Breslau N:The epidemiology of posttraumatic stress disorder what
is the extent of the problem? J Clin Psychiatry 2001; 62(suppi Of the options listed, long-term partial hospitalization
17):16-22
has the most empirical support. Studies of mood sta-
Resnick HS, Kiipatrick DG, Dansky BS, Saunders BE, Best CL:
bilizers have been m i x e d . Although not a listed
Prevalence of civilian trauma and posttraumatic stress disorder in
a representative national sample of women. J Consult Clin option, dialectical behavior therapy also has substan-
Psychol 1993; 61:984-991 tial empirical support for the treatment of borderline
Galea S, Ahem J, Resnick H, Kiipatrick D, Bucuvalas M, Gold J, personality disorder.
Vlahov D: Psychological sequelae of the September 11 terrorist
attacks in New York City. N Engl J Med 2002; 346:982-987 Bateman A, Fonagy P: Effectiveness of partial hospitalization in the
treatment of borderline personality disorder: a randomized con-
trolled trial. Am J Psychiatry 1999; 156:1563-1569
Bateman A, Fonagy P: Treatment of borderline personality disorder
with psychoanalytjcally oriented partial hospitalization: an 18-
345 month follow-up. Am J Psychiatry 2001; 158:36-42
W h i c h of the following statements is correct about the Ueb K, Zanarini MC, Schmahl C, Linehan MM, Bohus M: Borderline
personality disorder. Lancet 2004; 364:453-461
concordance of schizophrenia in the twin of an indi-
Practice Guideline for the Treatment of Patients With Borderline
vidual with schizophrenia?
Personality Disorder (2001), in American Psychiatric Association
(A) 50% if twin a monozygotic Practice Guidelines for the Treatment of Psychiatric Disorders,
Compendium 2004. Washington, DC, APA, 2004, pp 757-758
(B) 75% if twin is monozygotic
(C) Almost 100% if twin is monozygotic
(D) 50% if twin is dizygotic
(E) 75% if twin is dizygotic i
T h e correct response is option A: 50% if twin is
monozygotic

Between 50% and 60% of monozygotic twin pairs


are concordant for schizophrenia. In several studies
over recent decades, the concordance in dizygotic
twins has ranged from 4% to 15%.

Hales RE, Yudofsky SC (eds): The American Psychiatric Publishing


Textbook of Clinical Psychiatry, 4th ed. Washington, DC, American
Psychiatric Publishing, 2003, pp 18-19
Sadock BJ, Sadock VA (eds): Kaplan and Sadock's Synopsis of
Psychiatry: Behavioral Sciences/Clinical Psychiatry, 9th ed.
Philadelphia, Lippincott Williams & Wilkins, 2003, p 465

Section 2: Answers and Explanations


347 349
In addition to a stimulant trial for attention deficit hyper- A 29-year-old w o m a n is admitted to the hospital with
activity disorder symptoms, the parents of an 8-year-old acute herpes simplex encephalitis. W h i c h of the fol-
b o y ask what other treatment would be most helpful for lowing is the most common residual deficit upon
managing his refusal to cooperate at home. W h i c h of recovery?
the following is the best recommendation?
(A) Apraxia
(A) Biofeedback (B) Aphasia
(B) Behavior therapy (C) Amnesia
(C) Cognitive behavior therapy (D) Ataxia
(D) Family therapy (E) Dysarthria
(E) Psychodynamic psychotherapy The correct response is option C: Amnesia
T h e correct response is option B: Behavior therapy
Acute herpes simplex encephalitis d a m a g e s the
Behavior therapy has been documented to be helpful medial, temporal, and orbitofrontal regions of the cor-
as a component of the treatment of A D H D , especially tex. Amnesia is the. most common residual deficit. The
parent training a n d classroom behavioral modifica- regions of the brain for language, speech, and dis-
tion a p p r o a c h e s . crimination of touch are usually not affected.

Sadock BJ, Sadock VA (eds): Kaplan and Sadock's Comprehensive Wise MG, Rundell JR (eds): The American Psychiatric Publishing
Textbook of Psychiatry, 8th ed. Philadelphia, Lippincott Williams & Textbook of Consultation-Liaison Psychiatry: Psychiatry in the
Wilkins, 2005, pp 2545,3195 Medically III, 2nd ed. Washington, DC, American Psychiatric
Publishing, 2002, pp 86,688

348
350
A 15-year-old girl is brought in for an emergency eval-
uation because she has been out all night and refuses W h i c h of the following is most effective for the psy-
to tell her parents w h e r e she has been. T h f . oarents chotherapeutic treatment of obsessive-compulsive dis-
report that for several months the girl has been irritable order?
and oppositional with severe mood swings. She has
been leaving home and school without permission. The
(A) Biofeedback
girl admits that she has been sV'-newhat moody but
(B) Exposure and response prevention
insists that her parents are making "u vg deal about
(C) Psychodynamic psychotherapy
nothing. A preliminary diagnosis of bipolar disorder is
(D) Relaxation and visualization
made. W h i c h of the following is the most common
(E) Interpersonal therapy
comorbid condition with bipolar disorder? T h e correct response is option B: Exposure and
response prevention
(A) Conduct disorder
(B) Generalized anxiety disorder Exposure a n d response prevention is most effective for
(C) Oppositional defiant disorder the psychotherapeutic treatment of obsessive-compul-
(D) Posttraumatic stress disorder sive disorder. Relaxation techniques alone a r e not
(E) Substance use disorder
helpful a n d are often used as the control in research
T h e correct response is option E: Substance use disor- on obsessivfrcompulsive disorder.
der
Practice parameters for the assessment and treatment of children
Substance use or abuse is an important diagnosis to and adolescents with obsessive-compulsive disorder. J Am Acad
Child Adolesc Psychiatry 1998; 37(suppl 10):27S-45S
consider in adolescents w h o present with symptoms
Jenike MA: Clinical practice: obsessive-compulsive disorder. N Engl J
consistent with bipolar disorder, both as a possible
Med 2004; 350:259-265 _
cause of the symptoms and as an important potential American Psychiatric Association: Practice Guideline for the
coexisting problem. This diagnosis has significant Treatment of Patients With Obsessive-Compulsive Disorder. Am J
implications for treatment planning. 4
Psychiatry, expected 2007

Sadock BJ, Sadock VA (eds): Kaplan and Sadock's Comprehensive


• Textbook of Psychiatry, 8th ed. Philadelphia, Lippincott Williams &
Wilkins, 2005, p 3277
Lewis M (ed): Child and Adolescent Psychiatry: A Comprehensive
TextDOOk, 3rd ed. Philadelphia, Lippincott Williams & Wilkins,
2002, pp 783-786
351 353
Rebound insomnia is most severe after abrupt with- In clinical or forensic evaluations w h e n financial com-
d r a w a l of w h i c h of the following medications? pensation or special benefits may be available, a psy-
chiatrist must consider the diagnosis of:
(A) Alprazolam
(B) Clonazepam ; . (A) factitious disorder.
'(C) Diazepam (B) malingering.
(D) Chlordiazepoxide (C) somatization.
(E) Quazepam x (D) hypochondriasis.
T h e correct response is option A: A l p r a z o l a m The correct response is option B: Malingering

Abrupt withdrawal of any benzodiazepine will cause Malingering, the conscious attempt to fake or e x a g -
some degree of rebound anxiety and insomnia. Short- gerate an illness or symptom for personal g a i n , is the
acting compounds have been found to have a greater correct answer. Factitious disorder is a self-induced
effect on rebound insomnia on discontinuation. The medical problem w h e r e the personal goal is not evi-
elimination half-life of alprazolam is intermediate (6 to dent. Somatization disorder is a polysympfomatic dis-
20 hours) and is the shortest in comparison to clo- order that begins before age 30, extends over a
nazepam, long (>20 hours); diazepam, long period of years, and is characterized by a combina-
(>20 hours); q u a z e p a m , long (>20 hours); and chlor- tion of pain and gastrointestinal, sexual, and
diazepoxide, intermediate (6 to 20 hours) but with long pseudoneurological symptoms. Hypochondriasis is
(>20 hours) metabolites (demoxepam and nor- the fear of having a serious disease based on misin-
dazepam). terpretation of somatic signs or symptoms.

Hales RE, Yudofsky SC (eds): The American Psychiatric Publishing American Psychiatric Association: Diagnostic and Statistical Manual
Textbook of Clinical Psychiatry, 4th ed. Washington, DC, American of Mental Disorders, Fourth Edition, Text Revision (DSM-IV-TR).
Psychiatric Publishing, 2003, pp 1073-1075 Washington, DC, American Psychiatric Association, 2000,
Sadock BJ, Sadock VA: Kaplan and Sadock's Synopsis of Psychiatry, pp 485-489,739, 781-783
9th ed. Philadelphia, Lippincott Williams & Wilkins, 2003, p 1025.

354
352
Avoidance symptoms in posttraumatic stress disorder
All of the following ere symptom clusters of posttrau- (PTSD) include which of the following?
matic stress disorder (PTSD) EXCEPT:
(A) Hypervigilance
(A) reexperiencing. (B) Intrusive images of the event
(B) avoidance/numbing. (C) Sense of reliving the event or experience
(C) hyperarousal. (D) Difficulty recalling important aspects of the event
(D) derealization/depersonalization.
The correct response is option D: Difficulty recalling
T h e correct response is option D: D e r e a l i z a t i o n / important aspects of the event
depersonalization
Difficulty recalling is a form of avoidance. In the D S M -
Derealization and depersonalization are listed as cri- IV-TR, posttraumatic stress disorder symptoms are clus-
teria for acute stress disorder in DSM-IV-TR. tered into three categories: reexperiencing,
avoidance and numbing, and hyperarousal. O p t i o n
American Psychiatric Association: Diagnostic and Statistical Manual
of Mental Disorders, Fourth Edition, Text Revision (DSM-IV-TR). A is a symptom of hyperarousal, while options B a n d
Washington, DC, American Psychiatric Association, 2000, pp 468, C are symptoms of reexperiencing the event.
471
American Psychiatric Association: Practice Guideline for the Shalev AY: What is posttraumatic stress disorder? J Clin Psychiatry
Treatment of Patients With Acute Stress Disorder and 2001;62(suppl17):4-10
Posttraumatic Stress Disorder. Am J Psychiatry 2004; 161 (Nov American Psychiatric Association: Diagnostic and Statistical Manual
suppl):9 of Mental Disorders, Fourth Edition, Text Revision (DSM-IV-TR).
Washington, DC, American Psychiatric Association, 2000, p 468
Davidson JRT: Effective management strategies for posttraumatic
stress disorder. FOCUS 2003; 1:239-243

Section 2: Answers and Explanations 169


355 . 357
W e i g h t g a i n is LEAST likely to be a side effect of Trichotillomania is a difficult symptom to treat with
w h i c h of the following? either psychotherapy or medication. Emerging evi-
dence indicates that medication plus which of the fol-
(A) Lithium lowing types of psychotherapy is effective?
(B) Lamotrigine
(C) Divalproex (A) Exposure
(D) Olanzapine (B) flooding
(C) Hobil reversal
T h e correct response is option B: Lamotrigine (D) Interpersonal psychotherapy
W h e n lamotrigine w a s compared with valproate in a
(E) Psychodynamic psychotherapy
monotherapy study on epilepsy, weight remained sta- The correct response is option C: Habit reversal
ble among patients using the former but not the latter
medication (at 32 weeks, mean weight gain w a s Probably the best described and most effective psy-

12.8 pounds on valproate and 1.3 pounds on lamo- chotherapeutic technique for the trectment of trichotil-

trigine). W e i g h t g a i n is a well-established side effect lomania is habit reversal. The technique has been

of lithium, divalproex, and olanzapine. adapted to both individual a n d group therapies.


Habit reversal includes 13 components, including
Biton V, Mirza W, Montouris G, Vuong A, Hammer AE, Barrett PS: such things as self-monitoring, relaxation training,
Weight change associated with valproate and lamotrigine
habit interruption, overcorrection, and habit incon-
monotherapy in patients with epilepsy. Neurology, 2001;
56:172-177 _ 1 venience.
Russell JM, Mackell JA: Body weight gain associated with atypical
Hales RE, Yudofsky SC (eds): The American Psychiatric Publishing
antipsychotics: epidemiology and therapeutic implications. CNS
Textbook of Clinical Psychiatry, 4th ed. Washington, DC, American
Drugs 2001;15:537-551
Psychiatric Publishing, 2003, p 796
Mclntyre RS, Konarski JZ: Obesity and psychiatric disorders: fre-
Hautmann G, Hercogova J, Lotti T: Trichotillomania. J Am Acad
quently encountered clinical questions. FOCUS 2005; 3:511-519
Dermatol 2002; 46:807-821
(p516)

356
Trie following vignette applies to questions 358 and
In Erikson's epigenetic model, each \\h stage has an 359.
identity crisis that must be navigated. In.imncy vs. iso-
lation is the developmental crisis associated with: A 19-year-old w o m a n presents to a clinic for treat-
ment of c h a p p e d hands. She reports that for several
(A) school age. months she has h a d "this notion in my h e a d " that
(B) adolescence. there are germs e v e r y w h e r e . At first she w a s h e d her
(C) young adulthood. hands more frequently, but as the thoughts h a v e
(D) adulthood. become more prominent, she n o w usually w e a r s
(E) old oge. gloves and w a s h e s her hands with diluted bleach
several times a day. She says that if she does not
T h e correct response is option C: Young adulthood
complete her cleansing rituals, she cannot stand the
A c c o r d i n g to Erikson, the primary task at the life stage anxiety.

of y o u n g adulthood, between ages 20 and 40, is to


form strong friendships and to achieve a sense of love
a n d companionship or a shared identity with another
person. Feelings of loneliness or isolation are likely to
result from an inability to form friendships or an inti-
mate relationship. 1
\

Erikson EH: Trie Life Cycle Completed. New York, WW Norton, 1998,
pp 56-57 .'
Shaffer DR: Developmental Psychology: Childhood and Adolescence,
5th ed. Pacific Grove, Calif, Brooks/Cole, 1999, pp 45-47
' Weiner JM, Dulcan MK: Textbook of Child and Adolescent Psychiatry,
3rd ed. Washington, DC, American Psychiatric Publishing, 2004, p 36
358 360
T h e most common c o m o r b i d condition with this disor- In which of the following disorders has reduced vol-
der is: ume been observed in the prefrontal cortex?

(A) alcohol abuse. (A) ADHD


(B) generalized anxiety disorder; (B) Delusional disorder
(C) major depressive disorder. (C) Obsessive-compulsive disorder
(D) social phobia. (D) Panic disorder
(E) schizophrenia. , (E) Schizophrenia
The correct response is option C: Major depressive The'correct response is option E: Schizophrenia
disorder
Studies have demonstrated that patients diagnosed
Two-thirds of patients w h o h a v e obsessive-compul- with schizophrenia have decreased prefrontal g r a y
sive disorder will sometime in their life have an matter, decreased prefrontal white matter, and
episode of major depression, with about one-third increased ventricle size. The cognitive deficits persist
meeting the criteria for current comorbid depression, in patients w h o are not actively psychotic or experi-
making it the most c o m m o n c o m o r b i d disorder. All of encing negative symptoms.
the other anxiety disorders m a y be comorbid as
Hulshoff Pol HE, Schnack HG, Bertens MG, van Haren NE, van derTweel
well. Finally, obsessive-compulsive disorder can be
I, Staal WG, Baare WF, Kahn RS: Volume changes in gray matter in
comorbid with s c h i z o p h r e n i a , often making the treat- patients with schizophrenia Am J Psychiatry 2002; 159:244-250
ment more difficult.

Kaplan HI, Sadock BJ: Synopsis ot Psychiatry. 8th ed. Baltimore,


361
Williams & Wilkins, 1998, pp 610-611
Stein 0J, Hollander E (eds): American Psychiatric Publishing Textbook The first step in the evaluation of a patient with male
of Anxiety Disorders. Washington, DC, American Psychiatric erectile disorder is to:
Publishing, 2002, pp 183-186
(A) take a genetic history.
(B) rule out medical problems and substance use.
(C) refer the patient to a sex therapist.
359 (D) challenge with a test dose of a PDE-5 inhibitor.
(E) order a sleep study.
T h e structural brain abnormality that has been demon-
strated most consistently in this disorder is: T h e correct response is option B: Rule out medical
problems a n d substance use
(A) asymmetrical septal nuclei.
(B) decreased size of the caudate. Medical problems and substance use must be considered
(C) enlarged lateral ventricles. and carefully assessed during the evaluation of male erec-
(D) hypertrophy of the amygdala. tile disorder.^ From twin studies, the heritability of the risk
(E) shrinkage of the hippocampus. of dysfunction in having an erection is estimated to be
35%, and of maintaining an erection, 42%. The efficacy
T h e correct response is option B: Decreased size of
the caudate of the three available PDE-5 inhibitors (sildenafil, varde-
nafil, and tadalafil) is approximately 70%, and the three
This patient is suffering from obsessive-compulsive dis- have similar side effect profiles. Caution should be exer-
order. Functional brain imaging (e.g., positron emis- cised with PDE-5 inhibitors in patients with hypotension
sion tomography) has demonstrated increcsed and uncontrolled hypertension. The prognosis of male
metabolism and blood flow in the basal ganglia, erectile disorder has improved over the years by several
especially the caudate. Interestingly, structural studies groundbreaking treatments: sex therapy, penile prosthe-
(e.g., computed tomography a n d magnetic resonance sis, intracavernosal injection, and oral PDE-5 inhibitors.
imaging] have found bilaterally smaller caudates in
Fischer ME, Vitek ME, Hedeker D, Henderson WG, Jacobsen SJ,
patients with obsessive-compulsive disorder.
Goldberg J: A twin study of erectile dysfunction. Arch Intern Med
Kaplan HI, Sadock BJ: Synopsis of Psychiatry. 8th ed. Baltimore, 2004;164:165-168
Williams & Wilkins, 1998, p 610 Gresser II, Gleiter CH: Erectile dysfunction: comparison of efficacy
and side effects of the PDE-5 inhibitors sildenafil, vardenafil, and
tadalafil. Eur J Med Res 2002; 7:435-446
Farre JM, Fora F, Lasheras MG: Specific aspects of erectile dysfunc-
tion in psychiatry. Int J Impot Res 2004; 16:546-549

Section 2: Answers and Explanations


Hales RE, Yudofsky SC (eds): The American Psychiatric Publishing
362 Textbook of Clinical Psychiatry, 4th ed. Washington, DC, American
A 75-year-old w o m a n with Parkinson's disease devel- Psychiatric Publishing, 2003, p 1266
ops vivid dreams a n d night terrors. T h e most likely Sadock BJ, Sadock VA (eds): Kaplan and Sadock's Comprehensive
Textbook of Psychiatry, 8th ed. Philadelphia, Lippincott Williams &
explanation for these symptoms is:
Wilkins, 2005, pp 2621-2624
(A) the onset of dementia.
(B) a rapid progression of Parkinson's disease.
(C) a normal effect of aging.
(D) an anxiety disorder. 364
(E) side effects from corbidopa-levodopo. Heightened arousal in posttraumatic stress disorder
T h e correct response is option E: Side effects from (PTSD) is associated with an increase in which of the
carbidopa-levodopa following?

T h e encephalopathic side effects of carbidopa-le- (A) Heart rate


v o d o p a are essentially those of the levodopa. Nearly (B) Constridion of pupils
30% of patients with Parkinson's disease taking i-dopa
(C) Weight
have vivid dreams and 7% have night terrors. Delirium
(D) Tidal volume
occurs in 5%, and a delusional syndrome develops in The correct response is option A: Heart rate
as many as 3% of patients w h o take L-dopa for 2 or
T h e arousal in PTSD is largely due to an increased
more years.
autonomic response. Therefore, increased heart rate
Brown TM, Stoudemire GA Psychiatric Side Effects' of Prescription would be a natural occurrence. Constricted pupils, on
and Over-the-Counter Medications: Recognition and Management
the other h a n d , as well as weight gain a n d an
Washington, DC, American Psychiatric Press, 1998, pp 37-38
increase in tidal volume w o u l d more likely be associ-
Krahn LE, Richardson JW: Sleep disorders, in The American
Psychiatric Publishing Textbook of Psychosomatic Medicine. ated with parasympathetic stimulation. Other physio-
Edited by Levenson JL. Washington, DC, American Psychiatric logical findings associated with arousal in PTSD
Publishing, 2005, Table 16-10, pp 350-352 include muscle tension as measured by electromyog-
raphy and increased sweating.

American Psychiatric Association: Diagnostic and Statistical Manual


363 of Mental Disorders, Fourth Edition, Text Revision (DSM-IV-TR).
Washington, DC, American Psychiatric Association, 2000, p 465
In which of the following therapies, w h i c h has been Hollander E, Simeon D: Anxiety disorders; in The American
studied for the treatment of patients with borderline Psychiatric Publishing Textbook of Clinical Psychiatry, Fourth
personality disorder, is mindfulness training a central Edition. Edited by Hales RE, Yudofsky SC. Washington, DC,
component? American Psychiatric Publishing, 2004, p 600

(A) Cognitive behavior therapy


(B) Dynamic psychotherapy
(C) Dialectical behavior therapy
(D) Short-term group psychotherapy
(E) Interpersonal psychotherapy
T h e correct response is option C: Dialectical behavior
therapy

Mindfulness is considered a core skill in dialectical


behavior therapy, along with tolerance, emotion reg-
ulation, a n d interpersonal effectiveness. Dialeciical
behavior therapy is well studied a n d frequently cited
as an effective a p p r o a c h to the treatment of patients
with borderline personality disorder. Mindfulness
training addresses attentional control, described by
Linehan as being "in control of attentional processes."

• Tasman A, Kay J, Lieberman JA (eds): Psychiatry. Philadelphia, WB


Saunders, 1997, p 357

122 VnClK Vrvrhhrr.- R-IRW: 400 SDF-AASFTTMRNR fWrinnc


365 An elderly patient w h o develops acute mental status
changes while in the hospital for another problem (this
According to DSM-IV-TR, a mixed episode must meet
patienj is on the surgical service) is most likely suffer-
diagnostic criteria for a manic episode a n d w h i c h of
1
ing from delirium, w h i c h is a medical problem (thus
the following ?
options A, B, a n d D will likely turn out not to be the
(A) Panic attacks eventual intervention). Indeed, elderly postsurgical
(B) Rapid cycling patients pre especially at high risk of delirium.
(C) Brief psychotic episode • Although b e n z o d i a z e p i n e s can be useful in the man-
(D) Major depressive episode « agement of agitated delirium, the mainstay of delirium

The correct response is option D: M a j o r depressive management is dopamine blockade with an agent
episode such a s ' h a l o p e r i d o l . Of course, the most important
intervention is to identify and treat the cause of the
M i x e d episodes contain features of both mania and confusional state.
depression. W h i l e mixed episodes have been defined
by a number of different criteria, DSM-IV-TR requires Trzepacz PT: Delirium (confusional states), in The American
at least a w e e k during w h i c h criteria are met for both Psychiatric Publishing Textbook of Consultation-Liaison
Psychiatry: Psychiatry in the Medically III, 2nd ed. Edited by Wise
a manic episode a n d a major depressive episode.
MG, Rundell JR. Washington, DC, American Psychiatric Publishing,
McElroy SL, Keck PE Jr. Pope HG Jr, Hudson Jl, Faedda GL, Swann 2002, pp 266-268
AC: Clinical and research implications of the diagnosis of dys- Schouten R: Legal aspects of consultation, in Massachusetts General
phoric or mixed mania or hypomania. Am J Psychiatry 1992; Hospital Handbook of General Hospital Psychiatry, 5th ed. Edited
149:1633-1644 by Stem TA, Fricchione GL, Cassem NH, Jellinek MS, Rosenbaum
American Psychiatric Association: Diagnostic and Statistical Manual JF.St Louis, Mosby, 2004, pp 451-452
of Mental Disorders, Fourth Edition, Text Revision (DSM-IV-TR).
Washington, DC, American Psychiatric Association, 2000,
pp 362-363
American Psychiatric Association: Practice Guideline for the 367
Treatment of Patents With Bipolar Disordsr (Revision). Am J
Psychiatry 2002; 159(April suppl). Reprinted in FOCUS 2003; • W h i c h of the following variables is most important to
1:64-110 (p 81) take into account w h e n evaluating the score on a
Mini-Mental State E x a m (MMSE)?

(A) Educational level


(B) Gender
366
(C) History of alcohol use -,
A psychiatrist is called to see a 78-year-old female (D) Medical history
patient postoperatively on the surgical service w h o is (E) Past psychiatric history
said to be "manic." She is hardly sleeping, she is agi-
The correct response is option A: Educational level
tated and talking rapidly, a n d she believes she needs
to talk with the President of the United States. W h i c h of The formerfy used cutoff score of 23 to identify cog-
the following interventions is most likely to be effective?
nitive impairment has been s h o w n to h a v e poor sen-
(A) Transfer to a psychiatric unit sitivity for detecting cognitive impairment in
(B) Divalproex sodium better-educated cdults. At the same time, individuals
(C) Haloperidol with lower educational attainment will be overidenfi-
(D) ECT fied as cognitively impaired w h e n this cutoff is u s e d .
(E) A benzodiazepine A g e - a n d education-based norms have since b e e n
developed for the M M S E .
T h e correct response is option C: Haloperidol
Spar JE, La Rue A: Concise Guide to Geriatric Psychiatry, 3rd ed.
Washington, DC, American Psychiatric Publishing, 2002,
pp 158-159

Section 2: Answers md Explanations


173
The correct response is option E: Rett's disorder
368
T h e parents of a 5-year-old b o y bring their child to a In this vignette, development is normal until the age of
clinic with the complaint that he frequently awakens 8 months and then goes a w r y , with specific delays and
during the early part of the night screaming; he looks deviance in social, communicative, and cognitive
terrified, his pupils are dilated, a n d he hyperventi- development. Rett's disorder, which occurs only in
lates. He is also sweating, agitated, a n d confused, females, is characterized by an early onset of devel-
a n d he cannot be comforted. W h e n fully a w a k e n e d , opmental delays, with deceleration of head g r o w t h ,
the child has no recall of the event. This presentation
loss of purposeful hand movements and stereotypies,
is most consistent with:
and incoordination of gait and trunk movements. The
(A) narcolepsy. early age at onset suggests the presence of a pervasive
(B) nightmare disorder. developmental disorder rather than schizophrenia,
(C) primary insomnia. which typically does not present until closer to or dur-
(D) sleep disordered breathing. ing adolescence. In autistic disorder, qualitative impair-
(E) sleep terror disorder. ments are seen in social interaction and communication,
along with repetitive and stereotyped patterns of behav-
T h e correct response is option E: Sleep terror disorder
ior, interest, and activities, but head growth does not
T h e presentation is most consistent with sleep terror decelerate or stagnate, nor is there deterioration in
disorder. T h e most pertinent parts of this vignette are neurological functioning. The same is true for
a y o u n g child awakening during the early part of Asperger's syndrome, which presents with many of the
sleep in a state of heightened arousal,^ along with the signs and symptoms of autistic disorder but without
lack of recall of the event. This w o u l d suggest an inci- impairments in language or cognitive development.
dent most likely occurring in stages 3 - 4 of sleep, as Children with disintegrative disorder develop an autis-
this is the most prominent pattern in the early hours. tic-like condition after a longer period (2 or more years)
' T h e r e f o r e , consideration would be g i v e n to some type of unequivocally normal development.
of parasomnia associated with d e e p sleep. The lack
of recall for w h a t is happening during the episode Lewis M (ed): Child and Adolescent Psychiatry: A Comprehensive
Textbook, 3rd ed. Philadelphia, Lippincott Williams & Wilkins,
suggests that this is not a nightmare. T h e symptoms
2002, pp 587-597
a r e •lot consistent with the presentation of narcolepsy
or a breathing disorder, such as a p n e a .

Dulcan MK, Martini DR, Lake MB: Concise Guide to Child and
Adolescent Psychiatry, 3rd ed. Washington, DC, American 370
Psychiatric Publishing, 2003, pp 155-167
W h i c h of the following actions on the part of a psy-
chiatrist constitutes abandonment?

(A) Failing to show up for a scheduled appointment with a


369 patient
(B) Referring, with appropriate notification to the patient, an
A 6-year-old girl is brought to a clinic because of
extremely difficult patient to a colleague with more experi-
unusual stereotyped hand w a s h i n g . Pregnancy, labor,
ence in the treatment of the patient's disorder
a n d delivery w e r e unremarkable, as w e r e develop-
(C) Terminating the treating relationship when a patient threat-
mental milestones until the a g e of 8 months, w h e n the
ens to sue the psychiatrist
child seemed to lose interest in her social environ-
ment. Thereafter, significant delays in development'
(D) Prematurely discharging o patient from the hospital
w e r e noted. She did not walk until 2 y e a r s of age and T h e correct response is option D: Prematurely dis-
has h a d no spoken language. H e a d g r o w t h has stag- charging a patient from the hospital
nated. Recently she has d e v e l o p e d breath-holding
spells. Examination reveals a small, non'communica-
tive child w h o demonstrates truncal ataxia and non-
purposeful h a n d movements. E E G is abnormal. This
presentation is most consistent w i t h :

(A) Asperger's syndrome..


(B) autism.
(C) childhood schizophrenia.
(D) mild mental retardation. 4

(E) Rett's disorder.


A c c o r d i n g to Simon (2001], a n y of the following 372
actions can be construed as abandonment of the
W h i c h of the following classes of medications is sup-
patient: failure to inform the patient about medication
ported by* well-designed studies as the first-line phar-
side effects, failure to admit the patient to the hospital
macologic treatment of posttraumatic stress disorder
w h e n indicated, not attending (or arranging appropri-
(PTSD)?
ate attending] to the patient during a hospitalization,
prematurely discharging the patient from the hospital, (A) Mood stabilizers
inappropriate or improper referral of the patient, sex- (B) Benzodiazepines
ual relations with a patient;, anU termination of the (C) Tricyclic antidepressants
treatment based only on denial of benefits by a third-
(D) Selective serotonin reuptake inhibitors (SSRIs)
party payer. A b a n d o n m e n t is a breach of the fiduciary T h e correct'response is option D: Selective serotonin
duty of the psychiatrist to act in the patient's best inter- reuptake inhibitors (SSRIs)
est. W h e n the therapeutic relationship between the
Selective serotonin reuptake inhibitors are recom-
psychiatrist and the patient is "unilaterally and prema-
'mended as first-line pharmacological treatment for
turely terminated by the psychiatrist without reason-
posttraumatic stress disorder. SSRIs have been found
able notice," abandonment may have occurred.
to be effective not only in reducing PTSD symptoms
Terminating the treating relationship w h e n a patient
but also in treatment of comorbid disorders and asso-
threatens to sue may be in the patient's best interest
ciated symptoms. Double-blind placebo-controlled
because of the countertransference that almost invari-
studies support the use of SSRIs as first-line agents for
ably would occur (it is also difficult to imagine that a
the treafme'nt of PTSD. Sertraline a n d paroxetine have
patient w h o has threatened to sue would w a n t to con-
been approved by the F D A for the treatment of PTSD.
tinue in treatment with the same psychiatrist).
Other SSRIs a r e currently being studied for efficacy.
Simon Rl: A Concise Guide to Psychiatry and Law for Clinicians, 3rd ed. Open-label trials have suggested that nefazodone
Washington, DC, American Psychiatric Publishing, 2001, p 32-33 may be useful for reducing PTSD symptoms. T h e
monoamine oxidase inhibitors a n d tricyclic antide-
pressants have been shown in a number of double-
blind placebo-controlled studies to be effective, but
371 they are considered second- or third-line agents
C o m m o n side effects of selective serotonin reuptake because of their side effect profiles. M o o d stabilizers
inhibitors include: should be considered, especially w h e n there is
accompanying impulsivity or aggressiveness,
(A) ortiiv'tatic hypotension and dry mouth.
although further studies are needed to determine the
(B) confusion 8fw disorientation.
(C) priapism and arrhythmia. effectiveness of these agents for patients with PTSD.
(D) seizures and hallucinations. Hageman I, Andersen HS, Jorgensen MB: Posttraumatic stress disor-
(E) nausea and sexual dysfunction. der: a review of jpsychobiology and pharmacotherapy. Acta
Psychiatr Scand 2001;104:411-422
T h e correct response is option E: nausea a n d sexual
Maddux RM, Rapaport MH: Anxiolytic drugs, in Psychiatry, 2nd ed.
dysfunction
Edited by Tasman A, Kay J, Lieberman JA. New York, Wiley &
Sons, 2003
C o m m o n side effects of SSRIs include anxiety, nausea,
Hollander E, Simeon D: Concise Guide to Anxiety Disorders.
insomnia, sedation, and sexual dysfunction. T r a z o d o n e
Washington, DC, American Psychiatric Publishing, 2003 p 180
can cause priapism. Seizures are associated with Davidson J, Pearistein T, Londborg P, Brady KT, Rothbaum B, Bell J,
bupropion. Confusion a n d disorientation may occur Maddock R, Hegel MT, Farfel G: Efficacy of sertraline in preventing
with toxic levels of antidepressants. Orthostatic relapse of posttraumatic stress disorder: results of a 28-week
double-blind, placebo-controlled study. Am J Psychiatry 2001;
hypotension and d r y mouth occur more commonly with
158:1974-1981. Reprinted in FOCUS 2003; 1:273-281
tricyclic antidepressants.
Brady K, Pearistein T, Asnis GM, Baker D, Rothbaum B, Sikes CR,
Farfel GM: Efficacy and safety of sertraline treatment of posttrau-
Sadock BJ, Sadock VA (eds): Kaplan and Sadock's Comprehensive
matic stress disorder: a randomized controlled trial. JAMA 20CG;
Textbook of Psychiatry, 7th ed. Philadelphia, Lippincott Williams &
283:1837-1844
Wilkins, 2000, p 1381

Section 2: Answers and Explanation 175


degree of difficulty the patient has is usually related to
373
the number of time zones crossed.
A patient in early recovery from opiate dependence
has been maintained on 40 m g / d a y of oral meth- A dissociative fugue is characterized by sudden travel
a d o n e for the last month. W h i l e the patient has not a w a y from one's home with an inability to recall some
been experiencing any withdrawal symptoms at that or all of one's past. A dyssomnia is characterized by
dose, the weekly random urine drug tests begin show- a disturbance in the amount, quality, or timing of
ing a resumption of heroin use. Pharmacologically, the sleep. A parasomnia is characterized by abnormal
best change to make in medication would be to: behavioral or physiological events occurring in asso-

(A) increase the maintenance dose of methadone. ciation with sleep or the components of sleep.

(B) decrease the maintenance dose of methadone. Narcolepsy is one type of dyssomnia. It involves
(C) change the opiate agonist to levo-alpha-acetylmethadol repeated attacks of refreshing sleep, cataplexy, and
(LAAM). recurrent attacks of R E M sleep in the form of hypna-
(D) augment with buprenorphine. gogic or hypnopompic hallucinations.

T h e correct response is option A: Increase the main- American Psychiatric Association: Diagnostic and Statistical Manual
tenance dose of methadone of Mental Disorders, Fourth Edition, Text Revision (DSM-IV-TR).
Washington, DC, American Psychiatric Association, 2000, pp 523,
An oral methadone dose of 40 mg is a low dose. 622-629
Better outcomes have been achieved with higher
doses. There is no reason to change to L A A M at this
point. Decreasing or discontinuing tredtment would
likely lead to even poorer patient outcomes. Bupre- 375
norphine may precipitate withdrawal in opioid-depen-
C o m p a r e d with other dementias, the early presenta-
dent patients.. tion in Creutzfeldt-Jakob disease more often includes:

Bickel WK, Amass L: Buprenorphine treatment of opioid dependence: (A) choreoathetosis.


a review. Exp Clin Psychopharmacology 1995; 3:477-489
(B) dysarthria.
Preston KL, Umbricht A, Epstein DH: Methadone dose increase and
abstinence reinforcement for treatment of continued heroin use
(C) extrapyramidal symptoms.
during methadone maintenance. Arch Gen Psyc; iatry 2000; (D) frontal release signs.
57:395-404 (E) myoclonus.
Strain EC, Bigelow GE, Liebson IA, Stitzer ML: Moderate- vs high-
The correct response is option E: Myoclonus
dose methadone in the treatment of opioid dependence. JAMA
1999;281:1000-1005 „"
Myoclonus is a typical manifestation in the early
stages of Creutzfeldt-Jakob disease. Extrapyramidal
symptoms are found in Parkinson's disease; dysarthria
is a sign of head injury; choreoathetosis is a sign of
374
Huntington's disease; and frontal release signs are
A 45-year-old man w h o travels frequently finds that on found in Pick's disease.
returning from his most recent trip to a distant city, he
has had difficulty maintaining daytime alertness and Spar JE, La Rue A: Concise Guide to Geriatric Psychiatry, 3rd ed.
falls asleep easily and at inappropriate times. W h i c h Washington, DC, American Psychiatric Publishing, 2002, p 217
of the following is the most likely diagnosis?

(A) Grcadian rhythm sleep disorder


(B) Dissociative fugue 376
(C) Dyssomnia
(D) Parasomnia ' A 27-year-old man has a 4-month history of persecu-
(E) Narcolepsy tory delusions about being spied on at w o r k by
coworkers. A p a r t from the delusions, he functions rea-
T h e correct response is option A: C i r c a d i a n rhythm sonably well, a n d there \i no evidence of medical ill-
sleep disorder ness or substance abuse. T h e most likely diagnosis is:

Circadian rhythm sleep disorder is a persistent pattern (A) brief psychotic disorder.
of sleep disruption from a mismatch of the patient's (B) delusional disorder. .
endogenous sleep-wake cycle. T h e r e are four vari- (C) major depression with psychotic features.
eties: d e l a y e d sleep-phcse type, jet lag typ^e, shift (D) schizophrenia, paranoid type.
w o r k type, a n d unspecified type. In jet lag type, the (E) schizophreniform disorder.
T h e correct response is option B: Delusional disorder Grisso T, Appelbaum PS: Assessing Competence to Consent to
Treatment A Guide for Ptiysicians and Other Health Professionals.
According to DSM-IV-TR, nonbizarre delusions in the New York, Oxford University Press, 1998
absence of markedly impaired function and bizarre or
odd behavior w o u l d qualify for a diagnosis of delu-
sional disorder. Both schizophrenia a n d schizo-
phreniform disorder are characterized by features 379
such as prominent hallucinations, disorganized An 18-year-old w o m a n is starting her freshman y e a r in
1
speech a n d behavior, ana negative symptoms. An college. She is living at home with her parents. On
episode of brief psychotic disorder must have a dura- campus, she hopes to make friends but usually stays to
tion of less than 1 month. herself, fearing that she will be rejected by her peers.
W h e n called on in class, she avoids eye contact with
American Psychiatric Association: Diagnostic and Statistical Manual of the professor. Although she almost always knows the
Mental Disorders, Fourth Edition, Text Revision (DSM-IV-TR). Wash- answer to questions asked by the professor, she e x p e -
ington, DC, American Psychiatric Association, 2000, pp 323-329
- riences inordinate anxiety that she will make a mis-
take. In private moments, she refers to herself as "the
big nobody." This presentation is most consistent with:
377
N a u s e a a n d other gastrointestinal side effects with (A) avoidant personality disorder.
SSRIs a p p e a r to be related to which receptor subtype? (B) dependent personality disorder.
(Q paranoid personality disorder.
(A) 5-HT receptor
2 (D) schizoid personality disorder.
(8) DA-2 receptor (E) schizotypal personality disorder.
(C) DA-4 receptor
The correct response is option A: Avoidant personal-
(D) H receptor
2
ity disorder
The correct response is option A: 5-HT2 receptor
A number of personality disorders are characterized by
The short form of the promoter for the serotonin (5-HT). a paucity of interpersonal relationships. T h e cluster A
transporter has been reported to predict poor personality disorders (paranoid, schizoid,. a n d schizo-
response or intolerance ;o SSRIs in Caucasians. typal] are often described as "loners." However, patients
with these disorders are not particularly bothered by the
Schatzberg AF: Recent studies of the biology and treatment of
depression. FOCUS 2005;3:14-24 lack of relationships. Individuals with an avoidant per-
sonality disorder are hypersensitive to rejection by oth-
ers. Their main personality trait is timidity. Although they
desire human companionship, their inordinate fear of
378 rejection prevents them from developing relationships.

W h i c h of the following abilities is N O T directly rele- Their hypervigilance about rejection causes them to lack
vant to a person's capacity to make medical decisions? self-confidence and to speak in a self-effacing manner. In
contrast, individuals with dependent personality disorder
(A) Communicate or evidence a choice have a pattern of seeking and maintaining connections
(B) Understand the fads of the situation to important others rather than avoiding and withdraw-
(C) Appreciate how the fads of a situation apply to oneself
ing from relationships.
(D) Choose an option that reflects what most reasonable per-
sons in that situation would do Sadock BJ, Sadock VA: Kaplan and Sadock's Synopsis of Psychiatry,
9th ed. Philadelphia, Lippincott Williams & Wilkins, 2003,
The correct response is option D; Choose an option pp 812-813
that reflects w h a t most reasonable persons in that sit- Cloninger CR, Svrakic DM: Personality disorders, in Kaplan and
uation w o u l d do Sadock's Comprehensive Textbook of Psychiatry, 7th ed. Edited by
Sadock BJ, Sadock VA. Philadelphia, Lippincott Williams &
The currently accepted standards relevant to an indi- Wilkins, 2000, pp 1743-1747
vidual's capacity to make medical decisions do not American Psychiatric Association: Diagnostic and Statistical Manual
include whether or not the patient makes the "correct" of Mental Disorders, Fourth Edition, Text Revision (DSM-IV-TR).
Washington, DC, American Psychiatric Association, 2000, p 724
choice. It is still possible for a fully competent patient
to choose an option that few "reasonable" persons
would choose. W h i l e standards for assessing the
capacity to make a decision v a r y from state to state,
the abilities to communicate a choice, to understand,
and to appreciate are commonly accepted standards.

Section 2: Answers and Explanations


T h e D S M I V - T R criteria for the diagnosis of acute stress
380
disorder include the presence of at least three disso-
T h e oncology team is concerned because a patient
ciative symptoms (a sense of being in a d a z e , deper-
from another culture acts resigned w h e n faced with a
sonalization, derealization, a sense of numbing or
diagnosis of terminal cancer. T h e consulting psychia-
detachment, or dissociative amnesia). Hyperarousal
trist points out that in the patient's culture illness and
symptoms are a common feature of both acute stress
death are part of the normal cycle of life. W h i c h of
the following best describes the use of culture in this disorder a n d PTSD. T h e duration criteria for acute
psychiatric formulation? stress disorder state that the disturbance lasts for a
minimum of 2 days and a maximum of 4 weeks and
(A) Interpretive and explanatory tool occurs within 4 weeks of the traumatic event. Acute
[6] Pathogenic and pathoplastic agent PTSD requires a duration of symptoms less than 3
(C) Diagnostic and nosologic factor
months, and a diagnosis of chronic PTSD requires that
(D) Therapeutic and protective element
the symptoms have been present for at least 3 months.
(E) Management and service instrument
American Psychiatric Association: Diagnostic and Statistical Manual
T h e correct response is option A: Interpretive and
Of Mental Disorders, Fourth Edition, Text Revision (DSM-IV-TR).
explanatory tool
Washington, DC, American Psychiatric Association, 2000,
pp 469-472
There are several w a y s to look at the function of culture
Kaplan HI, Sadock BJ: Synopsis of Psychiatry: Behavioral
in contemporary psychiatry. Culture as interpretive and
Sciences/Clinical Psychiatry, 9th ed. Baltimore, Lippincott Williams
explanatory tool allows for describing nonpathologic & Wilkins, 2003, pp 626-627
behaviors within the context of an individual's culture.
Culture as pathogenic and pathoplastic agent demon-
strates that some psychopathology c a n result from cul-
tural practices. Culture as diagnostic and nosologic 382
factor frames a specific disease as unique to a culture.
Olfactory hallucinations are most commonly associ-
Culture can also be therapeutic a n d protective to one's ated with:
mental health. Culture as a management and service
instrument allows for cultural factors to piny a role in (A) grand mal seizures.
the w a y mental health services are delivered.
(B) hypoparathyroidism.
(C) parietal tumor.
Alarcon RD, Westermeyer J, Foulks EF, Ruiz P: Clinical relevance of (D) partial complex seizures.
contemporary cultural psychiatry. ,1 Nerv Ment Dis 1999; (E) psychotic depression.
187:465-471
T h e correct response is option D: Partial complex
seizures

O l f a c t o r y hallucinations are most commonly associ-


381
ated with partial complex seizures, although they c a n
According to DSM-IV-TR, which of the following char- be reported in patients with psychosis or somatization
acterizes acute stress disorder (ASD)? disorders. Olfactory tumors must also be ruled out.

(A) Lasts a maximum of 8 weeks Other types of hallucinations, such as taste or kines-
(B) Does not involve symptoms of hyperarousal thetic hallucinations, may also occur with partial com-
(C) Often occurs os a result of a minor threat plex seizures. Olfactory hallucinations may also occur
(D) Requires dissociative symptoms for a diagnosis in psychotic depression and typically involve odors of
decay, rotting, or death.
T h e correct response is option D: Requires dissocia-
tive symptoms for a diagnosis , Sadock BJ, Sadock VA (eds): Kaplan and Sadock's Comprehensive
Textbook of Psychiatry, 7th ed, vol 2. Philadelphia, Lippincott
Williams & Wilkins, 2000, p 811
383 For successful processing of traumatic events, three
processes must be accomplished: the person must
A n e w psychologist in t o w n approaches an estab-
engage emotionally with the memory of the trauma;
lished psychiatrist a n d proposes that the psychiatrist
the trauma story must be o r g a n i z e d and articulated in
refer therapy patients to the psychologist in return for
a sequenced a n d coherent fashion; a n d the dysfunc-
a small percentage of fees collected by the psycholo-
gist from treating those patients. This practice is: tional thoughts that commonly occur after trauma must
be addressed^and corrected.
(A) not acceptable because it does not put the patients' inter-
ests first. , Davidson JRT: Effective management strategies for posttraumatic
(B) not acceptable because psychiatrists should refer patients to stress disorder. FOCUS 2003; 1:239-243

psychiatrist therapists.
(C) acceptable because it provides incentives for all parties to
benefit.
(D) acceptable because the psychologist is fairly compensating 385
the psychiatrist. A hospital risk m a n a g e r speaks with y o u about devel-
oping an educational seminar on suicide prevention
The correct response is option A: N o t acceptable
contracts for e m e r g e n c y department staff. As part of
because it does not put the patients' interests first
the seminar, w h i c h of the following w o u l d be a most
Referrals need to be based on the patients' need, in appropriate point to emphasize?
order to preserve trust in the health care system. The
(A) A patient's willingness to enter into a suicide prevention
financial arrangement described in this question cre-
contract indicates readiness for discharge from an emer-
ates a financial incentive for the psychiatrist that could gency setting.
be in opposition to what is necessary for the welfare of (B) In emergency settings, suicide prevention contracts are a
the patient. For instance, there will be situations in helpful method for reducing suicide risk but should not be
which the referral to the therapist may be of financial used to determine readiness for discharge.
interest for the psychiatrist but not congruent with the (C) Using suicide prevention contracts in emergency settings is
patient's needs. A l s o , the therapist could try to recoup not recommended.
the costs for referrals by charging more for the services. (D) Suicide prevention contracts can be useful for assessing the
physician-patient relationship with individuals who are
Hundert EM, Appelbaum PS: Boundaries in psychotherapy: model intoxicated, agitated, or psychotic.
guidelines. Psychiatry 1995; 58:345-356
American Psychiatric Association: Principles of Medical Ethics With The correct response is option C: Using suicide pre-
Annotations Especially Applicable to Psychiatry. Washington, DC, vention contracts in e m e r g e n c y settings is not recom-
American Psychiatric Press, 2001 (section 2, annotation-7} mended

Suicide prevention contracts are only as reliable as


the state of the therapeutic alliance. Thus, with a n e w
384 patient, the psychiatrist m a y not have had sufficient
time to make an adequate assessment or to evaluate
In order for an individual to recover from PTSD after
the patient's c a p a c i t y to form a therapeutic alliance,
interpersonal violence, which of the following processes
is likely to be most helpful? creating little or no basis for relying on a suicide pre-
vention contract. As a result, the use of suicide con-
(A) Go to court and see the perpetrator brought to justice. tracts in e m e r g e n c y settings or with newly admitted
(B) Wait for symptoms to subside with time. and unknown inpatients is not recommended.
(C) Emotionally engage with the memory of the trauma.
(0) Restore sleep with a benzodiazepine. Practice Guideline for the Assessment and Treatment of Patients With
(E) Obtain treatment with eye movement desensitization Suicidal Behaviors (2003), in American Psychiatric Association
Practice Guidelines for the Treatment of Psychiatric Disorders,
techniques. .
Compendium 2004. Washington, DC, APA, 2004, p 905
The correct response is option C: Emotionally e n g a g e
with the memory of the trauma

Section 2: Answers and Explanations I


Dubovsky SL, Buzan R: Psychopharmacology, in Textbook of
386 Geriatric Neuropsychiatry, 2nd ed. Edited by Coffey CE,
A middle-aged man consults a psychiatrist at the rec- Cummings JL. Washington, DC, American Psychiatric Press,
ommendation of his primary care physician because 2000, pp 780-781,796
he has been unable to recover from his deep grief
a n d feelings of abandonment since his divorce 18
months a g o . He endorses many symptoms of major
depression and has w i t h d r a w n from the social activi- 388
ties that he used to enjoy, but he is not suicidal. Of the
A patient w h o is an artist is severely depressed a n d
following things that this patient reports, which would
has occasional passive suicidal thoughts. T h e patient
be the most positive indicator that he w o u l d be able
tells the psychiatrist that health insurance benefits
to benefit from psychodynamic psychotherapy?
have been discontinued and that the patient is no
longer able to p a y the psychiatric bills. T h e psychia-
(A) He is very angry at his ex-wife.
trist has decided not to provide free care to this
(B) He has no family history of psychiatric illness.
patient. T h e psychiatrist can avoid abandoning this
(C) He has been a successful writer.
patient by:
(D) He gets significant support from his two best friends.
(E) He is very religious. (A) giving the patient a written, 30-day notice of termination
T h e correct response is option D: He gets significant and terminating the patient at the end of the 30-day
support from his t w o best friends period.
(B) reducing the frequency of the patient's appointments to
O p t i o n D indicates a capacity for meaningful object help make the patient's bill more affordable.
relationships, a crucial indicator for psychodynamic (C) arranging to commission an artwork by the patient in lieu
psychotherapy. T h e other items listed are significant to of the professional fees.
his history but are not as relevant to suitability for psy- (D) continuing to see the patient until acute depression-related
chotherapy. Success at w o r k is a positive indicator to crises are resolved and then discharging the patient to the
some extent, but it might also indicate that the patient local state-funded community agency clinic.

w a s a workaholic w h o avoids painful feelings or T h e correct response is option D: Continuing to see


awareness by plunging himself into his w o r k . the patient until acute depression-related crises a r e
resolved a n d then discharging the patient to the local
Sadock BJ, Sadock VA (eds): Kaplan and Sadock's Synopsis of
state-funded community a g e n c y clinic
Psychiatry: Behavioral Sciences/Clinical Psychiatry, 9th ed.
Philadelphia, Lippincott Williams & Wilkins, 2003, pp 923-934
W h i l e an a d v a n c e notice of termination can some-
tirnss be sufficient, it is inadequate in the case pre-
sented in this question, because the patient's condition
is severe a n d may be worsening. O p t i o n B will not
387
provide the close monitoring that is necessary for ade-
Lorazepam may be a better choice of a benzodiazepine quate treatment of severe depression with suicidal
than diazepam for an elderly patient because the:
ideation. O p t i o n C describes a bartering arrange-
(A) volume of distribution decreases with age. ment that is highly questionable, as it creates a rela-
(B) hepatic oxidation is unaffected by age. tionship (artist-patron) that m a y not a l w a y s coincide
(C) hepatic conjugation is unaffected by age. with the goals of the doctor-patient relationship.
(D) glomerular filtration rate is unaffected by age.
American Psychiatric Association: Ethics Primer of the American
(E) hepatic blood flow is unaffected by age.
Psychiatric Association. Washington, DC, American Psychiatric
T h e correct response is option C: Hepatic conjugation Association, 2001, p 7
is unaffected by a g e Simon Rl: A Concise Guide to Psychiatry and Law for Clinicians, 3rd
ed. Washington, DC, American Psychiatric Publishing, 2001,
Lorazepam is primarily metabolized by conjugation, pp 32-34
a n d d i a z e p a m by oxidation. Conjugation is unaf- Gutheil TG, Appelbaum PS: Clinical Handbook of Psychiatry and
the Law, 3rd ed. Philadelphia, Lippincott Williams & Wilkins,
fected by a g e , whereas oxidation decreases with a g e , 1
2000, p 152
leading to increases in the half-life of d i a z e p a m .
Volume of distribution increases with acje, whereas
glomerular filtration rate and hepatic blood flow
decline—all of w h i c h would affect both drugs similarly.
389 391
A primary substance abuse prevention p r o g r a m is A 9-year-old b o y is seen in the emergency department
being developed for adolescent girls in a large, met- after attempting to jump out of a moving vehicle. His
ropolitan school district in the United States. T h e parents report that he has had a difficult time in the
school district is diverse, with youths from African, past year. Previously he had done well in school, but
A s i a n , C a u c a s i a n , M i d d l e Ec'stern, a n d N a t i v e n o w he is struggling academically. He often says he
A m e r i c a n families. Based on epidemiologic studies, does not w a n t to go to school, "because I am so stupid
w h i c h ethnic g r o u p of adolescent girls is at greatest and ugly." His teacher has contacted his parents a n d
risk of substance use? 5 informed them that he is falling asleep in class, seems
fatigued, has little to do with his peers, and often does
(A) African American not eat his lunch. T h e child used to play with friends in
(B) Asian American the neighborhood, but for the past 2 months has kept
(C) Caucasian to himself, playing alone in his room or just sitting a n d
(D) Middle Eastern looking out the w i n d o w . A few days earlier, he
(E) Native American ipformed his mother of what to do with his most impor-
tant belongings should he die, but she did not make
T h e correct response is option E: N a t i v e A m e r i c a n
anything out of it. He has generally seemed v e r y
The University of Michigan's annual Monitoring the grouchy a n d "on e d g e . " On questioning, he acknowl-
Future Study (MTF] is a survey of tens of thousands of edges that he w a s hoping to be killed w h e n he tried to
students in grades 8 through 12. T h e MTF documents jump out of the car. T h e most likely diagnosis is:
recent trends in substance use, among them a pro-
(A) borderline personality disorder.
gressively younger a g e of initiation, particularly for
(B) major depressive disorder.
girls. A m o n g girls, drug use is highest in Native (C) oppositional defiant disorder.
Americans a n d lowest in African Americans and (D) separation anxiety disorder.
A s i a n Americans. These differences are thought to be (E) somatization disorder.
attributable to sociocultural and genetic factors.
T h e correct response is option B: Major depressive
Romans SE, Seeman MV: Women's Mental Health: A Life-Cycle disorder
Approach. Philadelphia, Lippincott Williams & Wilkins, 2006, pp
133-137 T h e diagnostic criteria for major depressive disorder
National Institute on Drug Abuse: Gender and ethnic patterns in drug are the same for children and adolescents as adults,
use among high school seniors. NIDA Notes 2004; 18(2). Available except that youths are more likely to present with an
at http://www.nida.nih.gov/NIDA_notes/NNVol18N2/tearoff.html
irritable m o o d . This b o y exhibits loss of interest in
school or play, difficulty sleeping, impaired appetite,
fatigue, feelings of worthlessness, and suicidal
ideation with an attempt, in the presence of dyspho-
390
ria a n d irritability. This is consistent with major depres-
Posttraumatic stress disorder (PTSD] is considered to
sive disorder. [
be chronic PTSD after:
Dulcan MK, Martini DR, Lake MB: Concise Guide to Child and
(A) 1 month. Adolescent Psychiatry, 3rd ed. Washington, DC, American
(B) 3 months. Psychiatric Publishing, 2003, pp 129-131
(C) 6 months. Lewis M (ed): Child and Adolescent Psychiatry: A Comprehensive
(D) 1 year. Textbook, 3rd ed. Philadelphia, Lippincott Williams & Wilkins,
(E) 3 years. 2002, pp 768-770

The correct response is option B: 3 months

Someone w h o has suffered a life-threatening traumatic


event will be diagnosed as having an acute stress dis-
order in the first month after the trauma. If the duration
of the symptoms is less than 3 months, the diagnosis is
acute PTSD, and if it is 3 months or more, chronic PTSD.

American Psychiatric Association: Diagnostic and Statistical Manual


of Mental Disorders, Fourth Edition, Text Revision (DSM-IV-TR).
Washington, DC, American Psychiatric Association, 2000,
pp 463-468

Section 2: Answers and Explanations 181


American Psychiatric Association: Diagnostic and Statistical Manual
392 of Mental Disorders, Fourth Edition, Text Revision (DSM-IV-TR).
T h e family of a 40-year-old retired police officer Washington, DC, American Psychiatric Association, 2000,
pp 740-741
reports that in the past year he has been increasingly
isolative, w i t h d r a w n , a n d bizarre. He has accused his
family of trying to poison him. He put tarps over the
w i n d o w s in his house. He is disheveled and carries a
set of torn papers at all times. He has been observed 394
mumbling and talking to himself. He has no history of Involuntary hospitalization of a patient with schizo-
substance abuse or prior depressive episodes. W h i c h phrenia w h o is hearing voices is justified in w h i c h of
of the following is the most likely diagnosis? the following situations?
(A) Bipolar disorder (A) Hie patient hears a voice that he cannot resist telling him
(B) Delusional disorder to kill himself.
(() Dementia of the Alzheimer's type (B) Third-party payer deems hospitalization appropriate and
(D) Major depression with psychotic features or schizoaffective will pay.
disorder (C) The patient oppears dirty and disheveled.
(E) Schizophrenia (D) Hie patient lacks insight into the nature of his illness.
T h e correct response is option E: Schizophrenia The correct response is option A: T h e patient hears a
His diminished social function, evidence of delusional
voice that he cannot resist telling him to kill himself
thinking, a n d behavior related to delusjons are indica- If the patient's voices are telling him to kill himself and
tive of late-onset schizophrenia. Delusional disorder he feels he must act on these c o m m a n d s , then,
would be the second most likely diagnosis, but although it involves taking a w a y the patient's liberty
patients with this disorder generally do not have hal- (or autonomy), the psychiatrist m a y act in the patient's
lucinations or such extreme loss of social function. best interests (beneficence) a n d hospitalize the patient
American Psychiatric Association: Diagnostic and Statistical Manual against the patient's wishes. Involuntary hospitaliza-
of Mental Disorders, Fourth Edition, Text Revision (DSM-IV-TR). tion of mentally ill individuals brings together the often
Washington, DC, American Psychiatric Association, 2000, conflicting ethical principles of autonomy, benefi-
pp 298-313 cence, a n d informed consent, a m o n g others. T h e
patient w h o is hearing voices felling him to kill himself
m a y or m a y not require involuntary hospitalization.
The current standard for involuntary li. .spitalization in
393 .
most states is that of dangerousness to stl'- or others.
Clinical signs of major depression m a y e m e r g e for a Initially the specified amount of time of hospitalization
patient during bereavement after a parent's death. is determined by the state's law. T h e psychiatrist's
A c c o r d i n g to DSM-IV-TR criteria, w h a t is the earliest judgment of dangerousness is necessarily dependent
time interval after the parent's death that this diagno- on an adequate a n d appropriate clinical e x a m i n a -
sis is g e n e r a l l y made?
tion, w h i c h must be well d o c u m e n t e d . W h e t h e r or not
(A) 1 month a third-party p c y e r or g o v e r n m e n t a g e n c y deems hos-
(B) 2 months pitalization the appropriate c a r e , the psychiatrist's
(C) 3 months ethical obligation is to the patient. A p p e a r i n g dirty
(D) 6 months and disheveled does not in a n d of itself m e a n that the
T h e correct response is option B: 2 months patient is unable to care for himself. T h e concept of
g r a v e disability, if it is a result of mental illness, can
T h e diagnosis of major depressive disorder is not usu- be cause for involuntary hospitalization in some
ally m a d e until the symptoms of the disorder persist for states. G r a v e disability is usually defined as the inabil-
2 months after the death. Studies h a v e indicated that ity to p r o v i d e for one's o w n f o o d , clothing, or shelter.
if depression is not treated a r o u n d this time, it'is still
present 9 months to 1 y e a r after the death. American Psychiatric Association: Ethics Primer of the American
Psychiatric Association. Washington, DC, American Psychiatric
Zisook S, Shuchter SR, Sledge PA, Paulus M, Judd Lif The spectrum Association, 2001, pp 27-32
of depressive phenomena after spousal bereavement J Clin Simon Rl: The law and psychiatry. FOCUS 2003; 1:349-372 (p 358)
Psychiatry 1994; 55(April suppl):29-36
Zisook S, Shuchter SR: Uncomplicated bereavement J Clin
Psychiatry 1993; 54:365-372

182 FOCUS Psychiatry Review: 400 Self-Assessment Question*:


Stein DJ, Hollander E (eds): American Psychiatric Publishing Textbook
395 of Anxiety Disorders. Washington, DC, American Psychiatric
K i d n e y stones a r e most likely to be a side effect of Publishing, 2002, p 315
w h i c h of the following? >

(A) Gabapentin
(B) Lithium 397 •
'(C) Lamotrigine
(D) Topiramate A 49-yeot-old man with schizophrenia taking an
antipsychotic asks to change medication because of
T h e correct response is option' D: Topiramate intolerable side effects. He has had extrapyramidal
side effects and has experienced a 24-pound w e i g h t
T h e p a c k a g e insert for topiramate states that 1.5% of
g a i n . His b o d y mass index is n o w 32.4. His family
adults e x p o s e d to the drug during its development
history is significant for obesity, diabetes, hypercho-
had kidney stones, an incidence t w o to four times that
lesterolemia, hypertension, a n d sudden c a r d i a c
of the general population. T h e formation of kidney death. Of the following medications, which would be
stones m a y be related to reduced urinary citrate the next best one in the management of this patient?
excretion as a result of carbonic a n h y d r a s e inhibition
by the drug. T h e association w a s noted almost exclu- (A) Aripiprazole
sively in patients with epilepsy, although it has also
(B) Olanzapine
been reported in a patient with bipolar II disorder.
(C) Quetiapine
(D) Risperidone
Although lithium can adversely affect the kidneys in
(E) Ziprasidone
several w a y s , the formation of kidney stones is not
associated with lithium therapy. The correct response is option A: Aripiprazole

Takhar J, Manchanda R: Nephrolithiasis on topiramate therapy. Can J W e i g h t gain and metabolic syndromes are potential
Psychiatry 2000; 45:491-493 side effects of the atypical antipsychotics. W e i g h t
Jones MW: Topiramate: safety and tolerability. Can J Neurol Sci
gain tends to occur most frequently with olanzapine
1098; 25(suppl3):S13-S15
and clozapine, occurs moderately with quetiapine
and risperidone, and is least likely to occur with
ziprasidone. Aripiprazole tends to be weight neutral.

396 Of these medications, risperidone is the most likely to


cause extrapyramidal side effects. Ziprasidone m a y
T h e C E O of a large c o m p a n y is fearful of speaking at
cause Q T c interval prolongation and should be used
c; large stockholders' meeting. His fear of public
with caution in patients with a family history of sud-
speaking has been a lifelong disability, but he does
den cardiac death. Because it is weight neutral, ari-
not have anxiety in other social settings. W h i c h of the
piprazole is the next drug of choice for treating this
following is the most reasonable agent to prescribe?
patient's schizophrenia.
(A) A benzodiazepine
(B) A beta-blocker Schatzberg Af^Cole JO, DeBattista C: Manual of Clinical
Psychopharmacology. Washington, DC, American Psychiatric
(C) Buspirone
Publishing, 2005, pp 187-206
(D) A serotonin norepinephrine reuptake inhibitor (SNRI) Mclntyre RS, Konarski JZ: Obesity and psychiatric disorders: fre-
(E) An SSRI quently encountered clinical questions. FOCUS 2005; 3:511-519
T h e correct response is option B: A beta-blocker

T h e beta-adrenergic blockers have been used suc-


cessfully for management of this specific social pho-
bia. Both public speakers a n d music performers have
found them helpful because of the drugs' effectiveness
in decreasing manifestations of anxiety in the auto-
nomic nervous system. T h e y have an a d v a n t a g e over
b e n z o d i a z e p i n e s because the beta-blockers do not
impair concentration or coordination. The length of
time for fhercpeutic effect of the other agents (SSRIs,
SNRIs, a n d buspirone) makes them impractical for
occasional use.

Section 2: Answers and Explanations


Sadock BJ, Sadock VA (eds): Kaplan and Sadock's Comprehensive
398 Textbook of Psychiatry, 8th ed. Philadelphia, Lippincott Williams &
A 34-year-old man w h o is comatose, has myoclonic Wilkins, 2005, p 2829
twitching, and has a serum lithium level of American Psychiatric Association: Practice Guideline for the
Treatment of Patients With Schizophrenia, 2nd ed. Am J
4.2 m E q / L should respond best to w h i c h of the fol-
Psychiatry 2004; 161 (Feb suppl):1-56
lowing treatments?

(A) Activated charcoal


(B) Hemodialysis
(C) Intravenous sodium chloride 400
(D) Osmotic diuresis
The best legal protection for a psychiatrist w h o is
(E) Plasmapheresis
accused of malpractice after a patient's suicide is:
T h e correct response is option B: Hemodialysis
(A) the documentation of the patient's risk factors for suicide
Severe lithium toxicity, as evidenced in this case by recorded in the chart.
the markedly elevated serum level together with the (B) the patient's documented history of an axis II disorder.
(C) a doctor-patient suicide prevention ("no-harm'') contract.
level of neurologic impairment, is best treated with
(D) the patient's family having promised to supervise the
hemodialysis, which is the most effective w a y of
patient closely.
removing lithium rapidly from the body.
The correct response is option A: The documentation of
Jefferson JW, Greist JH: Lithium, in Kaplan and Sadock's
the patient's risk factors for suicide recorded in the chart
Comprehensive Textbook of Psychiatry, 8th ed. Edited by Sadock
BJ, Sadock VA. Philadelphia, Lippincott Williams & Wilkins, 2005,
The best legal protection is thorough documentation
pp 2839-2851
of a patient's risk factors for suicide. This should
include always asking a patient about suicidal
ideation a n d recording chronic a n d acute risk factors
399 as well as facilitating versus inhibiting factors. It has
been noted that as many as 25% of suicidal patients
W h i c h of the following is the most common extrapyra-
d e n y having suicidal ideation, so the risk assessment
midal side effect of antipsychotic medication?
must go b e y o n d simply recording the patient's answer
(A) Akathisia to a question about suicidal thoughts. T h e psychiatrist
(B) Torticollis should take appropriate steps, if indicated, such as
(C) Oculogyric crisis increasing the frequency of visits or even hospitalizing
(D) Neuroleptic ma!;unant syndrome
the patient involuntarily. W h i l e a comorbid axis II dis-
(E) Tardive myoclonus
order may, especially if it predisposes a patient to
T h e correct response is option A: Akathisia impulsivity or uncontrolled r a g e , represent a chronic
risk factor for suicide, documentation of an axis II dis-
T h e most common extrapyramidal side effect of
order in a n d of itself does not protect the psychiatrist
antipsychotic medications is acute neurolepfic-
from a malpractice allegation. A family's assurance
induced akathisia, w h i c h consists of a subjective feel-
that they will supervise a suicidal patient may be well-
ing of restlessness along with restless movements,
intentioned but does not mitigate the psychiatrist's
usually in the legs or feet. Patients often pace contin-
duty to assess thoroughly a patient's risk factors for
uously or move their feet. O v e r a third of patients
suicide. Although frequently used in psychiatric a n d
treated with high-potency dopamine receptor antago-
mental health practice, a suicide prevention or "no-
nists experience akathisia, particularly w h e n these
h a r m " contract generally provides minimal legal pro-
medications are administered in high doses. Akathisia
tection for a psychiatrist accused of malpractice after
appears to be less common w h e n lower doses or
a patient's suicide.
lower-potency dopamine receptor antagonists are
administered and is even more infrequent with atypi- Simon Rl: The suicidal patieritin A Concise Guide to Psychiatry and
cal antipsychotic agents. Dystonic reactions (including Law for Clinicians, 3rd ed. Washington, DC, American Psychiatric
torticollis and oculogyric crises) occur in up jo 10% of Publishing, 2001, pp 143-177.
Simon Rl: The law and psychiatry. FOCUS 2003; 1:349-372
patients treated with high-potency dopamine receptor
(pp 354-355)
antagonists, whereas tardive myoclonus and neu-
• roleptic malignant syndrome are relatively rare.

TR4
FOCUS Psychiatry Review: Answer Sheet

201. A> 301. D


202. C 302. C

203. A 303. C
204. B 304. A
205. B 305. C
206. B 306. A
207., D 307. A
208. E 308. A

209. D 309. A
210. B 310. C
211. A 311. C
212. D 312. D
213. A 313. D
214. E 314. B
215. B 315. B
216. D 316. A
217. A 317. D
1
218. C 318. A
219. A 319. E
220. C 320. B
221. D 321. C
222. C 322. A

223. B 323. D

224. C 324. C
225. B 325. D

226. A 326. A

227. C 327. C
228. D 328. D

229. B 329. B

230. C 330. D

231. C 331. C
m 232. C 332. D

233. A 333. D
234. C 334. B

235. C 335. C
236. C 336. D

237. E 337. A

238. B 338.
339. A
E mm
39. E 239. E

40. A 240. C 340. C


241. D 341. B
M s *
242. D 342. B

i l i i l 243. A 343. C
244. C 344. D
M L
245. A 345. A

46. 246. B 346. A

247. C 347. B

mm 248. B

249. A
348. E
349. C i 399: % -i
t|?|200:HC_?.4|| 250. C 350. B

Answer Sheet 185


FOCUS Psychiatry Review: Blank Answer Sheet

Blank Answer Sheet 187


FOCUS Self-Assessment Editorial Board
Disclosure of Financial Interests or Other Affiliations With Commercial Organizations

Deborah J . Hales, M . D .
Director, American Psychiatric Association Division of Education
No financial affiliations with commercial organizations.

r
Mark H y m a n Rapaport, M . D .
Chairman, Department of Psychiatry, and Polier Endowed Chair in Schizophrenia and Related Disorders, Cedars-
Sinai Medical Center, Los Angeles, California; Vice-Chairman and Professor in Residence, Department of
Psychiatry and Biobehavioral Sciences, David Geffen School of Medicine, UCLA
Grant/Research Support: AstraZeneca; Pfizer, Inc.; Janssen Pharmaceutiea; GlaxoSmithKine; Forest Labs; Lilly; Abbott
Laboratories; Corcept Therapeutics; Cyberonics, Inc.; Novartis; Pharmacia Upjohn; Sanofi Synthelabo; Solvay; Stanley
Foundation; Wyeth; UCB Pharma, Inc.; NIMH; NCCAM, NARSAD, VA Merit Award) Volskwagon Stiefhund. Speaker's
Bureau: Cyberonics, Inc.; Lilly; Porest Labs; GlaxoSmithKline; Janssen; Wyeth; Pharmaceutica; Novartis; Pfizer, Inc.
Consultant: Cyberonics, Inc; Forest Labs; Roche; Pfizer, Inc; Sanofi Synthelabo; Solvay; Wyeth; NIMH; NIDA;
GlaxoSmithKline; NCCR; Janssen Pharmaceutica; Neurocrine Biosciences; Lilly; Novartis; Sumitomo; Brain Cells, Inc.
Tanya R. Anderson, M . D . , Assistant Professor of Psychiatry; Director, Comprehensive Assessment and Response
Training System, University of Illinois at Chicago, Chicago, Illinoise
Speakers Bureau: AstraZeneca.

J. Michael Bostwick, M.D., Mayo Clinic, Rochester, Minnesota


No financial affiliations with commercial organizations.

John H. Coverdale, M . D . , Associate Professor of Psychiatry and Medical Ethics, Baylor College of Medicine,
Houston, Texas
No financial affiliations with commercial organizations.

Arden D. Dingle, M . D . , Associate Professor, Department of Psychiatry and Behavioral Sciences; Training
Director, Child and Adolescent Psychiatry, Emory University School of Medicine, Adanta, Georgia
No financial affiliations with commercial organizations.

Laura B. Dunn, M . D . , Assistant Professor, Department of Psychiatry, University of California, San Diego
No financial affiliations with commercial organizations.

i
Laura Fochtmann, M . D . , Practice Guidelines Medical Editor, American Psychiatric Association; Professor,
Department of Psychiatry and Behavioral Sciences, Department of Pharmacological Sciences, and Department of
Emergency Medicine, Stony Brook University, Stony Brook, New York
No financial affiliations with commercial organizations.
Waguih William IsHak, M.D., Director, Psychiatric Residency Training Program, Interim Medical Director,
Outpatient Psychiatry Service, Cedars-Sinai Medical Center, Los Angeles; Assistant Clinical Professor of
Psychiatry, UCLA
Grant/Research Support for Clinical Trials: AstraZeneca; Janssen. Speakers Bureau: Pfizer, Inc.

James W. Jefferson, M.D., Clinical Professor of Psychiatry, University of Wisconsin Medical School;
Distinguished Senior Scientist, Madison Institute of Medicine, Inc.; Director, Healthcare Technology Systems,
Inc., Madison, Wisconsin
Grant/Research Support: Abbott; Bristol-Myers Squibb; Forest; GlaxoSmithKline; Lilly; Novartis; Organon; Janssen;
Pfizer, Inc.; Solvay; Wyeth. Consultant: GlaxoSmithKline; Novartis; Solvay; UCB. Lecture Honoraria: Bristol-Myers
Squibb; Forest; GlaxoSmithKline; Lilly; Organon; Novartis; Pfizer, Inc.; Solvay; Wyeth. Minor Stock Shareholder:
Bristol-Myers Squibb; GlaxoSmithKline; Scios. Principal: Healthcare Technology Systems, Inc. Other Financial or
Material Support: Various, from time to time, from the pharmaceutical companies listed above.

Affiliations and Disclosures 189


Jerald Kay, M.D., Professor and Chair, Department of Psychiatr)', Wright State University School of Medicine,
Dayton, Ohio
No financial affiliations with commercial organizations.

Scott Y.H. Kim, M.D., Assistant Professor, Department of Psychiatry, Bioethics Program, and Center for
Behavioral and Decision Sciences in Medicine, University of Michigan Medical School, Ann Arbor, Michigan
No financial affiliations with commercial organizations.

Joan A. Lang, M.D., Professor and Chair, Department of Psychiatry, Saint Louis University, St. Louis, Missouri
No financial affiliations with commercial organizations.

Martin H. Leamon, M.D., Associate Professor of Clinical Psychiatry, University of California-Davis, Sacramento
No financial affiliations with commercial organizations.

Alan K. Louie, M.D., Director, San Mateo County Mental Health Services, Psychiatry Residency Training
Program, San Mateo, California
Consultant or Speaker: Abbott Laboratories; Bristol-Myers Squibb; Cephalon; Ciba-Geigy; Lilly; Forest; Glaxo
Wellcome; Janssen; Parke Davis; Sandoz; SmithKline Beecham; Wyeth Ayerst.
1
Annette M. Matthews, M.D., Psychiatrist, Pordand Veterans Affairs Medical Center, Pordand, Oregon; Assistant
Professor of Psychiatry, Oregon Health and Science University, Portland, Oregon; American Psychiatric
Association/Bristol-Myers Squibb Fellow in Public and Community Psychiatry
Other Financial or Material Support: APAJBristol-Myers Squibb fellowship.

Patricia I. Ordorica, M.D., Associate Chief of Staff for Mental Health and Behavioral Sciences, James A. Haley
VA Hospital; Clinical Director, Counterdrug Technology Assessment Center (CTAC) Drug Addiction Study;
Director, Addictive Disorders, and Associate Professor of Psychiatry, University of South Florida, Tair.ja
Consultant/Speaker: Bristol-Meyers Squibb; Pfizer, Inc.

David W. Preven, M.D., Clinical Professor in the Department of Behavioral Sciences and Psychiatry, Albert
Einstein College of Medicine, Montefiore Medical Center, Bronx, New York
Speaker: Pfizer, Inc.; Forest.

Rima Styra, M.D., Toronto General Hospital, University Health Network, Department of Psychiatry, Toronto,
Ontario, Canada
No financial affiliations with commercial organizations.

Christiane Tellefsen, M.D., Clinical Assistant Professor, University of Maryland School of Medicine and Johns
Hopkins University School of Medicine, Baltimore, Maryland
No financial affiliations with commercial organizations.

Eric R. Williams, M.D., Child and Adolescent Psychiatrist, Raleigh, North Carolina
No financial affiliations with commercial organizations.

Isaac Wood, M.D., Associate Professor of Psychiatry and Pediatrics; Associate Dean of Student Activities; Director
of Medical Student Education in Psychiatry, Virginia Commonwealth University School of Medicine, Richmond,
Virginia
No financial affiliations with commercial organizations.

Disclosure of Unapproved or Investigational Use of a Product


FOCUS examination questions may contain information on off-label uses of particular medications. Off-label use
of medications by individual physicians is permitted and common. Decisions about off-label use can be guided by
the evidence provided in the scientific literature and by clinical experience.
Index of Questions by Topic
This index provides a guide for review of questions by topic area. Many questions apply to more than one topic
area but are indexed by a single topic.

Anxiety Disorders:

3,43, 50, 57, 59, 60,63, 94, 138, 161, 164,179, 236,259, 271, 282, 294, 299, 315,318, 358, 359, 390, 396

Bipolar Disorder *
2, 22, 23, 64, 87,101,119, i128,163,171, 200,217,228, 230, 240, 250, 316, 327, 329, 341, 355, 365, 395
i
i i

Child and Adolescent Psychiatry:


31,38, 39,40, 67,177, 191, 203, 206, 220, 223, 245,248,251, 260, 261, 279, 280, 289, 310, 347, 348, 368,
369, 391

Forensic and Ethical Issues in Psychiatry.


8, 10, 26, 27, 30,37,78, 85, 96,182, 189, 190, 212, $21, 242, 244, 252, 253, 258, 273, 278, 306, 326, 342,
353, 370, 378,383, 385, 388, 394,400

Gender, Race, and Culture:

4, 13, 15, 28, 51, 56, 97, 146, 147, 151,167, 218, 239, 254, 266, 285, 290, 302, 304, 309, 321, 337, 380, 389

Geriatric Psychiatry: i
24, 35, 75, 82,99,106, 111, 118, 120, 131,137, 139,162,198, 227, 233,246,249, 287, 291, 295, 312, 338,
367, 375, 387
Major Depressive Disorder:
1, 11,32, 45, 48, 62,79, 84, 90, 92, 123, 126, 134, 140,141, 153,219, 232, 238, 256,286, 296, 300, 339,
371, 377

Neuroscience and Genetics in Psychiatry:

20, 25, 36, 49, 80,102, 112, 124, 156, 159, 165,172, 173,183, 197, 202, 222, 265, 276,281, 305, 313, 345

Personality Disorders:

53,55, 58,65, 86, 89, 105,108,125,129,160,180,185,193,194,211, 214, 263,308,320,322,340,346,379

Posttraumatic Stress Disorder:

41, 42,76,135, 204, 215, 225, 264, 272, 292, 344, 352, 354, 364, 372, 381, 384, 393

Psychopharmacology: {
7, 14, 21, 29,44, 54, 61, 73, 81, 93, 113, 122, 132,143,144,152, 154,168,174, 195, 196, 209, 213, 216,
274, 284, 293, 298, 325,397, 398
Psychosomatic Medicine:

33, 52, 66, 68, 71, 91,100, 103, 114, 115, 116, 148, 158,169,187, 231, 270, 288, 311, 324, 349, 362, 366

Psychotherapy:
19,72,74, 77, 98, 107, 130, 145, 181, 192, 235, 237, 243, 275, 297, 317, 319, 333, 334, 335, 336, 350, 356,
357, 363, 386
Schizophrenia and Other Psychotic Disorders:

6, 9, 17,18, 47, 69,104, 127,133, 142,176,18S, 199,210, 241, 255, 267, 269, 283, 307, 360, 376, 382, 392, 399

Sleep, Sex, and Eating Disorders:


5, 12, 16, 34, 83, 95, 109, 110, 121, 170, 201, 205, 207, 208, 224, 226, 234, 247, 268, 314, 331, 3 4 3 , 351,
361,374
Substance-Related Disorders:
46, 70, 83, 117, 136, 149, 150, 155, 157, 166, 175, 178, 184, 186, 229, 257, 262, 277, 301, 303, 323, 323,
330, 332, 373
Index 191

You might also like